You are on page 1of 279

Contents

1. Physical World, Units and Measurements 1-5


2. Kinematics 6-22
3. Laws of Motion 23-37
4. Work, Energy and Power 38-52
5. Rotational Motion 53-67
6. Gravitation 68-74
7. Mechanical Properties of Solids 75-81
8. Mechanical Properties of Fluids 82-89
9. Thermal Properties of Matter 90-101
10. Thermodynamics 102-113
11. Kinetic Theory of Gases 114-119
12. Oscillations 120-127
13. Waves 128-139
14. Electric Charges and Fields 140-151
15. Electrostatic Potential and Capacitance 152-164
16. Current Electricity 165-175
17. Magnetic Effects of Current 176-186
18. Magnetism and Matter 187-191
19. Electromagnetic Induction 192-197
20. Alternating Current 198-204
21. Electromagnetic Waves 205-209
22. Ray Optics and Optical Instruments 210-215
23. Wave Optics 216-221
24. Dual Nature of Radiation and Matter 222-226
25. Atoms 227-232
26. Nuclei 233-237
27. Semiconductor Electronics Materials, Devices and Simple Circuits 238-242
28. Communication Systems 243-245

Practice Sets (1-3) 247-278


1
Physical World, Units
and Measurements
1. Strength of gravitational force compared to Sol. (b)
strong nuclear force [17 Sep. 2020, Shift-II] Dimensional formula of work,
(a) 10− 39 (b) 10− 13 [W ] = [F] × [s]
(c) 10− 2 (d) 1 = [MLT−2 ] [L] = [ML2 T−2 ]
Sol. (a) Similarly, dimensional formulae of
The order of gravitational force and strong Angular momentum [L] = [m] [v] [r]
nuclear force is found to be 10−49 N and 10−10 N, = [M] [LT−1 ] [L]
respectively = [ML2 T−1 ]
i.e., FG = 10−49 N and Fnuclear = 10−10 N Torque [τ] = [F] × [r]
FG 10−49 = [MLT−2 ] [L] = [ML2 T−2 ]
∴ = −10 = 10−39
Fnuclear 10 Potential energy [U] = [m] [g] [h]
= [M] [LT−2 ] [L] = [ML2 T−2 ]
2. Dimensions of ε 0 are [17 Sep. 2020, Shift-II] Linear momentum [p] = [m] [v] = [M] [LT−1 ]
(a) [M − 1L−3 T4 A 2 ] (b) [M0L− 3 T3 A 3 ] = [MLT−1 ]
(c) [M−1L− 3 T3 A] (d) [M− 1L− 3 T A 2 ] 1
Kinetic energy [K ] = [m] [v2 ]
Sol. (a) 2
We know that, force between two charges q1 = [M] [LT−1 ]2 = [ML2 T−2 ]
and q 2 placed at distance r is given as Velocity [v] = [LT−1 ]
1 q q Clearly, dimensional formulae of work and
F= ⋅ 122
4 π ε0 r torque are same, hence option (b) is correct.
q1 q 2 [q ] [q ] L
⇒ ε0 = ⇒ [ε0 ] = 1 22 4. To estimate g from g = 4 π 2 , error in
4 π Fr 2 [F] [r ] T2
[AT] [AT] measurement of L is ± 2% and error in
= = [M −1 L−3 T 4 A 2 ]
[MLT−2 ] [L2 ] measurement of T is ± 3%. The error in
estimated g will be [18 Sep. 2020, Shift-II]
3. Choose the physical quantity pair with the (a) ± 8% (b) ± 5%
same dimensions. [18 Sep. 2020, Shift-I] (c) ± 3% (d) ± 6%
(a) Angular momentum and work Sol. (a)
(b) Work and torque L
Q g = 4π2
(c) Potential energy and linear momentum T2
(d) Kinetic energy and velocity Percentage error in the estimation of g is given as,
2 AP EAMCET Chapterwise Physics

∆g ∆L ∆T
× 100 = ±  × 100 + 2 × 100 Newton discovered laws of motion, belongs to
g  L T  England.
= ± (2% + 2 × 3%) Einstein belongs to Germany, who simplified
the laws of photoelectric effects.
= ± (2% + 6%) = ±8%
8. Which of the following is not a unit of time?
5. The SI unit of length is ‘metre’. Suppose we
adopt a new unit of length which equals [23 Sep. 2020, Shift-I]
x metre. Then, the area of 1 m 2 expressed in (a) Lunar month (b) Light year
terms of new unit has a magnitude (c) Leap year (d) Microsecond
[21 Sep. 2020, Shift-I] Sol. (b)
2 1 1 Physical quantity
(a) x (b) x (c) (d) 2 Unit
x x
Lunar month Time
Sol. (d)
SI unit of length = metre (m) Light year Distance
New unit of length = x metre Leap year Time
1
Hence, 1 m = new units
x Microsecond Time
∴ Area, 1 m 2 = 1 m × 1 m
1 1 1 9. Which among the following has dimensions
= × = 2 (in new units) of charge? [23 Sep. 2020, Shift-I]
x x x
(a) ε0 E / d s (b) ε0 E ⋅ d s
6. The unit of magnetic induction is µ ε
(c) 0 E ⋅ d s (d) 0 E ⋅ d s
[21 Sep. 2020, Shift-I] ε0 µ0
−2 −1
(a) Wb m (b) Wb m (c) Wb A (d) Wb Sol. (b)
Sol. (a) From the Gauss’s law, E ⋅ ds =
Qenc
Magnetic flux passing through a surface is given ε0
as or Qenc = ε0 E ⋅ ds
φ = BA
The above relation is true if dimension formula
where, A = area and B = magnetic induction. of LHS is equal to dimension formula of RHS,
φ then
∴ B = Wb/m 2
A [ε0 E ⋅ ds] = [Qenc ]
∴ Unit of magnetic induction B is Wbm −2 .
10. If the charge of electron e, mass of electron
7. Which of the following statement is correct? m, speed of light in vacuum c and Planck’s
(A) Becquerel, who discovered natural constant h are taken as fundamental
radioactivity, belongs to France. quantities, then the permeability of vacuum
(B) Marconi, who discovered wireless µ 0 can be expressed as [20 April 2019, Shift-I]
telegraphy, was an American. h hc h mc 2
(a) (b) (c) (d)
(C) Newton was an American, who mc 2 me 2 ce 2 he 2
discovered the laws of motion. Sol. ( c)
(D) Einstein belongs to England, who We can expressed the permeability of vacuum,
simplifies the laws of photoelectric effects. µ 0 ∝ e a mb c c hd
[22 Sep. 2020, Shift-I] or µ 0 = k e a mb c c hd …(i)
(a) A (b) B (c) C (d) D Where, k is a dimensional constant.
Sol. (a) As we know that,
Becquerel discovered natural radioactivity, Dimension of µ 0 = [M L T−2 A −2 ] ,
belongs to France. e = [A T] , m = [M] ,
Marconi discovered wireless telegraphy belongs c = [L T − 1 ] and h = [M L2 T − 1 ] ,
to Italy.
Physical World, Units and Measurements 3

Putting the dimension of various physical Sol. (b)


quantities in Eq. (i), we get
Here, ∆t 1 = (2. 00 ± 0.02)s and ∆t 2 = (4.00 ± 0.02)s
[M L T−2 A −2 ] = k[A T]a [M]b [L T−1 ]c ][M L2 T−1 ]d
Given, T = (∆t 1) (∆t 2)
[M L T−2 A −2 ] = k[M]b + d [L]c + 2 d [T]a − c − d [A]a
= (2.00) (4.00) = 2.828427 s
Compairing the powers of M, L, T and A on the
both sides, we get Now, according to the relative error of product
b + d =1 …(ii) ∆T  1 ∆t1 1 ∆t2 
± =±  + 
c + 2d = 1 …(iii) T  2 t1 2 t2 
a−c−d=−2 …(iv) Since, ∆t1 and ∆t2 have 3 significant figures,
a=−2 …(v) then T also has 3 significant figures. Hence,
After solving the Eqs. (ii), (iii), (iv) and (v), we get after rounding off,
a = − 2 , b = 0 , c = − 1 and d = 1 ∴ T = 2. 83 s
∴ µ0 = 2
h 1  0.02 0.02
Now, ∆T = ±  +  × 2. 8284
ce 2  2.00 4.00 
So, the permeability of vacuum µ 0 can be ∆T = 0.02121 s
h
expressed as, 2 . So, after rounding off,
ce
∆T = 0.02
11. A physical quantity obtained from the ratio Hence, T = (2. 83 ± 0.02) s
of the coefficient of thermal conductivity to So, most matched answer is (b).
the universal gravitational constant has a
dimensional formula [M 2a L4 b T 2c K d ], then
13. N divisions on the main scale of a vernier
calipers coincide with (N + 1) division of the
a+b
the value of − d is vernier scale. If each division of main scale is
c+b [20 April 2019, Shift-II] a units, then the least count of the calipers is
3 1 3 1 [21 April 2019, Shift-II]
(a) + (b) − (c) − (d) +
2 2 2 2 a a Na
(a) (b) a (c) (d)
Sol. (d) N N+1 N+1
Dimensional formula of thermal conductivity Sol. (c)
[k] = [M1 L1 T−3 K −1 ] . Given, N divisions of main scale,
Dimensional formula of universal gravitational N MSD = (N + 1) divisions of vernier scale,
constant, [G] = [M −1 L3 T−2 ] (N + 1)VSD
[k] N
Now, = [M 2 L−2 T−1 K −1 ] ⇒ Value of of 1VSD = MSD
[G] N +1
Compare above equation with [M 2 a L4 b T2 c K d ] Least count of vernier calipers,
1 1
This will give us, a = 1, b = − , c = − and d = −1 LC = 1 MSD − 1VSD
2 2 N
= 1 MSD − MSD
1−
1 N +1
a+ b 2 − (−1)
Now, − d=  N + 1 − N  1 
c+ b 1 1
− − =  MSD =   MSD
2 2  N +1   N + 1
a+ b 1 LC =
a
(Given,1MSD = a)
or −d= or
c+ b 2 N +1

12. Two intervals of time are measured as 14. If 1% and 2% are the errors in the
∆t1 = (2.00 ± 0 .02)s and ∆t2 = (4 .00 ± 0 .02)s . measurement of mass and density of a cube
The value of (∆t1) (∆t2) with correct respectively, then the error in the
measurement of length is [22 April 2019, Shift-I]
significant figures and error is
[21 April 2019, Shift-I] (a) 1% (b) 3%
(c) 2% (d) 4%
(a) (2.828 ± 0.01)s (b) (2.83 ± 0.01)s
(c) (2.828 ± 0.0075)s (d) (2.83 ± 0.0075)s
4 AP EAMCET Chapterwise Physics

Sol. (a) zeros on the right of the decimal point but to


Given, error in measurement of mass, the left of the first non-zero digit are not
∆m significant. [23 April 2019, Shift-I]
= 1%
m (a) Both (A) and (R) are true and (R) is the correct
∆d explanation of (A).
and error in measurement of density, = 2%
d (b) Both (A) and (R) are true but (R) is not the correct
mass m explanation of (A).
Now, density, d = ⇒d =
volume V (c) (A) is true but (R) is false.
∴Volume of cube, V = l 3 (d) (A) is false but (R) is true.
1

d = 3 ⇒ l =  
Then,
m m 3 Sol. (a)
l  d If the number is less than one, the zero on the
Then the error in measurement of length, right of decimal point but to the left of the first
∆l 1 ∆m ∆d non-zero digit are not significant, i.e. in
× 100 =  × 100 + × 100 0.00764, the underlined zeroes are not
l 3 m d 
significant. The number of significant digit is 3.
∆l 1
× 100 = (1 + 2) × 100 Both assertion (A) and reason (R) are true
l 3 and reason (R) is the correct explanation of
∆l assertion (A).
= 1%
l
17. Two resistance 60.36 Ω and 30.09 Ω are
15. If A represents density, B represents velocity, connected in parallel. The equivalent
C represents specific heat capacity and D resistance is [22 April 2018, Shift-I]
represents wavelength, then the quantity (a) 20 ± 0.08 Ω (b) 20 ± 0.06 Ω
having the dimensions of product of A , B, C (c) 20 ± 0.03 Ω (d) 20 ± 010
. Ω
and D is [22 April 2019, Shift-II]
Sol. (a)
(a) Stefan’s constant (b) Boltzmann’s constant
Equivalent resistance is
(c) thermal conductivity (d) universal gas
R1 R2 60 × 30
constant Rparallel = = = 20Ω
R1 + R2 60 + 30
Sol. (c)
and tolerance value is
According to the question,
 ∆R ∆R2 ∆R1 + ∆R2 
Given, A = density, B = velocity, C = specific heat ⇒ ∆RP = RP  1 + − 
capacity and D = wavelength  1R R2 R1 + R2 
0.36 0.09 0.36 + 0.09 
So, dimensions of A, B, C and D are = 20 + − 
A = [ML−3 T0 ], B = [M 0LT− 1 ] ,  60 30 90 
C = [M 0L2 T− 2K − 1 ] and D = [M 0LT0 ] = 20{0.006 + 0.003 − 0.005} = 0.085 Ω
Hence, dimension of ABCD = So, resistance in parallel is RP = 20 ± 0.08
[ML− 3 T0 ] [M 0LT− 1 ] [M 0L2 T − 2K − 1 ] [M 0LT0]
= [ML( − 3 + 1 + 2 + 1) T( − 1 − 2) K − 1 ] = [MLT− 3K − 1 ]
18. In a system, unit of mass is A kg, length is
or ABCD = [MLT− 3K − 1 ] … (i) B m and time is C s, then the value of 10 N in
Now, dimension of each given option, this system is [22 April 2018, Shift-II]
(a) Stefan’s constant = [ML0 T− 3K − 4 ] (a) 10 A −1 B−1C −2 (b) 10 A −1 B−1C 2
(b) Boltzmann’s constant = [ML2 T− 2K − 1 ] (c) 10 ABC −2 (d) 5 A −1 BC 2
(c) Thermal conductivity = [MLT− 3K − 1 ] Sol. (b)
(d) Universal gas constant = [ML2 T − 2K − 1 ] Using, N1 u1 = N 2 u2
So, the dimension of product of A, B, C and D is
A⋅ B
10 kg 2 = N 2  2 
m
same as dimension of thermal conductivity.
Hence, option (c) is correct. s  C 
10 ⋅ C 2
16. Assertion (A) The number 0.00764 has ⇒ N2 = = 10 A− 1 B− 1 C 2
A⋅ B
three significant figures.
So, numerical value of 10 N in given system is
Reason (R) If the number is less than 1, the 10 A− 1 B− 1 C 2 .
Physical World, Units and Measurements 5

19. Assertion (A) Energy per unit volume and Sol. (b)
angular momentum can be added In 74.083 number of significant figures = 5
dimensionally. In 0.029 number of significant figures = 2.
Reason (R) Physical quantities having same In 0.002407 number significant figures = 4
dimensions can be added or subtracted. In 2.47 × 107 number of significant figures = 3
[23 April 2018, Shift-I]
(a) Both (A) and (R) are true and (R) is the correct 21. If A represents Boltzmann constant, B
explanation of (A) represents Planck’s constant and C
(b) Both (A) and (R) are true but (R) is not the correct represents speed of light in vacuum, then
explanation of (A) the quantity having the dimensions of
(c) (A) is true but (R) is false A 4 B−3C −2 is [24 April 2018, Shift-I]
(d) (A) is false but (R) is true (a) universal gas constant
(b) specific heat capacity
Sol. (d)
(c) stefan’s constant
The units of energy per unit volume is J/m3 or (d) heat energy
kg / m- s2 and that for angular momentum is
kg - m2 / s. Sol. (c)
Since, they are not dimensionally equal, they Dimension of Boltzmann’s constant
cannot be added, so the Assertion (A) is false. J ML2 T−2
Only those physical quantities that have same A= = = [ML2 T−2K −1 ]
dimensions can be added or subtracted, so K K1
Reason (R) is true. Dimension of Planck constant
Hence, the correct option is (d). B = Js = [ML2 T − 1 ]
Dimension of speed of light,
20. Match the measurements given in List I with
the number of significant figures given in C = [LT − 1 ]
List II. [23 April 2018, Shift-II] Desired dimension = A4 B− 3 C − 2
⇒[M L2 T − 2 K −1 ]4 [M L2 T − 1 ]−3 [L T − 1 ]−2
List I List II ⇒[M1 L0 T − 3 K −4 ]
(A) 74.083 I. 3 Stefan constant (σ) is given by
E
(B) 0.029 II. 4 E = σT 4 ⇒ σ = 4
T
(C) 0.002407 III. 2 where, E = energy per unit area per unit time
and T = absolute temperature.
(D) 2.74 × 10 7
IV. 5
[M L2 T− 2 ]
⇒ Dimension, σ = 2
The correct answer is [L ] [T ] K 4 ]
A B C D A B C D σ = [M L0 T− 3K − 4 ]
(a) IV II III I (b) IV III II I So, desired dimension is of Stefan constant.
(c) III IV II I (d) I II III IV
2
Kinematics
N
1. An airplane flies 400 m north and 300 m j
south and flies 1200 m upwards, then net
displacement is [17 Sep. 2020, Shift-I] 45º 10√2 m/s
45º
(a) 1400 m W E
i
(b) 1500 m
(c) 1200 m
(d) 1100 m
Sol. (c) S
The various displacements of airplane is shown Initial velocity of body,
below u = 10 2 cos 45° $i + 10 2 cos 45° $j
= 10 $i + 10$j = 10 ($i + $j)
N Acceleration applied on body in south direction,
300 m
a S = 2 (− $j) = − 2 $j
400 m 1200 m W E
100 m Upward ∴Velocity of body after 5s is given as
v = u + a S t = 10($i + $j ) + (−2$j ) × 5
S
= 10$i + 10$j − 10$j = 10 $i
Net displacement travelled by airplane, Hence, body will move towards east with the
d = 1002 + 12002 velocity 10 ms −1 .
= 1450000 3. A particle is projected with a velocity v such
= 120416
. m that its range on the horizontal plane is
− 1200 m
~ twice the greatest height attained by it. The
range of the projectile is (g = acceleration
2. A body is moving with an initial velocity of due to gravity) [17 Sep. 2020, Shift-I]
10 2 ms −1 in the north-east direction. If it is 4v 2 4g v2 4v 2
(a) (b) (c) (d)
subjected to an acceleration of 2 ms −2 5g 5v 2 g 5g
directed towards the south, then velocity of
the body after 5s is [17 Sep. 2020, Shift-I] Sol. (a)
−1
(a) 10 ms , towards east Velocity of particle = v
(b) 10 ms −1 , towards north If θ is angle of projection such a way,
(c) 10 ms −1 , towards south R = 2H
(d) 10 ms −1 , towards north-east v2 sin 2θ 2⋅ v2 sin2 θ
⇒ =
g 2g
Sol. (a)
2sinθ cosθ = sin2 θ
The given situation is shown in the following sin θ
figure ⇒ 2= ⇒ 2 = tanθ ⇒ tanθ = 2
cosθ
Kinematics 7
v
0 = v − gt ⇒ t =
√5 g
2
Total time taken to reach the ball from point of
θ projection to reach at point B is given as
2v
1 t1 = t + t = 2t ⇒ t1 =
2 1 g
∴ sinθ = ⇒ cosθ =
5 5 If t2 be the time taken by the ball to reach from
v2 sin2θ v2 ⋅ 2sinθ cosθ point B to point C, then
∴ Range = =
1
g g h = v t2 + gt22 ⇒ gt22 + 2vt2 − 2h = 0
2 1 2
v × 2×
2
×
− 2v ± 4v2 + 8 gh
2
= 5 5 = 4v −v v2 + 2gh
∴ t2 = = +
g 5g 2g g 2
4. The ratio of displacement to distance for a [taking +ve sign because time is positive]
moving particle is [17 Sep. 2020, Shift-II] 2v v v2 + 2gh
∴ Total time, T = t1 + t2 = − +
(a) always less than one g g 2
(b) always greater than one v v 2gh v  2gh 
(c) always one = + 1+ 2 = 1 + 1 + v2 
(d) one or less than one
g g v g  

Sol. (d) 6. Which of the following options is correct for


When the body moves along straight line, then the object having a straight-line motion
distance and displacement between two points represented by the following graph?
is same. In this case, ratio of displacement to
[18 Sep. 2020, Shift-I]
distance is one.
If a body moves on curved path, then D
displacement is always less than distance
between any two points. In this case, ratio of C
displacement to distance is less than one. ∆t
B
5. A ball is projected upwards from a height h
above the surface of the earth with velocity A
v. The time at which the ball strikes the ground
is [17 Sep. 2020, Shift-II] O s
v  2 gh  v  2h  (a) The object moves with constantly increasing
(a) 1 + v 2 
(b) 1 − 1 + g  velocity from O to A and then it moves with
g  g  
constant velocity.
v 2 gh  v  2g  (b) Velocity of the object increases uniformly.
(c) 1+ 1+ (d) 1 + v2 +
g  v 2  g  v 2  (c) Average velocity is zero.
(d) The graph shown is impossible.
Sol. (c)
The given situation is shown in the following Sol. (c)
figure. According to given displacement-time graph, in
a certain time interval ∆t, displacement of the
A
object becomes zero.

v D
B v
C
∆t
h B

C v'
A
If time taken by the ball to reach at highest
point A is t, then O s
8 AP EAMCET Chapterwise Physics

Hence, average velocity of object 9. When a player thrown a ball, it reaches the
Total displacement 0
= = =0 other player in 4 s. If the height of each
Time interval ∆t player is 1.8 m, the maximum height
7. The displacement of a body when it covers a attained by the ball above the ground is
distance of C/4 (where, C is circumference) [18 Sep. 2020, Shift-II]
along the circumference of the circle of (a) 19.4 m (b) 20.4 m
radius r with a uniform speed u is (c) 21.4 m (d) 22.4 m
[18 Sep. 2020, Shift-I] Sol. (c)
(a) r (b) r 2 Time taken to reach the ball from one player to
r another is equal to time of flight.
(c) 2r (d)
2 Hence, time of flight, T = 4 s
2usinθ
Sol. (b) = 4 ⇒ usinθ = 2g …(i)
g
The given situation is shown in the figure
Height attained by the ball above the ground
B = height of either player + maximum height
C/4 u2 sin2 θ (usinθ)2
= 1.8 + = 1.8 +
2g 2g
(2g)2
O r A = 1.8 +
2g
= 1.8 + 2g = 1.8 + 2 × 9.8 = 1.8 + 19.6 = 21.4 m

10. A swimmer can swim in still water with


C v
When body covers distance, where C is speed v and the river flowing with velocity .
4 2
circumference, then it reaches from point A to B To cross the river in shortest time, he should
as shown in the figure. swim making angel θ with the upstream.
∴ Displacement, AB = OA2 + OB2 What is the ratio of the time taken to swim
across in the shortest time to that in
= r2 + r2 = 2r 2 = r 2
swimming across over shortest distance?
8. Simultaneously, from the top of a tower, [18 Sep. 2020, Shift-II]
when ball-1 is thrown horizontally and (a) cos θ (b) cot θ
ball-2 is just dropped, in the absence of air (c) sin θ (d) tan θ
resistance which among the following
Sol. (c)
options is correct? [18 Sep. 2020, Shift-I]
If d be the width of river and θ be the angle
(a) Ball-1 reaches the ground first.
made with the upstream.
(b) Ball-2 reaches the ground first.
Therefore, angle made with normal stream
(c) Both will reach the ground simultaneously.
= 90°− θ
(d) Either ball-1 or ball-2 reach the ground first
Hence, by shortest time method,
depending on which ever is heavier.
d
Sol. (c) time taken, t = …(i)
v
When ball-1 drops from the top of a tower By shortest distance method,
horizontally and at the same time, ball-2 is d
dropped vertically downward from the tower, time taken, t ′ =
v cos(90°− θ)
then both balls will move under the effect of
same value of gravitational acceleration g. d
t′ = …(ii)
Therefore, both balls will reach at the same time vsinθ
on the ground. t d/v
∴ = = sinθ
t ′ d / vsinθ
Kinematics 9

11. A train of 150 m length is going towards Sol. (d)


north direction at a speed of 10 ms − 1 . A According to question, the given situation is
parrot flies at the speed of 5 ms − 1 towards shown in the figure.
south direction parallel to the railway track.
The time for which the parrot flies alongside
the train is [21 Sep. 2020, Shift-I] 100 m Tower
(a) 12 s (b) 30 s (c) 10 s (d) 5 s
θ
Sol. (c)
200 m 200 m
According to question,
Relative velocity of parrot with respect to the Here, range, R = 200 + 200 = 400 m
train, vpt = vt − vp
Maximum height, H = 100 m
= 10 − (−5)
If θ be the direction of shot with respect to
= 10 + 5 = 15m/s horizontal, then
∴Time taken by the parrot to fly along side the u2 sin 2θ u2 sin2 θ
train is given as R= and H =
length of train 150 g 2g
t = = = 10 s
v pt 15 u2 sin 2θ
R 400 2sin 2θ
∴ = 2 g2 ⇒ =
12. A stone falls freely such that the distance H u sin θ 100 sin2 θ
covered by it in the last second of its motion 2g
is equal to the distance covered by it in the 2 × 2sinθ cosθ
⇒ 4= [Q sin 2θ = 2sinθ cosθ]
first 5 s. It is in air for ....... s. sin2 θ
[21 Sep. 2020, Shift-I] ⇒ 1 = tanθ ⇒ tan 45° = tanθ
(a) 12 (b) 13 (c) 25 (d) 26 ⇒ 45° = θ ⇒ θ = 45°
Sol. (b) 14. A ball is dropped from a spacecraft revolving
Since, stone falls freely, hence its initial velocity, around the earth at a height of 120 km.
u= 0 What will happen to the ball ?
Distance covered by the stone in first 5s is given [21 Sep. 2020, Shift-II]
as
(a) It will continue to move with same speed along
1 1
h1 = ut + gt 2 = 0 × 5 + × 9.8 × 52 the original orbit of spacecraft
2 2 (b) It will move with the same speed tangentially to
= 122.5m the original orbit
Since, distance travelled in last second (nth (c) It will fall down to the earth gradually
second) is equal to distance travelled in first 5 s, (d) It will go very far in space
hence
1 Sol. (a)
h1 = u + g(2n − 1)
2 When ball is dropped from a spacecraft
1 revolving around the earth at a height 120 km,
⇒ 122.5 = 0 + × 9.8 (2n − 1) then speed of ball is same as speed of spacecraft
2
because orbit of ball is same as spacecraft, hence
122.5 × 2
⇒ 2n − 1 = it will continue to move with same speed along
9.8 the original orbit of spacecraft.
⇒ 2n − 1 = 25 ⇒ 2n = 25 + 1
⇒ 2n = 26 ⇒ n = 13 15. Which of the following remains constant for
a projectile fired from the earth ?
13. A shot is fired from a point at a distance of [21 Sep. 2020, Shift-II]
200 m, from the foot of a tower 100 m high,
(a) Momentum
so that it just passes over it. The direction of
(b) Vertical component of velocity
shot with respect to the horizontal is
(c) Kinetic energy
[21 Sep. 2020, Shift-I]
(d) Horizontal component of velocity
(a) 30° (b) 60° (c) 70° (d) 45°
10 AP EAMCET Chapterwise Physics

Sol. (d) 18. A particle is projected with velocity 2 gh, so


There is no acceleration in the horizontal
that it just flies over two walls of equal
direction during projectile motion and only
gravitational acceleration g works in vertically
height h and 2h distance apart from each
downward direction. Hence, horizontal other. Find the time for which the particle
component of velocity remains constant. flies between the walls. [22 Sep. 2020, Shift-II]
4h h 4g g
16. The graph between two quantities P and Q is (a) (b) (c) (d)
g g h h
a straight line when [22 Sep. 2020, Shift-I]
(a) PQ = constant (b)
P
= constant
Sol. (a)
Q Velocity of projection,
P v = 2 gh
(c) PQ = constant
2
(d) 2 = constant
Q
∴ vx = v cosθ = 2 gh cosθ
Sol. (b) ∴ Time taken by the projectile to cover interwall
When the graph between two quantities P and Q distance,
is straight line as shown in the figure, then 2h 2h
slope of the graph remains constant. t= =
vx 2 gh cosθ
h
⇒ t= ⋅ sec θ …(i)
P g
Vertical velocity at the top of the wall is given as
P
v ′2y = v2y − 2gh = ( 2gh sinθ)2 − 2gh
θ
Q = 4 ghsin2 θ − 2gh
P v ′ = 2gh(2sin2 θ − 1)
2
y
i.e. slope = tanθ = = constant
Q ∴ v′y = 2gh (2sin2 θ − 1)
2v′ y
17. A body starts from rest and moves with ∴ t=
uniform acceleration. If the distance g
travelled by it in the first 2 s is x1 and in the 2 2gh(2sin2 θ − 1)
t= …(ii)
next 2 s is x 2, then x1 and x 2 are related as g
[22 Sep. 2020, Shift-I] From Eqs. (i) and (ii), we get
(a) x1 = x2 (b) x1 = 2 x2 (c) 2 x1 = x2 (d) 3 x1 = x2 h 2 2gh(2sin2 θ − 1)
secθ =
Sol. (d) g g
If a be the acceleration of the body, then Squaring both side,
distance travelled by the body in 2 s is given as,
h 4 × 2gh (2sin2 θ − 1)
1 1 ⇒ sec2 θ =
x1 = ut + at 2 = 0 × 2 + a × (2)2 g g2
2 2
x1 = 2a …(i) ⇒ 1 = 8 cos2 θ(2sin2 θ − 1)
Velocity of body at the end of 2s is given as, ⇒ 1 = 8 cos2 θ[2(1 − cos2 θ) − 1]
v = u + at = 0 + a × 2 ⇒ 16 cos4 θ − 8 cos2 θ + 1 = 0
v = 2a …(ii) (4 cos2 θ − 1)2 = 0
Distance travelled by the body in next 2s is given as, 1
1 ⇒ cos2 θ =
x 2 = vt + at 2 4
2
1
1 ⇒ cosθ = = cos 60° ⇒ θ = 60°
= 2a × 2 + a × (2)2 [from Eq. (ii)] 2
2
= 4a + 2a ∴ From Eq. (i),
x 2 = 6a = 3 × 2a h h 4h
t= sec 60° = ⋅2 =
⇒ x 2 = 3 × x1 [from Eq. (i)] g g g
⇒ x 2 = 3x1
Kinematics 11

19. The displacement of a particle moving with 22. A train is moving towards north. At one
uniform acceleration in time t is given by place, it turns towards north-east. Here, we
s = 30 t + 5 t2, its initial velocity is observe that [23 Sep. 2020, Shift-I]
[23 Sep. 2020, Shift-I] (a) the radius of curvature of outer rail will be greater
(a) 35 ms −1 (b) 30 ms −1 (c) 40 ms −1 (d) 45 ms −1 than that of the inner rail
(b) the radius of curvature of inner rail will be greater
Sol. (b) than that of the outer rail
Given that, displacement of a moving particle at (c) the radius of curvature of the outer and inner rails
any time t is given by will be the same
s = 30t + 5t 2 (d) the radius of curvature of inner rail will be infinite
ds d
As we know, velocity = = (30t + 5t 2) Sol. (a)
dt dt
A train is moving towards north. At one place it
v = 30 + 10t turns towards north-east. Here, we observe that
At t = 0, the radius of curvature of outer rail will be
i.e. initial velocity, u = 30 + 10 × 0 = 30 m/s greater than that of the inner rail.
20. The acceleration at the end of 2s, of a This is because the velocity of wheel on outer
particle whose motion is represented by the rail is greater than the velocity of wheel on
inner rail.
equation s = 4 t3 − 8 t2 + 5 t + 4 is
[23 Sep. 2020, Shift-I] 23. The velocity of an object moving in a straight
(a) 32 ms −2 (b) 40 ms −2 (c) 37 ms −2 (d) 35 ms −2 line path is given as a function of time by
v = 6 t − 3 t2, where v is in ms −1 , t is in s. The
Sol. (a)
Given that, s = 4t 3 − 8t 2 + 5t + 4
average velocity of the object between, t = 0
and t = 2 s is [20 April 2019, Shift-I]
At any instant velocity,
ds d (a) 0 (b) 3 ms −1 (c) 2 ms −1 (d) 4 ms −1
v= = (4t 3 − 8t 2 + 5t + 4)
dt dt Sol. (c)
= 12t 2 − 16t + 5 Given, velocity, v = 6t − 3t 2
As any instant acceleration, dx
As we know that, v =
dv d dt
a= = (12t 2 − 16t + 5) = 24t − 16
dt dt Here, x is the displacement of the particle.
At t = 2s, Now, dx = v dt
= 24 × 2 − 16 = 48 − 16 = 32 m/s 2 Integrate on the both sides, limit t = 0 to t = 2 ,
21. Four projectiles are fixed with the same we get
2 2
velocities at angles 25°, 40°, 55° and 70° with ∴ x = ∫ v dt = ∫ (6t − 3t 2) dt
the horizontal. The range of projectile will be 0 0
2 2
largest for the one projected at angle  6t 2   3t 3 
=  −   = [3t 2 ]20 − [t 3 ]20
[23 Sep. 2020, Shift-I]  2 0  3 0
(a) 25° (b) 40° (c) 55° (d) 70° = [3(2)2 − 3 (0 )2 ] − [(2)3 − (0)2 ]
Sol. (b) = [12 − 0] − [8 − 0] = 12 − 8 = 4 m
u2 sin 2 θ Total displacement
Range, R = Average velocity, vavg =
g Total time taken
4
For same projection velocity, = = 2 m/s
2
R ∝ sin 2θ
Hence, the average velocity of the object
i.e. higher value of sin2θ, range will also be between t = 0 to t = 2 s is 2m/s.
greater.
Among the given values of projection angle, the 24. A gun and a target are at the same
value of sin 2θ is greatest for θ = 40°. horizontal level separated by a distance of
So, range will be highest for projected angle 600 m. The bullet is fired from the gun with
θ = 40°. a velocity of 500 ms −1 . In order to hit the
12 AP EAMCET Chapterwise Physics

target, the gun should be aimed to a height h vx = ux = 140 cos 60° and v y = u y − gt
above the target. The value of h is 140 cos 60° = 140 sin 60° − 10t [Q v y = vx ]
(Acceleration due to gravity, g = 10 ms −2) 1 3
140 × = 140 × − 10 t
[20 April 2019, Shift-I] 2 2
(a) 2.4 m (b) 3.6 m 70 = 70 3 − 10t
(c) 7.2 m (d) 10.8 m 10t = 70 3 − 70
Sol. (c) 70 ( 3 − 1)
t= = 7 × (0.7320) = 5.124 s
Given, 10
distance between gun and target = 600 m
velocity of bullet = 500 ms −1 26. A body starting from rest at t = 0 moves
Now, distance = velocity of bullet × time along a straight line with a constant
600 = 500 × t acceleration. At t = 2 s, the body reverses its
600 6 direction keeping the acceleration same. The
t= = = 1.2 sec body returns to the initial position at t = t0,
500 5
From second equation of the motion,
then t0 is [20 April 2019, Shift-II]
1 (a) 4 s (b) (4 + 2 2 ) s
h = ut + gt 2 (c) (2 + 2 2 ) s (d) (4 + 4 2 ) s
2
Putting the given values, we get Sol. (b)
1 According to the question,
h = 0 × (1.2) + × (−10) × (1.2)2
2 u=0 u1=2a u2=0
1 t=0 t1=2s t2
h = × (−10) (1.2 × 1.2) s1 s2
2
h = − 7.2 m A B C
|h| = 7.2m
From first equation of the motion,
25. A projectile is thrown in the upward v1 = u + at1 ⇒ v1 = 2 a
direction making an angle of 60° with the Firstly, body decelerate with acceleration to the
horizontal with a velocity of 140 ms −1 . Then point C and then reverse it’s direction and
the time after which its velocity makes an accelerate with acceleration a to the point A.
angle 45° with the horizontal is Therefore for distance BC, from first equation of
the motion,
(Acceleration due to gravity, g = 10 ms −2)
v2 = v1 − at2 ⇒ 0 = 2 a − at2
[20 April 2019, Shift-I] or t2 = 2 s
(a) 0.5124 s (b) 51.24 s Hence, total time taken by body to covered
(c) 5.124 s (d) 512.4 s distance AC, t = 2 + 2 = 4s
Sol. (c) From second equation one motion,
1 1
Given, angle of projection, θ = 60° and velocity, s1 = AB = ut1 + at12 = 0 + a × 22
u = 140 ms −1 2 2
This velocity have divided into two components, s1 = 2a
horizontal component, ux = u cos 60° Q s1 = s 2 = 2 a
= 140 cos 60° ∴ AC = s1 + s 2 = 4 a
and vertical component, u y = u sin 60° Now, body returns from point C to point A.
= 140 sin 60° So, u1 = 0 , AC = ua
Let after time t, the inclination of particle with From second equation of the motion,
horizontal be 45° and at time t velocity along 1 1
x = vx and along y = v y. s = AC = u1 t + at 2 or ua = 0 + at 2
2 2
v ⇒ t2 = 8 ⇒ t = 2 2 s
Now, tan 45° = y = vx = v y
vx
Therefore, the total time taken by body,
Since, the horizontal component of velocity t0 = t2 + t = (4 + 2 2)s
remains constant, i.e.,
Kinematics 13

27. Two boys conducted experiments on the Sol. (d)


projectile motion with stopwatch and noted Given, vi = 5 ms− 1 , v f = 2 5 ms− 1 and
some readings. As one boy throws a stone in ∆v = 5 ms− 1
air at the same angle with the horizontal, the Since, both vi and v f are extreme speeds, i.e. at
other boy observes that after 4 s, the stone is t = 0 and t = t.
moving at an angle 30° to the horizontal and So, they can be considered as magnitude of the
after another 2 s it is travelling horizontally. velocities at time, t = 0 and t = t.
The magnitude of the initial velocity of the As we know that
stone is (Acceleration due to gravity, g = 10 R2 = A2 + B2 + 2AB cosθ
ms −2.) [20 April 2019, Shift-II] Hence, the angle between the velocities,
(a) 40 3 ms −1 (b) 20 3 ms −1 ∆v2 − vi2 − v2f
cosθ =
(c) 10 3 ms −1 (d) 50 3 ms −1 2vi v f
Sol. (a) Putting the given values, we get
Given, acceleration due to gravity, g = 10 m/s 2
(5)2 − ( 5)2 − (2 5)2
After 4s, angle between stone and horizontal cosθ =
2( 5)( 5)
plane, θ = 30°
After t = 4 s, equation of the vertical projectile 25 − 5 − 20 0
⇒ cosθ = = = 0 ⇒θ = 90°
motion, when θ = 30° 10 10
vsinθ − g()t
∴ tanθ = …(i) Hence, the correct option is (d).
v cosθ
29. If the maximum height and range of a
vsinθ − g(4)
tan 30° = …(ii) projectile are 3 m and 4 m respectively, then
v cosθ the velocity of the projectile is
Total time to reach the stone at horizontal (Take, g = 10 ms − 2) [21 April 2019, Shift-I]
surface, t = 2 + 4 = 6 s
6 3
After t = 6 s, equation of horizontal projectile (a) 20 ms− 1 (b) 10 ms− 1
motion, θ = 0°, 5 2
vsinθ − g(6) Q cos 0° = 1 (c) 10
2
ms− 1 (d) 20
5
ms− 1
tan 0° =  tan 0° = 0  3 6
v × cos 0  
or vsinθ − g(6) = 0 Sol. (c)
vsinθ = 60 …(iii) Here, maximum height, H = 3 m and range,
[Q Given, g = 10] R=4m
u2 sin2 θ
From Eq. (ii), At t = 4 s, when particles As, maximum height, H =
travelling in horizontal direction, 2g
∴ v cosθ = 20 3 …(iv) u2 sin 2θ
and maximum range, R =
g
Now, magnitude of initial velocity,
u2 sin2 θ
v = (v sinθ)2 + (V cosθ)2 then, H = 3= … (i)
2g
[From Eqs. (iii) and (iv)] u2 sin 2θ
and R=4=
or v = (60) + (20 3)
2 2
or v = 40 3 m/s g
H 3 sin2 θ
So, the magnitude of initial velocity of stone is The ratio, = = ⇒ 3sin 2θ = 2sin2 θ
v = 40 3 m/s. R 4 2sin 2θ
⇒ tanθ = 3 (Qsin 2θ = 2sinθ cosθ)
28. The speed of a particle changes from 5ms − 1 3
So, sinθ = …(ii)
to 2 5 ms − 1 in a time t. If the magnitude of 10
change in its velocity is 5 ms − 1 , the angle From Eqs. (i) and (ii), we get
2
between the initial and final velocities of the
3 × 2 × 10 = u2 sin2 θ ⇒ 3 × 2 × 10 = u2 × 
3 

particle is [21 April 2019, Shift-I]  10 
(a) 30° (b) 45° (c) 60° (d) 90°
14 AP EAMCET Chapterwise Physics
2
 10  100 × 2 Let ship A travel x A distance and ship B travel x B
u2 =   × 3 × 2 × 10 ⇒ u =
2
distance, in time t.
 3  3
x x
So, u A = A ⇒ t = A
2 20
⇒ u = 10 t
3 x x x x
and uB = B ⇒ t = B ⇒ A = B ⇒ x A = 2 x B
Hence, the correct option is (c). t 10 20 10
30. A body is projected at an angle other than So, AB = xA2 + (200 − xB )2
90° with the horizontal with same velocity. If = 4 x B2 + 40000 + x B2 − 400 x B
the time of ascent of the body is 1s, then the
maximum height it can reach is = 5x B2 − 400 x B + 40000
(Take, g = 10 ms − 2) [21 April 2019, Shift-I] Differentiate distance AB w.r.t. x B for finding
(a) 5 m (b) 10 m (c) 2.5 m (d) 75 m value of x B ,
d(AB) 1
Sol. (a) = (10 x B − 400) = 0
dx B 2 5x B2 − 400 x B + 40000
Given, angle of projection, θ = 90° and time of
ascent, t = 1s. or x B = 40 m
Since, time of ascent of a projectile body is half Again differentiating,
of the time of height, T = 2 × 1 = 2s  d 2 (AB) 
So,   >0
1  dx 2 B  x B = 40 m
Equation of motion, s = ut + gt 2
2
[Q Distance always greater than zero]
where, s = 0 for complete flight.
and gravitational acceleration, g = 10 m / s2 Hence x B at point, x B = 40 m distance, AB will
be shortest.
Putting the given values in above equation, we get
10 So, AB = 5 × 402 − 400 × 40 + 40000
0 = u × 2− × 22 ⇒ u = 10 m / s
2 AB = 32000
10
Now, the maximum height, h = ut − t 2 or AB = 80 5 km
2
The time after which the distance AB is shortest,
where, t = ls (time of ascent) x x 40
h = 10 × 1 − 5 × 1 ⇒ h = 5 m t= A = B = = 4 hr.
20 10 10
Hence, the correct option is (a).
31. Ship A is moving Westwards with a speed of 32. A body is projected at an angle of 60° with
20 km h −1 and another ship B which is at the horizontal such that the vertical
200 km South of A is moving Northwards component of its initial velocity is 40 ms −1 .
with a speed of 10 km h −1 . The time after The magnitude of velocity of the projectile at
which the distance between them is shortest one quarter of its time of flight is nearly,
and the shortest distance between them (Acceleration due to gravity, g = 10 ms −2)
respectively, [21 April 2019, Shift-II]
[21 April 2019, Shift-II]
(a) 4 h, 80 5 km (b) 50 2 h, 10 km
(a) 3.54 ms −1 (b) 35.40 ms −1
(c) 100 2 h, 2 10 km (d) 80 5 h, 4 km (c) 30.54 ms −1 (d) 34.5 ms −1
Sol. (a) Sol. (c)
According to the question, According to the question,
20 kmh–1
xA uy=u sin 60º
A C u
10 kmh–1

(200–xB)

θ 60º

B ux=u cos 60º


Kinematics 15

3 Let the time taken to travelled from A to B = t


Verticle component of velocity, u y = u = 40
2 second
or u=
80
ms−1 Now, distance covered from A to B = d AB
3 ∴Distance, d = Velocity × Time
2   sin 60°
80
∴ d AB = 7t …(i)
2 usinθ  3
Time of flight, T = = Now, distance covered from B to C = dBC
g 10 v + v3
∴Average velocity of the body = 2
80 3 1 2
⇒ T = 2×
× × ⇒ T = 8sec
3 2 10 14 + 21 35
or v′ = = m/s
T 8 2 2
at time, t = = = 2sec
4 4 35
∴ dBC = v′ × t ′ ⇒ dBC = t ′ …(ii)
80 2
40 T
vx = 3 = ms−1 at t = = 2sec , ∴ Distance travelled by the body from point A
2 3 4 to B = distance travelled by the body from point
From first equation of the motion, B to C.
3  80  d AB = dBC
v y = u y − gt =   − 10 × 2
2  3 From Eqs. (i) and (ii), we get
35 2
[Q u y = usin 60° ] 7t = t ′ or t ′ = t …(iii)
2 5
= 40 − 10 × 2 = 20 ms−1
Now, the average velocity from A to C, for
T
Hence, at t = , magnitude of velocity of projectile, finding distance d AC ,
4 v + v2 + v3 42
2 v= 1 = =14 m/s …(iv)
v= vx2 + v2y =  40  + (20)2 ≈ 30.54ms−1 3 3
 
 3 ∴Distance travelled from A to C,
d AC = v × t
or d AC = 14 t …(v)[From Eq. (iv)]
33. A body travelling along a straight line path
Total time taken from A to C,
travels first half of the distance with a
T = t + t′
velocity 7 ms −1 . During the travel time of the
2t
second half of the distance, first half time is or T=t+ [∴From Eq. (iii)]
5
travelled with a velocity14 ms −1 and the second
7t
half time is travelled with a velocity 21 ms −1 . T= …(vi)
5
Then the average velocity of the body during
Now, average velocity during the whole journey
the journey is [22 April 2019, Shift-I]
(From A to C),
(a) 14 ms−1 (b) 10 ms−1 (c) 9 ms−1 (d) 12 ms−1 d 14t
vavg = AC or vavg = ×5
Sol. (b) T 7t
According to the question, [∴From Eqs. (v) and (vi)]
or v avg = 10 m/s
t t′
34. Assertion (A) The range of a projectile is
A 7m/s B 14m/s 21m/s C maximum when the angle of projection
v1 v2 v3
is 45°.
T
Reason (R) The range of a projectile
Total time
depends only on the angle of projection.
Given, a body travelling along a straight line path. [22 April 2019, Shift-I]
Body travels first half of the distance (A to B) (a) Both (A) and (R) are true and (R) is the correct
with velocity, v1 = 7 m/s
explanation of (A).
Body travels second half of the distance (B to C) (b) Both (A) and (R) are true but (R) is not the correct
in first half time with velocity, v2 = 14 m/s explanation of (A).
and in the second half time with velocity, (c) (A) is true but (R) is false.
v3 = 21 m/s (d) (A) is false but (R) is true.
16 AP EAMCET Chapterwise Physics

Sol. (c) 4
sinθ1 4 sinθ1
Assertion is true. = ⇒ = 5
sinθ2 3 sinθ2 3
The range of projectile is maximum when the
angle of projection is 45°. 5
4 −1  4 
u2 sin (2 × 45) u2 Now, sinθ1 = or θ1 = sin   = 53° …(iii)
Rmax = = 5  5
g g 3
u2 sin 2 θ ∴ sin θ2 =
As range of a projectile, R = 5
g
θ2 = sin−1   = 37°
3
or …(iv)
Hence, the range of projectile depends on the  5
angle of projection θ and initial velocity u. Now, from Eq. (i), we get
∴ (A) → True   4 2 
 
(R) → False  u   sin 53
2 2
 u    5 
2
80 =     or 80 =    
35. A car moving with a certain velocity, jumps  g  2   g  2 
from an inclined plane placed at one bank of  
a river and reaches the other bank by u2 80 × 25 × 2
attained a maximum height of 80 m. If the =
g 16
same car, moving with the same velocity u2
jumps from another inclined plane having = 250 …(v)
g
different angle of inclination and reaches the
same point on the other bank by attaining ∴Width of the river given as
maximum height of 45 m, then the width of  u2  u2
d =   sin 2θ =` (2sinθ1 cosθ1 )
the river is [22 April 2019, Shift-I]  g g
(a) 80 m (b) 60 m (c) 125 m (d) 240 m (Qsin 2θ = 2sinθ cosθ and consider
θ = θ1 for first case )
Sol. (d)
From Eqs. (v) and (iii), we get
Given, the maximum height reached by the car
4 3 4 3
jumps from an inclined plane placed at one back ∴ d = 250 × 2 × × [∴ sinθ1 = then cosθ1 = ]
of river is H1 = 80 m 5 5 5 5
The second maximum height reached by car d = 20 × 2 × 4 × 3 = 240 m
jumps from another inclined plane placed at d = 240 m
other bank is H2 = 45 m Hence, the width of the river is 240 m.
Let the certain velocity of moving car = u
Let the inclination angle in first jump = θ1
36. Two food packets are thrown with the same
velocity in the same direction with different
Let the inclination angle in second jump = θ2
angles of projection simultaneously. The
We know that,
angle of projection of one packet is 15°. At
the maximum height of projectile given as the same moment one boy starts running
u2 sinθ
H= from rest from the point of projection with
2g
an acceleration of 10 ms − 2 to catch them. If
For first case,
u2 sin2 θ1 he caught one packet at a distance of 20 m
H1 = 1
2g and other packet in s later the first packet,
u2 2
80 = sin2 θ1 …(i) then the angle of projection of the second
2g
u 2 packet is
Similarly, for second case, H2 = sin2 θ2 …(ii)
2g (Acceleration due to gravity, g = 10 ms − 2)
Dividing Eqs. (i) by (ii), we get
u2 [22 April 2019, Shift-II]
sin2 θ1
(a) sin− 1   (b) sin− 1  
H1 80 sin2 θ1 1 25 1 8
∴ 2
= 2 g ⇒ = 2  32  2  9
H2 u 45 sin2 θ2
sin2 θ2
(c) sin− 1   (d) sin− 1  
1 7 1 5
2g
2  8 2  6
Kinematics 17

Sol. (a) dv
So, = − 2. 5 v
According to the question, the given situation dt
shown in the figure below Integrate on the both sides, we get
0
1
Y ⇒ ∫ v dv = − 2.5 ∫ dt
6. 25

⇒ [2 v]06. 25 = − 2 .5t
2 × 2.5
uB ⇒ t= = 2s
uA 2. 5
Hence, the correct option is (a).
15° P Q
X
O 38. A bullet fired from a gun falls at a distance
half of its maximum range. The angle of
From second equation of the motion, projection of the bullet is [23 April 2019, Shift-I]
1 Given, S A = 20 m 
s A = u At + at 2  (a) 45° (b) 60° (c) 30° (d) 15°
2  a = 10 ms2 
Sol. (d)
1
⇒ 20 = (10)t 2 ⇒ t = 2s As, we know that for a projectile motion,
2 maximum range,
u2 sin 2θ A u2 R
Range of projectile (R A) = A = 20 Rmax = and hmax = max
g g 4
Q initial velocity, Rmax R u2
 u= 0  Given, R= ⇒ max = =R …(i)
  2 2 2g
u2A u2 sin 2θ
⇒ sin 30° = 20 (Q θ A = 15°) where, R =
10 g
200
⇒ u2A = ⇒ u A = 20 m / s u2 sin 2θ u2
(1 / 2) Hence, = [From Eq. (i)]
g 2g
Range of projectile for B, 1
2 ⇒ sin 2θ =
RB = uB t + at 2 = × 10 ×  2 +  (Q uB = 0)
1 1 1 2
2 2  2 ⇒ 2θ = 30° ⇒θ = 15°
2
Hence, the correct option is (d).
= 5 ×   = 31.25 m
5
 2
39. A body is projected at an angle of 45° from a
u2 sin 2θB point on the ground at a distance of 30 m
Q RB = B
g from the foot of a vertical pole of height
(20)2 sin 2θB 20 m. The body just crosses the top of the
⇒ 31.25 = (Q u A = uB = 20 m / s)
10 pole and strikes the ground at a distance s
from the foot of the pole on the other side of
or θB = sin− 1  
312. 5 1 25
⇒ sin 2θB = the pole. Then, s
(20) 2
2  32 [23 April 2019, Shift-I]
(a) 20 m (b) 30 m (c) 50 m (d) 60 m
37. A car moving with a velocity 6.25 ms −1 is Sol. (d)
decelerated with 2.5 v ms −2 (v is According to the question, projection of a body
instantaneous velocity). Time taken by the shown in the figure below,
car to come to rest is [23 April 2019, Shift-I]
(a) 2 s (b) 3 s (c) 2.5 s (d) 4 s
20m
Sol. (a) u
45°
Given, u = 6.25 ms−1 and a = − 2. 5 v ms−2 ,
30m
v = instantaneous velocity s
dv Now, from the above figure, let t be the time of
As we know, a =
dt crossing of pole,
18 AP EAMCET Chapterwise Physics

then, 30 = t u cos 45° …(i) Sol. (c)


10 For a projectile projected at angle θ;
and 20 = u sin 45° t − t 2 …(ii)
2 u2 sin2 θ
Maximum height, Hmax =
Hence, 30 2 = ut and 2g
u u2 sin 2θ
⇒ 20 = t − 5t 2 and range, R =
2 g
⇒ t = 2 s and u = 30 m/s  u2 sin2 θ 
u sin 2θ 30 × 30 × sin 90°
2  
Hence, R = = = 90 m Hmax  2g  tanθ
∴ Ratio = = =
g 10 R  u2 sin 2θ  4
∴Distance between pole and the point at which  
 g 
body strike on the ground,
8
⇒ s = (90 − 30) = 60 m 8 8 2
So, the correct option is (d). Here, θ = tan−1 ⇒ tanθ = = 7 =
7 7 4 7
40. Assertion (A) The velocity of a projectile at 42. A body is projected with a speed u at an angle θ
a point on its trajectory is equal to the slope with the horizontal. The radius of curvature
at that point.  θ
of the trajectory, when it makes an angle  
Reason (R) The velocity vector at a point  2
always along the tangent to the trajectory at with the horizontal is (g-acceleration due to
that point. [22 April 2018, Shift-I] gravity)
(a) Both A and R are true and R is the correct [22 April 2018, Shift-I]
explanation of A θ θ
(b) Both A and R are true but R is not the correct u cos θsec  
2 2 3
u 2 cos 2 θsec 3  
2 2
explanation of A (a) (b)
3g 2g
(c) A is true but R is false
θ θ
(d) A is false but R is true 2u cos θsec 2  
2 3
u cos θsec 3  
2 2
2 2
Sol. (d) (c) (d)
g g
Trajectory of a projectice is of form
gx 2 Sol. (d)
y = f (x) = x tanθ − 2 θ
2u cos2 θ Let velocity of projectile is v at an angle with
dy dy / dt v y 2
∴ = slope = = horizontal
dx dx / dt vx
So, Assertion (A) is incorrect. θ/2 v
s in
Also, velocity of a projectile is always along mg θ/2
tangent to the trajectory (shown) θ/2
u mg cos θ/2
mg
θ
v u
v θ
∴ v cos = u cosθ
2
Hence, reason (R) is correct. u cosθ
or v=
θ
41. A body is projected from the ground at an cos
2
8
angle of tan −1   with the horizontal. The As horizontal component remains same.
7
Also, centripetal force is provided by the
ratio of the maximum height attained by it component of weight.
to its range is [22 April 2018, Shift-I] mv2 θ
So, = mg cos
(a) 8 : 7 (b) 4 : 7 (c) 2 : 7 (d) 1 : 7 r 2
Kinematics 19

Hence, radius of curvature of path, 45. A body projected with some velocity at an
v2 angle 45° with the horizontal from the origin
⇒ r=
θ in XY -plane passes through a point at (4, 3)
g cos
2
m. Its horizontal range is [22 April 2018, Shift-II]
u2 cos2 θ
2 (a) 10 m (b) 14 m (c) 18 m (d) 16 m
 cos θ  θ
u2 cos2 θ ⋅ sec3  
  Sol. (d)
 2  2
= ⇒ r=
θ g Equation of trajectory of projectile is
g cos
2 gx 2
y = x tanθ − 2
2u cos2 θ
43. Assertion (A) The angle between
acceleration and velocity of a body is Here, θ = 45°
gx 2
one-dimensional motion is always zero. ⇒ y= x− 2 … (i)
u
Reason (R) One-dimensional motion is 2
g(4 )
along a straight line. [22 April 2018, Shift-II] When y = 3, x = 4, so 3 = 4 −
(a) Both (A) and (R) are true and (R) is the correct u2
16 g
explanation of (A). ⇒ = 1 ⇒ u2 = 16 g
(b) Both (A) and (R) are true but (R) is not the correct u2
explanation of (A). Substituting the value of u2 in Eq (i), we get
(c) (A) is true but (R) is false. gx 2 x2
y= x− or y = x −
(d) (A) is false but (R) is true. 16 g 16
Sol. (d) How at maximum x, y = 0.
x2
= 0 ⇒ x 1 −  = 0
Assertion is false as during decceleration, i.e. x
Hence, x −
when body is slowing down, acceleration and 16  16 
velocity are directed opposite to each other.
⇒ x=0
Hence, angle between a and v may be 180° or 0°.
(initial position) and x = 16 m (final position).
Reason is true.
44. A projectile is given an initial velocity of 46. A body is projected vertically upwards with a
velocity u from the top of a tower. Time
($i + 2 $j) ms−1 . The equation of its path is
taken by it to reach the ground is n times,
(g = 10 ms−2) [22 April 2018, Shift-II] then the time taken by it to reach the
(a) y = 2 x − 5 x2 (b) y = x − 5 x2 highest point in its path. Height of the tower
(c) 4 y = 2 x − 5 x2 (d) y = 2 x − 25 x2 is [23 April 2018, Shift-I]
Sol. (a) nu 2 (n − 1) nu 2 (n − 2 )
(a) (b)
Velocity of particle is ($i + 2$j) ms− 1 initially. 2g g
nu 2 (n − 2 ) u2
So, ux = 1 ms− 1 and u y = 2 ms− 1 (c) (d) (n + 1)
2g 2g
Also, a x = 0 and a y = − 10 ms− 2
In time t, Sol. (c)
Horizontal distance covered by projectile is Let the time taken to reach the maximum
x = ux × t = t … (i) height, when thrown vertically upwards
And vertical distance covered by projectile is
u t1
1
y = u yt + a yt 2
2
⇒ y = 2t − 5t 2 … (ii) t2
Substituting the value of t from Eq (i) in Eq H
(ii), we get
y = 2x − 5x 2

u
t1 =
g
20 AP EAMCET Chapterwise Physics

If t2 be the time to hit the ground, then given Vertical displacement after 1 s,
t2 = nt1 = nu / g 1 1
s y = u yt + a yt 2 = 0 + × 10 × ()
1 2 = 5m
So, using equation of motion for distance, 2 2
1 Resultant velocity after 1 s,
i.e. s = ut + at 2
2 v= vx2 + v2y = (20)2 + (10)2 = 400 + 100
 u  1 n2 u2
− H = u ⋅ n  − g 2 = 500 = 22.36 = 22.4 m / s
 g 2 g
So, the correct option is (c).
[s = − H, t = t2 = nt1 ]
2 2 2
or − H=
nu

nu 48. Two towers A and B, each of height 20 m are
g 2g situated a distance 200 m apart. A body
or 2gH = nu (n − 2)
2 thrown horizontally from the top of the
nu2 (n − 2) tower A with a velocity 20 ms − 1 towards the
or H=
2g tower B hits the ground at point P and
another body thrown horizontally from the
47. A body is projected horizontally from the top top of tower B with a velocity
of a tower of height 180 m with a velocity of 30 ms − 1 towards the tower A hits the
20 ms − 1 . If acceleration due to gravity is ground at point Q. If a car starting from rest
10 ms − 2, then match the following. from P reaches Q in 10 seconds, then the
[23 April 2018, Shift-I] acceleration of the car is
List I List II (acceleration due to gravity = 10 ms − 2)
[23 April 2018, Shift-I]
(A) Velocity of the body after 1 second (I) 5
(in ms − 1 ) (a) 1 ms − 2 (b) 2 ms − 2 (c) 3 ms − 2 (d) 4 ms − 2

(B) Horizontal displacement of the body (II) 20


Sol. (b)
after 1 second (in metres) Given, height of both towers is same,
h1 = h2 = h.
(C) Vertical displacement of the body (III) 10 A B
after 1 second (in metres)

(D) Vertical velocity of the body after (IV) 22.4


1 second (in ms − 1 )
h=20m
A B C D A B C D
100m
(a) IV II III I (b) I II III IV
(c) IV II I III (d) II IV I III 40m 60m

Sol. (c) P Q
Given, initial horizontal component of velocity, 200m
ux = 20 m / s.
2h
Initial vertical component of velocity, Time of flight, t = will be same
g
uy = 0
2 × 20
Acceleration due to gravity, t= = 4 = 2s
g = a y = 10 m / s2 10
So, horizontal component of velocity after 1 s, ⇒ Displacement in horizontal direction from
vx = ux + a x t = 20 + 0 = 20 m/s tower A to point P = u At
and vertical component of velocity after 1 s, = 20 × 2 = 40 m
v y = u y + a yt = 0 + 10 × 1 = 10 m / s ⇒ Displacement in horizontal direction from
tower B to point Q = uB t
Horizontal displacement after 1 s,
1 = 30 × 2 = 60 m
s x = ux t + a x t 2 = 20 × 1 + 0 = 20 m So, distance between point P and Q
2
= 200 − (40 + 60) = 100 m
Kinematics 21

Given, distance between P and Q is covered by As per second condition,


car in 10 s, so using 1
1 30 t2 = × 10 t22 = t2 = 6s
s = ut + at 2 2
2
1 ∴ t2 − t1 = 3s
100 = 0 × 10 + a(10)2
2 51. A particle is projected at an angle of 60 ° with
[Q u = 0, as car starts from rest] the horizontal from the ground with a
a=2 velocity 10 3 ms −1 . The angle between
Acceleration, a = 2 m / s2 . velocity vector after 2s and initial velocity
v vector is (g = 10 ms −2) [23 April 2018, Shift-II]
49. The velocity-
displacement (v-s) (a) 0° (b) 30° (c) 60° (d) 90°
graph shows the Sol. (d)
motion of particle Initial velocity, vi = 2cosθ $i + 4sinθ $j = 5 3 $i + 15$j
moving in a straight 2
s Final velocity vector (after 2s),
line. 0
Velocity-displaceme vf = u cosθ $i + (usinθ − gt)$j = 5 3 $i − 5$j
nt graph is a circle of radius 2 m and centre Now, vi ⋅ vf = 25 × 3 − 15 × 5 = 0
is at (2, 0) m. ∴vi ⊥ vf
The value of acceleration for this particle at a 52. The motion of a particle along a straight line
point (2 − 2 , 2) m will be ............ms −2. is described by the function, x = (2 t − 3)2
[23 April 2018, Shift-II] where x is in metres and t is in seconds. The
(a) 2 (b) 4 (c) 2 (d) 3 acceleration of the particle at t = 2 s is
[24 April 2018, Shift-I]
Sol. (a) (a) 1 ms −2 (b) 4 ms −2 (c) 8 ms −2 (d) 7 ms −2
As, graph is a circle with centre (2, 0) and radius
2, its equation is Sol. (c)
(s − 2)2 + (v − 0)2 = 22 Given, x = (2t − 3)2
(s − 2)2 + v2 = 4 where, x = displacement and t = time.
Differentiating with respect to time, we get x = 4t 2 + 9 − 12t
ds dv Differentiate w.r.t. time, we get
2(s − 2) + 2v = 0
dt dt dx
Velocity, v = = 8t − 12
2(s − 2)v + 2v ⋅ a = 0 dt
At, s = 2 − 2 and v = 2 Acceleration, a =
dv
= 8 m / s2
2(2 − 2 − 2) ×
2 + 2 2a = 0 dt

⇒ −4 + 2 2a = 0 ⇒ a =
4
= 2 ms −2 53. A particle moves in the xy- plane with
2 2 x 3
velocity v = x$i + y t$j. At t = , the
50. A body is projected horizontally from the top y
of a tall tower with a velocity of 30 ms −1 . At magnitudes of tangential and normal
time t1 , its horizontal and vertical accelerations, respectively are
components of the velocity are equal and at [24 April 2018, Shift-I]
time t2, its horizontal and vertical 3y y 2 y 3y
(a) , (b) ,
displacements are equal. Then t2 − t1 is (take, 2 2 3 2
g = 10 ms −2) [23 April 2018, Shift-II] 3y 5y 11y
(c) , (d) 2 3 y,
(a) 1 s (b) 1.5 s (c) 2 s (d) 3 s 2 2 3
Sol. (d) Sol. (a)
As per first condition, Given, velocity of particle is v = x$i + yt$j
30 = 10 t1 So,|v| = v = x 2 + y2 t 2
t1 = 3s
22 AP EAMCET Chapterwise Physics

Magnitude of tangential acceleration is x 3


Now with t = , we get
dv 0 + 2ty 2 ty 2 y
at = = or a t =
dt 2 x 2 + y 2 t 2 x 2 + y2 t 2 xy xy y
a n = |a n| = = =
x 3 3x2
2x 2
Now, substituting t = , we get x 2 + y2 ⋅
y y2
x 3
× y2 54. Assertion (A) The speed of a body in
at = y
uniform circular motion is constant.
 x 2 × 3
x 2 + y2 ×   Reason (R) In uniform circular motion, the
 y2 
acceleration of the body is constant.
3y
⇒ at = [24 April 2018, Shift-I]
2 (a) Both (A) and (R) are true and (R) is the correct
dv
Also, total acceleration of particle is a = explanation of (A).
dt (b) Both (A) and (R) are true but (R) is not the correct
d $ explanation of (A).
⇒a = (x i + yt$j) ⇒ a = y$j
dt (c) (A) is true but (R) is false.
or magnitude of total acceleration is (d) (A) is false but (R) is true.
a = |a| = y Sol. (c)
Hence, normal acceleration is
(i) Assertion is correct : In uniform circular
a n = a total − a tangential motion, body moves with constant speed. It is
So, a n = |a n| = magnitude of normal acceleration magnitude of velocity, which is constant.
 y4 t 2  (ii) Reason is incorrect : In uniform circular
= a 2 − a t2 = y2 −  2 
 x + y2 t 2  motion, direction of acceleration (centripetal)
is always along the centre, which keep
x 2 y2 xy changing when particle moves. As,
= =
x + y2 t 2
2
x 2 + y2 t 2 acceleration is vector quantity and its direction
is changing, so it can not be constant.
3
Laws of Motion
1. When a body slides down on inclined plane Final velocity of the body,
with coefficient of friction µ, then its v = −2 ms −1
acceleration will be [17 Sep. 2020, Shift-I] Since, final velocity of the body decreases on
application of constant force, hence acceleration
(a) g(sin θ − µ cos θ) (b) g(sin θ + µ cos θ)
is negative.
(c) g(µ sin θ − cos θ) (d) gµ (sin θ − cos θ)
By first equation of motion,
Sol. (a) v = u + at
When a body of mass m slides down on inclined v − u −2 − 10 12
plane with acceleration a, then various forces ⇒ a= = = − = − 3 ms −2
t 4 4
acting on the body are shown in the figure.
3. One end of a string of length l as connected
R µR
a to a particle of mass m and the other to a
small peg on a smooth horizontal table. If
mg sin θ θ mg cos θ
the particle moves in a circle with speed v
mg
and T is the tension in the string, then net
force on the particle (directed towards the
θ
centre) is [17 Sep. 2020, Shift-II]
(a) T (b) 0
According to Newton’s second law of motion,
T + mv 2 T − mv 2
ma = mg sinθ − µR (c) (d)
l l
⇒ ma = mg sin θ − µmg cosθ
⇒ a = gsinθ − µg cosθ Sol. (a)
⇒ a = g (sinθ − µ cos θ) The given situation is shown in the following
figure
2. A body of mass 10 kg has constant velocity
of 10 m s − 1 . A constant force is applied for a
duration of 4s on the object and gives it a
m
speed of 2 m s − 1 in opposite direction. Then,
find acceleration produced in it.
[17 Sep. 2020, Shift-II] N
(a) 3 ms −2 (b) − 3 ms −2
T
(c) 0.3 ms −2 (d) − 0.3 ms −2
Sol. (b) mg
Mass of body, m = 10 kg
Velocity, v = 10 ms −1 mv2
Here, N = mg and T =
l
t = 4s
24 AP EAMCET Chapterwise Physics

Tension in the string provides necessary centripetal Since, the spring is stretched by an equal force of
force. Hence, net force on the particle towards 5N in opposite direction as shown in the figure.
centre of circular path is equal to T. Thus, it will be stretched by an equal amount in
both directions giving rise to a tension of
4. Which of the following sets of concurrent
5 + 5 = 10 N.
forces may be in equilibrium?
[17 Sep. 2020, Shift-II] 6. Find the apparent weight of a body of mass
(a) F1 = 3 N, F2 = 5 N, F3 = 10 N 1.0 kg falling with an acceleration of 10 ms − 2 .
(b) F1 = 3 N, F2 = 5 N, F3 = 9N (g ≈ 10 ms − 2 ) [18 Sep. 2020 Shift-I]
(c) F1 = 3 N, F2 = 5 N, F3 = 6N (a) 1 kg-wt (b) 2 kg-wt
(d) F1 = 3 N, F2 = 5 N, F3 = 15 N (c) 0 (d) 0.5 kg-wt
Sol. (c) Sol. (c)
According to triangle law of forces, resultant of Mass of body, M = 1 kg
two forces must be greater than or equal to the
Falling acceleration of body, a = 10 ms −2
third force.
In option (a), g = 10 ms −2
F1 (3N) + F3(1 N) < F2(5 N) [Q Here, a = g]
Hence, these sets of concurrent forces may not ∴ Apparent weight = m(g − a) = m × 0 = 0
in equilibrium. ∴When body is falling with gravitational
In option (b), acceleration, then its apparent weight will be
F1 (3 N) + F2(5 N) < F3(9 N) zero.
Hence, these sets of concurrent forces also may 7. A balloon of mass M descends with an
not in equilibrium.
acceleration a (< g). What mass need to be
In option (c),
removed from the balloon, so that it starts
F1 (3 N) + F2(5 N) > F3(6 N) ascending with acceleration, a?
Hence, these sets of concurrent forces may be [18 Sep. 2020, Shift-II]
in equilibrium. 2M 2Ma
In option (d), (a) (b)
(a + g ) (a + g )
F1 (3N) + F2(5N) < F3(15N) 2Ma 2Ma
(c) (d)
Hence, these sets of concurrent forces may not (a − g ) ( g − a)
in equilibrium.
Sol. (b)
5. The tension in the spring is [18 Sep. 2020 Shift-I] The given situation is shown in the figure
F
F F

60° µ= 1
√3 kg 2 √3
M a M–M′ a
(a) zero (b) 10 N (c) 2.5 N (d) 5 N
Sol. (b)
Mg (M–M′)g
in θ B
vs
In above figures, M is the mass of balloon and
v′ M′ is the mass remove from the ballon.
L y
According to Newton’s law of motion in first
case,
v θ Mg − F = Ma ⇒ F = Mg − Ma …(i)
A O According to Newton’s second law of motion,
x
for second case,
v cosθ
F − (M − M ′) g = (M − M ′)a
Laws of Motion 25

⇒ F − Mg + M ′g = Ma − M ′a will start motion due to application of 100 N


⇒ Mg − Ma − Mg + M ′g = Ma − M ′a force.
[from Eq. (i)] Due to motion, kinetic friction force
⇒ M ′g + M ′a = Ma + Ma fk = µ k R = 0.4 mg
⇒ M ′(g + a) = 2Ma = 0.4 × 10 × 9.8 = 39.2 N
2Ma
⇒ M′ = 40 kg body experiences a force of fk = 39.2 N
g+ a
∴Acceleration of 40 kg slab
f 39.2
8. A bomb at rest explodes into three parts of equal a= k = = 0.98 ms −2
mass. If the momentum of two parts are − 2p $i 40 40

and p $j, find the magnitude of momentum of 10. In a rocket, fuel burns at the rate of 1 kg/s.
the third part. [18 Sep. 2020, Shift-II] This fuel is ejected from the rocket with a
velocity of 60 km/s. The force exerted on the
(a) p (b) 3p
rocket by this is [21 Sep. 2020, Shift-I]
(c) 5p (d) 0
(a) 60 N (b) 600 N
Sol. (c) (c) 6000 N (d) 60000 N
$ be the momentum of third parts, then
If p′ n Sol. (d)
according to conservation of linear momentum.
dm
Total moment before explosion = Total For a rocket, = 1 kg/s
dt
momentum after explosion
Velocity of fuel ejected by rocket,
⇒ 0 = −2p$i + p$j + p ′ n
$
v = 60 km/s = 60000 m/s
⇒ p′ n
$ = 2p$i − p$j
∴Force exerted on the rocket is given as
$ = p′
∴Magnitude of p ′ n dm
F=v = 60000 × 1 = 60000 N
= (2p)2 + (− p)2 = 5p 2 = 5p dt

9. If 100 N force is applied to 10 kg block as 11. A rocket is fired vertically from the ground
shown in the diagram, the acceleration of with a resultant vertical acceleration of a
40 kg slab is [21 Sep. 2020, Shift-I] 10 ms −2 . Fuel is finished in 1 min and it
µs=0.6, µk=0.4 continues to move up. What is the maximum
height reached ? [21 Sep. 2020, Shift-II]
(a) 36.4 km (b) 42.3 km
100 N 10 kg
(c) 48.4 km (d) 25.6 km

40 kg
Sol. (a)
Resultant vertical acceleration,
−2 −2 a = 10 ms −2
(a) 1.65 ms (b) 0.98 ms
Height travelled by the rocket in 1 min (60 s),
(c) 0.5 ms − 2 (d) 0.25 ms − 2
1
h = ut + at 2
Sol. (b) 2
Static friction force between 10 kg and 40 kg 1
= 0 × 60 + × 10 × (60)2
block, 2
µs=0.6, µk=0.4
= 18000 m = 18 km
Velocity of rocket after 1 min (60s),
100 N 10 kg
v = u + at = 0 + 10 × 60 = 600 ms −1
After 1 min, when fuel of rocket is finished,
40 kg
then
u = 600 ms −1 , a = − g = −9.8 m/s 2
Fs = µ s R = 0.6 × mg Hence, if rocket goes at height h2, then
= 0.6 × 10 × 9.8 = 58.8 N v2 − u2 = 2gh2
Here, we see that the applied force (F = 100 N) is 0 − (600)2 = 2(−9.8)h2
greater than friction force, hence 10 kg block
26 AP EAMCET Chapterwise Physics

360000 Sol. (c)


⇒ h2 = = 18367.3 m = 18.4 km
2 × 9.8 Given, u = 30 $i + 40 $j ms −1
∴ Maximum height travelled by rocket from
Force, F = − ($i + 5$j) N
ground = h1 + h2 = 18 + 18.4 = 36.4 km
m = 5 kg
12. A block of mass m is placed on a smooth u y = 40 ms −1 , Fy = − 5 N
wedge of inclination θ. The whole system is
accelerated horizontally so that the block ∴ Acceleration of body along y-direction,
does not slip on the wedge. The force exerted F −5
ay = y = = − 1 ms −2
by the wedge on the block (g is acceleration m 5
due to gravity) will be [21 Sep. 2020, Shift-II] Using equation of straight line motion,
(a) mg cosθ (b) mg sinθ v = u + at
(c) mg (d) mg sec θ v y = u y + at
Sol. (d) 0 = 40 + (−1) t ⇒ t = 40 s
The given situation is shown in the following 14. A block P of mass MP is in contact with
figure,
another block Q of mass MQ as shown in the
figure and they are placed on a smooth floor.
θ
s

Force on block Q is
co

R a [22 Sep. 2020, Shift-I]


a
m

θ
ma
mg sin θ F P
(mg cos θ + ma sin θ) (MP) Q
(MQ)
θ
mg
MP MQ F
(a) (b)
M P + MQ M P + MQ
The acceleration given to wedge to the left is a.
The block has a pseudo acceleration to the right, M F M F
(c) P (d) Q
pressing against the wedge because of which MQ MP
the block does not move.
In this case,
Sol. (b)
∴ mgsinθ = ma cosθ If a be the common acceleration of system of
blocks on application of force F.
gsinθ
⇒ a= …(i) Free body diagram of block P and Q are given as
cosθ
Force exerted by the wedge on the block is a a
equal to normal reaction on the block,
F R R
i.e. R = mg cosθ + ma sinθ P (MP) Q (MQ)
sinθ
= mg cosθ + m ⋅ g ⋅ sinθ [From Eq. (i)]
cosθ
Hence, for block P, F − R = M P a …(i)
mg
= [cos2 θ + sin2 θ] For block Q, R = M Qa …(ii)
cosθ
mg Adding Eqs. (i) and (ii), we get
⇒ R= = mg sec θ
cosθ F = (M P + M Q) a
F
13. A body of mass 5 kg starts from the origin ⇒ a= …(iii)
MP + MQ
with an initial velocity u = 30 $i + 40 $j ms −1 .
∴ Force on block Q is given as
When a constant force F = − (i$ + 5 $j) N acts F
R = MQ a = MQ ⋅
on the body, the time in which the MP + MQ
y-component of the velocity becomes zero is MQ F
[22 Sep. 2020, Shift-I] =
MP + MQ
(a) 5 s (b) 20 s (c) 40 s (d) 80 s
Laws of Motion 27
Retarding force 100
15. A body whose momentum is constant must ∴ Retardation = =
have constant [22 Sep. 2020, Shift-II] Mass of body 20
(a) acceleration (b) force −a = 5
(c) velocity (d) mass or a = −5 m/s 2
When body come to rest, then v = 0
Sol. (c) From first equation of motion, v = u + at
Momentum of a moving body with velocity v is 0 = 15 + (−5) × t
given as
5t = 15 or t = 3s
p = mv
Since mass m of a body remains always 19. The maximun value of the applied force F
constant. Hence, if momentum of body is such that the block as shown in the
constant, then its velocity must be constant. arrangement does not move is
16. A boy travelling in an open car moving at (Acceleration due to gravity, g = 10 ms −2 )
constant velocity throws a ball vertically up [20 April 2019, Shift- I]
into air. The ball falls [22 Sep. 2020, Shift-II] F
(a) outside the car
(b) in the car ahead of the boy
(c) in the car beside the boy 60° µ= 1
√3 kg 2 √3
(d) exactly in his hand
Sol. (d) (a) 20 N (b) 15 N
When the boy travelling in an open car moving (c) 25 N (d) 10 N
at constant velocity throws a ball vertically up
into air, then ball also moves with same velocity Sol. (a)
of open car due to law of inertia. Hence, the ball Applied force on the block on a horizontal
falls exactly in his hands. surface, as shown in the figure , we have
R
17. The time taken by a force of 2 N to produce a
−1 F
change of momentum of 0.4 kg ms in a
body is [23 Sep. 2020, Shift-I] F cos 60°
60°
√ 3 kg µ= 1
(a) 0.2 s (b) 0.02 s 2 √3

(c) 0.5 s (d) 0.05 s


Sol. (a) F sin 60 W = mg = 10 √ 3
Given that, force, F = 2 N
As we know that, force of friction, f = µ R
Change in momentum, ∆p = 0.4 kgms −1
From the diagram,
∴ We know that, force, f = µ (W + F sin 60°)
change in momentum ∆p
F= = F cos 60° =
1
(10 3 + F sin 60°)
time taken ∆t 2 3
∆p 0.4
or ∆t = = = 0.2 s Q cos 60° = 1 
F 2 1 1  3  2 
⇒F × = 10 3 + F ×   
18. When will a body of mass 20 kg moving at 2 2 3 2  sin 60° = 3 
15 ms −1 , subjected to a retarding force of  2 
× 310 + 
100 N, come to rest? [23 Sep. 2020, Shift-I]

F
=
1 F
(a) 3 s (b) 6 s 2 2 3  2
(c) 1.5 s (d) 9 s F 1 (20 + F) 20 + F
⇒ = ⇒F =
Sol. (a) 2 2 2 20
Given that, mass of body, m = 20 kg ⇒ 2F = 20 + F ⇒ 2F − F = 20
Initial velocity,u = 15 ms −1 ∴ F = 20 N
Retarding force, F = 100 N
28 AP EAMCET Chapterwise Physics

 25  Q cosθ = 1 − sin2 θ = 1 − (0.6)2


20. A rough inclined plane BCE of height  m
6 ∴ cosθ = 0.8
is kept on a rectangular wooden block ABCD Form Eq. (ii),
of height 10 m, as shown in the figure. A 1
a = 10 (0.6) − × 10 (0.8 ) or a = 5 ms−2
small block is allowed to slide down from the 8
top E of the inclined plane. The coefficient of when, block reached to point C, then from
kinetic friction between the block and the third equation of the motion,
inclined plane is 1/8 and the angle of v2 = u2 + 2as
inclination of the inclined plane is sin −1 (0 .6). where, v = final velocity of the block at point C
If the small block finally reaches the ground at u= initial velocity of block at point E
a point F, then DF will be (Acceleration due to or v = 2as (Q u = 0)
gravity, g = 10 ms −2 ) [20 April 2019, Shift-I] From ∆ EBC, EC =
BE
⇒s =
25 / 6
=
25
sinθ 0.6 6 × 0.6
E
125
25 or s= m
m 18
6
θ C 125 25 −1
B Hence, v = 2 × 5 × = ms
18 3
10 m From second equation of motion of the block at
point C,
Ground θ v cosθ
A D F C θ
5 10 13 20
(a) m (b) m (c) m (d) m
3 3 3 3 v sinθ
Sol.(d) v
According to question, a small block is slide down
from top E of inclined plane as shown in figure, In y-direction,
1
E
R ⇒ h = ut + gt 2
f 2
a
mg c

1 2
25 m
θ
⇒ 10 = usinθt + gt
6 mg sinθ 2
osθ

mg θ
B 25 1
⇒ 10 = × 0.6t + × 10 × t 2
3 2
10 m 25 6 1
⇒ × t + × 10 × t 2 = 10
3 10 2
Ground ⇒ 5t + 5t 2 = 10
A D F ⇒ t 2 + t − 2 = 0 or t = 1 sec
./

Force equation of a block, Now, again from second Eqs. of motion in


⇒ mg sinθ − f = ma x-direction,
25 20
Q friction force applied one block, f = µ k R ⇒ DF=v cosθ. t + 0 or DF = × 0.8 = m
3 3
or f = µ k R (mg cosθ) (From figure)
where, µ k = coefficient of kinetic friction 21. A thin uniform rod of length L is resting
from Eq. (i), we get against a wall and the floor as shown in the
⇒ mg sinθ – µ k mg cosθ = ma figure. Its lower end A is pulled towards left
⇒ a =10sinθ − 1 / 8 × 10 cosθ … (ii) with a constant velocity v. Then the
1
(Q µ k = , given) downward velocity v ′ of the other end B when
8 the rod makes an angle θ with the floor is
Q Given, angle of the inclined plane, θ = sin−1 (0.6) [20 April 2019, Shift-II]
or sinθ = 0.6
Laws of Motion 29

(a) 1 s (b) 2 s (c) 3 s (d) 6 s


B
Sol. (d)
Given, force acts on a body,
L v′ F = (2.6$i + 1.6$j) N
mass of a body, m = 2 kg
v θ 90°
At t = 0, velocity of the body, v = (36
. $i − 4.8 $j) m/s
A
We know that,
(a) v (b) v cosθ (c) v sinθ (d) v cotθ
Force = Mass × Acceleration
Sol. (d)
∴ F= m×a
According to the question,
F (2.6 $i + 1.6 $j)
or a = or a =
m 2
inθ
B
vs or a = (1.3$i + 0.8 $j) m/s 2
dv
v′ Now, the velocity vector, =a
L y dt
∫ dv = ∫ a dt or v = (1.3$i + 0.8$j)t + c
v θ
at t = 0, c = v = (36
. $i - 4.8$j) ms–1
A O
x
∴ v = (36
. + 1.3 t)$i +`(−4.8 + 0.8t) $j m/s
v cosθ
∴ v y = 0 ( because body will just have a velocity
Given, length of the uniform rod = L along x-axis.)
4.8
the angle between rod and floor = θ − 4.8 + 0.8t = 0 ⇒ 0.8t = 4.8 ⇒ t = = 6s
the angle between wall and floor = 90° 0.8
Let the distance between point O to A is x and This is the time at which the body will just have
the distance between point O to B is y. velocity along x-axis
t = 6s
Now, By the velocity displacement relation w.r.t t
dx 23. The force required to move a body up a
v= …(i)
dt rough inclined plane is double the force
dy required to prevent the body from sliding
and v′ = − …(ii)
dt down the plane. If the angle of inclination of
(− negative sign denoted that y is decreasing.) the plane is 60°, then the coefficient of
Futher, x 2 + y 2 = L2 [From the figure] friction is [20 April 2019, Shift-II]
Differentiating above equation w.r.t., time t, 1 1 1 1
(a) (b) (c) (d)
dx dy 3 2 3 2
∴ 2x + 2y =0
dt dx Sol. (c)
dx dy
Now, putting the values of and from Eqs.
dt dt According to the question, angle of inclination
(i) and (ii), we get θ = 60°.
F

x
∴ xv = yv′ or v′ = v
N

y
x
or v′ = v cotθ [From the figure, Q cotθ = ]
y
α

θ
m

So, the downward velocity of the other end B


θ

mg cosθ
when the rod makes an angle θ with the floor is
sin

mg
v′ = v cotθ.
g
m

22. A force of (2.6 $i + 16


. $j) N acts on a body of
N
f=

mass 2 kg. If the velocity of the body at time, θ=60°


t = 0 is (3.6 i$ − 4 .8 $j) ms −1 , the time at which N=mg cosθ
the body will just have a velocity along Now, force of friction, f = µN = µ mg cosθ
x-axis only is [20 April 2019, Shift-II] and net retarding force, (F1) = mg sinθ + f
30 AP EAMCET Chapterwise Physics

∴Net accelerating force down the inclined 25. A block of mass m is lying on a rough
plane is
inclined plane having an inclination
F = mg sinθ − µmg cosθ …(i)  1
∴External force needed (up the inclined plane) α = tan − 1   . The inclined plane is moving
to maintain sliding motion is (net retarding
 5
force) horizontally with a constant acceleration of
F1 = mg sinθ + µmg cosθ …(ii) a = 2 ms − 2 as shown in the figure. The
It is given as, F1 = 2F …(iii) minimum value of coefficient of friction, so
From Eqs. (i),(ii) and (iii) we get that the block remains stationary with
mg sinθ + µmg cosθ = 2(mgsinθ − µmg cosθ) respect to the inclined plane is
or 3µ mg cosθ = mg sinθ (Take, g = 10 ms − 2 ) [21 April 2019, Shift-I]
sinθ
or = 3µ
cosθ
or tanθ = 3µ (Q θ = 60°)
or tan 60° = 3µ m
1 a
or 3 = 3µ or µ =
3
When inclination of plane is 60° then the α
1
coefficient of friction, µ = .
3 2 5 1 2
(a) (b) (c) (d)
9 12 5 5
24. The position-time (x-t) graph of a moving
body of mass 2 kg is shown in the figure. Sol. (b)
The impulse on the body at t = 4 s is The block-plane system is shown in the figure,
[21 April 2019, Shift-I] a sin α (m
)
l o ck
x (m) B
a
α
g sin α cos α a
3 a g cos α
2 g
1 α
0 t (s)
1 2 3 4 5 6 7 8

(a) 1.5 kg ms −1 (b) − 1.5 kg ms − 1 So, from the above free body diagram (FBD),
−1 −1 downward acceleration,
(c) 1 kg ms (d) 2 kg ms
a down = gsinα + a cosα … (i)
Sol. (b) and the upward or opposing acceleration,
Here, mass of the body, m = 2 kg and time, aup = µ (g cosα − a sinα) … (ii)
t = 4s
For block to be stationary,
Impulse = p f − pi
a down = aup [from Eqs. (i) and (ii)]
where, pi = initial momentum, gsinα + a cosα
⇒ µ= … (iii)
p f = final momentum g cosα − a sinα
Impulse = mv f − mvi = m(v f − vi) (Q p = mv)
Divided the Eq. (iii) bysinα and putting the values,
As from the graph, 1
3− 3 3− 0 3 10 + 2
vf = = 0 and vi = = µ= tan α = 10 + 2 × 5 = 20 = 5
8−4 4−0 4
− 2 10 × 5 − 2
1 48 12
10
Impulse = 2 ×  0 −  = − 1.5 kg-ms− 1
3 tanα
 4 Hence, the correct option is (b).
Hence, the correct option is (b).
Laws of Motion 31

26. A block of mass 48 kg kept on a smooth According to the question,


horizontal surface is pulled by a rope of
length 4 m by a horizontal force of 25 N
applied to the other end. If the linear density
of the rope is 0.5 kg m −3 , the force acting on
the block is [21 April 2019, Shift-II] a A
(a) 24 N (b) 25 N (c) 12 N (d) 13 N a
80 cm=0.8m
Sol. (a) B
According to the question,
4m Given, m A = 1.5 kg, mB = 0.5 kg
48 kg 25 N The common acceleration of two masses,
 m − mB  1.5 − 0.5
a= A g = × 10 or a = 5 ms−2
 m A + mB  1.5 + 0.5
Given, linear density of rope, λ = 0.5 kgm −3
According to the question,
hence, mass of rope, m = λ ⋅ l block A is lifted until
= 0.5 × 4 = 2 kg block B touches the
Therefore, the total mass of system, ground a shown in the
M = 48 + 2 = 50 kg figure below, when block A
A is released then it
a h
50 kg 25 N falling freely under
gravity for h height,
therefore from third 0.8 m
F 25 1 B
∴Acceleration, a = = = ms −2 equation of the motion,
M 50 2 v2 = u2 + 2gh or
Hence, the force acting on the block is, v = 0+ 2 × 10h = 20h
2
1
F = 48 × = 24 N But for height 0.8 m block A falls
2
with common acceleration, therefore from
27. Two blocks A and B of masses 1.5 kg and third equation of the motion,
0.5 kg, respectively are connected by a v12 = u12 – 2 a × 0.8
massless inextensible string passing over a here, v1 = 0, u12 = v2 = 20h
frictionless pulley as shown in the figure. ⇒ 20h – 2 × 5 × 0.8 (Q a = 5ms–2)
Block A is lifted until block B touches the ⇒ h = 0.4m = 40 cm
ground and then block A is released. The
Hence, the maximum height reached by block B
initial height of block A is 80 cm when
= 80 + 40 = 120 cm
block B just touches the ground. The
maximum height reached by block B from 28. A system containing masses and pulleys
the ground after the block A falls on the connected on an inclined plane is shown in
ground is [21 April 2019, Shift-II] the figure. If the system is in equilibrium
then the value of m is [22 April 2019, Shift-I]

A 1.5 kg 1 kg

2k
0.5 kg B g

m
(a) 80 cm (b) 120 cm (c) 140 cm (d) 160 cm 30°
Sol. (b) (a) 1 kg (b) 0.5 kg (c) 0.75 kg (d) 0.25 kg
32 AP EAMCET Chapterwise Physics

Sol. (b) If f be the friction force between 2 kg and 4 kg


According to the question, the complete block then the static friction force on 2 kg block
situation is shown in the following figure, .
R1

f' 2 kg 2N
T T1
T1
2g
1 kg
R f ′ = µR1 = 0.5 × 2 g = 0.5 × 2 × 10 [Given, µ = 0.5]
T 1g
2 kg f ′ =10 N
m Since, the static friction on 2 kg block is more
(2g
(2g–T1) 30° –T than force applied on it.
mg 1 )s
cos 30° (2g–T1) 30° in
3 0°
i.e, f ′> 2 N
Hence, 2 kg body will move along the direction
Since, system is in equilibrium, hence of 4 kg body.
T = mg …(i) Hence, net friction force on the block of 2 kg,
T = (2g − T1)sin 30° …(ii) f = f ′ − 2N = 10 − 2 = 8 N
T1 = 1 g …(iii) 30. A body is acted on by a force given by
∴From Eqs. (ii) and (iii), we get F = (15 + 3 t2) N. The impulse received by the
T = (2g −1 g)sin 30° = g sin 30° body during the first 2 seconds is
g
T= …(iv) [22 April 2019, Shift-II]
2
(a) 28 N-s (b) 38 N-s
∴From Eqs. (i) and (iv), we get, (c) 30 N-s (d) 19 N-s
g
mg = ⇒ m = 0.5 kg Sol. (b)
2
Given,
29. In the arrangement shown in the figure, the Force, F = (15 + 3t 2) N
coefficient of friction between two blocks is t t
0.5. The force of friction between the two ∴Impulse, I = ∫ F ⋅ dt = ∫0 (15 + 3t ) dt
2
0
blocks is (Assume that the 4 kg block is For, t = 2s
placed on a smooth horizontal surface). 2
 t3 
(Acceleration due to gravity = 10 ms −2 ). I = 15t + 3 × 
 3 0
[22 April 2019, Shift-I]
I =15× 2+ (2)3 = 30 + 8 = 38 Ns
2 kg 2N 31. A body starts sliding down from the top of
an inclined plane at an angle θ with the
20 N 4 kg horizontal direction. The first one third of
the incline is smooth, the next one third has
(a) 8 N (b) 10 N µ
coefficient of friction and the last one
(c) 6 N (d) 4 N 2
Sol. (a) third has coefficient of friction µ.
According to the question, the given condition is If the body comes to rest at the bottom of
shown in the figure, the plane then the value of µ is
[22 April 2019, Shift-II]
2 kg 2N tanθ 3 tanθ
(a) (b)
20 N 4 kg 2 2
(c) tanθ (d) 2 tanθ
Laws of Motion 33

Sol. (d) 32. An explosion blows a stationary rock into


According to the question, the given situation is three parts. Two parts of masses 1 kg and
shown below, 2 kg moves at right angles to one another
with velocities 12 ms −1 and 8 ms −1 , respectively.

N
If the velocity of third part is 4 ms −1 , the
mass of the rock is

A
sin a [23 April 2019, Shift-I]
θ
θ (a) 8 kg (b) 5 kg (c) 17 kg (d) 3 kg
g
x m

B mg cosθ
mg Sol. (a)
Given, m1 = 1 kg, m2 = 2 kg,
C v1 = 12 ms−1 , v2 = 8 ms−1 and v3 = 4 ms−1
x

θ
Since, in the explosion of stationary rock, the
D momentum is conserved,
so pi = p f
Let mass of body = m and AB = BC = CD = x
0 = p f = p1 + p 2 + p 3
[Given in AB, BC and CD plane coefficient of
µ where, p1 = m1 v1 , p 2 = m2v 2 and p 3 = m3v 3
friction respectively 0, and µ]
2 p 3 = − (p1 + p 2)
For inclined plane AB, friction force, F = 0 p3 = p12 + p 22 + 2 p1 ⋅ p 2 cosθ
So, the net force equation,
p 3 = 122 + 162 + 2 × 12 × 16 cos 90°
mgsinθ − 0 = ma or a = gsinθ
Hence, from third equation of the motion, p 3 = 20
v2 = u2 + 2ax m3v3 = m3 × 4 = 20 ⇒ m3 = 5 kg
Hence, the mass of the rock is,
or v2 = 2 × gsinθ × x = 2gx sinθ [Q u = 0]
m = m1 + m2 + m3
For inclined plane BC, friction force,
m = 1 + 2 + 5 = 8 kg
F = ( µ /2 ) mg cosθ
So, the correct option is (a).
So, net force equation,
⇒ mgsinθ − (µ /2) mg cosθ = ma ′ 33. Four blocks A , B, C and D of masses 6 kg,3kg,
µ 6 kg and 1 kg respectively are connected by
⇒ a ′ = gsinθ − g cosθ
2 light strings passing over frictionless pulleys
From third equation of the motion, as shown in the figure. The strings P and Q
v′2 = u′2 + 2a ′ x are horizontal. The coefficient of friction
⇒ v′2 = u2 + 2a ′ x [Q in this case u′ = v] between the horizontal surface and the block
B is 0.2 and the blocks A and B move
µ
⇒ v′2 = 2gx sinθ + 2  gsinθ − g cosθ x together. If the system is released from rest
 2 
then the tension in the string Q is
v′2 = 4 g x sinθ − µ gx cosθ (Acceleration due to gravity, g = 10 ms −2 )
For inclined plane CD, friction force,
F = µ mg cosθ [23 April 2019, Shift-I]
So, net force equation, P
A
⇒ mgsinθ − µmg cosθ = ma ′′ B
Q
⇒ a ′′ = gsinθ − µg cosθ
From third equation of the motion,
⇒ v″ = u′′ + 2a ′′x = u′ + 2a ′′x (Q u′′ = v′) C D
⇒ 0 = 4 gx sinθ − µg x cosθ +2(gsinθ − µ g cosθ) x
⇒ 0 = 6 g x sinθ = 3µgx cosθ
⇒ 6 gx sinθ = 3ugx cosθ
6 gx sinθ
⇒ µ= ⇒ µ = 2tanθ (a) 48 N (b) 24 N (c) 12 N (d) 6 N
3gx cosθ
34 AP EAMCET Chapterwise Physics

Sol. (c) ( 3 − 1)
From above equation we get, a = g
Given, m A = 6 kg, mB = 3 kg, mC = 6 kg, and 4
mD = 1 kg and coefficient of friction between QMagnitude of the acceleration of centre of mass,
block B and horizontal surface, µ = 0.2 ma + ma 2
Now, the acceleration of complete system, a CM =  1 
 m+ m 
m g − mD g − µ (m A + mB) g
a= C
m A + mB + mC + mD  3 − 1  $  3 − 1  $
gi + 
 4   gj
 4  
6 × 10 − 1 × 10 − 0.2 (6 + 3) × 10 =
= Here, a CM 
6 +3 + 6 + 1  2 
60 − 10 − 18 32  
= = = 2 ms−2
16 16 2 2
g  3 − 1  3 − 1
Now, the tension in the string Q =   +  
is given by 2  4   4 
T
mD a = T − mD g g 3 − 1  3 − 1
= × 2× = g
T = mD a + mD g mD a 2 4  4 2 
T = 1 × 2 + 1 × 10 = 12 N Hence, option (d) is correct.
Hence, the correct answer is (c).
mDg 35. Sand is to be piled up on a horizontal ground
34. In the figure shown, the blocks in the form of a regular cone of a fixed base
have equal masses. Friction, of radius R. Coefficient of static friction
mass of the string and the mass of the pulley between the sand layers is µ. Maximum
are negligible. The magnitude of the volume of the sand can be piled up in the
acceleration of the centre of mass of the two form of cone without slipping on the ground
blocks is (Acceleration due to gravity = g). is [22 April 2018, Shift-I]
[23 April 2019, Shift-I] µR 3 µR 3 πR 3 µπR 3
(a) (b) (c) (d)
3π 3 3µ 3
90° Sol. (d)
m Let h = critical height of sand cone of radius R.
m Then, for a sand particle to be in equilibrium
(it must no slips to the ground)
60° 30°
f cos φ
 3 − 1 g N sin φ
(a)   g (b) θ f φ
 2  2 N
f sin φ N cos φ
 3 − 1
(c) ( 3 − 1) g (d)   g
 4 2  w

Sol. (d) R
For a pulley and block system on a smooth
double inclined plane as shown below f sin φ = N cos φ
where, f = friction,
N = normal reaction and f = µN
T 90º T ⇒ µN sin φ = N cos φ
a
a 1 R
m m ⇒ tan φ = = (from figure)
µ h
mg cos 30º mg cos 60º
60º 30º So, maximum volume of sand cone that can be
formed over level ground is
Force equation for both the blocks,
1 1 µπR3
⇒ mg cos 30° − T = ma …(i) Vmax = πR2h = πR2(µR) =
3 3 3
⇒ T − mg cos 60° = ma …(ii)
Laws of Motion 35

36. A block of mass 2 kg is being pushed against Sol. (b)


a wall by a force F = 90 N as shown in the T T
figure. If the coefficient of friction is 0.25, 200g 300g
m1 m2
then the magnitude of acceleration of the
 3
block is (Take, g = 10 ms −2) sin 37 ° =  30º 30º
 5
From free body diagram
[22 April 2018, Shift-I]
T − m1 gsinθ = m1 a .............(i)
and m2 gsinθ − T = m2a .............. (ii)
Adding Eqs (i) and (ii), we get
(m2 − m1) gsinθ = (m1 + m2)a
(300 − 200) g sin 30° 1
F ⇒ a= ⇒a = g
(300 + 200) 10
37º
∴ a = 10% of g
−2 −2
(a) 16 ms (b) 8 ms 38. A particle of mass 4 M which is initially at
(c) 38 ms −2 (d) 54 ms −2 rest explodes into three pieces of masses M,
M and 2 M. The equal masses move along X
Sol. (b) and Y -axes with velocities 4 ms − 1 and
Block’s weight (downward), w = 2 × 10 = 20 N
6 ms − 1 , respectively. The magnitude of the
Vertical component of applied force (upwards), velocity of the heavier mass is
FV = F sinθ = 90 × 3 / 5 = 54 N [23 April 2018, Shift-I]
Maximum frictional force, (a) 17 ms − 1 (b) 2 13 ms − 1
Fr = µF cosθ = 0 ⋅ 25 × 90 × 4 / 5 13
= 18N (c) 13 ms − 1 (d) ms − 1
2
∴Net vertical force, Fnet = FV − (Fr + w) = ma
Sol. (c)
∴ Fnet = 54 − 18 − 20 = 16N = ma
16 Initial mass of particle = 4M
⇒ a= = 8 m / s2 Initial velocity of particle = 0.
2
After explosion into three pieces of masses
37. A double inclined plane as shown in the M , M and 2M particle of equal masses move
along X and Y axes with velocities 4m/s and
figure has fixed horizontal base and smooth
6m/s, respectively. So, according to law of
faces with the same angle of inclination of conservation of momentum mass 2M particle
30°. A block of mass 300 g is on one face and will move in the direction shown in the figure.
is connected by a cord passing over a Mvy
frictionless pulley to a second block of mass
200 g kept on another face. The acceleration
with which the system of the blocks moves
is ........ % of acceleration due to gravity. Mvx
[22 April 2018, Shift-II]
2Mv
Law of conservation of momentum,
200g 300g
Momentum before collision
= Momentum after collision.
4 M × 0 = 2Mv + Mv x + Mv y
30° 30°
⇒ −2Mv = M(v x + v y)
⇒ −2v = 4$i + 6$j ⇒ v=− 2$i + 3$j
(a) 5 (b) 10
(c) 15 (d) 20 So,|v|= 22 + 32 = 4 + 9 = 13 ms−1
36 AP EAMCET Chapterwise Physics

39. A rocket with an initial mass m0 is going up (a) 2 ms −2 (b) 4 ms −2


−2
with a constant acceleration a by exhausting (c) 6 ms (d) 8 ms −2
gases with a velocity v relative to the rocket Sol. (c)
motion, then the mass of the rocket at any
instant of time is (assume that no other T sin 30°
forces act on it) [23 April 2018, Shift-II] T
T
at 2a t
− −
(a) m = m0e v (b) m = m0e v 30º
a
at a2t 2
− −
(c) m = m0e 2v (d) m = m0e v2

Sol. (a) Let T is tension in the string.


dv dm Maximum vertical force on clamp, Tsin 30° = 40
For a rocket, m = Fext + v
dt dt 1
⇒ T ⋅ = 40 ⇒ T = 80N
2
Let maximum acceleration of monkey for sate
vg m (t) F climbing is a. Then, FBD of monkey

v (t) T

Without any external force (Fext = 0),


dv dm M a
m =v
dt dt
dv dm mg
⇒ ∫ v =∫ m T − mg
T − mg = ma ⇒ a =
⇒ log v = log m + C m
When, v = 0, m = m0 80 − 5 × 10
a=
∴ C = − log m0 5
So, we have a = 6 m / s2
∆v m
= log . dt 41. In the arrangement shown in the figure, if
v m0
the blocks of masses m and 2m are released
− at / v
or m = m0 e from the state of rest, tension in the string is
(µ = coefficient of friction, string is massless
40. One end of a light string is fixed to a clamp and inextensible, pulley is frictionless)
on the ground and the other end passes over [24 April 2018, Shift-I]
a fixed frictionless pulley as shown in the
figure. It makes an angle of 30° with the
ground. The clamp can tolerate a vertical
force of 40 N. If a monkey of mass 5 kg were m 2m
to climb up the rope, then the maximum µ= 2 µ= 1
3 45º 45º 3
acceleration in the upward direction with
which it can climb safely is (g = 10 ms−2 )
(a) mg (b) 2 mg
[24 April 2018, Shift-I]
2 2 mg 2 mg
(c) (d)
3 3
Sol. (c)
T
Let N1 , N 2 are normal reaction force and f1 , f2
Clamp are the friction force on two blocks. acceleration
30º a
is a and tension is T.
Laws of Motion 37

The respective FBD Roof


a
N1 T Hollow
cylinder

mg sin 45º m String


mg cos 45º
f1 mg m
45º
2g g 5g
(a) (b) (c) (d) g
T N2 3 2 6
f2
Sol. (b)
2m

a Let tension in string is T and tension T is


rotating the hollow cylinder. Torque produced
2mg cos 45º 2 mg sin 45º
2 mg in hollow cylinder,
45º
τ = Iα

Friction force, f1 = µN1 , f2 = µN 2

2 2 Hollow
As, N1 = mg cos 45°, f1 = ⋅ mg cos 45° = mg
3 3 cylinder
and N 2 = 2mg cos 45°, T
2 2 2
f2 = ⋅ 2mg cos 45° = mg T
3 3
⇒ Now, by second law of motion for 2m mass, a m
2mg cos 45° − T − f2 = 2ma
mg
2 2
2mg − T − mg = 2ma … (i)
3 Moment of inerita of cylinder, I = MR2
For m1 , T − f1 − mgcos 45° = ma
where, M = mass of cylinder
2 mg
T− mg − = ma … (ii) and R = radius of cylinder.
3 2 a
Multiplying by 2 in Eq. (ii), we get Angular acceleration, α = , where
R
2T −
2 2
mg − 2 mg = 2ma … (iii) a = acceleration.
3 a
⇒ τ = MR2 ⋅ = MRa … (i)
Subtracting Eq. (iii) from Eq. (i), we get R
2 2mg Also, torque, τ = F ⋅ R = TR … (ii)
3T − 2 2 mg = 0 ⇒ T =
3 Equating Eqs. (i) and (ii), we get TR = MRa
⇒ T = Ma
42. In the figure shown, acceleration with which
Now, Mg − T = Ma
the mass m falls down when released is
Mg − Ma = Ma
(consider the string to be massless,
g-acceleration due to gravity) ⇒ 2Ma = Mg
[24 April 2018, Shift-I] ⇒ a = g/ 2
4
Work, Energy
and Power
1. Work done [17 Sep. 2020, Shift-I] and displacement (directed tangentially) are
perpendicular to each other.
(a) can only be positive
∴ Work done W = F s cos 90° = 0 [Q cos 90° = 0]
(b) can only be negative
(c) can either be positive or negative 3. In a circus, a stuntman rides a motor bike in
(d) cannot be assigned a sign a vertical circular track of radius r. Find the
Sol. (c) minimum speed, he must maintain at
Work done by a constant force is given as highest point of track. [17 Sep. 2020, Shift-II]
W = Fs cosθ (a) 2 gr (b) 2 gr
(c) gr (d) 5 gr
where, F = applied force
s = displacement Sol. (c)
and θ = angle between the direction of force For motion in vertical circle, to complete the
and displacement. revolution of moving body, the minimum speed
When, θ = 0°, then W = F s (positive) of body at lowest point and highest point should
When, θ = 90° , then W = F s cos 90° be 5gr and gr respectively, where r is the
= 0 (zero) radius of circular path.
When, θ = 180°, then W = F s cos180° Hence, in circus, for completing vertical circular
= − F s (negative) track, minimum speed of stuntman at highest
point of track should be gr.
Hence, work done may be positive, negative or
zero. 4. A cyclist comes to a skidding stop in 10 m.
2. When a body moves in a circular path, no During this process, the force on the cycle
work is done by the force, since due to the road is 200 N and is directly
opposite to the motion. How much work
[17 Sep. 2020, Shift-II]
does the road do on the cycle?
(a) there is no net force [17 Sep. 2020, Shift-II]
(b) there is no displacement
(a) 2000 J (b) − 2000 J
(c) force is always away from the centre
(c) − 1000 J (d) 0
(d) force and displacement are perpendicular to
each other Sol. (b)
Sol. (d) Displacement travelled by cyclist to reach
skidding stop, s = 10 m
When a body moves in a circular path, then
centripetal force is always directed towards Force on the cyclist due to road,
centre of circular path. Hence, centripetal force F = 200 N
Work, Energy and Power 39

Since, force is applied directly opposite to Sol. (c)


motion (or displacement), hence θ = 180° Potential energy,
∴ Work done by the road on the cycle U = 2 − 20 x + 5x 2 J
W = F s cosθ = 200 × 10 × cos180° = − 2000 J

5. A person of mass M = 90 kg standing on a


A B
smooth horizontal plane of ice throws a body x
of mass m = 10 kg horizontally on the same x=–3m
surface. If the distance between the person
and body after 10 s is 10 m, then the KE of
the person (in J) is [18 Sep. 2020, Shift-I]
When particle gains whole of potential energy
(a) 0.55 J (b) 4.5 J (c) 0.90 J (d) 0 in the form of kinetic energy at x = − 3m, then
Sol. (a) it will travel maximum distance till whole of its
kinetic energy becomes zero.
According to the question body travels
10 m in 10 s, hence its velocity is given as i.e., K =U = 0
10 m ⇒ 2 − 20 x + 5x 2 = 0
v= = 1 m/s
10 s 5x 2 − 20 x + 2 = 0
∴According to conservation of linear −(−20) ± (−20)2 − 4 × 5 × 2
∴ x=
momentum. 2× 5
Initial total moment = Final total moment 20 ± 360 20 ± 18.97
= =
0 + 0 = M ⋅ v′ + mv 10 10
where, v′ is velocity of person. x = 2 ± 1.9
mv 10 × 1 1 For maximum value of x, taking + ve sign
v′ = − =− = − m/s
M 90 9 x = 2 + 1.9 = 3.9
1
∴ Kinetic energy of person = Mv′2 ∴Total distance = |−3| + 39
. = 6.9 m ~−7m
2
2 8. When two bodies collide elastically, then
= × 90 ×  −  = 0.55 J
1 1
2  9 [18 Sep. 2020, Shift-I]
(a) kinetic energy of the system along is conserved
6. If a force F is applied on a body and it moves
(b) only momentum is conserved
with a velocity v, the power will be
(c) both kinetic energy and momentum is
[18 Sep. 2020, Shift-I] conserved
F F (d) Neither kinetic energy nor momentum is
(a) F ⋅ v (b) (c) 2 (d) F ⋅ v 2
v v conserved
Sol. (a) Sol. (c)
We know that, power is given as In elastic collision, both kinetic energy and
dW d momentum are conserved. In this collision, total
P= = [F ⋅ s] [Q W = F ⋅ s] energy is also conserved.
dt dt
ds
= F⋅ [For constant force F] 9. The kinetic energy of a body is increased by 4
dt times. Its momentum will [18 Sep. 2020, Shift-II]
⇒ P = F⋅v Q v = ds  (a) increase twice (b) increase four times
 dt  (c) decrease twice (d) remains constant
7. A particle is moving an X -axis has potential Sol. (a)
energy U = 2 − 20 x + 5 x 2 J along X -axis. The Given, initial kinetic energy,K 1 = K
particle is released at x = − 3. The maximum Final kinetic energy, K 2 = 4K
value of x will be (x is in metre and U is in Momentum and kinetic energy are related as,
joules) p2
[18 Sep. 2020, Shift-I] K = ⇒ p = 2mK
(a) 5m (b) 3m (c) 7m (d) 8m 2m
40 AP EAMCET Chapterwise Physics

∴ Initial momentum, p1 = 2mK 1 ∴ By conservation of momentum,


⇒ = 2mK …(i) m1 × 10 + m2 × (−10) = m1 v1 f + m2 v2 f
⇒ 2 × 10 + 4 × (−10) = 2v1 f + 4v2 f
Final momentum, p2 = 2mK 2 = 2m ⋅ 4K
⇒ −20 = 2(v1 f + 2v2 f )
⇒ = 2 2mK = 2p1 [from Eq. (i)]
⇒ v1 f + 2v2 f = − 10 …(i)
10. A bullet strikes against a wooden block and In perfectly elastic collision,
Velocity of separation = Velocity of approach
is embedded in it, the nature of collision is
⇒ v2 f − v1 f = 10 − (−10)
[18 Sep. 2020, Shift-II] ⇒ v2 f − v1 f = 20 …(ii)
(a) elastic (b) perfectly inelastic Adding Eqs. (i) and (ii), we get
(c) inelastic (d) perfectly elastic 3v2 f = 10
Sol. (b) v2 f =
10
ms −1
When a bullet strikes a wooden block and 3
embedded in it, then such type of collision is an From Eqs. (ii), we get
example perfectly inelastic collision. In this type 10
of collision, maximum loss of kinetic energy − v1 f = 20
3
occurs.
10 50
⇒ − v1 f = 20 − =
11. A machine which is 70% efficient raises a 3 3
10 kg body through a certain distance and −50 −1
⇒ v1 f = ms
spends 100 J energy. The body is then 3
released. On reaching the ground, the kinetic
13. An automatic gun fires 360 bullets per
energy of the body will be [18 Sep. 2020, Shift-II]
minute with a speed of 360 km/h. If each
(a) 0 (b) 70 J (c) 50 J (d) 35 J
bullet weighs 20 g, the power of the gun is
Sol. (b) [21 Sep. 2020, Shift-I]
Energy spent by the machine to raise the body
(a) 75 W (b) 150 W (c) 300 W (d) 600 W
of mass 10 kg at certain height, is E = 100 J
Since, machine is only 70% efficient. Sol. (d)
Hence, energy used by machine, Speed of each bullet, v = 360 km/h
70 5
E ′ = 70 % of E = × 100 = 70 J = 360 × m/s = 100 m/s
100 18
When body is released, then whole energy Number of bullets fired per minute, N = 360
spent by machine is converted into kinetic Kinetic energy of 360 bullets,
energy on reaching the ground.
k′ = N × kinetic energy of each bullet
∴ Kinetic energy, K = E ′ = 70 J 1 1
= 360 × mv2 = 360 × × 0.02 × 1002
12. Two balls X (2 kg) and Y (4 kg) approach 2 2
each other with equal speeds of 10 ms − 1 . If = 36
. × 104 J
the collision is perfectly elastic, then the new Total energy 36 . × 104
∴Power of gun = = = 600 W
velocities of balls X and Y are respectively Time 60
[21 Sep. 2020, Shift-I] [Q Time, 1 minute = 60 s]
50 10 50 10
(a) ms − 1 , − ms − 1 (b) − ms − 1 , − ms − 1 14. The power utilised when a force of
3 3 3 3
50 10 50 10 (2 $i + 3 $j + 4 k$ ) N acts on a body for 4 s,
(c) − ms − 1 , ms − 1 (d) ms − 1 , ms − 1
3 3 3 3 producing a displacement of
(3 $i + 4 $j + 5 k$ ) m , is [21 Sep. 2020, Shift-I]
Sol. (c)
Given, m1 = 2 kg, m2 = 4 kg (a) 9.5 W (b) 7.5 W (c) 6.5 W (d) 4.5 W
v1 = v2 = 10 ms −1 Sol. (a)
In perfectly elastic collision, momentum and $
Force, F = 2$i + 3$j + 4k
kinetic energy is conserved. $
Displacement, s = 3$i + 4$j + 5k
Work, Energy and Power 41
Displacement
∴ Velocity = 17. When a moving body collides with a
Time stationary body of n times its mass, then the
s 1 $)
⇒ v = = (3$i + 4$j + 5k amount of kinetic energy transferred to the
t 4 stationary body is [21 Sep. 2020, Shift-II]
∴Power utilised is given as 4n n n2 4n2
P = F⋅ v (a) (b) (c) (d)
(1 + n)2 (1 + n)2 (1 + n)2 (1 + n)2
$ ) ⋅ 1 (3$i + 4$j + 5k
= (2$i + 3$j + 4k $)
4 Sol. (a)
3 4 5
= 2× + 3× + 4 × Suppose, mass of moving body is m1 .
4 4 4 Mass of stationary body, M 2 = nm1
= 1.5 + 3 + 5 = 9.5 W For elastic collision,
15. The minimum horizontal speed with which a velocity of separation = velocity of approach
body must be projected, so that it goes around v2 − v1 = u − 0
a smooth vertical circular track of radius 4 m v2 − u = v1 …(i)
is (g = 9.8 ms −2) [21 Sep. 2020, Shift-I] By the law of conservation of momentum,
(a) 7 ms − 1 (b) 14 ms − 1 (c) 0.7 ms − 1 (d) 1.4 ms − 1 m1 u = m1 v1 + M 2 v2
Sol. (b) ⇒ m1 u = m1 v1 + nm1 v2
Radius of circular vertical track, u = v1 + nv2 …(ii)
r = 4 m, g = 9.8 ms −2 u = v2 − u + nv2 [From Eq. (i)]
∴Minimum horizontal speed at lowest point to ⇒ 2u = (n + 1)v2
2u
complete the revolution in vertical circle, v2 =
Ge v = 5gr = 5 × 9.8 × 4 = 196 = 14 ms −1 n+1
Putting this value in Eq. (i)
16. A bomb of mass 9 kg explodes into two 2u 2u − nu − u (1 − n)u
v1 = v2 − u = − u= =
pieces of mass 3 kg and 6 kg. The velocity of n+1 n+1 n+1
mass 3 kg is 16 m/s. The kinetic energy of ∴ Kinetic energy of moving body of mass m1 ,
mass 6 kg (in joule) is [21 Sep. 2020, Shift-II] before collision,
(a) 96 (b) 384 (c) 192 (d) 768 1
K 1 = m1 u2
Sol. (c) 2
Given, mass of bomb, M = 9 kg Kinetic energy of stationary body after collision,
2
K 2 = M 2 v22 = nm1 
m1=3kg 1 1 2u 

2 2  n + 1
1 4n
= m1 u2 ⋅
v1=16 m/s 2 (n + 1)2
M 9 kg ∴ Amount of KE transferred to stationary body
1 4n
m2=6kg m1 u2 ×
K2 2 (n + 1)2 4n
= = =
K1 1
m1 u2 (n + 1)2
2
According to conservation of linear momentum,
M × 0 = m1 v1 + m2 v2 18. A man pushes the wall and fails to displace
⇒ 9 × 0 = 3 × 16 + 6 × v2 it. He does [21 Sep. 2020, Shift-II]
−3 × 16 (a) negative work
⇒ v2 = = −8 m/s
6 (b) positive but not maximum work
∴ Kinetic energy of mass, m2 = 6 kg (given) (c) no work at all
1 1 (d) maximum work
K = m2 v22 = × 6 × (−8)2
2 2 Sol. (c)
1
= × 6 × 64 = 192 J When man pushes the wall, then he does not
2 produce any displacement in the wall.
42 AP EAMCET Chapterwise Physics

i.e., displacement = 0 Similarly, m2 =


4
π r23 ρ
∴ Work done = Force × Displacement 3
= Force × 0 = 0 m1 r 3 (2 × 10−2)3 1
∴ = 13 = =
m2 r2 (4 × 10−2)3 8
19. A string of a pendulum of length l is
displaced through 90° from its vertical and ⇒ m2 = 8 m1 …(i)
released. Then, the minimum strength of the For elastic collision, velocity of first ball,
string needed to withstand the tension as,  m − m2   2m2 
v1 =  1  u1 +   u2
the pendulum passes through its mean  m1 + m2   m1 + m2 
position is [22 Sep. 2020, Shift-I]
Here, u1 = 0 and u2 = 81 cms −1
(a) mg (b) 3 mg (c) 5 mg (d) 6 mg  m − m2   2m2 
∴ v1 =  1  ×0+   × 81
Sol. (b)  m1 + m2   m1 + m2 
Length of pendulum = l 2 × 8 m1
l =0+ × 81 [from Eq. (i)]
Centre of gravity of pendulum’s string = m1 + 8m1
2
16
When pendulum is displaced through 90° and = × 81 = 144 cm s −1
released, then according to law of conservation 9
of energy. 21. A bullet of mass m and velocity v when fired
Gravitational potential energy = Kinetic energy at a sand bag of mass M, suspended by a string,
l 1
⇒ mg ⋅ = mv2 ⇒ v2 = gl gets embedded into the bag. The loss of kinetic
2 2 energy in this process is [22 Sep. 2020, Shift-I]
⇒ v = gl …(i) mv 2 mv 2 Mv 2 mMv 2
(a) (b) (c) (d)
At the mean position, tension in the string is 2 2(M + m) 2 2(M + m)
balanced by the weight as well as the
centrifugal force. Sol. (d)
mv2 Q r = l  Mass of the bullet = m
Hence, T = mg +
l/2  2 Speed of bullet = v
2m 2 According to question, bullets gets embedded
= mg + ⋅v into the bag, then they will move with common
l
velocity v1 (say).
2m
= mg + ⋅ ( gl)2 [from Eq. (i)] This is the case of perfectly enelastic collision.
l
∴Initial kinetic energy of bullet,
2m gl
= mg + = mg + 2 mg = 3 mg 1
l K i = mv2 …(i)
2
20. A steel ball of radius 2 cm is at rest on a By the conservation of linear momentum,
frictionless surface. Another steel ball of mv = (M + m) v1
radius 4 cm moving with velocity of 81 cms −1 ⇒ v1 =
mv
…(ii)
collides elastically with the ball which is at M+ m
rest. After collision the ball with radius of 1
∴Final kinetic energy, K f = (M + m) v12
2 cm moves with speed of [22 Sep. 2020, Shift-I] 2
(a) 144 cm s −1 (b) 72 cm s −1 1
= (M + m)
m2 v2
[from Eq. (ii)]
(c) 216 cm s −1 (d) 36 cm s −1 2 (M + m)2
Sol. (a) 1 m2 v2
=
Radius of first steel ball, 2 M+ m
r1 = 2 cm = 2 × 10−2 m Loss in kinetic energy = K i − K f
Radius of second steel ball, 1 1 m2 v2
= mv2 −
r2 = 4 cm = 4 × 10−2 m 2 2 M+ m
∴ Mass of first steel ball,
1  m  mMv2
4 = mv2 1 − =
m1 = V1 × ρ = π r13 ρ [ρ = density] 2  M + m  2(M + m)
3
Work, Energy and Power 43
W F×s
22. A bullet of mass 0.01 kg travelling at a speed P= = [Q W = Fs]
−1
of 500 ms strikes a block of mass 2 kg t t
50 × 60
which is suspended by a string of length 5m. = = 50 W
The centre of gravity of the block is found to 60
rise a vertical distance of 0.1 m. What is the 24. A uniform chain of mass M and length L is
speed of the bullet after it emerges from the lying on a smooth horizontal table, with half
block? [22 Sep. 2020, Shift-I] of its length hanging down. The work done
(a) 200 ms −1 (b) 220 ms −1 is pulling the entire chain up the table is
(c) 204 ms −1 (d) 284 ms −1
[23 Sep. 2020, Shift-I]
Sol. (b) MgL MgL MgL MgL
(a) (b) (c) (d)
According to question, the given situation is 2 4 8 16
shown in the following figure.
O Sol. (c)
Let λ be linear mass density of chain. To pull the
chain we have to do work against the weight of
5m
hanging part of chain.
Let at any instant length of hanging part be x.
M Therefore, the weight of hanging part of chain is
M
0.1 m F = λxg
The work done in pulling the chain by small
Mass of block, M = 2 kg distance dx is dW = − Fdx = −λxgdx
Mass of bullet, m1 = 0.01 kg
Speed of bullet, v = 500 ms −1
F
h = 0.1 m
L/2
If v1 and v2 be the velocities of the bullet and
blocks after collision, then by conservation of x
energy,
1
Mv22 = Mgh
2 The work done to pull the whole of hanging
v2 = 2gh = 2 × 9.8 × 0.1 = 1.4 ms −1 part of chain is
0
According to the law of conservation of W = ∫ dW = ∫ − λgx dx
momentum, L/2

m1 v = m1 v1 + Mv2
0
 x2 
0

m v − Mv2 0.01 × 500 − 2 × 1.4 = − λg ∫ xdx = − λg


⇒ v1 = 1 =  2
L/2  L / 2
m1 0.01
 (0)2 (L / 2)2 
5 − 2.8 2.2 = − λg −
= = = 220 ms −1  2 2 
0.01 0.01 
λgL 2
( λL) gL
23. An electric motor exerts a force of 50 N on a W= = (Q m = λL)
8 8
a cable and pulls it through 60 m in 1 min. mgL
The power supplied by the motor is or W=
8
[22 Sep. 2020, Shift-II]
(a) 50 W (b) 3000 W (c) 1 W (d) 100 W 25. Two particles P and Q each of mass 3m lie at
rest on the X-axis at points (−a , 0) and (+a , 0),
Sol. (a)
respectively. A third particle R of mass 2 m
Force exerted by electric motor, initially at the origin moves towards the
F = 50 N particle Q. If all the collisions of the system
Displacement, s = 60 m of 3 particles are elastic and head on, the
Time, t = 1 min = 60 s total number of collisions in the system is
∴Power supplied by the motor, [20 April 2019, Shift-I]
(a) 2 (b) 3 (c) 4 (d) 5
44 AP EAMCET Chapterwise Physics

Sol. (a) particle moves from (0 , a) to (a , 0) along a


According to question , three particles are circular path of radius a about the origin is
situated as shown below, [20 April 2019, Shift-II]
2Kπ Kπ Kπ
(a) (b) (c) (d) 0
a a 2a
P R v Q
X-axis Sol. (d)
3m 2m 3m According to the question,

Q All the collision of the system of 3 particles


are elastic and head on. So, collision of R and Q, r y=r sinθ
θ
R Q x=r cosθ
P v1 v2
2m 3m Force on a particle moves in the x - y plane,
3m
 x y $j
F = k 2 $i +
2 3/ 2 
 ( x + y 2 3/ 2
) ( x 2
+ y ) 
Hence, R and Q will never collide after 1st collision. Now, Putting, x = r cosθ, y = r sinθ
∴Total numbers of collision is 2. [Q From figure]
26. A motor engine pumps 1800 L of water per  r cosθ
F = k 2 $i
 (r cos θ + r sin θ)
2 2 2 3/ 2
minute from a well of depth 30 m and allows
to pass through a pipe of cross-sectional area + 2
r sinθ $j
30 cm 2. Then the power of the engine is (r cos2 θ + r 2 sin2 θ)3 / 2 
(Acceleration due to gravity, g = 10 ms −2) Q sin2 θ + cos2 θ = 1
F = k  3 (cosθ $i + sinθ $j)
[20 April 2019, Shift-I] r
(a) 20.5 kW (b) 15.5 kW (c) 10.5 kW (d) 9.5 kW  r 
k
Sol. (c) F = 2 [cosθ $i + sinθ $j]
r
Given, depth of a well, d = 30 m
∴Work done, W = F × s (Q s = 0, for circle)
water quantity per minute = 1800 litre
Now, we can say that the direction of force is
cross-section area of pipe, A = 30 cm 2
along the radius of circle. Hence, the work done
= 30 × 10−4 m 2 by this force will be zero.
Now,
1800 × 10−3 28. Potential energy of a body of mass 1 kg free
velocity of water a jet = = 10 m/s to move along X -axis is given by
(30 × 10−4 ) × 60
 x2 
and work done by engine = mgd + mv2
1 U(x) =  − x  J. If the total mechanical
2 2 
1
= 1800 × 10 × 30 + × 1800 × (10)2
energy of the body is 2 J, then the maximum
2 speed of the body is (Assume only
= 540000 + 90000 = 630000 J conservative force acts on the body)
W 630000 [21 April 2019, Shift-I]
Power of engine, P = = − 1 − 1
t 60 (a) 5 ms (b) 5 ms (c) 3.5 ms − 1 (d) 8 ms − 1
= 10500 W = 10.5 kW Sol. (a)
27. A particle moves in the x-y plane under the Key Idea Total mechanical energy of a system is the
action of a force, addition of potential and kinetic energy
 E = U + KE
F = K 2
x
i$ + 2
y $j where, K is
2 2/1  Here, mass of the body, m = 1 kg,
 (x + y 2 3/ 2
) (x + y )  Emech = 2 J
a constant. Work done by the force when the
Work, Energy and Power 45

dU(x) d  x2  = 104 × 3.14 × (2.5)2 × 182 ,


So, for Umin = = − x =0
dx dx  2 
 U = 3571
. × 106 J
⇒ x −1 = 0 ⇒ x =1 Now, the time taken by pump of power 10 HP
work done
12
() 1 time = ,
Hence, Umin = −1 = − power
2 2
35.71 × 106
1 2 v2 t= ≈ 80 min [Q1 HP = 746W]
Kinetic energy, KE = mv = (Q m = 1 kg) 10 × 746 × 60
2 2
Hence, the correct option is (b).
Now, putting the values in above expression,
1 v2 30. A body is released from a height of 30 m
⇒ 2= − +
2 2 vertically downwards. The speed of the body
⇒ v2 = 5 at which potential energy is twice that of
⇒ v = 5 m/s kinetic energy is
Hence, the correct option is (a). (Acceleration due to gravity, g = 10 ms −2)
[21 April 2019, Shift-II]
29. A cylindrical well of radius 2.5 m has water −1
upto a height of 14 m from the bottom. If (a) 20 2 ms (b) 10 2 ms −1
the water level is at a depth of 6 m from the (c) 10 ms −1 (d) 20 ms −1
top of the well, then the time taken (in Sol. (b)
minutes) to empty the well using a motor of Total energy at height 30 m = mg(30) = 30 mg
10 HP is approximately, (Take, g = 10 ms − 2) Q TE = PE + KE ⇒ 30 mg = PE + KE …(i)
[21 April 2019, Shift-I] According to the question, PE = 2KE
(a) 30 (b) 80 From Eq. (i),
(c) 98 (d) 90 ⇒ 30mg = 2KE + KE
Sol. (b) 1
⇒ KE = 10mg ⇒ mv2 = 10mg
A cylindrical well of radius 2.5 m is shown in 2
the figure, Hence, speed of the body, v = 200 = 10 2 ms−1

31. The work done by a force F = − 5 x 4 i$ N in


6m
2.5m displacing a body from, x = 2 m to x = − 2 m
x is [21 April 2019, Shift-II]
20m (a) 6 J (b) 8 J (c) 64 J (d) 0 J
Sol. (c)
Given, force on the body, F = − 5x 4 $i N
dx
Body is displacing from x = 2m to x = –2m .
Bottom Hence, the work done by a force in displacing a
Let a small volume element dx at a distance x body,
x = −2 x = −2 −2
from the surface, then mass of the element,  x5 
W= ∫ F ⋅ dx or W = ∫ (−5x )dx = (−5)
4

dm = ρdV = ρπr 2 dx  5
x =2 x = 2m  2
So, the potential energy of the mass dm,
⇒ = (−1)[(−2) − (2) ⇒ W = 64 J
5 5

dU = dmg x ⇒ dU = gρπr 2 xdx


Integrating on the both sides, we get 32. In the arrangement shown in the figure,
20 work done by the string on the block of mass
U = gρπr ∫
2
x dx
6 0.36 kg during the first second after the
Here, g = 10 m / s2 , density of water, blocks are released from state of rest is
ρ = 103 kg / m3 (Ignore friction and mass of the string.)
r = 2.5 m
(Acceleration due to gravity, g = 10 ms −2)
 202 62 
U = 10 × 103 × 314
. × (2 . 5)2 − [22 April 2019, Shift-I]
 2 2 

46 AP EAMCET Chapterwise Physics

36
.
T = 36
. + 0.36 ×
1.08
T = 4.8 …(iv)
Distance travelled by mass m1 block,
By displacement-time relation,
1
0.36 kg Displacement, s = ut + at 2 [Q u = 0]
2
[by kinetic equation for uniformly motion]
0.72 kg
1 36 .
s= × × ()
1 2 [∴t = 1 second given]
(a) 8 J (b) 4 J (c) 12 J (d) 2 J 2 1.08
1 36 .
Sol. (a) s= × …(v)
2 1.08
According to the question,
Now, work done by the string on the block of
mass (m1 ) is 0.36 kg.
∴Work done,
W = Tension force (T) × displacement(s)
1
W = T × at 2
T 2
T
From Eqs. (iv) and (v), we get
1 36 .
m1=0.36kg W = 4.8 × ×
2 1.08
16
m2=0.72kg W= ⇒ W =8J
2
Given, mass of block 1, m1 = 0.36 kg
mass of block 2, m2 = 0.72 kg 33. A man who is running has half the kinetic
and acceleration due to gravity, g =10 m / s2 energy of a boy of half his mass. The man
Now, Dynamic equation of block m1 (m2 > m1 ). speeds up by 1 ms −1 and then has the same
∴ T − m1 g = m1 a …(i) kinetic energy as the boy. The initial speed of
Dynamic equation of block m2 (m2 > m1 ). the boy is [22 April 2019, Shift-I]

∴ m2 g − T = m2 a …(ii) (a) 2 + 1 ms −1 (b) 2( 2 + 1) ms −1


(c) 2 ms −1 (d) 2 ms −1
Thus, from adding Eqs. (i) and (ii), we get
∴ T − m1 g + m2 g − T = m1 a + m2 a
Sol. (b)
According to the question,
m2 g − m1 g = a (m1 + m2)
let the mass of man be m.
(m − m1 ) g
or a= 2 m
(m1 + m2) So, mass of boy is .
2
 m − m1  Let speed of man be v1 and that of boy be v2 .
Hence, acceleration, a =  2 g
 m1 + m2  1
∴Kinetic energy of man = kinetic energy of boy
(0.72 − 0.36) 2
∴ Acceleration, a = × 10 1
1.08 KEman = KEboy
.
36 2
a= m / s2 …(iii) 1 2 1 1 m 2
1.08 or mv1 = × × v2
Now, the tension force in the string is, 2 2 2 2
2
for block 1st, m1 = 0.36 v1 1
or =
T − m1 g = m1 a v22 4
T − m1 g v1 1
=a ⇒ =
m1 v2 2
T − 36
. 36. 1
= ∴ v1 = v2 …(i)
0.36 1.08 2
Work, Energy and Power 47

If the velocity of man increased by 1 m/s then, When ball is dropped from point P, and collides
new velocity of man, v1′ =  2 + 1
v
at point Q , from third equation of the motion,
 2 
vQ2 = u2 + 2gh = 0 + 2 × 10 × (12 − 1) = 220
[∴From Eq. (i)] ∴Velocity at a point Q, VQ = 220 m / s
Now, kinetic energy of man (KE) = kinetic
energy of Boy (KE B) In perfect elastic collision at point Q, there is no
2 loss of kinetic energy.
× m ×  2 + 1 = × × v22
1 v 1 m
∴ Hence, the height gain by ball after collision,
2  2  2 2
1 2
2 2 ⇒ mvQ = mgh′
 v2 + 1 =  v2 
    2
 2   2 1
⇒ ( 220)2 = 10 × h′
v2 v 2
+1= 2
2 2 ⇒ h′ = 11m
2v2 + 2 2 = 2v2 Let velocity of the ball before inelastic collision
= vs
2 2 = 2v2 − 2 v2 ⇒ 2 v2 ( 2 −1) = 2 2
From energy conservation law,
2
v2 = ⇒
1 2
mvs = mg (11 + 0.5)
( 2 −1) 2
2 ( 2 + 1) 2 ( 2 + 1) ⇒ vs2 = 2 × 10 × 11.5
⇒ v2 = × =
( 2 −1) ( 2 + 1) (2 − 1) ⇒ vs = 230 m / s
v2 = 2( 2 + 1) Q coefficient of restitution,
velocity after collision (v)
So, the initial speed of the boy, v2 = 2( 2 + 1). e = 0.5 =
velocity before collision (vs)
34. A ball dropped from a building of height ⇒ 0.5 =
v
⇒ v = 0.5 230 m / s
12 m falls on a slab of 1 m height from 230
the ground and makes a perfect elastic Again from energy conservation law,
collision. Later the ball falls on a wooden 1 2
mv = mgh′ ′
table of height 0.5 m, makes inelastic 2
collision and falls on the ground. If the 1
⇒ (0.5 230)2 = 10 × h′′
coefficient of restitution between the ball 2
and the table is 0.5, then the velocity of the 0.5 × 230 × 0.5
⇒ h′′ = = 2. 875 m
ball while touching the ground is about 20
(Acceleration due to gravity, g = 10 ms − 2) Hence, the total height from point T,
[22 April 2019, Shift-II] h = h′ ′ + 0.5 = 2.875 + 0.5 = 3.375 m
(a) 15.5 ms − 1 (b) 14.5 ms − 1 From energy conservation law,
(c) 9.2 ms − 1 (d) 8.2 ms − 1 1 2
mvT = mgh
2
Sol. (d)
Velocity of ball while touching the ground,
According to the question,
vT = 2gh
R P
= 2 × 10 × 3375
. = 8.215 m / s
O
35. A motor pump a liquid of density ρ through a
pipe of cross-sectional area A. If the liquid
Q 12m moves with a speed v in this pipe, then the
rate of kinetic energy imparted to the liquid
S
1m is proportional to [22 April 2019, Shift-II]
0.5m

(a) v 2 (b) v 3
T (c) v 4 (d) v
48 AP EAMCET Chapterwise Physics

⇒ OS = R  =
Sol. (b) OS 1 R
In ∆OPS, ⇒ cos 60° =
dm OP  2 2
Q Mass flow of rate, = ρAv
dt Velocity of the particle 1, v1 = 2g(OS) = gR
Here, A = cross sectional area of pipe, Velocity of particle 2, v2 = 2g(OQ) = 2 gR
ρ = density of liquid
v1 gR 1
and v = speed of the liquid Hence, = =
dv dm v2 2 gR 2
Q Force F = ma = m = v = v(ρAv)
dt dt 37. A constant power of 7 W is supplied on a toy
QKE P = F ⋅ v = ρAv3 car of mass 15 kg. The distance travelled by
dk dω the car when its velocity increases from
Rate of change KE = = =P
dt dt 3 ms −1 to 5 ms −1 is [23 April 2019, Shift-I]
where, P = power and P = Fv or P ∝ v3 (a) 56 m (b) 7 m (c) 61 m (d) 70 m
Hence, the rate of kinetic energy imparted to the
liquid is proportional to v3 . Sol. (d)
Given, P = 7 W, mass, m = 15 kg,
36. Two particles 1 and 2 are allowed to descend vi = 3 ms−1 and v f = 5 ms−1
on two frictionless chords OP and OQ as From work-energy theorem,
shown in the figure. The ratio of the speeds 1 1
Work done = m(v2f − vi2) = × 15 (25 − 9)
of the particles 1 and 2, respectively when 2 2
they reach the circumference is ⇒ W = 120 J
[22 April 2019, Shift-II] So, the time of work done,
work 120
O t= = = 1714
. s
power 7
60°
P 1 R Hence, the acceleration,
v − vi 5− 3
a= f = = 0116
. m / s2
t 1714
.
2
Distance travelled by car,
v2 − vi2 52 − 32
D= f = = 68.96 ≈ 70 m
Q 2a 2 × 0116
.

1 3 2 Hence, the correct option is (d).


(a) (b) 2 (c) (d)
2 2 3 38. A body A moving with momentum P collides
Sol. (a) one-dimensionally with another stationary
According to the question, body B of same mass. During impact, A gives
impulse J to B. Then which of the following
O
is/are correct? [23 April 2019, Shift-I]
1 60° R (A) The total momentum of A and B is P
P 90° S before and after impact and (P − J )
during the impact.
2 (B) During the impact, B gives impulse of
magnitude J to A.
2 J 
Q (C) The coefficient of restitution is  − 1.
P 
triangle made in a semicircle is right angle 2 J 
(D) The coefficient of restitution is  + 1.
triangle, P 
In ∆OPQ, (a) Only (A) is correct.
OP 1
⇒ cos 60° = ⇒ OP = 2R × ⇒ OP = R (b) (A) and (C) are correct.
OQ 2 (c) (B) and (C) are correct.
(d) Only (C) is correct.
Work, Energy and Power 49

Sol. (c) Work done on block of 3 kg by gravity


According to the question, a collision between W = Mgs = 3 × 10 × 4 = 120 J
two bodies (A and B) is shown in the figure
below,
41. A canon shell fired breaks into two equal
parts at its highest point. If one part retraces
So, the coefficient of restitution the path to the canon with kinetic energy E1
Relative velocity of separation and kinetic energy of the second part is E 2 ,
e=
Relative velocity of approach then [22 April 2018, Shift-II]
J−p J (a) E2 = 15 E1 (b) E2 = E1 (c) E2 = 4 E1 (d) E2 = 9 E1
+
⇒ e= m m = 2J − 1 Q v = mv = p  Sol. (d)
 
p P  m m
−0
m
v1= –u cos θ
v
During the impact, B gives impulse
JB = p + J − p = J
Hence, the statement (B) and (C) are correct.

39. A body of mass 2 kg thrown vertically from Momentum before explosion = ucosθ × 2m
−1
the ground with a velocity of 8 ms reaches Momentum after explosion = − mu cosθ + mv2
a maximum height of 3 m. The work done by Momentum is conserved, so
the air resistance is (acceleration due to 2mu cosθ = − mu cosθ + mv2 ⇒ v2 = 3u cosθ
gravity = 10 ms −2) [22 April 2018, Shift-I]
Kinetic energy of first part
1 1
(a) 4J (b) 60J (c) 64J (d) 8J = E1 = mv12 = m(− u cosθ)2
2 2
Sol. (a) 1
⇒ E1 = mu2 cos2 θ
Initial energy of particle 2
1 1 Now, kinetic energy of second part
= mv2 = × 2 × 82 = 64 J
2 2 1 1
= E 2 = mv22 = 9 × mu2 cos2 θ = 9E1
Final energy of particle at maximum height 2 2
= mgh = 2 × 10 × 3 = 60 J So, E 2 = 9E1
Work done against air friction
42. A uniform chain of mass m and length l is on
= Loss of energy = 64 − 60 = 4 J th
 1
40. The system of two masses 2 kg a smooth horizontal table with   part of
 n
and 3 kg as shown in the
its length is hanging from one end of the
figure is released from rest.
table. The velocity of the chain, when it
The work done on 3 kg block
completely slips off the table is
by the force of gravity during
[22 April 2018, Shift-II]
first 2 seconds of its motion is 2 kg
(a) gl  1 − 2  (b) 2 gl  1 + 2 
(g = 10 ms −2) 1 1
 n   n 
[22 April 2018, Shift-I] 3 kg
(c) 2 gl  1 − 2 
1
(a) 120 J (b) 80 J (d) 2gl
 n 
(c) 30 J (d) 40 J
Sol. (a) Sol. (a) l (1–1/n)
This is the example of Atwood machine, so
acceleration is the system
 M − m 1 l/n
a=  g ⇒ a = × 10 = 2 m / s
2

 M + m 5
1 2 1
Now, s = ut + at = 0 + × 2 (2)2 ⇒ s = 4 m
2 2
50 AP EAMCET Chapterwise Physics

Taking surface of table as zero level of potential kinetic energy of the bullet, when it is
energy, between the plates A and B is _____
1
Potential energy of chain with th part hanging (Neglect any loss of material of the plates
n
− Mgl during the collision) [23 April 2018, Shift-I]
=
2n2 (a) 25 (b) 56.25 (c) 43.75 (d) 75
Potential energy of chain or it leaves table Sol. (c)
− Mgl
= Let v1 be the initial velocity of bullet and v2 be
2
the velocity with which each plate moves.
Kinetic energy = Loss of potential energy
So, applying law of conservation of momentum,
1 Mgl  1  1
⇒ Mv2 = 1 − 2  ⇒ v = gl 1 − 2  mv1 = M1 v2 + (M 2 + m)v2
2 2  n   n  0. 01v1 = 0. 5v2 + (1. 49 + 0. 01)v2
2
43. A particle is released from a height H. At a v1 = v2 = 200 v2 … (i)
certain height, its kinetic energy is half of its 0.01
potential energy with reference to the Let v3 be the velocity of bullet, when it comes
surface of the earth. Height and speed of the out of plate M.
particle at that instant are respectively So, momentum of bullet between plate M1 and
M 2 = Sum of momentum of plate M 2 and bullet.
[23 April 2018, Shift-I]
mv3 = (M 2 + m)v2
H 2 gH H gH
(a) , (b) , 2 0.01v3 = (1.49 + 0.01)v2 = 1.5v2
3 3 3 3
2H 2 H 2 gH or v3 = 150v2 …(ii)
(c) , 2 gH (d) , (1 / 2) mv12 − (1 / 2) mv33
3 3 3 % loss in KE = × 100
(1 / 2) mv12
Sol. (d)
v12 − v32
Total mechanical energy of particle, i.e. = × 100
v12
PE + KE = mgH … (i)
1 KE 1   v  
2
 150  
2
Given, KE = PE i.e. = or PE = 2KE = 1 −  3   × 100= 1 −    × 100
2 PE 2   v1    200 
  
Substituting in this Eq. (i), we get
= 1 −  × 100 =
9 7 7
PE + KE = mgH ⇒ 2KE + KE = mgH × 100 = × 25 = 43. 75%
 16  16 4
mgH
3KE = mgH ⇒ KE =
3 45. A bead of mass 100 g is attached to one end of
2 a spring of natural length L and spring
Similarly, PE = mgH
3 ( 3 + 1)mg
So, height from ground at that instant, constant k = , where m is the mass
L
2H
h= of bead. The other end of the spring is fixed at
3 point A on a smooth vertical ring of radius R
So, speed of particle,
v = 2gh = 2gH / 3
as shown in the figure. The normal reaction at
B just after it is released to move is (take,
44. A bullet of mass 10 g g = 9 .8 ms −2) [23 April 2018, Shift-II]
pierces through a plate
B
A of mass 500 g and
Bullet
then gets embedded
into a second plate B of Plate A Plate B 30°
mass 1.49 kg as shown A
in the figure. Initially, the two plates A and B R
are at rest and move with same velocity after
collision. The percentage loss in the initial
(a) 1.73 N (b) 2.23 N (c) 2.44 N (d) 2.55 N
Work, Energy and Power 51

mgR 2
sin  
Sol. (d) mgl l
(a) (b)
Normal reaction = Resultant of spring force kx 2 l  R
mgR 2 mgl 2
sin   sin  
and weight mg. R l
(c) (d)
= (k2 x 2 + m2 g2) = 2. 55 N l  l R  R

Sol. (b) dl
46. A particle is released freely from a height H.
At a certain height, its kinetic energy is two
times of its potential energy. Then, the dθ h
height and the speed of the particle at that θ
instant are respectively [23 April 2018, Shift-II]
(g = acceleration due to gravity) h
H 2 gH H gH We have, = sinθ or h = Rsinθ
(a) , (b) , 2 R
3 3 3 3 Also, dl = Rdθ
2 H 2 gH H m
(c) , (d) , 2 gH Mass of dl length of chain = dm = . dl
3 3 3 l
Sol. (b) PE of dm mass
x mgh mgh mgR2
= dU = dmgh = . dl = Rdθ = sinθ dθ
H l l l
H-x So, PE of complete chain is
mgR2
sin 
π / 2− θ l
U=∫ dU =
π /2 l  R
If particle falls by a distance x, then
1 1
K E = mv2 = m(2gx) = mgx 48. A stone of mass 2 kg tied to a light
2 2 5
PE = mg(H − x) inextensible string of length m is whirling
3
2H
As, KE = 2(PE) ⇒ mgx = 2mg (H − x) ⇒ x = in a circular path in a vertical plane. If the
3
ratio of the maximum tension to the
H
So, height of particle is H − x = . minimum tension in the string is 4, then the
3 speed of the stone at the highest point of the
Speed of particle at
H
distance = 2g
2H 
 circle is (g = 10 ms−2) [24 April 2018, Shift-I]
3  3
(a) 20 ms −1 (b) 10 3 ms −1
gH (c) 50 ms −1 (d) 10 ms −1
=2
3
Sol. (c)
47. A uniform chain of length l and mass m lies Tension is maximum at bottom of vertical circle
on the surface of a smooth hemisphere of and it is
radius R(R > l) with one end tied to the top of T1 =
mv2
+ mg
the hemisphere as shown in the figure. r
Gravitational potential energy of the chain
with respect to the base of the hemisphere is
[23 April 2018, Shift-II]
T
Chain
θ
Tθ = mg cosθ + mv2
r
R u

Tension is minimum at top and it is


mu2
T2 = − 5mg
r
52 AP EAMCET Chapterwise Physics

T1 mu2 Total time before it ceaser to rebound


Given, ratio = 4 ⇒ 21mg = 3
T2 r T = t0 + 2t1 + 2t2 + ………
⇒ u2 = 7rg 2v 2v
T = t0 + 1 + 2 + ………
Now, velocity at top, g g
v= u2 − 4rg = 7rg − 4rg = 3rg = 2ev0 2e 2 v0
50 m/s T = t0 + + + ………
g g
49. A ball falls freely from a height of 180 m on {as v2 = ev1 = e 2 v0 }
to a hard horizontal floor and repeatedly 2h
bounces. If the coefficient of restitution is T= [1 + 2e(1 + e + e 2 + e 3 + …)]
g
0.5, the average speed and average velocity
of the ball before it ceases to rebound are  2h 
as v0 = 
respectively  g
(acceleration due to gravity = 10 ms −2) It forms a GP,
[24 April 2018, Shift-I] 2h   1  2h  1 + e 
T= 1 + 2e   =  
 1 − e  
50
(a) 10 ms −1 , 10 ms −1 (b) 50 ms −1 , ms −1 g g 1 − e 
3
50 20 50 Given, h = 180 m and e = 0.5
(c) ms −1 , 10 ms −1 (d) ms −1 , ms −1
3 3 3 2 × 180  1 + 0.5
T=   = 6 × 3 = 18 s
Sol. (c) 10  1 − 0.5
Calculation of distance in entire motion initial
height h0 = h
After first collision = h1
v0 v 2gh1 h
h As e= = = 1
v1 u 2gh0 h
v2
v3 h1 = e 2 h, similarly h2 = e 2 (e 2 h)
t0 t1 t2 t3
h2 = e 4 h
Total distance,
H = h0 + 2h1 + 2h2 + ……
When ball dropped from height h, then time
taken to reach the ground H = h + 2e 2 h + 2e 4 h + ……
2h H = h[1 + 2e 2 (1 + e 2 + e 4 + ……)]
t0 = and speed, v0 = 2gh
g   1 
H = h1 + 2e 2  
After first collision, its speed will become   1 − e 2  
v1 = ev0 = e 2gh 1 + e 2 
H = h 
where, e = coefficient of restitution. 1 − e2 
Now, the ball will go up and will take time t1 , 1 + 1/ 4  1
when it stops H = 180  , e = 0.5 =
 1 − 1/ 4  2
v = u + at
5
o = v1 − gt1 H = 180 ×
v 3
t1 = 1 H = 300 m
g
Total distance
It will come down and take same time t1 before Average speed, v =
second collision. Time
300 50
So, time taken between first and second v= = m/s
collision is 2t1 . 18 3
Total displacement
Similarly, time taken between second and third Average velocity, v =
2v Time
collision will be 2t3 = 2 180
g v= = 10 m/s
18
5
Rotational Motion
1. Four point masses, each of mass M are Sol. (a)
placed at the corners of a square of side L. Initial angular velocity of flywheel,
The moment of inertia of the system about ω0 = 0
one of its diagonals is [17 Sep. 2020, Shift-I] Angular acceleration,
(a) 2 ML2 (b) ML2 (c) 4ML2 (d) 6ML2 α = 2 rad/s 2
Sol. (b) Angular displacement in t = 10 s is given as
1
The given masses are shown in the figure. θ = ω0 t + α t 2
2
M M 1
D C = 0 × 10 + × 2 × 102 = 100 rad
2
θ 100 100
Number of revolution = = =
O 2π 2π 2 × 22
7
700
= = 15.9 − 16
~
44
A B
M M 3. Three identical uniform solid spheres each of
L mass m and radius r are joined as shown in
the figure. with centres lying in the same
Mass of each point mass = M
plane. The moment of inertia of the system
AB = BC = CD = DA = L
about an axis lying in that plane and passing
Length of diagonal
through the centre of sphere C is
BD = DA2 + AB2 = L2 + L2 = 2L2 = L 2
[17 Sep. 2020, Shift-I]
BD L 2 L
∴ OD = OB = = = P
2 2 2
∴ Moment of inertia of given system about the A B
diagonal AC
I AC = I A + I B + I C + I D
2 2 C
= 0 + M   + 0 + M  
L L
 2  2
Q
ML2 ML2
= + = ML2
2 2 16 2 12 2 3
(a) mr (b) mr (c) 4 mr 2 (d) mr 2
2. A flywheel starts from rest and rotates at a 5 5 5
constant acceleration of 2 rad s −2. The number Sol. (a)
of revolutions that it makes in first 10 s is Moment of inertia of each solid sphere,
2
[17 Sep. 2020, Shift-I] I = mr 2
5
(a) 16 (b) 24 (c) 32 (d) 8
54 AP EAMCET Chapterwise Physics

P ∴Angular velocity,
ω = 2πf = 2 π × 10 = 20 π rad/s
A B ∴Centripetal force,
F = mrω2 = 0.1 × 0.1 × (20 π)2
= 0.01 × 400 π 2 = 4 π 2 N
C
6. The speed of a uniform solid sphere after
rolling down from rest without slipping
Q along a fixed inclined plane of vertical height
h is [17 Sep. 2020, Shift-II]
where, m = mass and r = radius.
10 gh 6 gh 4 gh
The moment of inertia of the system about axis (a) (b) gh (c) (d)
PQ as shown in the figure. 7 5 3
I PQ = I A + I B + I C Sol. (a)
 2   2 
=  mr  + mr 2 +  mr 2  + mr 2 +  mr 2 
2 2 If v is the speed of solid sphere when it reaches
  5     5    5  at point O, then by the law of conservation of
7 7 2 16 energy,
= mr 2 + mr 2 + mr 2 = mr 2
5 5 5 5

4. A disc of moment of inertia I1 is rotating


with an angular velocity ω1 . Another disc of
moment of inertia I 2 which is not rotating, is h
gently put on the first disc. The angular speed
of the system will be [17 Sep. 2020, Shift-II]
O
 I + I2   I 
(a)  1  ω1 (b)  1  ω1 mgh =
1 2 1
Iω + mv2
 I1   I1 + I2  2 2
 I − I2   I  2
= ⋅ mR2   + mv2
(c)  1  ω1 (d)  1  ω1 1 2 v 1
 I1   I1 − I2  2 5  R 2
1 2 1
Sol. (b) ⇒ mgh = mv + mv 2

5 2
Since, no external torque acts on the system,
v2 v2 7v2
hence by the law of conservation of angular ⇒ gh = + ⇒ gh =
momentum, 5 2 10
I1 ω1 = (I1 + I 2) ω2 10 gh 10 gh
⇒ v =
2
⇒ v=
[ω2 = final angular velocity of system] 7 7
 I1 
⇒ ω2 =   ω1 7. A ballet dancer suddenly folds her
 I1 + I 2  outstretched arms. Her angular velocity
5. Find the centripetal force acting on a coin [18 Sep. 2020, Shift-I]
weighing 0.1 kg placed 0.1 m from the centre (a) increases
on a gramophone disc rotating at 600 rpm. (b) decreases
[17 Sep. 2020, Shift-II] (c) remains the same
π2 4 (d) may increase or decrease
(a) 4 π N
2
(b) 40 π N (c)
2
N (d) N
4 π2 Sol. (a)
Sol. (a) According to law of conservation of angular
Weight of coin, m = 0.1 kg momentum,
r = 0.1 m Iω = constant …(i)
Angular frequency, where, I = moment of inertia
600 and ω = angular velocity.
f = 600 rpm = rps = 10 rps = 10 Hz
60
Rotational Motion 55

When a ballet dancer suddenly folds her 2 kg C 2 kg


outstretched arms, then her moment of inertia 1/2 m 1/2 m
decreases, hence from Eq. (i), we observe that
her angular velocity will increases. Moment of inertia about centre of mass,
2 2
I = 2  + 2  = + = 1 kg-m 2
8. A mass m is in rest on an inclined plane of 1 1 2 2
mass M which is further resting on a smooth  2  2 4 4
horizontal plane. Now, if the mass m starts ∴ Rotational kinetic energy
moving under gravity, the position of centre 1 1 025
.
Krot = Iω2 = × 1 × (0.5 )2 = = 0125
. J
of mass of system will [18 Sep. 2020, Shift-I] 2 2 2
(a) remain unchanged 10. If the raw egg and boiled egg are subjected
(b) change along the horizontal direction to spin on a table by applying the equal
(c) move up in vertical direction torque, then the egg that spins with the
(d) move down in the vertical direction and changes greater speed is [18 Sep. 2020, Shift-II]
along the horizontal
(a) raw egg
Sol. (c) (b) boiled egg
The given situation is shown in the following (c) Both eggs have equal speed
figure. (d) Both eggs never spin
Sol. (b)
m
When same torque is applied on boiled egg and
on raw egg, the boiled egg would have higher
spin than a raw egg. It is due to the fact that the
M
raw egg resist to get its spin on account of its
internal fluid. Thus, if the raw egg and boiled
egg are subjected to spin on a table by applying
the equal torque, then boiled egg spins with
greater speed than raw egg.
Horizontal plane 11. A solid cylinder of mass M and radius R rolls
on a flat surface. Its moment of inertia about
Clearly when mass m starts moving under the line of contact is [18 Sep. 2020, Shift-II]
gravity, hence net force on the system in
(a)   MR 2
3
horizontal direction is zero. Therefore, centre of (b) MR 2
2
mass of the system will not move in horizontal
(c)   MR 2
direction. Now, for vertical direction there is a 2
(d) 2 MR 2
net force that is the mass of the wedge and the  3
block and hence the centre of mass changes in
this direction. Sol. (a)
Moment of inertia of solid cylinder of radius R
9. A light meter rod has two-point masses each and mass M about its axis is given as
of 2 kg fixed at its ends. If the system rotates 1
I CM = MR2 …(i)
about its centre of mass with an angular 2
speed of 0.5 rad s − 1 , its rotational KE is Moment of inertia about contact line AB is given
[18 Sep. 2020, Shift-II]
(a) 0.125 erg (b) 1.25 erg ICM
(c) 1.25 J (d) 0.125 J R

Sol. (d) A B
Total mass of light meter rod including two
point masses, m = (2 + 2) kg = 4 kg According to parallel axes theorem,
Angular speed, I = 0.5 rad s −1 1 3
i.e. I AB = I CM + MR2 = MR2 + MR2 = MR2
Length of rod = 1 m 2 2
56 AP EAMCET Chapterwise Physics

12. The moment of inertia of a rectangular plate of Sol. (c)


mass M, length L and breadth B, about an axis The given situation is shown in the following
passing through its centre and perpendicular figure.
to its plane is [21 Sep. 2020, Shift-I]
M(L + B) M(L2 )
(a) (b)
12 12
M(L2 + B2 ) M(B2 ) h
(c) (d)
12 12
Sol. (c)
The moment of inertia of rectangular plate of L
mass M, length L and breadth B is given as When cylinder reaches at bottom, then its whole
M(L2 + B2) potential energy is converted into its rotational
I= kinetic energy and linear kinetic energy of its
12
centre of mass.
1 1
Hence, Mgh = Iω2 + MvCOM 2

2 2
2
1 MR2  vCOM  1
= × ×  + MvCOM
2

2 2  R  2
B
 MR2 vCOM 
Q I = 2 and ω = R 
 
2 2
L MvCOM MvCOM
= +
4 2
13. A body of mass 5 kg acquires an acceleration 3 2 4 gh
of 10 rads − 2 due to an applied torque of 2 Nm. ⇒ Mgh = MvCOM ⇒ vCOM 2
=
4 3
Its radius of gyration is [21 Sep. 2020, Shift-I]
4 gh
(a) 2.5 m (b) 2.5 m ⇒ vCOM =
3
(c) 0.2 m (d) 0.2 m
Sol. (d) 15. A uniform circular disc has radius r. A square
Mass of body, m = 5kg portion of diagonal r is cut from it. The
Angular acceleration, α = 10 rad s −2
centre of mass of the remaining disc from
the centre of disc is [21 Sep. 2020, Shift-II]
Torque, τ = 2N-m
r r r 2r
We know that, (a) (b) (c) (d)
2 − 4π 3 − 3π 2 − 5π 1− 2 π
Torque = Moment of inertia × Angular
acceleration Sol. (a)
⇒ τ = Iα Suppose, the mass of circular disc is M.
τ 2 A square portion of diagonal r is cut from it.
⇒ I= = = 0.2 kg-m 2 Assuming centre of mass of circle to be origin.
α 10
Centre of mass of square portion will be at
If k be the radius of gyration, r
distance from it.
then I = mk2 2
I 0.2 r
⇒ k= = = 0.04 = 0.2 m ∴ Side of square =
m 5 2
2
14. A solid cylinder of mass M and radius R rolls  r 
Area of square =   =
r2
down an inclined plane of length L and  2 2
height h, without slipping. Find the speed of r2 / 2 M
Mass of square,m = M × =
its centre of mass when the cylinder reaches πr 2 2π
its bottom. [21 Sep. 2020, Shift-I] Centre of a mass of remaining disc from the
3 gh 4gh centre of disc,
(a) 2gh (b) (c) (d) 4gh
4 3
Rotational Motion 57
M r Since, all spheres are identical with mass 1 kg
− ×
m× r / 2 2π 2 = −r and radius 10 cm each, hence according to
x = M × 0− =
M−m M−
M 4π − 2 figure, it is clear that all spheres are
2π symmetrically arranged, hence centre of mass
r will be at intersection point of diagonals of
⇒ x= square ABCD.
2 − 4π
∴ AC = 202 + 202 = 20 2 cm
16. How many revolutions does a wheel with
20 2
angular speed 88 rad s −1 make in one second? ∴ AO = BO = CO = DO = = 10 2 cm
2
[21 Sep. 2020, Shift-II]
(a) 7 (b) 14 (c) 28 (d) 44 18. The centre of a wheel rolling on a plane
Sol. (b) surface moves with a speed v0. A particle on
Angular speed of wheel, ω = 88 rad s −1 the rim of the wheel at the same level as the
∴Number of revolution per second is equal to its
centre will be moving at a speed
frequency f. [21 Sep. 2020, Shift-II]
i.e. ω = 2πf (a) 0 (b) v 0
ω 88 88 × 7 (c) 2 v 0 (d) 2 v 0
⇒ f = = = = 14 rev/s
2π 2 × 22/ 7 2 × 22 Sol. (c)
17. Four identical spheres each of radius 10 cm According to question, speed of the centre of
and equal mass 1 kg each are placed on wheel is v0 .
horizontal surface touching each other, so ω ω
that their centre are located at the vertices of
a square of side 20 cm. What is the distance R
of their centre of mass from the centre of O vo O P
either sphere? [21 Sep. 2020, Shift-II] R R r
20 cm v
M
(i) (ii)
D C Particle is situated on the rim of wheel as
shown in Fig. (ii).
20 cm

20 cm

∴ v = rω = (OP)2 + (OM)2 ⋅ ω = R2 + R2 ⋅ ω
v = 2Rω …(i)
A B But v0 = Rω …(ii)
From Eqs. (i) and (ii), we get
20 cm v = 2 v0
(a) 20 2 (b) 30 2 (c) 10 2 (d) 40 2 19. Two cars A and B are going around
Sol. (c) concentric circular paths of radii R A and RB .
The given situation is shown in the figure, If the two cars complete the circular paths in
20 cm the same time, then the ratio of angular
speed of A and B is [22 Sep. 2020, Shift-I]
D C (a) 1 : 1 (b) R A : R B (c) R B : R A (d) 1 : 2
Sol. (a)
20 cm

20 cm

Angular speed of a body moving on circular


O path is given as

ω=
A B T
where, T is the time-period.
20 cm ω A TB
AB = BC = CD = DA = 20 cm ∴ =
ω B TA
58 AP EAMCET Chapterwise Physics

Given, TA = TB We know that, torque, τ = I α …(i)


ω A TA ωA 1 If T is the tension in the string, then
∴ = ⇒ =
ω B TA ωB 1 τ = TR …(ii)
∴ ω A : ωB = 1 : 1 From Eqs. (i) and (ii), we get
I α = TR
20. When the torque acting upon a system is Under no slipping condition,
zero, the parameter that remains constant is a
α=
R
[22 Sep. 2020, Shift-II]
a I⋅a
(a) force (b) linear momentum Hence, I ⋅ = TR ⇒ T = 2
R R
(c) angular momentum (d) linear impulse
MR2
For cylinder, I =
Sol. (c) 2
Torque acting upon a rotating body is given as MR2 a ma
∴ T= ⋅ ⇒ T= …(iii)
dL 2 R2 2
τ=
dt For the motion of the block,
where, L = angular momentum. Mg − T = Ma
Ma
When τ = 0, then ⇒ Mg − = Ma [from Eq. (iii)]
dL 2
= 0 ⇒ L = constant 3Ma 2g
dt ⇒ Mg = ⇒ a=
2 3
Hence, when the torque acting upon a system is
zero, then its angular momentum remains constant. 22. A 20 kg flywheel in the form of a uniform
21. A mass M is supported by a massless string circular disc, 1 m in diameter is making
wound around a uniform cylinder of mass M 120 rpm. What is its angular momentum?
and radius R. On releasing the mass from [22 Sep. 2020, Shift-II]
rest, it will fall with acceleration (a) 3.14 kg m 2 s −1 (b) 31.4 kg m 2 s −1
[22 Sep. 2020, Shift-II] (c) 314 kg m 2 s −1 (d) 0.314 kg m 2 s −1
Sol. (b)
Mass of fly wheel,
m = 20 kg
R
M Diameter, D = 1 m
1
∴ Radius, R = = 0.5 m
2
120
M Angular velocity, ω = 2π × = 4π rad s −1
60
g 2g 3g ∴ Moment of inertia of uniform circular disc,
(a) g (b) (c) (d)
2 3 2 mR2 20 × (0.5)2
I= =
Sol. (c) 2 2
The given situation is shown in the figure, = 10 × 0.25 = 2.5 kg-m 2
∴ Angular momentum,
L = Iω = 2.5 × 4π
= 2.5 × 4 × 3.14 = 31.4 kg m 2 s −1
M R
O
23. If the radius of a sphere is doubled by
keeping its mass constant, compare the
T
a moment of inertia of the old sphere with
that of the new sphere, about any diameter.
M
[23 Sep. 2020, Shift-I]
(a) I1 : I2 = 1 : 4 (b) I1 : I2 = 1 : 2
Mg
(c) I1 : I2 = 4 : 1 (d) I1 : I2 = 2 : 1
Rotational Motion 59

Sol. (a) 1 2  k2 
= mvCM  2 
Moment of inertia of a solid sphere about its 2 R 
2
diameter is given by I = MR2 ∴Total kinetic energy, KE = KE T + KE R
5
1 2 1 2 k2
If mass is constant, then I ∝ R2 = mvCM + mvCM
2 2 2 R2
I R 
For two sphere of same masses, 1 =  1  1 2  k 
2
I 2  R2  KE = mvCM 1 + 2 
2  R 
Here, R2 = 2R1
2 Fraction of total energy associated with its
I1  R1  I1 1
=  or = rotation, i.e.
I 2  2R1  I2 4 1 2  k2 
mvCM  2 
KE R 2 R  k2
24. Kinetic energy of rotation of a flywheel of = = 2
radius 2 m, mass 8 kg and angular speed KE 1 2  k  k + R2
2
mvCM 1 + 2 
4 rad s −1 about an axis perpendicular to its 2  R 
plane and passing through its centre is
26. A solid sphere of 100 kg and radius 10 m
[23 Sep. 2020, Shift-I] moving in a space becomes a circular disc of
(a) 128 J (b) 196 J (c) 256 J (d) 392 J radius 20 m in one hour. Then the rate of
Sol. (a) change of moment of inertia in the process is
Given that, radius, R = 2 m [20 April 2019, Shift-I]
40 10
Mass, m = 8 kg (a) kg m 2 s −1 (b) kg m 2 s −1
Angular speed, ω = 4 rad/s 9 9
50 25
Moment of inertia of flywheel about an axis (c) kg m 2 s −1 (d) kg m 2 s −1
9 9
perpendicular to its plane and passing through
mR2 8 × (2)2 Sol. (a)
its centre, I = = = 16 kg-m 2
2 2 Given, mass of solid sphere, M s = 100 kg
Therefore, kinetic energy of rotating fly wheel radius of solid sphere, Rs = 10 m
1 1 radius of circular disc, Rc = 20 m
KE = Iω2 = × 16 × (4)2 =128 J
2 2 and time = 1 hour = 60 minute = 60 × 60 sec
25. A uniform solid sphere of radius R and radius Moment of inertia of the solid sphere,
2 2
of gyration k about an axis passing through I s = M s Rs2 = × 100 × (10)2 = 4000 kg-m 2
the centre of mass is rolling without slipping. 5 5
Then, the fraction of total energy associated Similarly,
with its rotation will be [23 Sep. 2020, Shift-I] 1
moment of inertia of the disc, I c = M c R2
k2 + R 2 k2 k2 R2 2
(a) (b) 2 (c) (d) 2 1
k 2
R k + R
2 2
k + R2 = × 100 × (20) = 20,000 kg- m 2
2

2
Sol. (c) I − Is
Rate of change of moment of inertia = c
For a body rolling without slipping, t
vCM = Rω …(i) 20000 − 4000 16000 160 40
= = = = kg-m 2 s −1
Translational kinetic energy, KE T =
1 2
mvCM 60 × 60 60 × 60 36 9
2
Rotational kinetic energy, KE R =
1 2
Iω 27. A semicircular plate of mass m has radius r
2 and centre c. The centre of mass of the plate
1 is at a distance x from its centre c. Its
= (mK 2)ω2
2 moment of inertia about an axis passing
(Q I = mK 2 , where k is radius of gyration.) through its centre of mass and perpendicular
2

= mk2  CM 
1 v
[from Eq. (i)]
to its plane is [20 April 2019, Shift-I]
2  R  mr 2 mr 2 mr 2 mr 2
(a) (b) (c) + mx2 (d) − mx2
2 4 2 2
60 AP EAMCET Chapterwise Physics

Sol. (d) or u= 2as (Qv = 0)


Given, Q Acceleration of a block on a horizontal,
mass of a semicircular plate = m a =µg
radius of semicircular plate = r ⇒ a = 0.2 × 10 = 2 m/s 2 (Given, µ = 0.2)
According to the question we can drawn the After putting the value of a in Eq. (i), we get
following diagram, ∴ u = 2 × 2 × 1 = 2 m/s (Qs = 1 m, given)
From above diagram,
the frictional force applied on the disc inclined
plane r is
x F = µ N = µ mgcos 30°
and the net acceleration force down the inclined
r plane,
C
mg sin 30° − f = mg sin 30°− µ mg cos 30° = ma1
Now, from the parallel axis theorem,
or a1 = g(sin 30° − µ cos 30°) = 51
( − 3µ) …(i)
I c = I cm + mx 2
I cm = I c − mx 2 Hence, from the third equation of motion,
mr 2 v12 = u12 + 2a1 s
Here, Ic =
2 When s is distance travelled by the disc,
Hence, moment of inertia of semicircular plate v1 = u = 2 m/s, u1 = 0
about an axis passing through its centre of mass 2
∴ (2) = 2a1 s or s =
2

and perpendicular to it’s plane is a1


mr 2
I cm = − mx 2 Putting the value of a1 from Eqs. (i), we get
2 2 2
⇒s = = = 0.612 m
28. A disc of mass 100 g slides down from rest ( − 3µ) 5(1 − 3 × 0.2)
51
on an inclined plane of 30° and come to rest Hence, the work done by the frictional force,
after travelling a distance of 1m along the W = work done on inclined plane + work done
horizontal plane. If the coefficient of friction on horizontal plane.
is 0.2 for both inclined and horizontal 1
= (µ mg cos 30°) s + mu2
planes, then the work done by the frictional 2
force over the whole journey, approximately, Putting the given values, we get
is (Acceleration due to gravity, g = 10 ms −1 ) = 0.2 ×
100 1 100
× 10 × cos 30° × 0.612 + × × (2)2
[20 April 2019, Shift-II] 1000 2 1000
= 0.306J
(a) 0.106 J (b) 0.05 J (c) 0.306 J (d) 0.2 J
Sol. (c) 29. Two identical discs are moving with the same
kinetic energy. One rolls and the other slides.
According to the question, acceleration of a
block sliding down an inclined plane is shown The ratio of their speeds is [20 April 2019, Shift-II]
in the following figure. (a) 1 : 2 (b) 1 : 1 (c) 2 : 3 (d) 2 : 3
From third equation of the motion, velocity of Sol. (d)
disc when it leaves the inclined plane,
According to the question,
µN ∴Kinetic energy for the rolling disc,
F=
N

1 1
or KE r = mv12 + Iω2
ma
1
° 2 2
s 30 mR2
° co h (∴Moment of inertia, I = )
30 g 2
in 30° m
gs 2
m 1 1 mR2  v1 
mg or = mv12 +  
30° 2 2 2  R
1m 3
or KE r = mv12 …(i)
4
v2 = u2 − 2as
Rotational Motion 61

Now, KE for the sliding disc, 31. A flywheel of mass 1 kg and radius vector
1
∴ KE s = mv22 …(ii) (2 i$ + $j + 2 k$ ) m is at rest. When a force
2
Given, KE of rolling disc = KE of sliding disc
(3 $i + 2 $j − 4 k$ ) N acts on it tangentially, it can
3 1 v2 2 rotate freely. Then, its angular velocity after
or, mv12 = mv22 or 12 = 4.5 s is [21 April 2019, Shift-I]
4 2 v2 3
2 3
v1 2 (a) 261 rad s − 1 (b) 261 rad s − 1
or = or v1 : v2 = 2: 3 9 2
v2 3 5
(c) 261 rad s − 1 (d) 261 rad s − 1
9
30. A tangential force F acts at the top of a thin
spherical shell of mass m and radius R. The Sol. (c)
acceleration of the shell if it rolls without Given, mass of flywheel, M = 1 kg,
slipping is ( f = rolling friction) radius vectors R = (2$i + $j + 2k$ ) m,
[20 April 2019, Shift-II] $
F = (3$i + 2$j − 4k) N and time, t = 4.5 s
F
R Magnitude of radius
(R) = (2)2 + ()
1 2 + (2)2 = 9=3m
f Similarly, F = 29 N
Torque on the flywheel,
5F 6F 3F F MR2
(a) (b) (c) (d) τ = Iα = F ⋅ R = α
6m 5m 2m 6m 2
2F 2 29 2
Sol. (b) α= = = 29
F MR 1 × 3 3
Now, the angular velocity,
2R 2
ω = ω0 + αt ⇒ω = 0 + 29 × 4.5 (Qω0 = 0)
3
f
⇒ ω = 261 rad s− 1
Hence, the correct option is (c).
Torque due to the force F on a thin spherical
shell, 32. Three identical spheres each of diameter
τ=r×F 2 3 m are kept on a horizontal surface such
= 2R F sin 90° = 2RF [Qsin 90° = 1] that each sphere touches the other two
QAngular acceleration,
spheres. If one of the sphere is removed,
τ then the Shift-in the position of the centre of
α= …(i) mass of the system is [21 April 2019, Shift-I]
I
3
Where, I is moment of inertia of a thin spherical (a) 12 m (b) 1 m (c) 2 m (d) m
2
shell.
From parallel axes’s theorem, moment of inertia Sol. (b)
of spherical shell, The centre of mass of the spheres is given by
I = I cm + Mr 2 m x + m2 x 2 + m3 x 3
x CM = 1 1
or
2
I = MR2 + MR2 (Qr = R) m1 + m2 + m3
3
From Eqs. (i), we get
=  
2RF 6 F
α=
2
MR + MR
2 2  5 RM C (√3, 3)
3
Hence, the tangential acceleration, a T = Rα
a T =  
6 F 6 F
or × R=
 5 RM 5 M A B
(0, 0) (2√3, 0)
62 AP EAMCET Chapterwise Physics

As the spheres are identical, We know that the given body is performing
m 3 3 angular SHM,
⇒ x CM = (0 + 2 3 + 3) = = 3 1 1
3m 3 KE = k(A2 − x 2) ⇒ KE = k(θ20 − θ2)
m 2 2
Similarly, yCM = (y1 + y2 + y3)
3m Hence, reduced kinectic energy ∆E is,
(0 + 0 + 3) or ∆ E ∝ θ2
= =1
3
35. A uniform rod of length l and density ρ is
So, the centre of mass, CCM = ( 3, 1)
revolving about a vertical axis passing through
If one sphere is removed (say C), then its one end. If ω is the angular velocity of the
0+ 2 3 rod then the centrifugal force per unit area
′ =
x CM = 3
2 of the rod is [21 April 2019, Shift-II]
′ =0
yCM ρω2 l 2 ρω2 l 2 ρω2 l 2 ρω2 l 2
(a) (b) (c) (d)
4 12 2 8
Sol. (c)
According to the question, a uniform rod of
(0, 0) B length l and density ρ as shown in the figure
(2√3, 0) below,
ω

So, CCM = ( 3, 0)
Hence, the centre of mass shifted by 1 m in dx
− y direction. The correct option is (b).

33. A wheel having moment of inertia 2 kg-m 2


about its axis, rotates at 50 rpm about the l
same axis. The torque required to stop the .

wheel in one minute is [21 April 2019, Shift-II] Q Centrifugal force per unit area of rod for
π π π 3π small length dx is
(a) Nm (b) Nm (c) − Nm (d) Nm
10 18 12 8 F l
mω2 dx
Sol. (b) ∫ dF = ∫ A
0 0
Given, moment of inertia of a wheel, Let A is the area of rod and x is variable length
I = 2Kg-m 2 and angular velocity of wheel, m
50 5 of the rod then density of rod , ρ = or
ω = 50 rpm = × 2π = π rad s −1 A⋅ x
60 3 m = ρAx
The torque required to stop the wheel, l
ω2 2 x  ρω2 l 2
l 2
ω
⇒ τ = Iα = I   ⇒ τ = 2 × 
5/ 3 π

⇒ F =ρ
A ∫ A ⋅ x dx = ρω  2  0 = 2
 t  60  0

[Qt = 1 min = 60] 36. A solid sphere rolls down without slipping
π
= N-m on a smooth inclined plane of inclination
18
sin −1 (0 .42). If the acceleration due to gravity
34. The angular retardation of a rotating is 10 ms −2, the acceleration of the rolling
flywheel is proportional to the angle through sphere is [22 April 2019, Shift-I]
which it rotates. If its kinetic energy gets (a) 1 ms −2 (b) 2 ms −2 (c) 3 ms −2 (d) 4 ms −2
reduced by ∆E while it rotates through at an
Sol. (c)
angle θ, then [21 April 2019, Shift-II]
Given, inclination of inclined plane,
(a) ∆E ∝ θ2 (b) ∆E ∝ θ
3 θ = sin−1 (0.42) ⇒ sinθ = 0.42
(c) ∆E ∝ θ (d) ∆E ∝ θ 2
Acceleration due to gravity,
Sol. (a) g = 10 m/s2
Rotational Motion 63

Acceleration of rolling solid sphere on the I AB = MR2 …(i)


inclined plane without slipping is given by ∴Moment of inertia axis passing through the
gsinθ centre of circular loop about its diameter ,
a=
I 1
1+ I CM = MR2 …(ii)
MR2 2
where, I = moment of inertia Now, the total moment of inertia for circular
M = mass of sphere loop of wire. If R is the distance between the
R = radius of sphere axis, I CM and I AB are the respective moments of
2 inertia about these axies,
But I = MR2 Then, I = I CM + I AB
5
gsinθ 5 From Eqs. (i) and (ii), we get
∴ a= = gsinθ 1 3
2
MR 2 7 I = MR2 + MR2 ⇒ I = MR2 …(iii)
2 2
1+ 5 2
MR We know that,
5
= × 10 × 0.42 = 3 m / s2 Mass = length × density
7 Mass of the wire, M =ρl …(iv)
Thus, the length of the wire, l = circumference
37. A thin wire of length l having a linear of the circular loop of wire
density ρ is bent into a circular loop with C as l
its centre as shown in the figure. The ∴ l = 2πR or R = …(v)

moment of inertia of the loop about the line
Now, from Eq. (iv) and (v) putting the value of
AB is [22 April 2019, Shift-I]
M and R in Eq. (iii), we get
3ρl 3
2

I = (ρl) ×   ⇒ I =
3 l
2  2π  8π2
C So, the moment of inertia of a circular loop of
3ρl 3
wire is, I = .
8π2
A B
5 ρl 3 1 ρl 3 1 ρl 3 3 ρl 3 38. A uniform rod of mass m and length l is
(a) (b) (c) (d) pivoted smoothly at point O as shown in
16 π 3 16 π 3 8 π3 8 π2
figure. If a horizontal force F acts at the
Sol. (d) bottom of the rod and ω is the angular
Given, length of a thin wire = l velocity of the rod which is a function of
Linear density of the wire = ρ angle of rotation θ, then the maximum
Let the mass of the wire = M angular displacement of the rod is
Now, according to the question (Acceleration due to gravity, g)
[22 April 2019, Shift-II]

C
ICM O

I
A B

R
F
 2F   2F 
Here, circular loop of the wire with centre, C. (a) θ = 2 sin− 1   (b) θ = 2 cos − 1  
 mg   mg 
R is the radius of circular loop of wire.
 2F   2F 
∴Moment of inertia axis passing through the (c) θ = 2 tan− 1   (d) θ = 2 cot − 1  
centre and perpendicular to the plane AB.  mg   mg 
64 AP EAMCET Chapterwise Physics

Sol. (c) QPower of an electric motor, P = τ ωi


According to the question, we drawn the Torque, τ = Iα
following figure, Where, I is moment of inertia,
∴ P = I α ωi
P
⇒ α= … (ii)
φ I ωi
CM From Eqs. (i) and (ii), we get
ωi Iω
2
mg cos φ t= = i
mg sin φ φ
(P / I ωi) P
mg φ
F 0.36 × (100)2
or t= = 48 s
75
As for maximum angular displacement,
equilibrium of torque exists about the centre 40. A wheel of radius 8 cm is attached to a
mass of rod. support so as to rotate about a horizontal
So, τ1 + τ 2 = 0 … (i) axis through its centre. A string of negligible
Torque due to the gravitational force, mass wrapped around its circumference
τ1 = mgsin φ × l carries a mass of 0.4 kg attached to its free
⇒ τ1 = mglsin φ … (ii)
end. When the mass is released, it descends
through 1 m in 10 seconds, then its moment
Similarly, torque due to horizontal force,
of inertia is (Acceleration due to gravity,
τ 2 = − F 2l cos φ
g = 10 ms −2) [23 April 2019, Shift-I]
⇒ τ 2 = 2Fl cos φ … (iii)
From Eqs. (i), (ii) and (iii), we get
2F 2F
tan φ = ⇒ φ = tan− 1
mg mg
Since, the angular velocity of the rod is a
function of angle of rotation θ,
As putting, θmax = 2 φ
θ
So, φ = max
2
2F 0.4 kg
Hence, θmax = 2tan− 1
mg (a) 1.277 kg m2 (b) 2.177 kg m2
∴The correct option is (c). (c) 21.77 kg m2 (d) 12.77 kg m2
39. An electric motor of power 75 W rotates a Sol. (a)
flywheel of moment of inertia 0 .36 kg - m 2 at Given, radius of wheel, R = 8 × 10−2 m, mass of
a constant rate of 100 rad s − 1 . If the power is weight, m = 0.4 kg, descending length, L = 1 m
switched off, the time taken for the wheel to and time, t = 10 s
come to rest is [22 April 2019, Shift-II] As, torque τ = F. R = mgR
(a) 12 s (b) 24 s (c) 36 s (d) 48 s putting the given values, we get
= 0.4 × 10 × 8 × 10−2
Sol. (d)
So, τ = 0.32 Nm
Given, I = 0.36 kg-m 2 , ωi = 100 rad / sec and
Let the wheel is rotated by an angle θ,
P = 75W arc 1
(ω − ωi) θ= = = 12 .5 rad
QAngular acceleration, α = f radius 0.08
t So, from the equation of rotational motion,
ωi
or α=− (Qω f = 0) αt 2
t θ = ω0 t +
2
ω
or t= i …(i) 2θ 2 × 12 .5
α ⇒ α= 2 = = 0.25 rad s −2 [Qω0 = 0]
t 10 × 10
Rotational Motion 65

Now, as Torque, τ = Iα Sol. (d)


∴ Moment of inertia of a wheel, In this case, angular momentum, J = Iω
τ 0.32
I= = = 1.28 kg-m2 Taken, v =1
α 0.25
v′ = 1.09 and ∆v = 0 ⋅ 09
Hence, the correct option is (a). 1
⇒ ω∝
41. A body of mass 1 kg is suspended from a I
spring of negligible mass. Another body of But I ∝ r 2 and ω ∝ v1 / 3
mass 500 g moving vertically upwards hits 1
∴ ω ∝ 2 / 3 (or v− 2 / 3 )
the suspended body with a velocity of 3 ms −1 v
and gets embedded in it. If the frequency of ∴Change in angular speed
oscillation of the system of the two bodies ∆ω 2 ∆v
= × 100 = − × 100
10 ω 3 v
after collision is Hz, the amplitude of the
π 2
= − × 0.03 × 100 = − 6%
motion and the spring constant are 3
respectively, [23 April 2019, Shift-I] So, angular speed will decrease with 6%.
(a) 5 cm, 300 Nm −1 (b) 10 cm, 300 Nm −1 43. Two spheres P and Q, each of mass 200 g are
(c) 10 cm, 600 Nm −1 (d) 5 cm, 600 Nm −1 attached to a string of length one metre as
Sol. (d) shown in the figure. The string and the
Given, mass of body, M = 1 kg, frequency of spheres are then whirled in a horizontal
10 circle about O at a constant angular speed.
oscillation, f = Hz,
π The ratio of the tension in the string between
mass of hitting body, m = 0.5 kg and speed of P and Q to that of between P and O is (P is at
body, v = 3 ms−1 mid-point of the line joining O and Q)
As frequency of oscillation of spring mass [22 April 2018 Shift-I]
system,
1 k
f =
2π Mbody
Q
O P
Since, hitting body get embedded to the initial
mass, so mass of the body changed to
Mbody = M + m = 1 + 0.5 = 1.5 kg
Spring constant, k = (2πf)2 Mbody 1 2 3 2
2 (a) (b) (c) (d)
=  2π ×  (1 .5) = 600 Nm−1
10 2 3 2 1
 π
Sol. (b)
Now, velocity of body after collision,
Tension between P and Q is
(M + m) v′ = mv ⇒ (1 + 1 / 2)v′ = 1 / 2 × 3
T1 = centripetal force on Q = mrω2
v′ = 1 m/s
 rad2 
The maximum amplitude of the sping-mass = 200 × 1 × ω2  g ⋅ m ⋅ 2 
system is v′ = Aω  s 
v′ 1
⇒ A= = = 0.05m = 5 cm Tension between O and P is
2πf 20 T2 = centripetal force on Q + centripetal force on
Hence, the correct answer is (d). P
1
42. A rigid metallic sphere is spinning around its = 200 × 1 × ω2 + 200 × × ω2
2
own axis in the absence of external torque. If
2 rad2 
the temperature is raised, its volume = 300 × 1 × ω  g ⋅ m ⋅ 2 
 s 
increases by 9%. The change in its angular
speed is [22 April 2018 Shift-I] Ratio of tension is
T1 200 × 1 × ω2 2
(a) increases by 9% (b) decreases by 9% = =
(c) increases by 6% (d) decreases by 6% T2 300 × 1 × ω2 3
66 AP EAMCET Chapterwise Physics

44. Two particles of masses in the ratio 1 : 2 are 46. The moment of inertia of a body about a
placed along a vertical line. The lighter given axis is 12 kg-m 2. Initially, the body is
particle is raised through a height of 9 cm. at rest. In order to produce a rotational
To raise the centre of mass of the system by kinetic energy of 15000 J, an angular
2 cm, the heavier particle should be acceleration of 10 rads − 2 must be applied
[22 April 2018 Shift-II]
about that axis for a duration of
(a) moved 1.5 cm downward [23 April 2018 Shift-I]
(b) moved 2 cm upward
(a) 2 s (b) 4 s (c) 10 s (d) 5 s
(c) moved 1.5 cm upward
(d) moved 2 cm downward Sol. (d)
Sol. (a) Given, moment of inertia, I = 12 kg - m 2
1
Let initially particles are at origin, their masses Rotational kinetic energy, E k = Iω2
are m and 2m. 2
1
m1 15000 J = × 12 × ω 2

2
9 cm 30000
COM of system So, ω= = 2500 = 50 rad / s
12
2 cm
Angular acceleration,
m2 α = 10 rad / s2
ω
As α=
m × 9 + 2m × y2 9 + 2y2 t
2= =
3m 3 ω 50
So, time = = = 5s
y2 = − 1 ⋅ 5 cm α 10
So, second particle must be moved 1.5 cm
downward. 47. A light rope is wound around a hollow
cylinder of mass 4 kg and radius 40 cm. If
45. A solid sphere and a ring of same radius roll the rope is pulled with a force of 40 N, its
down an inclined plane without slipping. angular acceleration is [23 April 2018 Shift-I]
Both start from rest from the top of the (a) 0.40 rads − 2 (b) 0.25 rads − 2
inclined plane. If the sphere and the ring (c) 25 rads − 2 (d) 40 rads − 2
reach the bottom of the inclined plane with
v2 Sol. (c)
velocities vs and vr respectively, then r2 is Torque, τ = Iα
vs
F × r = Mr 2α
[22 April 2018 Shift-II]
(a) 0.2 (b) 0.5 (c) 0.7 (d) 0.9
Sol.(c)
40 cm
When a body rolls down an inclined plane,
energy conservation gives
1 1 1  k2 
mgh = mv2 + Iω2 = mv2 1 + 2 
2 2 2  R 
where, I = mk2 and k = radius of gyration. 40N
  40 × 0.4 = 4 × (0.4)2 α
 2gh 
So, vbottom =   16 = 0.64α
 k2 
1 + 2  α=
16
 R  0.64
k2 k2 2
Now, for ring, 2 = 1 and for sphere, 2 = 16
R R 5 = × 100
2
2
vbottom 1+ 64
∴ ring
= 5 = 0⋅ 7 = 25 rad / s2
2
vbottom sphere 1+1
Rotational Motion 67
2mg
48. A particle of mass 15 kg is moving with a α=
−1
uniform speed 8ms in xy-plane along the MR
So, acceleration of rope, a = Rα
line 3 y = 4 x + 10 , then the magnitude of its
2mg 2 mg 2 × 1 × 10 10
angular momentum about the origin in = R⋅ = = = ms −2
MR M 8 4
 4
kg -m 2s −1 is ... sin53 ° =  Speed of bucket as it falls by 16 m is
 5
10
[23 April 2018 Shift-II] v = 2× a × s = 2× × 16 = 80 = 8.6 ≈ 8 ms −1
4
(a) 240 (b) 80 (c) 120 (d) 280
Sol. (a) 50. A circular disc of radius R is removed from
Y one end of a bigger circular disc of radius 2R.
3y=4x+10 The centre of mass of the new disc is at a
m
v=8ms–1
distance αR from the centre of the bigger
disc. The value of α is [24 April 2018 Shift-I]
1 1 1 1
(a) (b) (c) (d)
2 3 4 6
r1
Sol. (b)
X

Angular momentum = mvr1 2R


 10  R
= 15 × 8 ×   = 15 × 8 × 2 = 240 kg-m 2 s −1 αR
 32 + 42  C
  C2 C1

49. An empty bucket of mass 1 kg attached by a


light cord passed over a pulley of a water well
is released from rest. If the pulley assembly is
assumed to be a uniform solid cylinder of Let, mass of entire disc = M
mass 8 kg and free to rotate about its axis M M
Mass per unit area = =
without any friction, then the speed of the π(2R)2 4 πR2
bucket as it hits the water 16 m below is Mass of removed disc of radius R,
(take, g = 10 ms −2) [23 April 2018 Shift-II] M M
M1 = × R2 =
(a) 4 ms −1 (b) 8 ms −1 (c) 16 ms −1 (d) 20 ms −1 4 πR 2 4
Sol. (b) Mass of remaining disc,
M 3M
Torque on pulley = Iα ⇒ M2 = M − =
4 4
Center of mass of removed disc is C1 and centre
of mass of remaining new disc is C2 .
a And centre of mass of combination of M1 and
M 2 will be at C(0, 0).
M1 x1 − M 2 x 2
⇒ =0
M1 − M 2
⇒ M1 x1 = M 2 x 2
M 3M
⋅R = (αR)
4 4
MR2α 1
⇒ mgR = ⇒ α=
2 3
6
Gravitation
According to figure, r be the position vector of
1. Acceleration due to gravity [17 Sep. 2020, Shift-I] the planet w.r.t sun and F be the gravitational
(a) increases with altitude force on the planet due to the sun. Then, torque
(b) decreases with altitude exerted on the planet by this force about the
(c) is independent of altitude sun is
(d) first decreases and then increases with altitude τ = r × F= 0
[Q r and F are oppositely directed]
Sol. (b) But τ=
dL
Acceleration due to gravity above the surface dt
(at altitude) is given as dL
−2
∴ = 0 ⇒ L = constant
g  h dt
g′ = = g 1 +  Angular momentum = constant. Hence, Kepler’s
 Re 
2
 h
1 +  second law (law of areas) is equivalent to law of
 Re  conservation of angular momentum.
 2h 
g ′ = g 1 −  …(i) 3. Using the data given below, find the height
 Re  at which a communication satellite can reside.
[By binomial theorem (1 + x)n = 1 + nx, when (G = 6.67 × 10 −11 N-m 2 kg −2, M = 5.98 × 10 24 kg,
x < 1]
From Eq. (i), it is clear that acceleration due to R = 6.4 × 10 6 m) [17 Sep. 2020, Shift-II]
gravity decreases with altitude. (a) 35850 km (b) 3585 km
(c) 358.5 km (d) 35.85 km
2. Kepler’s second law (law of areas) is nothing
but a statement of [17 Sep 2020, Shift-I] Sol. (a)
(a) work-energy theory Given, G = 6.67 × 10−11 N-m 2 kg −2
(b) conservation of linear momentum M = 5.98 × 1024 kg, R = 6.4 × 106 m
For communication satellite,
(c) conservation of angular momentum T = 24 h = 24 × 60 × 60 s = 8.64 × 104 s
(d) conservation of energy We know that, time period of communication
Sol. (c) satellite is given as
2 π (R + h) (Re + h)3
According to Kepler’s second law of planetary T= = 2π
motion, the radius vector joining planet to the GM GM
sun sweeps out equal areas, in equal interval of Re + h
time. 1 /3
∆A (Re + h)3  T 2 GM 
i.e., = constant ⇒ T2 = 4 π2 ⇒ Re + h =  2 
∆t GM  4π 
1 /3
v  T 2 GM 
v ⇒ h=  2 
−R
 4π 
1 /3
 8.64 × 104 × 6.67 × 10−11 × 598
. × 1024 
= 
∆A 4 × (3.14) 2
P (Planet)  
r F
Sun − 6.4 × 106
= 42.25 × 106 − 6.4 × 106 = 35.85 × 106 m
= 35850 × 103 m = 35850 km
Gravitation 69

4. The escape velocity for a planet whose radius But given that,
v 2gR 2gR
is 1.7 × 10 m and acceleration due to gravity
6
v= e = ⇒ v2 =
2 2 4
is 1.7 ms − 2 is [18 Sep. 2020, Shift-I] gR
v2 = …(ii)
(a) 1.7 kms − 1 (b) 2.89 kms − 1 2
(c) 1.7 2 kms − 1 (d) 3.4 kms − 1 From Eqs. (i) and (ii), we get
Sol. (c) gR gR2
⋅R
R
Radius of planet, R = 1.7 × 106 m h= 2 = 2 ⇒ h=
Acceleration due to gravity, g = 1.7 ms −2 gR 3gR 3
2gR −
∴Escape velocity on the surface of planet is given as 2 2
ve = 2gR = 2 × 1.7 × 1.7 × 106 7. A point mass of 10 kg is placed at the
= 1.7 2 × 10 ms −1 = 1.7 2 km s −1
3
centre of earth. The weight of the point
mass is [21 Sep. 2020, Shift-I]
5. At what height from surface of earth the (a) zero (b) 98 N (c) 49 N (d) 10 N
value of acceleration due to gravity will fall to
half that on the surface of the earth? Sol. (a)
At the centre of earth, gravitational
[18 Sep. 2020, Shift-I] acceleration g is zero.
(a) 2625 m (b) 2625 km (c) 2526 m (d) 2526 km Hence, weight of point mass,
w = mg = 10 × 0 = 0
Sol. (b)
If g be the gravitational acceleration on the surface 8. A body is projected with a velocity greater
of earth, then gravitational acceleration at height h than orbital velocity but less than escape
given as velocity. Its path is [21 Sep. 2020, Shift-I]
g
gh = 2 (a) circular (b) elliptical
 h
1 +  (c) parabolic (d) hyperbolic
 Re 
Sol. (b)
g
But, gh = When the velocity of satellite (body) is more
2 than orbital velocity but less than escape
g g
∴ = 2 velocity, then total energy of satellite is
2  h negative and the path is elliptical. When the
1 + 
 Re  orbital velocity of satellite is equal to the escape
2 velocity, then path of satellite will be parabolic.
 h h
⇒ 1 +  = 2 ⇒ 1 + = 2 Again, when the orbital velocity of satellite is
 Re  Re greater than escape velocity, the path of
⇒ h = ( 2 − 1)Re = (1.414 − 1) 6400 satellite will be hyperbolic.
= 2649.6 km ~ − 2625 km 9. The distance between the centre of moon
6. A body is projected vertically upwards from and earth is D and mass of earth is 81 times
the surface of a planet of radius R with a the mass of moon. At what distance from
velocity equal to half the escape velocity of the centre of the earth, the gravitational
that planet. Then, the maximum height force will be zero ? [21 Sep. 2020, Shift-II]
attained by the body is [18 Sep. 2020, Shift-II] D 2D 4D 9D
(a) (b) (c) (d)
R R R R 2 3 3 10
(a) (b) (c) (d)
3 2 4 5 Sol. (d)
Sol. (a) Let a unit mass m is present at a distance x from
If a body is projected vertically upwards with a the earth, where gravitational force is zero.
velocity of v from the surface of planet, then GmM e GmM
= …(i)
maximum height attained by the body is x2 (D − x)2
v2 R
h= …(i) where, M e is mass of earth and M is mass of
2gR − v2 moon.
Given, M e = 81 M
70 AP EAMCET Chapterwise Physics

∴From Eq. (i), we have where, M = mass of earth.


Gm81 M GmM 81 1 and R = radius of earth.
= ⇒ = Gravitational potential energy at height h = 3R is
x2 (D − x)2 x 2 (D − x)2
2 given as
⇒  9 = 1 9
=⇒
1
U2 = −
GMm
=−
GMm
 
 x (D − x)2 x D− x R+ h R + 3R
9D GMm
⇒ 9D − 9 x = x ⇒ 9D = 10 x ⇒ x = U2 = − …(ii)
10 4R
∴Change in gravitational potential energy,
10. Acceleration due to gravity at a height h is GMm  GMm 
equal to that at a depth d below the surface ∆U = U 2 − U1 = − − − 
4R  R 
of the earth, if [21 Sep. 2020, Shift-II] [from Eqs. (i) and (ii)]
(a) d = h (b) 2d = h (c) d = 2 h (d) 3d = h GMm GMm 3 GMm
=− + = ⋅
Sol. (c) 4R R 4 R
3 gR2 ⋅ m
According to question, = ⋅ [Q GM = gR2 ]
4 R
Acceleration due to gravity at a height (h) 3
= mg R
= Acceleration due to gravity below depth (d) 4
 2h   d
⇒ g1 −  = g1 − 
 Re   Re
13. Two bodies of masses m1 and m2 initially at
2h d 2h d rest at infinite distance apart move towards
⇒ 1− =1− ⇒ =
Re Re Re Re each other under gravitational force of
⇒ 2h = d attraction. Their relative velocity of approach
when they are separated by a distance r is
11. The value of acceleration due to gravity g is (G = universal gravitational constant.)
maximum at [22 Sep. 2020, Shift-I] [20 April 2019, Shift-I]
(a) poles (b) centre 2G(m1 − m2 )
1/ 2
2G(m1 + m2 )
1/ 2

(c) equator (d) surface of earth (a)  (b) 


 r   r 
Sol. (a)  
1/ 2 1/ 2
(d)  mm 
r r
The acceleration due to gravity on the surface of (c)    2G 1 2 
earth is given as  2G(m1 m2 )
g′ = g − ω2 Re cos2 λ …(i) Sol. (b)
where, λ is the angle of lattitude.
Initially when the two masses are at an infinite
At poles, λ = 90°
distance from each other, their gravitational
∴ cos 90° = 0
potential energy is zero. When they are at a
From Eq. (i), g′ = g
Hence, at poles, the acceleration due to gravity distance r from each, the gravitational PE is
− Gm1 m2
is maximum. PE =
r2
12. A body of mass m is placed on the earth’s The minus sign-indicates that there is a
surface. It is taken from the earth’s surface decrease in PE. This gives rise to an increase in
to a height h = 3 R ,(R is radius of earth). The kinetic energy. If v1 and v2 are their respective
velocities when they are a distance r apart then,
change in gravitational potential energy of
from the law of conservation of energy, we have
the body is [22 Sep. 2020, Shift-II]
1 Gm1 m2 2Gm2
m1 v12 = or v1 =
(a)   mgR (b)   mgR
2 3
 3  4 2 r r
1 Gm1 m2 2Gm1
m2 v2 = or v2 =
(c)   mgR (d)   mgR
2
1 1 and
2  4 2 r r
Therefore, their relative velocity of approach is
Sol. (b) 2Gm2 2Gm1 2G
Gravitational potential energy on the surface of v1 + v2 = + = (m1 + m2)
r r r
earth,
2G (m1 × m2) 
1 /2

= 
GMm
U1 = − …(i) 
R  r 
Gravitation 71
mass of earth (M e)
14. A planet is revolving around the Sun as Q Density of earth, ρ =
shown in the figure. The radius vectors volume of earth(V)
or M e = ρ ⋅ V = ρ  πR3 
joining the Sun and the planet at points A 4
and B are 90 × 10 6 km and 60 × 10 6 km, 3 
 4 3
G  πR  ρ m G  π R ρ m
4
respectively. The ratio of velocities of the planet
3  3 
at the points A and B when its velocities ∴ F= or F =
(R + h)2
2
make angle 30° and 60° with major-axis of 1 + 
h
 
the orbit is [20 April 2019, Shift-I]  R
−2
π GRρ m 1 + 
4 h
vB or F =
3  R
B
By using Binomial expansion,
F = πGRρm 1 − 
4 2h
60°
30° S 3  R
n(n − 1) 2
[Q (1 + x) = 1 + nx +
n
.x + …
A 2!
and neglecting higher terms.]
vA Difference in weight
= π GRρ m − πGρ mR  
4 4 2h
3 2 1 3
(a) (b) (c) (d) 3 3  R
2 3 3 3 2
8
Sol. (b) Hence, error in weight = πρGmh.
3
According to the law of conservation of angular
momentum, angular momentum (J) of a planet 16. Two masses 90 kg and 160 kg are separated
is constant. by a distance of 5 m. The magnitude of
⇒ mu ArA sinθ A = muB rB sinθB intensity of the gravitational field at a point
u A rB sinθB
or = which is at a distance 3 m from the 90 kg
vB rA sinθΑ mass and 4 m from the 160 kg mass is
Given, rA = 90 × 10 km, rB = 60 × 106 km
6
(Universal gravitational constant,
θ A = 30°, θB = 60°
G = 6 .67 × 10 − 11 N - m 2 kg − 2).
u A 60 × 106 sin 60° 2 3/ 2
or = × = × [21 April 2019, Shift-I]
uB 90 × 106 sin 30° 3 1 / 2
uA 2 (a) 94.3 × 10− 10 N kg − 1 (b) 9.43 × 10− 10 N kg − 1
or = (c) 9.43 × 10− 12 N kg − 1 (d) 94.3 × 10− 12 N kg − 1
uB 3
Hence, the ratio of velocities of the planet is 2 3 . Sol. (b)
A system of two masses is shown in the figure,
15. Two bodies each of mass m are hung from a
C
balance whose scale pans differ in a vertical
height by h. If the mean density of the earth 3m 4m
is ρ, the error in weighing is
[20 April 2019, Shift-II]
4 πρ Gmh 3 πρ Gmh
(a) (b)
3 4 A B
8 πρ Gmh 3 πρ Gmh m1 5m m2
(c) (d)
3 8
Here, m1 = 90 kg and m2 = 160 kg
Sol. (c) Gravitational field intensity due to mass A,
Gravitational force on a body of mass m at height GM 90
h due to earth, E A = 2 A = G 2 = 10G $rCA
GM e m rCA (3 )
F= 160
(R + h)2 Similarly, EB = G × 2 = 10G $rCB
where, M e is mass of the earth. 4
72 AP EAMCET Chapterwise Physics

In ∆ABC, (AB)2 = (AC)2 + (BC)2 18. The gravitational potential difference


(5)2 = (3)2 + (4)2
between the surface of a planet and a point
Hence ∆ABC is a right angle triangle.
Hence, the resultant of E A and EB , 20 m above it is 16 J kg − 1 . The work done in
E = E 2A + E B2 = (10G)2 + (10G)2 = 2 × 10 G
moving a 4 kg body by 8 m on a slope of 60°
from the horizontal is [22 April 2019, Shift-II]
Putting the value of G, we get
⇒ E = 2 × 10 × 6.67 × 10− 11 (a) 22.17 J (b) 2.217 J
(c) 221.7 J (d) 0.2217 J
= 9.43 × 10− 10 Nkg −1
Hence, the correct option is (b). Sol. (a)
V
17. A rocket is launched straight up from the Q Gravitational field, E =
r
1 16
surface of the earth. When its altitude is of Given, E = J kg − 1 m− 1
3 20
the radius of the earth, its fuel runs out and Q Work done in moving a body,
therefore it coasts. If the rocket has to escape W = mEh
from the gravitational pull of the earth, the 8 3
minimum velocity with which it should Q h = 8sin 60° = = 4 3m
2
coast is (Escape velocity on the surface of the Hence, work done in moving a 4 kg body by 8 m
earth is 11.2 kms −1 .) [22 April 2019, Shift-I] on a slope of 60° from the horizontal is
(a) 11.2 kms−1 (b) 10.7 kms−1
W = 4 ×   × 4 3 = 22.17 J
16
(c) 9.7 kms−1 (d) 8.7 kms−1  20 
Sol. (c)
19. The gravitational field in a region is given by
Given, escape velocity on the surface of the earth.
E = (5 $i + 12 $j) Nkg −1 . If a particle of mass 2 kg
ve = 2gRe =11.2 km/s …(i)
is moved from the origin to the point (12 m,
where, R is the radius of earth.
15 m) in this region, the change in gravitational
Height of the rocket from the surface of earth,
R potential energy is [23 April 2019, Shift-I]
h = e . If the rocket has to escape from the
3 (a) − 450 J (b) − 480 J
R (c) − 240 J (d) − 500 J
gravitational pull of the earth from h = e .
3 Sol. (b)
Then, the potential energy at the altitude, Given, E = (5$i + 12$j) Nkg −1 , m = 2kg and
Re
= kinetic energy r = (12$i + 15$j) m
3
GM e m 1 2 GM e 1 Gravitational potential,
= mve1 ⇒ = ve2 r
Re + h 2 Re + (Re / 3) 2 1 V = − ∫ E ⋅ dr
R
[ve1 = escape velocity of rocket from e ] 0
r
3
= − ∫ (E x $i + E y$j) (drx $i + dry$j)
 M e = mass of the earth and
  0
 m = mass of the rocket 
x y

gRe2 1 2 V = −  ∫ E x drx + ∫ E dr 
= v 0
y y

4Re 2 e1 0

3  12 15

V = −  5 ∫ drx + 12∫ dry 
3 v2  GM e   0 
⇒ gRe = e1 Q g = R2 
0
4 2  e  V = − [5 (12 − 0) + 12 (15 − 0)]
3
.2gRe = ve21 ⇒
3
. 2gRe = ve1 V = − 240 J kg −1
4 2 Hence, the change in gravitational potential
3 energy of particle of 2 kg mass,
11.2 = ve1 [From Eq. (i)]
2 E = mV = − 2 × 240 = − 480 J
9.7 = ve1 ⇒ ve1 = 9.7 km/s So, the correct option is (b).
Gravitation 73

20. Three masses m, 2m and 3m are arranged in Let object rises upto height h, when it is thrown
with speed x ⋅ ve , then from conservation of
two triangular configurations as shown in
energy, we get
figure 1 and figure 2. Work done by an 1 GMm GMm
external agent in changing, the configuration m(x ⋅ ve)2 − = 0−
2 R R+ h
from figure 1 to figure 2 is [22 April 2018, Shift-I]
3m 2m
h
x⋅ve
a a a m

M
90º
R
m a 2m m a 3m
Fig. 1 Fig. 2
1 GMm GMm
⇒ m(x ⋅ ve)2 = −
6Gm  2
6  2 R R+ h
(a) 2− (b) 0
a  2  ⇒

(x ⋅ ve) = 2GM 
2 h 

Gm2  6  Gm2  6   R(R + h) 
(c) 6+ (d) − 6−
a  2  a  2  2GM 2GM  h 
⇒ x2 =  
Sol. (d) R R  R + h
h Rx 2
Work done to change configuration from 1 to 2 ⇒ x2 = or h =
can be calculated by R+ h 1 − x2
W12 = − (U f − U i)
2m
22. An artificial satellite of mass m is moving
3m along an elliptical path around the earth.
The areal velocity of the satellite is
proportional to [23 April 2018, Shift-I]
a a
(a) m (b) m− 1 (c) m0 (d) m1 / 2
a 1 √2a 2
Sol. (c)
dA
Areal velocity, = constant
dt
m a 2m m a 3m dA L
=
 Gm dt 2m
 2 + 3 + 6  −  − Gm (2 + 3 + 6) 
2 2
= −−     (where, L is angular momentum)
 a  2  a  mvr sinθ vr sinθ
= =
Gm2  6 
=− 6 −  2m 2
a  2 This shows areal velocity does not depends on mass.
dA
So, ∝ m0
21. An object is thrown vertically upwards from dt
the surface of the earth with a velocity x
times the escape velocity on the earth (x < 1), 23. Two bodies of equal masses are some
then the maximum height to which its rises distance apart. If 20% of mass is transferred
from the centre of the earth is (radius of from the first body to the second body, then
earth is R) [22 April 2018, Shift-II] the gravitational force between them
R 1 − x2 x2 [23 April 2018, Shift-II]
(a) R(1 − x)2 (b) (c) (d)
(1 − x )
2
R 1− R (a) increases by 4% (b) increases by 14%
(c) decreases by 4% (d) decreases by 14%
Sol.(*) Sol. (c)
(No option is matching.)
Initially force,
2GM Gm2
Escape velocity is ve = F1 = 2
R r
74 AP EAMCET Chapterwise Physics

v2 = 2gR + 2gR − 1


When 20% mass is transferred from one body to R
another, then their masses are r 
80 120
m1 = m and m2 = m 2gR2 dr 2g
100 100 v= , So, v = =R ...(i)
So, force is finally r dt r
Gm2 2g
F2 = 2 ×  
24 =R
r  25 r
F − F2
Hence, reduction in F2 is 1 × 100 % = − 4% Integrating Eq. (i), we get
F1 t 1 R +h

∫0 dt = R 2g ∫R r ⋅ dr
1 /2

24. A body is projected vertically upwards from


2 1
the surface of the earth with a velocity ⇒ t= [(R + h)3 / 2 − R3 / 2 ]
sufficient to carry it to infinity. The time 3 R 2g
taken by it to reach a height of three times
1 2R   
3/ 2
h
the radius of the earth is t= 1 +  − 1
3 g  R  
(acceleration due to gravity = 9 .8 ms−2 and
radius of the earth = 6400 km) According to question, h=3R
[24 April 2018, Shift-I] 1 2R   3R 
3/ 2

(a) 44.44 min (b) 22.22 min t=  1 +  − 1
3 g  R 
(c) 18.76 min (d) 37.52 min
1 2R
Sol. (a) t= ×7
If at a distance r from the centre of the earth, the 3 g
body has velocity v, then by conservation of Putting all values, we get
mechanical energy 1  2 × 6400 × 103 
(TE)at weight = (TE)at surface t=   × 7 [Q R = 6400 × 10 ]
3

3 9.8 
mv2 +  −
1 GMm  1 2  GMm 
 = mve +  −  t=
80
×14.2 × 7s
2  r  2  R 
3
where, ve = escape velocity.
t = 2666.65s
2GM  R
− 1 Q v = GM 
v2 = ve2 +  e 2gR , g =  t = 44 . 44 min
R  r   R2 
7
Mechanical Properties
of Solids
mgl 10 × 10 × 3
1. A 3 m long steel wire is stretched to increase Y = =
π r 2 ∆l π × (2 × 10−4 )2 × 2.4 × 10−2
its length by 0.3 cm. Poisson’s ratio for steel
300
is 0.26. The lateral strain produced in the = = 1011 N/m 2
wire is [17 Sep. 2020, Shift-I] 3 × 10−9
(a) 0.26 × 10−4 (b) 0.26 × 10−2 When d ′ = 2d = 2 × 0.4 = 0.8 mm
(c) 0.26 × 10−3 (d) 0.26 × 10−1 d ′ 0.8
r= = = 0.4 mm = 4 × 10−4 m
2 2
Sol. (c) mgl 10 × 10 × 3
Length of steel wire, l = 3 m Hence, ∆l = =
πr 2 Y π × (4 × 10−4 )2 × 1011
Increased length, ∆l = 0.3 cm = 3 × 10−3 m
= 5.96 × 10−3 m = 0.596 cm ~
− 0.6 cm
Poisson’s ratio = 0.26
Lateral strain 3. The diagram shows stress versus strain curve
⇒ = 0.26
Longitudinal strain for the materials A and B. From the curves,
⇒ Lateral strain = 0.26 × longitudinal strain we can infer that [17 Sep. 2020, Shift-II]
∆l
= 0.26 × Stress
l
3 × 10−3
= 0.26 × = 0.26 × 10−3 A
3
B
2. When a weight of 10 kg is suspended from a
copper wire of length 3m and diameter
0.4 mm, its length increases by 2.4 cm. If the
diameter of the wire is doubled, then the Strain
extension in its length will be (a) A is brittle but B is ductile
[17 Sep. 2020, Shift-II] (b) A is ductile but B is brittle
(a) 9.6 cm (b) 4.8 cm (c) 1.2 cm (d) 0.6 cm (c) Both A and B are ductile
(d) Both A and B are brittle
Sol. (d)
Mass of the suspended body, m = 10 kg Sol. (b)
Length of copper wire, l = 3 m According to stress-strain curve, we see that for
same value of applied stress, strain produced is
Diameter, d = 0.4 mm
more in material B than in material A in linear
d 0.4
∴ Radius, r = = = 0.2 mm = 2 × 10−4 m region. Therefore, A is ductile but B is brittle. In
2 2 ductile materials, yield point exists whereas in
Increased length, ∆l = 2.4 cm = 2.4 × 10−2 m brittle material failure occurs without yielding.
Young’s modulus of the wire is given as A has yield point hence it is ductile. B doesn't
have yield point hence B is brittle.
76 AP EAMCET Chapterwise Physics

4. A force F is needed to break a copper wire Sol. (c)


having radius R. The force needed to break a For copper wire, Y = 12 × 1010 Nm −2
copper wire of radius 2R will be Length, l = 5 m,
[18 Sep. 2020, Shift-II] F = mg = 40 × 10 = 400 N
F F Fl
(a) (b) 2F (c) 4F (d) We know that, Y =
2 4 A∆l
Sol. (c) Fl 400 × 5
⇒ ∆l = =
Force needed to break a wire is directly AY πr 2 × 12 × 1010
proportional to its cross-sectional area. 2000
∆l = …(i)
i.e. F∝A 12πr 2 × 1010
F2 A2 πR22 For aluminium wire, Y = 7 × 1010 Nm −2
= =
F1 A1 πR12 Diameter, d = 3 mm = 3 × 10−3 m
2
R  2
d 3 × 10−3
=  2  =  
2R ∴ r= = m = 1.5 × 10−3 m
[Q R1 = R, R2 = 2R]
 R1   R 2 2
=4 Fl
Similarly, Y =
⇒ F2 = 4F1 = 4F [Q F1 = F] A∆l
Fl 400 × 5
5. If a pressure of 8 × 10 N m 8 −2
is applied to a ⇒ ∆l = =
AY π(1.5 × 10−3)2 × 7 × 1010
lead block, so that its volume reduces by 2000
20%. The Bulk modulus of lead block is ∆l = …(ii)
. π × 104
1575
[21 Sep. 2020, Shift-I]
According to given condition, ∆l is same in both
(a) 4 × 107 Nm − 2 (b) 4 × 108 Nm − 2 cases.
(c) 4 × 109 Nm − 2 (d) 4 × 1010 Nm − 2
From Eqs. (i) and (ii), we get
Sol. (c) 2000 2000
∴ =
Pressure applied on the block, 12πr 2 × 1010 1575. π × 104
p = 8 × 108 Nm −2 . × 10−6
12r 2 = 1575
If V1 be initial volume, then final volume of
⇒ r = 1.15 × 10−3 m = 115
. mm
block,
20 ∴ Diameter, d = 2r = 2 × 1.15 = 2. 3 mm
V2 = V1 − 20% of V1 = V1 − V1
100 7. The energy stored in a strained wire is given
4
V2 = V1 by [21 Sep. 2020, Shift-II]
5 1 1
(a) × load × extension (b) × extension × stress
∴ Change in volume, 2 2
4 V
∆V = V1 − V2 = V1 − V1 ⇒ = 1 1
(c) × stress × strain
1
(d) × strain × load
5 5 2 2
∴ Bulk modulus,
Sol. (a)
p pV 8 × 108 × V1
B= = = Energy stored in a strained wire is equal to work
∆V / V ∆V V1 / 5 done by the load to increase the length of wire.
= 4 × 109 Nm −2 ∴ Energy, U = Work done
6. What should be the diameter of a copper = Average force (load)
wire (Y = 12 × 10 10 Nm −2) of length 5 m to × Extension in the wire
 0 + F
produce the same elongation produced by a =  × Extension
 2 
5 m long aluminium wire (Y = 7 × 10 10 Nm −2)
F
of diameter 3 mm with the same 40 kg = × Extension
2
mass ? [21 Sep. 2020, Shift-II]
1
(a) 1.5 mm (b) 5 mm (c) 2.3 mm (d) 10 mm = × Load × Extension
2
Mechanical Properties of Solids 77
Stress
8. A material has Poisson’s ratio 0.50. If a ⇒ = Young’s modulus
uniform rod made of this material suffers a Strain
longitudinal strain of 2 × 10 −3, then the F/ A FL
⇒ =Y ⇒ =Y
∆l A∆l
percentage change in volume is
L
[22 Sep. 2020, Shift-I] Y ⋅ A∆l
F=
(a) 0.6 (b) 0.4 (c) 0.2 (d) 0 L
Sol. (d) Since, Y , A, L are constant.
Given, Poisson’s ratio, Hence, F ∝ ∆l
σ = 0.5 But ∆l = l
Longitudinal strain, ∴ F∝l
∆l
= 2 × 10−3 11. A wire of length 1 m and radius 2 mm is
l vertically clamped. The lower end is twisted
∆V 
Volumetric strain   and longitudinal strain through an angle of 45°. The angle of shear
 V  is [23 Sep. 2020, Shift-I]
 ∆l  are related as
  (a) 0.09° (b) 0.9° (c) 9° (d) 90°
 l 
∆V ∆l Sol. (a)
= (1 − 2 σ) Given that, length, l = 1 m
V l
Radius, r = 2 mm = 2 × 10−3 m
⇒ = (1 − 2 × 0.5) × 2 × 10−3
Twisted angle, θ = 45°
= (1 − 1) × 2 × 10−3 = 0 × 2 × 10−3 = 0
rθ 2 × 10−3 × 45°
∆V Angle of shear, φ ≈ tan φ = =
∴ × 100 = 0 × 100% = 0 l 1
V
φ = 0.09°
9. Young’s modulus for perfectly rigid body is
12. What is the pressure required to reduce the
[22 Sep. 2020, Shift-II] given volume of water by 1%?
(a) zero (b) infinite (Bulk modulus (K) = 2 × 10 8 Nm −2)
(c) 1 ×1010 Nm −2 (d) 10 × 1010 Nm −2 [23 Sep. 2020, Shift-I]
Sol. (b) (a) 2 × 105 Nm −2 (b) 2 × 106 Nm −2
In perfectly rigid body, there is no change in its (c) 2 × 107 Nm −2 (d) 2 × 108 Nm −2
dimensions on applying a deforming force,
Sol. (b)
therefore strain in the perfectly rigid body is
zero. Given that, bulk modulus, K = 2 × 108 Nm −2
Stress Stress % change in volume = 1%
∴ Young’s modulus = = =∞ ∆V
Strain 0 or = 0.01
Therefore, Young’s modulus of a perfectly rigid V
body is infinite. As we know, bulk modulus of a liquid,
p ∆V
10. According to the Hooke’s law the force B= or p = B ×
∆V V
required to change the length of a wire by l V
is proportional to [22 Sep. 2020, Shift-II]
= 2 × 108 × 0.01 = 2 × 106 Nm −2
(a) l −2 (b) l −1 (c) l (d) l 2
13. A solid copper cube of 7 cm edge is subjected
Sol. (c)
to a hydraulic pressure of 8000 kPa. The
Given, change in length, ∆l = l
volume contraction of the copper cube is
L = original length (say) (Bulk modulus of copper = 140 GPa)
According to Hooke’s law, [20 April 2019, Shift-I]
Stress ∝ Strain (a) 196 × 10−3 cm 3 (b) 19.6 × 10−6 cm 3
Stress (c) 19.6 × 10−3 cm 3 (d) 196 × 103 cm 3
⇒ = constant
Strain
78 AP EAMCET Chapterwise Physics

Sol. (c) 15. The following four wires are made of the
Given, same material. If same tension is applied to
edge of solid copper cube, l = 7 cm each, the wire having largest extension is
hydraulic pressure, p = 8000 kPa = 8000 × 103 Pa [21 April 2019, Shift-I]
and Bulk modulus of copper, β = 140 GPa (a) length 0.5 m, diameter 0.5 mm.
= 140 × 109 Pa (b) length 1 m, diameter 1 mm.
As we know that, (c) length 2 m, diameter 2 mm.
p pV (d) length 3 m, diameter 3 mm.
Bulk modulus, β = or β =
 ∆V  ∆V Sol. (a)
 
 V  Given, that material is same, so Young’s
pV 8000 × 103 × () l3 modulus = constant and tension = constant.
∴∆V = =
β 140 × 10 9 Hence, the extension,
FL
8000 × 103 × (7)3 ∆L =
= = 19.6 × 10−3 cm 3 YA
140 × 109
4F L  πD 2 
So, ∆L = × 2 Q A = 
14. A one metre steel wire of negligible mass Yπ D  4 
and area of cross-section 0.01 cm 2 is kept on L
⇒ ∆L ∝ 2
a smooth horizontal table with one end D
fixed. A ball of mass 1 kg is attached to the So, now checking the options,
other end. The ball and the wire are rotating (a) L = 0.5 m, D = 0.5 mm = 0.5 × 10− 3 m
with an angular velocity of ω . If the 4F 0.5 4F
elongation of the wire is 2 mm, then ω is ∆L = × = 2 × 106 ×
Yπ (0.5 × 10− 3)2 Yπ
(Young’s modulus of steel = 2 × 10 11 Nm −2)
(b) L = 1 m , D = 1 mm
[20 April 2019, Shift-II] 4F
(a) 5 rad s −1 (b) 10 rad s −1 ∆L = × 1.0 × 106

(c) 15 rad s −1 (d) 20 rad s −1
(c) L = 2m, D = 2 mm
Sol. (d) ∆L =
4F
× 0.5 × 106
Given, elongation of the wire, ∆l = 2 mm Y 4π
= 2 × 10−3 m (d) L = 3m, D = 3 mm
Mass of the ball, m = 1 kg ∆L =
4F
× 0.33 × 106
Length of wire, l = 1 m Yπ
4F
Area of cross-sectional of wire, Hence, is a constant in all quantities. Now
A = 0.01 cm 2 = 0.01 × 10−4 m Yπ
4F
Young’s modulus of steel, Y = 2 × 1011 Nm −2 after checking the multiplier of . The highest

Q Tension force in wire, T = mω2 l value of ∆L is in option (a).
Tension mω2 l
Q Stress = = 16. Two metal plates P and Q of same material
Area A
∆l stress are arranged as shown in the figure. If both
Strain = =
l Young ′s modulus the plates are uniformly heated through the
same range of temperature, then
mω2 l 2 YA ∆l
or ∆l = or ω = [21 April 2019, Shift-II]
YA ml 2
Putting the given values, we get y
2 × 1011 × 0.01 × 10−4 × 2 × 10−3
=
1 × ()
12 Plate P Plate Q
ω = 20 rad/sec −1 x
Mechanical Properties of Solids 79

(a) both x and y increase mg


stress mgl
(b) both x and y decrease and Y2 = = 2A = …(ii)
(c) x decreases and y increase strain ∆ l 2A∆l
(d) x increases and y decrease l
Sol. (b) If Y be the equivalent Young’s modulus of the
combination,
According to the question, both the plates P mgl
and Q are uniformly heated through the same then, Y = ...(iii)
range of temperature. So, both the plates A∆l
expands outward. Therefore, both the distance x From Eqs. (i), (ii) and (iii), we get
and y decreases. Y + Y2
Y = 1
2
17. Two wires of equal length and equal cross
sectional areas are suspended as shown in 18. The area of cross-section of steel wire is
the figure. Their Young’s modulii are Y1 and 0 .1 cm 2 and Young’s modulus of steel is
Y2, respectively. The equivalent Young’s 2 × 10 11 N m − 2. The force required to
modulus is [22 April 2019, Shift-I]
stretched by 0.1% of its length is
[22 April 2019, Shift-II]
(a) 1000 N (b) 2000 N (c) 5000 N (d) 4000 N
Y1 Y2 Sol. (b)
Q Young’s modulus of steel,
F
Y =
∆l

A 
 l 
Y1 + Y2 Y1 Y2 Q Given, Y = 2 × 1011 Nm− 2 ,
(a) Y1 + Y2 (b) (c) (d) Y1 Y2
2 Y1 + Y2 . cm2 = 10− 5 m2
A = 01
∆l 01
.
Sol. (b) and = = 1 × 10− 3
l 100
Given, two wires have same length and equal
cross-sectional area. Hence, the force required to stretched by 0.1% of
it’s length,
∆l
F = YA 
 l 
Y1 Y2
⇒ F = 2 × 1011 × 10− 5 × 1 × 10− 3 N
T T or F = 2000 N

19. A uniform wire of length 10 m and diameter


0.6 mm is stretched by 6 mm with certain
force. If the Poisson’s ratio of the material of
the wire is 0.3, then the change in diameter
mg
of the wire is [23 April 2019, Shift-I]
i.e., l1 = l 2 = l and A1 = A2 = A
(a) 108 × 10−8 m (b) 108 × 10−6 m
Suppose, m is the mass of the hanging load. (c) 10.8 × 10−8 m . × 10−8 m
(d) 108
From figure, 2 T = mg
mg Sol. (c)
i.e., T= Given, length of the wire, L = 10 m, diameter,
2
T mg D = 0.6 × 10−3 m, poisson’s ratio, σ = 0.3 and
Hence, stress on each wire = = change in wire length, ∆L = 6 × 10−3 m
A 2A
∆D
mg
lateral strain
stress mgl As poisson’s ratio, σ = = D
∴ Y1 = = 2A = …(i) longitudinal strain ∆L
strain ∆l 2A∆l
L
l
80 AP EAMCET Chapterwise Physics

∆DL (Young’s modulus of steel = 2 × 10 11 Nm −2


∴ σ=
D∆L and acceleration due to gravity = 10 ms−2)
σD∆L
⇒ ∆D = [22 April 2018, Shift-I]
L
(a) 10 ms −1 (b) 12 ms −1
0.3 × 0.6 × 10−3 × 6 × 10−3 (c) 15 ms −1 (d) 18 ms −1
=
10
Sol. (a)
⇒ ∆D = 10.8 × 10−8 m
Hence, the change in diameter of wire is
10.8×10−8 m. So, the correct option is (c).

20. Two equal and opposite forces each F act on 200 cm


205 cm
a rod of uniform cross-sectional area a as
shown in the figure. Shearing stress on the
4.5 cm
section AB will be [22 April 2018, Shift-I]
A
Extension in wire due to load and centrifugal
force
F F = ∆l = (205 − 204.5) cm = 0.5 cm
θ If velocity of sphere at lowest point is v, then
B  Mv2 
 Mg +  L
F sinθcos θ F sinθ  R 
(a) (b) Y =
a a A∆l
F cosθ F sin2 θ Mv2 YA∆l
(c) (d) ⇒ Mg + =
a a R L
Sol. (a) where, R = 202 . 25 cm
a
2 × v2
⇒ 2 × 10 +
202. 25 × 10− 2
F cos θ 2 × 1011 × 0. 24 × 10− 6 × 0. 5 × 10− 2
θ =
ax = a F 2
cosθ = 0.12 × 103
θ 2 × v2
⇒ = 100
202 . 25 × 10− 2
F sin θ
⇒ v2 = 101.125
Shearing stress
F sinθ F ⇒ v = 10 ⋅ 05 ms− 1
=− = − sinθ cosθ
ax a or v = 10 ms− 1
Magnitude of shearing stress 22. A rubber cube of side 5 cm has one face fixed,
F while a tangential force 1800 N is applied on
= sinθ cosθ
a its opposite face. If modulus of rigidity of
rubber is 2.4 × 10 6 Nm − 2, then the lateral
21. A sphere of mass 2 kg and diameter 4.5 cm is
attached to the lower end of a steel wire of displacement of the strained face is ______
2 m length and area of cross-section [23 April 2018, Shift-I]
0 .24 × 10 −6 m 2. The wire is suspended from (a) 3 mm (b) 5 mm
(c) 15 mm (d) 1.5 mm
205 cm high ceiling of a room. When the
system is made to oscillate as a simple Sol. (c)
pendulum, the sphere just grazes the floor at Shear stress
its lowest position. The velocity of the sphere =
Tangential force 1800 N
=
at the lowest position is Area (0.05)2
Mechanical Properties of Solids 81

Shear strain = tanθ ∆l F


=
x Lateral displacement l YA
= =
h Height of cube ∆l
where, = longitudinal strain
x l
= cm
5 Given, F = 22N, Y = 1 .1 × 1011 N - m 2 ,
xm A = 0. 01 cm 2 = 10− 6 m 2
=
0 ⋅ 05 m ∆l 22
= = 2 × 10− 4
Modulus of rigidity, l . × 1011 × 10− 6
11
Shear stress Now, Poisson ratio
η=
Shear strain Lateral strain ∆d / d
σ= =
N 1800 0.05 Longitudinal strain ∆l / l
2.4 × 106 2 = × N / m2
m 25 × 10− 4 x ∆d ∆l
= σ⋅ = 0. 32 × 2 × 10− 4
1800 × 0.05 d l
or x =
25 × 10− 4 × 2.4 × 106 ∆d
Change in diameter, = 6. 4 × 10− 5
d
= 15 × 10− 3 m = 15 mm
∆r
or change (decrease) in radius, = 6. 4 × 10− 5
23. A copper wire of cross-sectional area 0.01 cm 2 r
is under a tension of 22 N. The decrease in Area, A = πr 2
the cross-sectional area is Fractional change in area,
(Young modulus = 1.1 × 10 11 Nm −2, Poisson’s ∆A ∆r
= 2⋅
ratio = 0 .32) [24 April 2018, Shift-I] A r
∆A −5
(a) 0128
. × 10−6 cm2 (b) 128 × 10−6 cm2 = 2 × 6.4 × 10
(c) 12 .8 × 10−6 cm2 . × 10−6 cm2
(d) 128 A
∆A=(12.8 × 10−5 )A
Sol. (d)
Decrease in area,
Young’s modulus,
∆ A=(12.8 × 10−5 )× (0.01)cm 2
F/ A
Y = ∆ A=1.28 × 10−6 cm 2
∆l / l
8
Mechanical Properties
of Fluids
1. A pure number which determines the type of 3. A fully loaded Boeing air craft has a mass of
flow of a liquid through a pipe is known as 3.3 × 10 5 kg. Its total wing area is 500 m 2. It
[17 Sep. 2020, Shift-I] is in level flight with a speed of 960 km/h.
(a) Reynold’s number (b) Bernoulli’s number Estimate the pressure difference between the
(c) Pascal’s number (d) Torricelli’s number lower and upper surfaces of the wings.
Sol. (a) [17 Sep. 2020, Shift-II]
A pure number which determines the type of (a) 4.5 × 103 Nm − 2 (b) 5.5 × 103 Nm − 2
flow of a liquid through a pipe is known as (c) 6.5 × 103 Nm − 2 (d) 7 .5 × 103 Nm − 2
Reynold’s number. Sol. (c)
(i) If R < 2100 , then flow is laminar. Mass of boeing air craft,
(ii) If 2100 < R < 4000, then flow is stream line. M = 3.3 × 105 kg
(iii) If R > 4000, then flow is turbulent. Wing’s area, A = 500 m 2
2. If a vessel containing a fluid of density ρ The weight of boeing aircraft is balanced by
upward force due to the pressure difference.
upto height h is accelerated vertically
Hence, ∆ p × A = mg
downwards with acceleration a 0. Then, the
mg 3.3 × 105 × 9.8
pressure by fluid at the bottom of vessel is ⇒ ∆p = =
given by the equation ……… (p 0 denotes the A 500
atmospheric pressure and g denotes the − 6.5 × 103 N/m 2
= 6.47 × 103 N/m 2 ~
acceleration due to gravity). 4. A weightless bag is filled with 5 kg of water
[17 Sep. 2020, Shift-I] and is weighed in water using a spring
(a) p = p0 + ρgh + ρha0 (b) p = p0 + ρgh balance. The reading of spring balance is
(c) p = p0 + ρh(g − a0 ) (d) p = p0 − ρgh
[17 Sep. 2020, Shift-II]
Sol. (c) (a) 5 kg (b) 2.5 kg (c) 1.25 kg (d) 0
When a vessel containing a fluid of density ρ
upto height h is accelerated downwards with Sol. (d)
acceleration a 0 , then net gravitational When water is placed in water, then its
acceleration acting on the fluid apparent weight is zero.
g′ = g − a 0 …(i) Hence, weight measured by spring balance of
Hence, the pressure by the fluid at the bottom of weightless bag filled with 5 kg in water is zero.
vessel is given as 5. The radius of the bore of a capillary tube is r
p = p0 + ρg ′ h = p0 + ρ (g − a0 ) h [From Eq. (i)] and the angle of contact of the liquid is θ.
When the tube is dipped in the liquid, the
Mechanical Properties of Fluids 83

radius of curvature of the meniscus of liquid A1 v1 = A2 v2 ⇒ πr12 v1 = πr22 v2


2 2
rising in the tube is [18 Sep. 2020, Shift-I] r   10−2 
r r ⇒ v2 =  1  v1 =   v1
(a) r sin θ (b) (c) r cos θ (d)  r2   2 × 10−2 
sin θ cos θ
v1
⇒ v2 = ⇒ v1 = 4v2
Sol. (d) 4
The given situation is shown in the figure,
8. Water is flowing through a horizontal pipe
in streamline flow. At the narrowest part of
the pipe [21 Sep. 2020, Shift-I]
(a) velocity is max and pressure is min
(b) pressure is max and velocity is min
R
(c) Both pressure and velocity are max
θ (d) Both velocity and pressure are min
r
Sol. (a)
According to Bernoulli’s theorem, for stream
line flow through horizontal pipe.
1
p + ρV 2 = constant …(i)
2
where, r = radius of capillary tube, where, p = pressure,
R = radius of meniscus v = velocity of liquid
and θ = angle of contact. and ρ = density of liquid.
r r Hence, according to principle of continuity, at
From figure, = cosθ ⇒ R = narrowest part of the pipe, velocity is maximum.
R cosθ
Hence from Eq. (i), pressure is minimum at
6. The coefficient of viscosity of an ideal fluid is narrowest part of the pipe.
[18 Sep. 2020, Shift-II]
9. A 0.5 kg block of brass (density = 8 × 10 3
(a) equal to 1 (b) zero
kg m − 3) is suspended from a string. What is
(c) infinite (d) greater than 1
the tension in the string if the block is
Sol. (b) completely immersed in water? (g = 10 ms −2)
The coefficient of viscosity of an ideal fluid is zero. [21 Sep. 2020, Shift-I]
7. Two water pipes of diameter 2 cm and 4 cm 0.5
(a) 5 N (b) N
are separately connected to a main supply 8 × 103
line. The velocity of flow of water in the pipe (c)
5
N (d)
35
N
of 2 cm diameter is [18 Sep. 2020, Shift-II] 8 8
(a) 4 times that in the other pipe Sol. (d)
1
(b) times that in the other pipe Mass of block, m = 0.5 kg
4
Density, ρ = 8 × 103 kg m −3
(c) 2 times that in the other pipe
1 Volume of block,
(d) times that in the other pipe m 0.5
2 V= = = 6.25 × 10−5 m 3
ρ 8 × 103
Sol. (a)
When block is fully immersed in water, then
Diameter of first pipe, d1 = 2 cm upthrust force on the block
∴ Radius, r1 = 1 cm = 10−2 cm F1 = mg = V.ρw × g
Diameter of second pipe,d2 = 4 cm = 6.25 × 10−5 × 103 × 10 = 0.625 N
∴ r2 = 2 cm = 2 × 10−2 m ∴Tension in the string
If v1 and v2 are the velocities of water in first T = mg − F1 (upthrust force)
and second pipe respectively, then according to 35
principle of continuity, = 0.5 × 10 − 0.625 = 4.375 = N
8
84 AP EAMCET Chapterwise Physics

10. Water flows through a hose pipe whose ∴ Pressure, p = ρ gh


internal diameter is 4 cm at a speed of = 13.6 × 103 × 9.8 × 10−3
1 ms −1 . If water has to emerge at a speed of = 133.28 Pa ~− 133.3 Pa
4 ms −1 , then the diameter of the nozzle 13. Find the apparent weight of a metallic block of
should be [21 Sep. 2020, Shift-II]
density 5 g cm −3 and dimensions 5 cm × 5 cm
(a) 1 cm (b) 2 cm (c) 4 cm (d) 0.5 cm × 5 cm, in water. [22 Sep. 2020, Shift-II]
Sol. (b) (a) 5 × 5 × 5 × 5 gf (b) 4 × 4 × 4 × 4 gf
Internal diameter for hose pipe, (c) 5 × 4 × 4 × 4 gf (d) 4 × 5 × 5 × 5 gf
d 4 Sol. (d)
d1 = 4 cm, r1 = = = 2 cm = 2 × 10−2 m
2 2 Density of metallic block, ρ = 5g cm −3
Speed of water through hose pipe, v1 = 1 ms −1 Volume of block,
Speed of water through nozzle, v2 = 4 ms −1 V = 5 cm × 5 cm × 5 cm
Diameter, d2 = ? Upthrust force on the metallic block in water,
According to principle of continuity, F1 = m ⋅ g = ρw V × g [ρw = 1 g cm −3 ]
A1 v1 = A2 v2 = Vg = 5 × 5 × 5 × g = 5 × 5 × 5 gf
⇒ πr12 v1 = πr22 v2 ∴ Apparent weight = Weight of block − Upthrust
r 2v (2 × 10−2)2 × 1 force
⇒ r22 = 1 1 ⇒ r22 =
v2 4 = mg − 5 × 5 × 5 gf = (m − 5 × 5 × 5) gf
⇒ r22 = 10−4 = (ρ V − 5 × 5 × 5) gf
⇒ r2 = 10−2 m ⇒ r2 = 1 cm = (5 × 5 × 5 × 5 − 5 × 5 × 5) gf
∴ Diameter of nozzle, d2 = 2r2 = 2 × 1 = 2 cm = 5 × 5 × 5 (5 − 1) gf = 5 × 5 × 5 × 4 gf

11. The terminal velocity v of a spherical ball of 14. Bottom of a cylindrical vessel has a hole of area
lead of radius R falling through a viscous liquid A. If water is filled up to a height h, it flows
varies with R such that [21 Sep. 2020, Shift-II] out in t seconds. If water is filled to a height
v 4 h, it will flow out in time [22 Sep. 2020, Shift-II]
(a) = constant (b) vR = constant
R t
(a) t (b) 4t (c) 2t (d)
v 4
(c) v = constant (d) 2 = constant
R
Sol. (c)
Sol. (d) The given situation is shown in the figure,
Terminal velocity of spherical ball falling
through viscous liquid is given as
2 R2 (ρ − σ) g
v= × …(i)
9 η
A
where, R = radius of ball, ρ = density of ball,
h
σ = density of liquid
and η = coefficient of viscosity.
From Eq. (i),
v 2(ρ − σ) g
= = constant Time taken to flow out of water upto height h
R2 9η from hole completely,
12. A pressure of 1 mm of mercury is equivalent to 2h
t= …(i)
[22 Sep. 2020, Shift-II] g
(a) 1 mPa (b) 13.33 Pa (c) 133.3 Pa (d) 1.08 Pa When, h′ = 4h, then
Sol. (c) 2h′ 2 × 4h 2h
t′ = = =2 = 2t
Height of mercury column h = 1 mm = 10−3 m g g g
. × 103 kg m −3
Density of mercury, ρ = 136 [from Eq. (i)]
−2
g = 9.8 ms
Mechanical Properties of Fluids 85

15. Two thin circular discs A and B of radii 2 cm 17. A cylindrical tank has a hole of area 2 cm 2 at
and 4 cm are in a liquid at the same depth. its bottom, If water is poured into the tank
T A is the thrust on A and TB thrust on B. from a tube above it at the rate of
Then, T A : TB [23 Sep. 2020, Shift-I] 100 cm 3 s −1 , then the maximum height upto
(a) 2 : 1 (b) 1 : 2 (c) 4 : 1 (d) 1 : 4 which water can rise in the tank is
(Acceleration due to gravity, g = 10 ms −2)
Sol. (d)
The thrust acting on the circular discs is [20 April 2019, Shift-II]
dependent on the area covered by the circular (a) 2.5 cm (b) 1.25 cm (c) 5.5. cm (d) 3.5 cm
disc and the density of the fluid. In this case, Sol. (b)
the radii of the two circular discs are given as 2
Given, area of hole in tank, A = 2 cm −2
cm and 4 cm and density of the liquid remains
the same. ⇒ A = 2 × 10−4 m −2
The area covered is given by πr 2 . Volume flow rate = 100 cm 2 /sec
2 = 100 × 10−6 m 3 /sec = 10−4 m 2 sec –1
TA  r A 
2
=   =   =
2 1
So, ratio of their thrust, At maximum height h, velocity of water flowing
TB  rB   4 4 through hole, v = 2gh
16. A long cylindrical glass vessel has a pinhole ∴From principle of continuity of flow of liquid,
of diameter 0.2 mm at its bottom. The depth volume flow rate = Av
to which the vessel can be lowered vertically 10−2 = Av = 2 × 10−4 2gh {Q v = 2gh}
in a deep water bath without the water 10−4
⇒10–4 = 2 × 10–4 2gh ⇒ = 2gh
entering into the vessel is 2 × 10–4
(surface tension of water, T = 0 .07 Nm −1 , 2
2gh ⇒   = 2gh
1 1
⇒ =
acceleration due to gravity, g = 10 ms −2) 2  2
[20 April 2019, Shift-I] 1 1 1
⇒ × = h ⇒ h= = 0.125m
(a) 14 cm (b) 7 cm 4 2g 80
(c) 21 cm (d) 28 cm ⇒ h = 1.25 cm
Sol. (a) 18. A liquid drop of density ρ is floating half
Given, diameter of a pinhole, d = 0.2 mm
immersed in a liquid of surface tension S and
∴ radius of pinhole,
ρ
r=
0.2
= 01
. mm = 01 . × 10−3 m
density . If the surface tension S of the
2
2
liquid is numerically equal to 10 times of
When the hydrostatic pressure is equal to the
excess pressure, then water cannot enter the acceleration due to gravity, then the
pinhole diameter of the drop is : [21 April 2019, Shift-I]
2T 20 80 60 40
Hence, hρg = (a) (b) (c) (d)
r ρ ρ ρ ρ
where, h = depth up to which cylinder is
immersed.
Sol. (b)
ρ = density of water A figure given here, shows a liquid drop half
immersed in a liquid,
T = surface tension of water
and r = radius of a pinhole P0 P0
2T 2 × 0.07
∴ h= = A
ρgr 103 × 10 × 01 . × 10−3 x B
= 014
. m = 14 cm O′ O
Hence, the depth to which the vessel can be
lowered vertically in a deep water bath without the
water entering into the vessel is 14 cm.
86 AP EAMCET Chapterwise Physics

where density of drop, ρd = ρ, density of liquid, RI ∝


1

1
…(iii)
ρ (rI )4 16 × 16 × 10−12
ρL = and surface tension, T = 10 g
2 1 1
.
RII ∝ ∝ …(iv)
(rII )4 16 × 10−12
From Eqs. (iii) and (iv), we get
RII = 16RI
If RI = R then RII = 16R
Hence, the pressure difference between ends of
p p
the first tube, ×R=
R + 16R 17
Force acting on the drop due to surface tension,
F = Tl = 10g(πD) …(i) 20. A rain drop of radius r is falling through air,
The observed weight of the drop, starting from rest. The work done by all the
ρV 3 forces on the drop, when it attains terminal
w = wd − wL = ρVg − g = ρVg … (ii)
2 2 4 velocity, is proportional to [22 April 2019, Shift-I]
4 (a) r 3 (b) r 7 (c) r 5 (d) r 4
where, V = volume of the drop = πr 3
3 Sol. (b)
Since, the ball is in equilibrium. Given, radius of rain drop = r
∴ F=w Since, rain drop starts falling from rest, hence
3 π D3 its initial speed, u = 0
10 g π D = ρg
4 6 Final velocity of rain drop is equal to the
80 terminal velocity v, which is given by
D= m [From Eqs. (i) and (ii)]
ρ 2gr 2 (ρ − σ)
v=
Hence, the correct option is (b). 9η
where, ρ → density of the rain drop
19. Two capillary tubes of same length each of
50 cm but of different radii 4 mm and 2 mm σ → density of air
are connected in series. When water flows, η → coefficient of viscosity
the pressure difference between the ends of ∴According to the work-energy theorem,
the first tube is [21 April 2019, Shift-II] work done by all the forces on the drop
p p p p = change in its kinetic energy
(a) (b) (c) (d)
1 1 1
2 17 4 8 W = mv2 − mu2 = mv2 [Q u = 0]
2 2 2
Sol. (b) 2
1 4 3  2gr 2 (ρ − σ)  Q ρ = m 
According to the question, = × π r .ρ  
2 3  9η   V 
I
II 8 πρg2 (ρ − σ)2 r 7
k= ⇒ W = kr 7 or W ∝ r 7
243 η2

21. A sphere of radius R has a concentric


50 cm 50 cm
spherical cavity of radius r. The relative
density of the material of the sphere is σ. It
According to poiseuille’s equation, water
8ηl just floats when placed in tank full of water.
resistance, R = …(i) R
πr 4 The value of is [22 April 2019, Shift-II]
1 r
or R∝ 4 …(ii) 1 1
r  σ 3 σ − 1 3
Given, radius of first capillary tube, rI = 4 mm (a)   (b)  
= 4 × 10−3 m and radius of second capillary tube,  σ − 1  σ 
rII = 2 mm = 2 × 10–3 m 1 1
 σ 2 σ − 1 2
From Eq. (ii), we get (c)   (d)  
 σ − 1  σ 
Mechanical Properties of Fluids 87
P0 P0
Sol. (a)
Buoyancy force due to water, on a sphere of
A
radius R, x B
Fw = ρw gVs O′ O
where, ρw = density of water
Vs = volume of sphere
Fw = ρw g πR3  Q V = 4 πR3 
4
∴  
3   s
3 
weight of the sphere, Given, radius of first tube, R1 = 2 mm and radius
of second tube, R2 = 4 mm.
Fs = ρs g π(R3 − r 3)
4
3  density of water, ρ = 1.0 × 103 kg-m −3
where ρs = density of material of sphere Since, the tube is open at both the ends.
According to Archimedes’ principle for floating, So, p A = pB = p0 (atmospheric pressure)
Pressure in capillary tube B,
∴ Fw = Fs
2T
pB = = p0
⇒ ρw g πR3  = ρs g π(R3 − r 3)
4 4
R2
3  3 
[Here, T = Surface tension of water]
⇒ ρw R3 = ρs (R3 − r 3)
Similarly pressure at capillary tube A,
⇒ R3 (ρs − ρw ) = r 3ρs 2T
1 /3 pA = = p0
  R1
R  1 
1 /3
R  ρs 
⇒ =  or =   Hence, for same height OO′,
r  ρs − ρw  r  1 − ρw  2T 2T 2T  1 1 
  −ρgx + = ⇒ x= − 
 ρs 
R1 R2 ρg  R1 R2 
Q Relative density of material of the sphere,
ρ Putting the given values, we get
σ= s 2 × 7.3 × 10−2 1 1 
ρw x= 3 − = 365
. mm
1 /3 10 × 10 × 10−3  2 4 
 
R  1 
1 /3
 σ  Hence, the difference between the levels of
or =  =  water in the two limbs of the tube is 3.65 mm.
r 1 − 1   σ − 1
 So, the correct option is (a).
σ
23. A body is suspended by a light string. The
22. Two tubes of same length and diameters tensions in the string when the body is in
4 mm and 8 mm are joined together to form
air, when the body is totally immersed in
a U-shaped tube open at both the ends. If
water and when the body is totally
the U-tube contains water, then the
immersed in a liquid are respectively 40.2N,
difference between the levels of water in the
28.4N and 16.6N. The density of the liquid is
two limbs of the tube is
[22 April 2018, Shift-I]
(Surface tension of water at the temperature (a) 1200 kg - m−3 (b) 1600 kg - m−3
of experiment is 7 .3 × 10 −2 Nm −1 , angle of (c) 2000 kg - m−3 (d) 2400 kg - m−3
contact = 0 °, density of water = 10
. × 10 3 kg m
−3
Sol. (c)
and acceleration due to gravity = 10 ms −2) Density of water−Density of air ∝ Twater − Tair
[23 April 2019, Shift-I] Density of liquid−Density of water ∝
Tliquid − Twater
(a) 3.65 mm (b) 36.5 mm
Now, from given values,
(c) 0.365 mm (d) 365 mm Twater − Tair = Tliquid − Twater
Sol. (a) Air density = 1 kg / m 3
A figure of U-shaped tube of different diameter and water density = 1000 kg / m 3
is given below, So, liquid density ≈ 2000 kg / m 3
88 AP EAMCET Chapterwise Physics

24. A spherical body of density ρ is floating half Sol. (b)


immersed in a liquid of density d . If σ is the
surface tension of the liquid, then the (b–h)
diameter of the body is [22 April 2018, Shift-II]
3σ 6σ b
(a) (b)
g (2ρ − d ) g (2ρ − d ) h

4σ 12 σ
(c) (d)
g (2ρ − d ) g (2ρ − d ) a O

Sol. (a) Force on dam = Pressure of water × Area


Buoyant force = {(density of water) (acceleration due to
gravity) (height of water/2} × {(width of dam)
Force of surface tension (height of water)}
F =  ρg  × [(b − h) ⋅ h]
h
 2
ρg
= (bh2 − h3)
Weight 2
At equilibrium,
Weight of body atmospheric pressure = pressure on dam
= Buoyant force + Force of surface tension dF
So, =0
4 3 2
πr ρ × g = πr 3 dg + 2πrσ dh
3 3 ∂ ρg 3 
2 3 ⇒  (bh − h ) = 0
2

πr g(2ρ − d) = 2πrσ ∂h  2 
3
ρg
3σ ⇒ (2bh − 3h ) = 0
2
So, r2 = 2
g(2ρ − d)
⇒ 2bh − 3h2 = 0

So, r= ⇒ 2b = 3h
g(2ρ − d)
⇒ b = 3h / 2
25. Water stands upto height h behind the dam b− h=
3h
− h= h/ 2
as shown in the figure. The front view of the 2
dam gate is also shown in the adjoining is the location where the total weight of water
figure. Density of water is ρ and acceleration acts at a particular point.
due to gravity is g. If atmospheric pressure To find the point of action of total force,
force is also considered, then the point of I
yR = xc + yc
application of total force acting on the dam yc A
due to water above O is [23 April 2018, Shift-I]
where, yR = location where point of force acts,
A = area,
yc = location where total weight acts = h / 2
and I xc = moment of inertia
(here rectangular plate).
h
1 1
= Ah2 = bh3
12 12
O a (1 / 12) bh3 1
Side view of Front view of So, yR = + h
(1 / 2)h⋅ bh 2
dam gate dam gate
1 1 2
h h h = h+ h= h (from the top)
(a) (b) (c) h (d) 6 2 3
4 3 2
Mechanical Properties of Fluids 89
h 2 When system achieves terminal velocity,
So, yR = (above the base) = h (from the top)
3 3 Fnet = 0
4
⇒ πρ f g (rΑ3 + rΒ3) + 6 πηv(rA + rB) = (m A + mB) g
3
With values, we get v = 4 cms −1
h
27. When a soap bubble of radius 0.2 mm is
FR
h charged, it experiences an outward
3 σ2
electrostatic pressure of magnitude ,
2ε0
b
rh where σ = 20 µCm −2 is the surface charge
density. If the excess pressure inside the
26. Two solid sphere of radii 2 mm and 4 mm soap bubble due to the surface tension is
are tied to the two ends of a light string and same as this electrostatic pressure, then the
surface tension of the soap solution is
released in a liquid of specific gravity 1.3 and
coefficient of viscosity 1 Pa-s. The string is (ε 0 = 8 .85 × 10 −12C 2N −1 m −2) [24 April 2018,
just taut, when the two spheres are Shift-I]
completely in the liquid. If the density of the (a) 8.85 × 10−4 Nm−1 (b) 12 .4 × 10−4 Nm−1
materials of the two sphere is 2800 kgm −3, (c) 11.3 × 10−4 Nm−1 (d) 90 × 10−4 Nm−1
then the terminal velocity of the system of Sol. (c)
the sphere is Excess pressure inside soap bubble due to
(take, g = 10 ms −2) [23 April 2018, Shift-II] surface tension,
(a) 2 cms −1 (b) 4 cms −1 p =   , where S = surface tension and
4S
(c) 4 ms −1 (d) 2 ms −1  R
R = radius.
Sol. (b)
σ2
Free body diagram of system is ⇒ Electrostatic pressure, p =
2ε0
F=4/3 πrA3 ρf g+6πηrAv According to the question,
4S σ 2
A =
R 2ε0
mA g σ 2R
S= .........(i)
F=4/3 πrB3 ρf g+6πηrBv 8ε0
Putting all values in Eq. (i), we get
B
(20 × 10− 6)2 × (0. 2 × 10− 3)
S=
(8 × 8. 85 × 10− 12)
mB g
⇒ S = 11. 3 × 10− 4 N / m
9
Thermal Properties
of Matter
1. 5 kg of water at 20°C is added to 10 kg of Heat lost by steam,
water at 60°C. Neglecting heat capacity of Q′ = mL f + ms c ∆t
vessel and other losses, the resultant = 103 × 540 + 103 × 1 × (150 − 90)
temperature will be nearly [17 Sep. 2020, Shift-I] = 54 × 104 + 6 × 104
(a) 35°C (b) 40°C (c) 47°C (d) 28°C = 104 (54 + 6) = 60 × 104 cal
Heat gained by 20 L water,
Sol. (c) Q′ ′ = mw × c × ∆T
If T be the resultant temperature, then heat
Mass of 20 L water, mw = 20 kg = 20 × 103 g
gained by 5 kg (at 20°C) water
∴ Q′ ′ = 20 × 103 × 1 × ∆T
Hgain = mc (T − 20) (Q given, m = 5 kg)
By the principle calorimetry,
⇒ Hgain = 5c(T − 20) …(i)
Heat lost = Heat gained
Heat lost by 10 kg water at 60°C is given as
60 × 104 = 20 × 103 × ∆T
Hloss = 10c (60 − T) …(ii)
60 × 104
By the principle of calorimetry ⇒ ∆T = = 30° C
20 × 103
Hgain = Hloss
⇒ 5c (T − 20) = 10c(60 − T) 3. When 50 g of water at 10°C is mixed with
⇒ T − 20 = 120 − 2T 50 g of water at 100°C. The resultant
⇒ 3T = 140 temperature is [17 Sep. 2020, Shift-I]
T = 46.67° C ~− 47° C (a) 80°C (b) 55°C (c) 25°C (d) 45°C
2. 1 kg of steam at 150°C is passed from a Sol. (b)
steam chamber is to a copper coil immersed If resulting temperature of the mixture is T°C,
in 20 L of water. The steam condenses in the then
coil and is returned to the steam chamber as Heat gained by 50 g water at 10°C,
water at 90°C. Latent heat of steam is Hgain = mc ∆T = 50 c(T − 10)
540 cal g −1 , specific heat of the steam is
Heat lost by 50 g of water at 100°C
1 cal and g −1 °C −1 . Then, the rise in
temperature of water is [17 Sep. 2020, Shift-I]
Hloss = mc ∆T = 50 c (100 − T)
(a) 75°C (b) 60°C (c) 30°C (d) 20°C According to principle of calorimetry,
Heat gain = Heat loss
Sol. (c)
⇒ Hgain = Hloss
Mass of steam, ms = 1 kg = 1000 g = 10 3

Temperature of steam, T1 = 150°C ⇒ 50c(T − 10) = 50 c (100 − T)


Latent heat of steam, ⇒ 2T = 110 ⇒ T = 55°C
Ls = 540 cal g −1 ⇒ c = 1 cal g −1 °C −1
Thermal Properties of Matter 91

4. A calorimeter contains 0.5 kg of water at = Bulk modulus × Volumetric strain


∆V
30°C. When 0.3 kg of water at 60°C is added =B× = B × γ ⋅ ∆t [Q ∆V = Vγ ∆t]
to it, the resulting temperature is found to be V
40°C. The water equivalent of the calorimeter = B × 3α × ∆t [Q γ = 3α]
is [17 Sep. 2020, Shift-I] = 3Bα ∆t = 3B α (t2 − t1 )
(a) 0.25 kg (b) 0.1 kg (c) 0.2 kg (d) 0.25 kg = 3 × 140 × 109 × 1.7 × 10−5 (57 − 37)
= 1.4 × 108 Nm −2
Sol. (b)
Temperature of cold water in calorimeter, 7. If 60% of the kinetic energy of water falling
T1 = 30°C, m1 = 0.5 kg = 500 g from 210 m high water fall is converted into
Temperature of hot water, T2 = 60°C, heat. The raise in temperature of water at
m2 = 0.3 kg = 300g the bottom of the falls is nearly (specific
Resulting temperature, T3 = 40° C heat of water = 4.2 × 10 3 J kg − 1K − 1 )
Let water equivalent of the calorimeter is Wgram . [18 Sep. 2020, Shift-I]
According to principle of calorimetry. (a) 0.6 °C (b) 0.3 °C
Heat lost by warm water = Heat gained by cold (c) 1.2 K (d) 2.4 K
water + Heat gained by the calorimeter Sol. (b)
⇒ m2 (T2 − T3) = m1 (T3 − T1 ) + W (T3 − T1 ) Given, h = 210 m
⇒ 300(60 − 40) = 500(40 − 30) + W (40 − 30) Specific heat of water, c = 4.2 × 103 J kg −1 K −1
⇒ 300 × 20 = 500 × 10 + 10W When water falls on the surface of earth, then
⇒ 6000 = 5000 + 10W its potential energy is converted into kinetic
⇒ 10W = 6000 − 5000 ⇒ 10W = 1000 energy.
100 ∴Kinetic energy = Potential energy
⇒ W = 100 g = = 01
. kg
1000 KE = mgh …(i)
According to question,
5. The triple point of water is [17 Sep. 2020, Shift-II] Heat produced = 60% of KE
(a) 273.16 °C (b) 273.16 K 60
(c) 273.16 °F (d) 0.15 K ⇒ mc∆T = × mgh [from Eq. (i)]
100
Sol. (b) 0.6 gh 0.6 × 10 × 210
∆T = = = 0.3° C
In thermodynamics, the triple point of water is c 4.2 × 103
that temperature and pressure at which water
can exist in equilibrium in the liquid, solid and 8. For a perfect black body, the absorption
gaseous states. The triple point of pure water is coefficient is [18 Sep. 2020, Shift-I]
temperature of 273.16 K and pressure of 611.2
(a) a = 1 (b) a < 1 (c) a > 1 (d) a = 0
Pa.
Sol. (a)
6. Find the stress developed inside a tooth A perfectly black body absorbs 100% of
cavity filled with copper when hot tea at radiation incident on it, hence absorption
temperature of 57°C is drunk. You can take coefficient (a r) is given as
body (tooth) temperature to be 37°C and amount of absorb radiation (Qa)
ar =
α = 1.7 × 10 − 5 °C − 1 , bulk modulus for copper amount of incident radiation (Qi)
B = 140 × 10 9 Nm − 2. [17 Sep. 2020, Shift-II] Q Q
= a = a [Q Qa = Qi ]
(a) 1.4 × 108 Nm −2 (b) 1.9 × 108 Nm −2 Qi Qa
(c) 2.0 × 108 Nm −2 (d) 3.4 × 107 Nm −2 =1
Sol. (a)
9. 300 g of water at 25 °C is added to 100 g of
Temperature of hot tea, t2 = 57°C
ice at 0°C. The final temperature of mixture
Normal temperature of tooth, t1 = 37°C will be [18 Sep. 2020, Shift-II]
α = 1.7 × 10−5 C −1
(a) 25 °C (b) 0 °C
Bulk modulus, B = 140 × 109 Nm −2 (c) 12.5 °C (d) 30 °C
Thermal stress in tooth cavity
92 AP EAMCET Chapterwise Physics

Sol. (b) 11. Wein’s displacement law states


Given, mass of water, mw = 300g
[21 Sep. 2020, Shift-I]
Temperature of water, Tw = 25° C
λ
Mass of ice, mi = 100 g, Ti = 0° C (a) λ mT = constant (b) m = constant
Heat required to convert the ice T
T
at 0°C to water at 0°C (c) = constant (d) λ m + T = constant
λm
Q1 = mi Li = 100 × 80 [Li = 80 cal / g]
= 8000 cal …(i) Sol. (a)
Heat given by water from 25°C water to 0°C According to Wien’s displacement law, the
water wavelength (λ m) corresponding to which the
Q2 = mw c∆T = 300 × 1 × (25 − 0) energy emitted by a black body maximum is
inversely proportional to its absolute
⇒ = 7500 cal …(ii)
temperature (T).
From Eqs. (i) and (ii), we observed that 1 b
Q2 < Q1 i.e., λ m ∝ ⇒ λ m = ⇒ λm ⋅ T = b
T T
Hence, total ice will not be melt, so final [where, b = Wien’s constant]
temperature of mixture will be 0°C with some
unmelted ice in mixture. 12. The rate of radiation of a black body at 0°C is
10. The temperature of a body is measured both E Js − 1 . The rate of radiation of the black
in °C and °F. A graph is plotted with °F on body at 273°C will be [21 Sep. 2020, Shift-I]
E −1
X-axis and °C on Y-axis. Then, the cosine of (a) E Js − 1 (b) 4 E Js − 1 (c) Js (d) 16 E Js − 1
angle between the graph and the X-axis is 2

[18 Sep. 2020, Shift-II]


Sol. (d)
9 5 9 Given, T1 = 0° C = (0 + 273)K = 273 K
(a) 0 (b) (c) (d)
5 106 106 T2 = 273° C = (273 + 273)K = 546 K
According to Stefan-Boltzmann’s law,
Sol. (d)
rate of radiation,
The relation between celsius scale temperature
E ∝ T4
and fahrenheit scale temperature is given as 4 4
E 2  T2  546 
C F − 32
= ⇒ 9C = 5F − 160 ∴ =   =   = (2) = 16
4

E1  T1   273
5 9
5 160 ⇒ E 2 = 16E1 = 16E Js −1 [Q E1 = E Js−1 ]
⇒ C = F− …(i)
9 9
13. A steel wire of length 20 cm and area of
The graph between C and F is shown in the figure
cross- section 1 mm 2 is tied rigidly at both
C
the ends. When the temperature of the wire
is changed from 40°C to 20°C, find the change
θ
F in its tension. Given, the coefficient of linear
expansion for steel is 1.1 ×10 − 5 °C − 1 and
Young’s modulus of steel is 2.0 × 10 11 Nm −2.
Comparing Eq. (i) with line y = mx + C, we get [21 Sep. 2020, Shift-I]
5
Slope of graph, m = tanθ = (a) 22 N (b) 44 N (c) 16 N (d) 8 N
9
9 Sol. (b)
∴ cosθ =
106 Given, Young’s modulus of steel wire,
Y = 2 × 1011 Nm −2
Length of wire, l = 20 cm = 0.2 m
√106 5 Area of cross-section, A = 1 mm 2 = 10−6 m 2
Change in temperature of wire,
θ ∆t = (40 − 20)° C = 20° C
9 α = 1.1 × 10−5 ° C−1
Thermal Properties of Matter 93

∴Change in tension in the steel wire, 8 4


A2 = cm × cm
T = Y A α ∆t 2 2
= 2 × 1011 × 10−6 × 11
. × 10−5 × 20 = 44 N = 4 × 2 cm 2 = 8 × 10−4 m 2
T2 = 327 + 273 = 600 K
14. A body at 3000 K emits maximum energy at Putting these values in Eq. (i), we have
a wavelength of 9660 Å. If the sun emits 4
E 2  8 × 10−4   600  1 81
maximum energy at a wavelength of 4950 Å, = 
−3 
  = ×
E1  . × 10
32  400  4 16
what would be the temperature of the sun ?
E 2 81 81 81
[21 Sep. 2020, Shift-II] ⇒ = ⇒ E2 = E1 =   E Js −1

E1 64 64  64 
(a) 5855 K (b) 7000 K
(c) 4250 K (d) 8000 K [Q E1 = E Js−1 ]

Sol. (a) 16. When a body is heated, then maximum rise


Temperature of body, T1 = 3000 K will be in its [22 Sep. 2020, Shift-I]
Body emits maximum energy at wavelength, (a) length (b) surface area
λ1 = 9660 Å (c) volume (d) density
Wavelength of radiation emitted by sun, Sol. (b)
λ 2 = 4950 Å The ratio of coefficients of linear, surface and
Sun's temperature, T2 = ? volume expansions are given as
According to Wien’s displacement law, α : β : γ = 1 : 2: 3
1 Hence, we see that coefficient of volume
λ∝
T expansion is maximum.
b Therefore, when a body is heated, then
⇒ λ = [b = Constant of proportionality]
T maximum rise will be in its volume.
λ1 T2
⇒ = 17. Two blocks of ice when pressed together join
λ 2 T1 to form one block because [22 Sep. 2020, Shift-I]
Tλ 3000 × 9660
⇒ T2 = 1 1 = (a) of heat produced during pressing
λ2 4950 (b) of cold produced during pressing
= 5854.5 K ≈ 5855 K (c) melting point of ice decreases with increase of
pressure
15. A rectangular metal plate 8 cm × 4 cm at (d) melting point of ice increases with increase in
127°C emits E Js −1 . If both length and pressure
breadth are halved and the temperature is
Sol. (c)
raised to 327°C, the rate of emission is
When two blocks of ice are pressed against each
[21 Sep. 2020, Shift-II] other, then the pressure applied on the blocks of
ice increases and the melting point decreases.
(a)   E Js −1 (b)   E Js −1
9 81
 4  64  Thus, two blocks of ice fuse together and form
one block.
(c)   E Js −1 (d)   E Js −1
27 10
 8 7
18. Cylindrical rod of copper of length 2 m and
Sol. (b) cross-sectional area 2 cm 2 is insulated at its
According to Stefan-Boltzmann’s law, rate of curved surface. The one end of rod is
emission of radiation from metal surface is maintained in steam chamber and other is
given as, maintained in ice at 0°C. The thermal
E = σAT 4 conductivity of copper is 386 Js −1 m −1 °C −1 ).
4 Find the temperature at a point which is at a
E 2  A2   T2 
⇒ =    …(i) distance of 120 cm from the colder end.
E1  A1   T1 
[22 Sep. 2020, Shift-II]
Given, A1 = 8 cm × 4 cm
(a) 80°C (b) 50°C
. × 10−3 m 2
= 32
(c) 60°C (d) 70°C
T1 = 127 + 273 = 400 K
94 AP EAMCET Chapterwise Physics

Sol. (c) Sol. (d)


For cylindrical wire, Given,
l = 2m, A = 2cm 2 = 2 × 10−4 m 2 length of rod, l = 10 cm = 01 . m
θ1 = 100° C, θ2 = 0°C area of cross-section of rod, A = 2.8 × 10−4 m 2
θ=? temperatuer at one end, T1 = 80° C
At steady state, temperature gradient of rod temperature at other end, T2 = 0°C
remains constant. quantity of melted ice, m = 20 gm
time taken to melt ice, t = 5 min = 300 sec
θ1=100ºC θ θ2=0ºC
and latent heat of ice, s = 80 cal g −1
m × s × 4184
.
x1=80 cm x2=120 cm Now, rate of the heat flow =
t
θ1 − θ θ − θ2 20 × 80 × 4184
.
i.e., = = = 22.314 J/s
x1 x2 300
kA.∆T
100° − θ θ − 0° Rate of heat flow in the rod, ∆Q =
⇒ = l
80 × 10−2 120 × 10−2
k(2.8 × 10−4 ) × 80
80 22.314 =
⇒ 100° − θ = ×θ 01.
120
2 5θ ∴ k = 99.61 ≈ 100 Js −1 m −1 K −1
⇒ 100°− θ = θ ⇒ = 100°
3 3 21. The densities of wood and benzene at 0°C are
100° × 3 880 kg m −3 and 900 kg m −3, respectively. The
⇒ θ= = 60° C
5 coefficient of volume expansion is
19. The wavelength of maximum emitted energy . × 10 −3C−1 ° for wood and 15
12 . × 10 −3°C −1 for
(λ m) of a body at 700 K is 4.08 µm. If the benzene. Then the temperature at which a
temperature of the body is raised to 1400 K, piece of wood just sinks in benzene is
then the value of λ m will be [20 April 2019, Shift-II]
[22 Sep. 2020, Shift-II] (a) 88°C (b) 90°C
(a) 1.02 µm (b) 16.32 µm (c) 83.3°C (d) 90.3°C
(c) 8.16 µm (d) 2.04 µm Sol. (c)
Sol. (d) Given, density of wood at 0°C, ρw = 880 Kg m −3
Given, λ m1 = 4.08 µm, T1 = 700 K, T2 = 1400 K density of benzene at 0°C, ρb = 900 kg m −3
λ m2 = λ m = ? coefficient of volume expansion of wood,
According to Wien’s displacement law, γ w = 1.2 × 10−3 ° C −1
1 λ m2 T coefficient of volume expansion of benzene,
λm ∝ ⇒ = 1
T λ m1 T2 γ b = 1.5 × 10−3 °C −1
T1 λ m1 700 × 4.08 and initial temperature, T1 = 0° C
⇒ λ m2 = = = 2.04 µm Let T2 be the temperature at which pieces of
T2 1400
wood will just sink in benzene and ∆T = T2 − T1
∴ λ m = λ m2 = 2.04 µm The piece of wood begins to sink when it weight
20. A metal rod of length 10 cm and area of is equal to the weight of benzene displaced.
cross-section 2.8 × 10 −4 m 2 is covered with a Mass = Volume × Density
Therefore, V ρw g = V ρb g
non-conducting substance. One end of it is
ρw ρb
maintained at 80°C, while the other end is ∴ =
put in ice at 0°C. It is found that 20 gm of ice 1 + γ w ∆T 1 + γ b ∆T
melts in 5 min. The thermal conductivity of 880 900
=
the metal in Js −1 m −1 K −1 is (Latent heat of 1 + 1.2 × 10−3 ∆T 1 + 1.5 × 10−3 ∆Τ
ice is 80 cal g −1 .) [20 April 2019, Shift-I] 880 + 880 × 1.5 × 10−3 ∆T = 900 + 900 × 1.2 × 10−3 ∆T
(a) 70 (b) 80 (c) 90 (d) 100 (1320 − 1080) × 10−3 ∆T = 20
Thermal Properties of Matter 95
20 T T
∆T = = 833
. °C
240 × 10−3
T2 − T1 = 833
. °C (a) (b)
Hence, T2 = 833
. °C
t t
22. A window used to thermally insulate a room T T
from outside consists of two parallel glass
sheets each of area 2.6 m 2 and thickness (c) (d)
1 cm separated by 5 cm thick stagnant air. In
the steady state, the room glass interface is
at 18°C and the glass-outdoor interface is at t t
−2 °C. If the thermal conductivities of glass
and air are respectively 0.8 Wm −1 K −1 and Sol. (b)
0.08 Wm −1 K −1 , the rate of flow of heat Key Idea Newton’s law of cooling is given by
through the window is [20 April 2019, Shift-II] expression,
dT
(a) 15 W (b) 40 W (c) 60 W (d) 80 W − = k′(T − T0)
dt
Sol. (d) k
where, k′ = and negative sign shows the rate of
According to the question, room-glass interface ms
and glass-outdoor interface are connected in heat loss.
series combination so, the equivalent thermal The given expression can be rearranged by
resistance, integrating as,
Req = R1 + R2 + R3 dT
l l l ∫ T − T0 = − k′ ∫ dt
Q R1 = 1 , R2 = 2 , R3 = 3
k1 A1 k2 A2 k3 A3 log e (T − T0) = − k′ t + log e A
Where, k1 and k3 are thermal conductivity of (Q log e A = Constant)
glass and k2 is thermal conductivity of air. dT
− = k′ dt
l
or Req = 1 + 2 + 3
l l (T − T0)
k1 A1 k2 A2 k3 A3 ⇒ ln(T − T0) = − k′ t
Given, l1 = l 3 = 1 cm = 10−2 m, ⇒ T = e − k ′ t + T0 at t → ∞
l 2 = 5 cm = 5 × 10−2 m ⇒ T = T0 , at t = 0 ⇒ T → ∞
k1 = k3 = 0.8 Wm −1 K −1 , Hence, the graph as shown below, shows
k2 = 0.08 Wm −1 K −1 temperature of a body (done) varies
and A1 = A2 = A3 = 2.6 m 2 exponentially with time from T to T0 (T0 < T).
Putting the given values, we get
T
10−2 5 × 10−2 10−2
+ +
0.8 × 2.6 0.08 × 2.6 0.8 × 2.6
 10−2   52  1 T0
=   = t
 2.6   0.8  4
Hence, flow of heat, Thus, the correct option is (b).
∆ T [18 − (−2)]
H= = = 80 W
Req 1  24. A solid copper sphere of density ρ, specific
 
 4 heat capacity C and radius r is initially at
200K. It is suspended inside a chamber
23. A block of metal is heated to a temperature whose walls are at 0 K. The time required (in
much higher than the room temperature and µ s) for the temperature of the sphere to drop
placed in an evacuated cavity. The curve to 100 K is
which correctly represents the rate of cooling
(T is temperature of the block and t is the (σ is Stefan’s constant and all the quantities
time.) [21 April 2019, Shift-I] are in SI units.) [21 April 2019, Shift-I]
96 AP EAMCET Chapterwise Physics

rρC 1 rρC
(a) 48 (b) 26. A liquid of mass 250 g is kept warm in a
σ 48 σ vessel using an electric heater. The liquid is
27 rρC 7 rρC
(c) (d) maintained at 57°C when the power supplied
7 σ 27 σ
by the heater is 30 W and surrounding
Sol. (b) temperature is 27°C. As the heater is switched
Here, T = 200K and T0 = 0K off, it took 10 s time for the temperature of
As the rate of fall of temperature, the liquid to fall from 47°C to 46.9°C. The
∆T σAe (T 4 − T04 ) specific heat capacity of the liquid is
=
∆t ms [21 April 2019, Shift-II]
where, σ = Stefan’s constant, A = area of (a) 8000 J kg −1K −1 (b) 9000 J kg −1K −1
sphere, and e = emissivity = 1 and S = specific (c) 6000 J kg −1K −1 (d) 12000 J kg −1K −1
heat capacity.
ms∆T Sol. (d)
So, t= (Q T0 = 0K) dQ
σA(T 4 − T04 ) Given, power supplied by heater, = 30W
(ρV) C(200K − 100K) dt
⇒ t= 250
σ(A) (2004 − 04 ) mass of liquid, m = = 250 × 10−3 kg
1000
4
ρ πr 3 C × 100 Q Rate of heat flow,
dQ msdT
=
⇒ t= 3 2 dt dt
σ 4 πr × (200)4 putting the given values, we get
1 rρC 1 ρrC 250 × 10−3 s × (47 − 46.9)
⇒ t= × 10− 6 s = µs ⇒ 30 =
48 σ 48 σ 10
Hence, the correct option is (b). Specific heat capacity of the liquid,
30 × 10
25. A hammer of mass 200 kg strikes a steel s=
250 × 10−3 × (01.)
block of mass 200 g with a velocity 8 ms − 1 . If
23% of the energy is utilized to heat the steel = 12000 J Kg −1 K −1
block, the rise in temperature of the block is 27. The amount of heat that must be supplied to
(specific heat capacity of steel, 35 g of oxygen at room temperature to raise
= 460 J kg − 1 K − 1 ) [21 April 2019, Shift-I] its temperature by 80°C at constant volume is
(a) 8 K (b) 16 K (c) 12 K (d) 24 K (Molecular mass of oxygen is 32 and
Sol. (b) R = 8 .3 J mol −1 K −1 ) [21 April 2019, Shift-II]
Given, mass of hammer, m = 200 kg, (a) 2.84 kJ (b) 1.68 kJ
steel block of the mass = 200 g = 0.2 kg and (c) 1.81 kJ (d) 2.88 kJ
specific heat capacity of steel, s = 460 J kg −1 K –1
Velocity of hammer, v = 8 ms−1
Sol. (c)
As we know that, Given, weight of oxygen = 35g, molecular
1 weight of oxygen = 32 , change in temperature
Kinetic energy, KE = mv2
2 dT = 80° C and R = 8.3J mol –1 K −1
Putting the given values, we get Heat supplied at constant volume, Q = nCv dT
1
= × 200 × 82 = 6400 J Qn =
weight of oxygen
=
35
2 gram molecular weight of oxygen 32
Hence, the 23% of this energy is converted to
heat. QO 2 is a diatomic gas therefore, the specific
6400 × 23 heat capacity at constant volume,
⇒ H= = 1472 J
100 5
Cv = R
The rise in temperature of steel, 2
H 1472 35 5 35 5
∆T = = = 16 K ∴ Q =    R (80) =     × 8.3 × 80

ms 460 × 0.2  32  2   32  2
Hence, the rise in temperature is 16 K. ⇒ Q = 1815625
. J or Q = 1.81 kJ
Thermal Properties of Matter 97

28. Two metallic spheres P and Q are made of


same material have same smoothness but
the weight of P is 8 times that of Q. If the r1
two are heated to same temperature and left r2
to cool, then the ratio of rate of cooling of Q T1
to that of P is [22 April 2019, Shift-I] T2
(a) 4 (b) 8
(c) 2 (d) 1 r  r2 − r1 r1 r2
(a) r2 − r1 (b) ln  2  (c) (d)
Sol. (c)  r1  r1 r2 r2 − r1
Given, weight of material P is 8 times the
weight of material Q.
Sol. (d)
i.e., mp = 8 mQ Q Radial rate flow of heat in a substance,
dQ dθ
From stefan’s law, the rate of heat radiation = k4 πr 2 ⋅ = c (say)
from an object is given by dt dr
∆Q Q dQ = kA dT 
= eσAT 4 …(i)  
 dt dr 
∆t
where, e = emissivity, σ = Stefan’s constant where, A = area of surface
A = area and T = Temperature and k = thermal conductivity.
But, ∆Q = mc∆T ...(ii) c
or dr = k dθ
∴From Eqs. (i) and (ii), we get 4 πr 2
r2 dr
mc∆T
T2
= eσAT 4 or c∫ = k∫ dθ
r1 4 πr 2
∆t
T1

c  − 1 2
r
∆T eσAT 4 ⇒   = k(θ)T21
T
= ...(iii)
∆t mc 4 π  r  r1
4 c 1 1
But, m =` πr 3 . σ ⇒  −  = k(T2 − T1 )
3 4 π  r1 r2 
1
3m  3
∴ r =   ⇒ c = 4 πk
(T2 − T1 )
 4 πσ   r2 − r1 
2  
3m  3  r1 r2 
∴ A = 4 πr = 4 π 
2
 …(iv)
 4 πσ  r1 r2
or c = 4 πk (T2 − T1 )
∴From Eqs. (iii) and (iv), we get (r2 − r1 )
 2
 1
c∝
r1 r2
∆T eσT 4 3m  3  1 3 or
=  4 π    = k  r2 − r1
∆t mc   4 πσ    m
  dQ rr
or ∝ 1 2
 eσT 4   3  
2/ 3
dt r2 − r1
where, k =   4π   
 4π  
 c   Hence the radial rate of slow of heat in a
substance filled between the two concentric
 ∆T   r r 
  1 1
 ∆t  Q  mP  3  8mQ  3 1
sphere is proportional to  1 2  .
∴ =  =  = (8) 3 = 2  r2 − r1 
 ∆T   mQ   mQ 
 
 ∆t  P
30. A composite bar of uniform cross-section is
29. Figure shows a system of two concentric made of 25 cm of copper, 10 cm of nickel
spheres of radii r1 and r2 at the temperatures and 15 cm of aluminium with perfect
T1 and T2, respectively. The radial rate of flow thermal contacts. The free copper end of the
of heat in a substance filled between the two rod is at 100°C and the free aluminium end
concentric spheres is proportional to is at 0°C. If K Cu = 2 K Al and K Al = 3 K Ni , then
the temperatures of Cu-Ni and Ni-Al
[22 April 2019, Shift-II] junctions are respectively, (Assume no loss
98 AP EAMCET Chapterwise Physics

of heat occurs from the sides of the rod, Heat lost by B = Heat gained by A
K-thermal conductivity). [22 April 2019, Shift-II] (Q temperature B = 30° and temperature of
(a) 82.3 °C, 31.3 °C (b) 78.3 °C, 26.1 °C A = 15°)
(c) 70 °C, 23.3 °C (d) 90.3 °C, 30.1 °C So, B loses heat and A gains heat
mB c B (30 − T1 ) = m Ac A(T1 − 15)
Sol. (b)
∆Q KA(T1 − T2) So, m × 2(30 − T1 ) = m × 1(T1 − 15)
Heat transfer, = ⇒ 2(30 − T1 ) = 1(T1 − 15)
∆t L
K Cu (A) (100 − T1 ) K Ni (A) (T1 − T2) ⇒ T1 = 25°C
or =
25 10 Here, m1 = m2 = m3 = 1
K (A) (T2) and c1 : c 2 : c 3 = 1 : 2 : 3
= Al
15 For (B),
Q K Cu = 2K Al and K Al = 3 K Ni Let resultant temperature of mixture B and C is
2K Al (A) (100 − T1 ) K Al (A) (T2) T2 then,
∴ =
25 15 heat lost by C = heat gained by B
⇒ 6T1 + 5T2 = 600 …(i) ⇒ 3(45 − T2) = 2(T2 − 30)
K Ni (A) (T1 − T2) 3K Ni (A) (T2) ⇒ T2 = 39°C
⇒ =
10 15 For (C),
⇒ T1 = 3T2 …(ii) Let resultant temperature of mixture C and A is
From Eqs. (i) and (ii), we get T3 then,
T2 = 26.08° C, heat lost by C = heat gained by A
T1 = 78.26° C ⇒ 3 (45 − T3) = (T3 − 15)
Hence, the temperature of Cu-Ni and Ni-Al ⇒ T3 = 37.5° C
junctions are respectively, 78.26° and 26.08°C. Hence, A → (i), B → (iv), C → (iii), D → (ii).

31. The specific heat capacities of three liquids 32. A uniform metal bar of length 10 m with a
A , B and C are in the ratio, 1 : 2 : 3 and the crack at its midpoint is clamped between two
masses of the liquids are in the ratio 1 : 1 : 1. rigid supports. The bar buckles upward due to
The temperatures of the liquids A , B and C are temperature rise of 40°C. If the coefficient of
15 °C, 30 °C and 45 °C, respectively. Then linear expansion of the metal is 2.5 × 10 −6 °C −1 ,
matched the resultant temperature of the the maximum displacement of the mid-point
mixture given in list-II with the of the bar is [23 April 2019, Shift-I]
corresponding mixture given in list-I. (a) 11.3 cm (b) 22.3 cm (c) 33.3 cm (d) 44.3 cm
[22 April 2019, Shift-II]
Sol. (b)
List I List II Given, length of bar L = 10 m, rise in
temperature, ∆T = 40°C and coefficient of linear
A. Mixture of liquids A and B (i) 25 °C
expansion, α = 25 × 10−6 °C −1
B. Mixture of liquids B and C (ii) 35 °C As, linear expansion,
C. Mixture of liquids C and A (iii) 37.5 °C ∆L = L α ∆t = 10 × 2. 5 × 10−6 × 40
⇒ ∆L = 0.1 cm
D. Mixture of liquids A, B and C (iv) 39 °C
In the figure below, let the displacement of mid
Codes point of bar is x,
A B C D A B C D L+∆L
(a) (i) (ii) (iii) (iv) (b) (ii) (i) (v) (iii) B
(c) (i) (iv) (iii) (ii) (d) (iv) (i) (iii) (ii) A C x
5m O
Sol. (c) 10m
For (A),
Let the resultant temperature of mixture A and Now, from ∆OBA,
B is T1 then, from principle of colorimetry x = (AB)2 − (OA)2
Thermal Properties of Matter 99
2 keq A(Ti − Tf ) k1 A1 (Ti − T1 )
⇒ x=  10 + 0.001  − (5)2 ⇒ =
 
 2  l l1
⇒ x = 0.223 m here, l = l1 + l 2 + l 3 = l1 + l1 + l1 = 3l1 and
A = A1 + A2 + A3 = A1 + A1 + A1 = 3A1
Hence, x = 22 .3 cm
2k 3A1 2k A1 (100 − T1 )
Hence, the correct option is (b). So, × (100 − 0) =
7 3l1 l1
33. Three rods each of length l and cross 1 100
or × 100 = 100 − T1 ⇒ T1 = 100 −
sectional area A joined in series between two 7 7
heat reservoirs as shown in the figure. Their 600°C
K ⇒ T1 =
conductivities are 2 K , K and , respectively. 7
2 Similarly,
Assuming that the conductors are insulated From Eq. (i), we get
from surroundings, the temperatures T1 and 2k(100 − T1 ) = 0.5 kT2
T2 of the junctions in steady state condition 600
are respectively. [23 April 2019, Shift-I] ⇒ 200 − 2 × = 0. 5 T2
7
TA TB 400
⇒ T2 = °C
7
T1 T2
Hence, the correct option is (a).
K
100°C 2K K
2
0°C 34. Steam at 100°C is passed into 1 kg of water
contained in a calorimeter at 9°C till the
l l l temperature of water and calorimeter is
increased to 90°C. The mass of the steam
(a)
600
°C,
400
°C (b)
600
°C,
700
°C condensed is nearly
7 7 7 4 (water equivalent of calorimeter = 0.1 kg,
500 600 600 400
(c) °C, °C (d) °C, °C specific heat of water = 1 calg −1 ° C−1
6 5 4 7
Sol. (a) and latent heat of vaporisation = 540 calg −1)
Key idea In a series combination of heat conductors, [22 April 2018, Shift-I]
the rate of heat flow remains constant. (a) 81g (b) 162 g (c) 243 g (d) 486 g
According to the question, the figure given Sol. (b)
below shows the conduction of heat through Let mass of the steam condensed is x.
series combinations of rods.
Heat released = Heat gained by water
where, T1 , T2 = temperature of junctions, ⇒ x × 540 + x × 1 × (100 − 90)
I = heat current =
dQ = 1 × 1 × (90 − 9) + 01
. × 1 × (90 − 9)
dt ⇒ 540 x + 10 x = 81 + 81 .
dQ2 dQ3 Ti − Tf .
891
As,
dQ1
= = = ⇒ x= = 0 ⋅162 kg, x = 162g
dt dt dt keq 550

k1 A1 (Ti − T1 ) k2 A2 (T1 − T2) k3 A3 (T2 − Tf ) 35. Three very large plates of same area are kept
So, = =
l1 l2 l3 parallel and close to each other. They are
Q A1 = A2 = A3 and l1 = l 2 = l 3 considered as ideal black surfaces and have
very high thermal conductivity. First and
⇒ 2 k (100 − T1 ) = k(T1 − T2) = 0.5(T2 − 0) ...(i)
third plates are maintained at absolute
Equivalent coefficient of thermal conductivity,
temperatures 2T and 3T respectively.
1 1 1 2
= + + Temperature of the middle plate in steady
keq 2k k k
state is [22 April 2018, Shift-I]
2k
⇒ keq =
1 1 1
1

(a)   T (b)   T (c)   T (d) (97 )4 T


65 4 97 4 97 4
7
dQ dQ1 2   4  2 
Q Heat current, =
dt dt
100 AP EAMCET Chapterwise Physics

Sol. (c) Codes


Let assume temperature of middle plate is T0 . A B C D
As temperatures are constant, (a) I II III IV
∴ Heat gained by middle surface = Heat lost by (b) II I IV III
middle surface. (c) III II IV I
So, σA[(3T)4 − T04 ] = σA[T04 − (2T)4 ] (d) II I III IV
1 /4
97 4 97
⇒ T04 = T ⇒ T0 =   T Sol. (b)
2  2
When ice melts, its’ density increases or volume
36. As shown in the figure, an equilateral decreases.
triangle ABC is formed by joining three rods Water when changes into steam, its volume
of equal lengths and D is the midpoint of AB. increases.
Coefficient of linear expansion of the Melting point of ice decreases on increasing
material of AB is α1 and that of AC and BC is pressure.
α 2. If the length DC remains constant for Boiling point of water increases with pressure.
small changes in temperature, then 38. The time taken for a calorimeter containing
[22 April 2018, Shift-II] 75 g of water at 62°C to cool to 58°C is
D 9 minutes. When the calorimeter contains
A B
105 g of water, it takes 12 minutes to cool
from 62°C to 58°C. The water equivalent of
the calorimeter is [23 April 2018, Shift-I]
(a) 10 g (b) 15 g
(c) 20 g (d) 30 g

C Sol. (b)
α Heat lost = (w + m) CV dT
(a) α1 = α 2 (b) α1 = 4α 2 (c) α 2 = 4α1 (d) α1 = 2
2 where, w = water equivalent of calorimeter,
Sol. (b) m = mass of water,
For a change of temperature by t, change in CV = specific heat of water
length DC is ∆DC, then ∆DC 2 = ∆AC 2 − ∆AD 2 and dT = temperature difference = T2 − T1 .
2
= l(1 + α 2 t)2 −  (1 + α 2 t)
l Time taken to cool down is proportional to
2  amount of heat lost.
l2 Case I 9 minutes
= l 2 (2α 2 t) −
(2α1 t)
4 ∝ (w + 75) CV × (62° − 58°) … (i)
(After neglecting terms α 22 t 2 and α12 t 2 , being very Case II 12 minutes
small) ∝ (w + 105) CV × (62° − 58°) … (ii)
l2 Dividing Eq. (i) by Eq. (ii), we get
As, ∆DC = 0 ⇒ l 2 2α 2 t − (2α1 t) = 0
4 9 (75 + w) CV × 4°
=
⇒ α1 = 4α 2 12 (105 + w) CV × 4°
37. Match the following List I with List II. 3 × (105 + w) = 4(75 + w)
[22 April 2018, Shift-II] or 4w − 3w = 315g − 300 g
w = 15g
List I List II
A. When ice melts into water I. Volume increases
39. Three rods of same dimensions have thermal
conductivities 3K, 2K and K. They are
B. When water changes II. Volume decreases arranged as shown in the figure below. Then
into steam
in the steady state the temperature of the
C. Melting point of ice III. Increases with junction P is [23 April 2018, Shift-I]
increase of pressure
200 100 50
D. Boiling point of water IV. Decreases with
(a) °C (b) °C (c) 75°C (d) °C
3 3 3
increase in pressure
Thermal Properties of Matter 101

Sol. (a) 41. The temperature of a spherical black body is


50ºC
inversely proportional to its radius. If its
2k
3k radius is doubled, then the power radiating
100ºC from it will be [24 April 2018, Shift-I]
k (a) doubled
0ºC 1
(b) times of initial value
Amount of heat transmitted from one point to 4
another is (c) halved
kAt(T1 − T2) (d) four times of initial value
Q=
d Sol. (b)
where, k is thermal conductivity, t is time and Power radiating from a black body is given by
T1 − T2 is temperature difference. P = σAT 4 …(i)
At junction, For a spherical body,
Q = Q1 + Q2 A = 4 πR 2 …(ii)
3kA(100 − T)t 2kA(T − 50)t kA(T − 0)t and it is also given that,
= + 1 k
d d d T ∝ or T = …(iii)
R R
300kAt − 3kAtT = 2kAtT − 100 kAt + kATt
From Eqs. (i), (ii) and (iii), we have,
400kAt = 6kAtT  k4  1
P = σ 4 πR2 ⋅  4  or P ∝ 2
400 200° R  R
T= = C
6 3 So, when radius is doubled, radiating power of
sphere becomes one fourth.
40. A wire of 20 Ω is immersed in ice. If 10 A
current is passed through this wire for 42. A metal sphere immersed in water weighs w1
1 minute, ice completely melts. The mass of at 0°C and w 2 at 50°C. The coefficient of
the ice is nearly (L ice = 79 .7 calg −1) cubical expansion of the metal is less than
[23 April 2018, Shift-II] that of water. Then [24 April 2018, Shift-I]
(a) 3.5 g (b) 359 g (c) 540 g (d) 3.5 kg (a) w1 > w2 (b) w1 < w2 (c) w1 = w2 (d) w1 = 2 w2
Sol. (b) Sol. (b)
Heat gained = Heat generated Apparent weight of object in water,
mL = I 2 Rt density of object (sphere) d
w∝ ∝ s
I 2 Rt density of water dw
⇒ m=
L As, cubical expansion (Y) of metal is less than
100 × 20 × 60 water, thats why ratio of density of sphere and
= (Q1 cal = 4.2J)
79.7 × 4.2 water will greater at 50°C than 0°C.
= 359 g  ds  d 
  > s
 dw  50 ° C  dw  0 ° C
w2 > w1
10
Thermodynamics
1. The ratio of the slopes of isothermal and Temperature of sink, T2 = ?
adiabatic curves is [17 Sep. 2020, Shift-I] We know that, efficiency of Carnot engine,
T
(a) 1 (b) γ (c)
1
(d)
3 η =1 − 2
γ 2 T1
T
Sol. (c) ⇒ 0.3 = 1 − 2
500
For an isothermal change, T2
⇒ = 1 − 0.3 ⇒ T2 = 0.7 × 500
pV = K 500
Differentiating both side, we get = 350 K = (350 − 273)° C = 77°C
p ⋅ dV + V dp = 0 ⇒ Vdp = − p dV
3. First law of thermodynamics is represented
∴ Slope of an isothermal curve
by [17 Sep. 2020, Shift-II]
 dp  = − p
  (a) dQ = dU + dW (b) dU = dW + pdV
 dV  iso V
(c) dW = dQ + dU (d) dU = dQ + pdV
For an adiabatic change,
Sol. (a)
pV γ = K ′
Differentiating both side, we get According to first law of thermodynamics,
p ⋅ γV γ − 1 ⋅ dV + V γ ⋅ dp = 0 dQ = dU + dW
⇒ γp dV + V dp = 0 where, dQ = heat supplied to the system ,
or Vdp = − γ ⋅ p dV dU = change in internal energy of the system
∴Slope of an adiabatic curve, and dW = work done by the system.
 dp  γp
  =− 4. The door of an operating refrigerator is kept
 dV  adia V open. As a result, the temperature of the
 dp  room will [18 Sep. 2020, Shift-I]
 
 dV  iso −p / V 1
∴ = = (a) remain unchanged
 dp  −γ p / V γ (b) increases
 
 dV  adia (c) decreases
(d) depends on the contents inside the refrigerator
2. An ideal Carnot’s engine with an efficiency
of 30% operates between a source and a sink. Sol. (c)
If the temperature of the source is 500 K, A refrigerator is a heat engine which extracts
that of the sink is [17 Sep. 2020, Shift-II] heat from low temperature reservoir and
transfer it to high temperature. If a
(a) 27 °C (b) 57 °C
refrigerator’s door is kept open, then room will
(c) 77 °C (d) 107 °C become hot, because in this situation,
Sol. (c) refrigerator exhaust more heat into the room
Efficiency of Carnot’s engine, η = 30% = 0.3 than earlier. Hence, temperature of the room
increases and room becomes hot.
Temperature of source, T1 = 500 K
Thermodynamics 103

5. There is no change in internal energy of an Sol. (c)


ideal gas when it undergoes For ideal heat engine, efficiency is given as
[18 Sep. 2020, Shift-I] W Q1 − Q2
η= =
(a) isothermal expansion (b) adiabatic expansion Q1 Q1
(c) free expansion (d) isobaric expansion Q2 T2
⇒η =1 − =1 −
Sol. (c) Q1 T1
In free expansion, the ideal gas is allowed to T2
⇒ =1 − η
expand into a vacuum. This happens quickly, so T1
there is no heat transferred. No work is done, T1 1
because the gas does not displace anything. ⇒ = …(i)
T2 1 − η
According to first law of thermodynamics, this
means that When heat engine operated in backward
∆Einternal = 0 direction, then coefficient of performance is
given as
Hence, there is no change in internal energy of
Q Q2 T2
an ideal gas when it undergoes free expansion. β= 2 = =
W Q1 − Q2 T1 − T2
6. Which of the following parameters does not 1 1
characterise the thermodynamic state of ⇒ β= = [from Eq. (i)]
T1 1
−1 −1
matter? [18 Sep. 2020, Shift-I] T2 1−η
(a) Temperature (b) Pressure 1−η 1−η 1
= = = −1
(c) Work (d) Volume 1 −1 + η η η
Sol. (c)
Pressure, volume and temperature are
9. In a thermodynamic process pressure of a
thermodynamic state variables which are fixed mass of a gas is changed in such a
independent on the way how a state was manner that the gas releases 30 J of heat
achieved. But work is not a state variable and 10 J of work was done on the gas. If the
because it depends on the way it was achieved. initial internal energy of the gas was 10 J,
7. When the temperature difference between then the final internal energy will be
[21 Sep. 2020, Shift-I]
the source and sink increases, the efficiency
of the heat engine [18 Sep. 2020, Shift-II] (a) 2 J (b) − 18 J (c) 10 J (d) 58 J
(a) decreases Sol. (c)
(b) increases Given, heat, Q = − 30 J
(c) is not affected Work done on the gas, W = −10 J
(d) may increase or decrease Initial internal energy, U i = −10 J
Sol. (b) Final internal energy, U f = ?
Efficiency of heat engine is given as According to first law of thermodynamics,
T − T2 Q = W + ∆U ⇒ −30 = −10 + U i − U f
η= 1 …(i)
T1 ⇒ −30 = − 10 − 10 − U f
where, T1 = temperature of source ⇒ 30 = 10 + 10 + U f
and T2 = temperature of sink. ⇒ 30 − 20 = U f ⇒ U f = 10 J
From Eq. (i), it is clear that when
temperature difference between source and sink 10. A gas is compressed from a volume of 2 m 3
(T1 − T2) of heat engine increases, then its to a volume of 1 m 3 at a constant pressure of
efficiency η increases. 100 Nm − 2. Then it is heated at constant
volume by supplying 150 J of energy. As a
8. An ideal heat engine has an efficiency η. The
result, the internal energy of the gas
coefficient of performance of the engine when [21 Sep. 2020, Shift-I]
driven backward will be [21 Sep. 2020, Shift-I]
(a) increases by 250 J (b) decreases by 250 J
 1  1  1 1
(a) 1 −   (b) η −   (c)   − 1 (d) (c) decreases by 50 J (d) increases by 50 J
 η  η  η 1− η
104 AP EAMCET Chapterwise Physics

Sol. (a) amount of heat required to raise the


Since, gas is compressed from 2m3 to1m3 . temperature of the same gas through the
Hence, work done on the gas is negative. same range at constant volume is
W = − p∆V = − 100(2 − 1) = −100 J [21 Sep. 2020, Shift-II]
Since, heat supplied, Q = 150 J (a) 306 J (b) 153 J (c) 140 J (d) 80 J
Hence, according to first law of Sol. (c)
thermodynamics,
Given, at constant pressure heat
Q = W + ∆U
Qp = 306J
150 = − 100 + ∆U
Number of mole, n = 2
⇒ ∆U = 150 + 100 = 250 J
∆T = T2 − T1 = 35 − 25 = 10° C
11. The efficiency of a Carnot’s engine is 100% We know that,
only when [21 Sep. 2020, Shift-II] Qp = nCp ∆T
(a) ideal gas is used as a working substance ⇒ 306 = 2Cp × 10
(b) temperature of the sink is equal to absolute zero 306
⇒ Cp =
(c) source temperature is equal to the temperature 2 × 10
of the sink ⇒ Cp = 15.3 J/mol K
(d) source temperature is equal to absolute zero According to Mayer’s formula,
Sol. (c) Cp − CV = R ⇒ CV = Cp − R = 15.3 − 8.314
The efficiency of Carnot’s engine is given as ⇒ CV = 6.986 J/mol K
T The amount of heat required to raise the
η =1− 2 …(i)
T1 temperature of gas at constant volume through
where, T1 = temperature of source same range of temperature.
QV = nCV ∆T = 2 × 6.986 × (35 − 25)
and T2 = temperature of sink.
= 2 × 6.986 × 10 = 139.72~
−140 J
For 100% efficiency, η = 1
∴ From Eq. (i), we get 14. An electrical refrigerator with β = 5 extracts
T T
1 = 1 − 2 ⇒ 2 = 0 ⇒ T2 = 0 5000 J from the contents of the refrigerator.
T1 T1 During this process, find the electrical energy
Hence, temperature of sink T2 is equal to utilised by its motor. [22 Sep. 2020, Shift-I]
absolute zero. (a) 1 kJ (b) 0.5 kJ (c) 0.8 kJ (d) 1.2 kJ
12. The condition dQ = dW holds good in which Sol. (a)
of the following ? [21 Sep. 2020, Shift-II] Coefficient of performance of refrigerator,
(a) Adiabatic process (b) Isothermal process β=5
(c) Isochoric process (d) Isobaric process Amount of heat removed,
Sol. (b) Q = 5000 J
According to first law of thermodynamics, Electrical energy utilised by the motor
dQ = dW + dU …(i) = Work done by the motor (W)
Q
For isothermal process, there is no change in ∴ β=
internal energy. W
Q 5000
Hence, dU = 0 ⇒ W= = = 1000 J = 1 kJ
β 5
∴ From Eq. (i), we get
dQ = dW 15. A Carnot engine takes 3 × 10 6 calories of heat
Hence, above equation holds good for from reservoir at 627°C and gives it to a sink
isothermal process.
at 27°C. The work done by the engine is
13. 306 J of heat is required to raise the [22 Sep. 2020, Shift-I]
temperature of 2 moles of an ideal gas at (a) zero (b) 8.4 × 106 J
constant pressure from 25°C to 35°C. The (c) 4.2 × 106 J (d) 16.8 × 106 J
Thermodynamics 105

Sol. (b) ⇒ T1 = 400 K


Heat taken by Carnot engine, When efficiency, η′ = 40% = 0.4
Q = 3 × 106 cal = 4.2 × 3 × 106 J T′ T′
then, 1 − 2 = 0.4 ⇒ 2 = 1 − 0.4 = 0.6
Temperature of reservoir (source) T1 T1
T1 = (627 + 273)K = 900 K ⇒ T2′ = 0.6 T1 = 0.6 × 400 = 240 K
Temperature of sink,
18. The efficiency of an ideal Carnot engine
T2 = (27 + 273) K = 300 K
working between temperatures T1 and T2 is
We know that, efficiency of Carnot engine, 1/3. If the temperature of the sink is reduced
W T W T
η= =1− 2 ⇒ =1 − 2 by 40%, then its efficiency will be
Q T1 Q T1
[22 Sep. 2020, Shift-II]
W 300 W 2
⇒ =1 − ⇒ = (a) 50% (b) 25% (c) 60% (d) 75%
4.2 × 3 × 106 900 4.2 × 3 × 106 3
2 Sol. (c)
⇒ W = × 4.2 × 3 × 106 = 8.4 × 106 J 1
3 Efficiency of Carnot engine, η =
3
16. During phase change, entropy T2
But η = 1 −
[22 Sep. 2020, Shift-II] T1
(a) remains constant where, T1 = and temperature of fource
(b) always increases
T2 = temperature of sink.
(c) always decreases
1 T T 1 T 2
(d) may increase or decrease ∴ =1− 2 ⇒ 2 =1− ⇒ 2 = …(i)
3 T1 T1 3 T1 3
Sol. (d)
The entropy of a system during phase change When temperature of sink is reduced by 40%,
(phase transition) increases, if the phase then its new temperature, T2′ = T2 − 40% of T2
transition is towards higher internal energy = T2 − 0.4T2
(e.g., during melting) and decreases if the phase ⇒ T2′ = 0.6 T2
transition is towards lower internal energy (e.g.,
during freezing). Therefore, during phase Now, efficiency of Carnot's engine
change, entropy of the system may increase or  T′   T 
η′ = 1 − 2  × 100 = 1 − 0.6 2  × 100
decrease.  T1   T1 

= 1 − 0.6 ×
17. A Carnot’s engine has an efficiency of 25% 2
 × 100 = 0.6 × 100 = 60%
when its sink is at 27°C. If it has to be  3
increased to 40%, what should be the
temperature of the sink keeping the 19. The first law of thermodynamics confirms
temperature of the source constant? the law of [23 Sep. 2020, Shift-I]
(a) conservation of momentum of molecules
[22 Sep. 2020, Shift-II]
(b) conservation of energy
(a) 320 K (b) 375 K (c) 340 K (d) 300 K (c) flow of heat in a particular direction
Sol. (*) (d) conservation of heat energy and mechanical
Efficiency of Carnot’s engine, energy
η = 25% = 0.25 Sol. (b)
Temperature of sink, The first law of thermodynamics is a form of the
T2 = 27°C = 273 + 27 = 300 K law of conservation of energy, adapted for
Temperature of source = T1 thermodynamic processes, distinguishing two
kinds of transfer of energy as heat and as
T T
∴ η = 1 − 2 ⇒ 0.25 = 1 − 2 thermodynamic work, and relating them to a
T1 T1 function of a body’s state called internal energy.
T2
⇒ = 1 − 0.25 = 0.75 The law of conservation of energy states that the
T1 total energy of an isolated system is constant,
T 300 energy can be transformed from one form to
⇒ T1 = 2 = = 400 K another but can be neither created nor destroyed.
0.75 0.75
106 AP EAMCET Chapterwise Physics

p
20. One mole of an ideal diatomic gas undergoes
a transition from A to B along a path AB as
shown in figure. The change in internal II
energy of the gas during the transition is
A B
[23 Sep. 2020, Shift-I]
p (kPa) I

5
A (a) ∆U1 = ∆U 2 (b) ∆U1 > ∆U 2
(c) ∆U1 < ∆U 2 (d) ∆U1 ≠ ∆U 2
Sol. (a)
For a thermodynamic system, change in
2 B internal energies in processes is function of
states only, i.e. it depends only on their initial
state and final state and completely
4 6 V (m3) independent from the thermodynamic paths
(a) – 20 kJ (b) 20 J (c) –12 kJ (d) 20 kJ through which system has been taken.
So, in the figure given in question, for processes
Sol. (a) I and II initial and final states are same,
p (kPa) therefore change in internal energies will also
A (T A ) be same for both processes I and II.
5
i.e. ∆U1 = ∆U 2

2 B (TB) 22. A gas expands with temperature according


to the relation, V = kT 2/ 3, where k is a
constant. Work done when the temperature
4 6 V (m3 ) changes by 60 K is (R = universal gas
Given that, p A = 5kPa = 5 × 10 Pa 3
constant.) [20 April 2019, Shift-I]
pB = 2kPa = 2 × 103 Pa (a) 10 R (b) 20 R (c) 50 R (d) 40 R
VA = 4 m 3 Sol. (d)
VB = 6 m 3 Given, V = kT 2 / 3
µ = 1 mole From definition of work done,
Change in internal energy, ∆U = µCV ∆T RT RT
dW = P dV = dV = dV …(i)
= µ  R (TB − TA)
5
V k T 2/ 3
2 
Now, V = k T 2 / 3
5
(Q For diatomic gas, CV = R) Taking derivative on the both sides, we get
2
2
5 5 dV = K T −1 / 3 dT …(ii)
∴ ∆U = µR(TB − TA) = (µRTB − µRTA) 3
2 2
5 Substituting the value of Eqs. (ii) in Eqs. (i) and
= (pB VB − p AV A) (using pV = µRT ) taking integration on the both sides, we get
2 T
2 2 2
3 T∫1
5
= (2 × 103 × 6 − 5 × 103 × 4) W= R dT = R(T2 − T1 )
2 3
5 2 2
= × (−8 × 103) = −20 × 103 = −20 kJ = R(60 − 0) = × R × 60 = 40 R
2 3 3

21. A system goes from A to B via two processes I 23. An ideal gas is taken through the cycle
and II as shown in the figure. If ∆U1 and ∆U 2 A → B → C → A as shown in the figure. If
are the changes in internal energies in the the net heat supplied to the gas in the cycle
processes I and II respectively, then is 5 J. The magnitude of work done during
the process C → A is [20 April 2019, Shift-I]
[23 Sep. 2020, Shift-I]
Thermodynamics 107

Now, change in internal energy due to adiabatic


process is
2 C B R
∴ ∆ U = nCv ∆T or ∆U = 5 × (T2 − T1 )
V (m3) γ −1
1 A  R 
Q Specific heat, Cv = 
 γ − 1
(0,0) p(Nm–1) 10
8.3 8.3
or 5× (673 − 273) or = 5 × × 400
(a) 5 J (b) 10 J (c) 15 J (d) 20 J 1.4 − 1 0. 4
Sol. (a) or ∆U = 41.5 kJ
An ideal gas is taken through the cycle So, the increase in internal energy of the gas is
A → B → C → A as shown in the figure, ∆U = 41.54 kJ.

25. One mole of a monatomic ideal gas undergoes


V(m3)
2 C B the process A → B in the given p-V diagram.
Specific heat capacity in the process is
[20 April 2019, Shift-II]
1 A
p

(o,o) 10
p (Nm–1 )
B
6p0
From the graph,
W AB = p × dV = 10 × (2 − 1) = 10 J
3p0
Similarly, A
WBC = p × (0) = 0 J
According to the first law of thermodynamics V
∆Q = ∆ U + W V0 5V0
`Here, ∆U = 0 13R 13R 7R 2R
(a) (b) (c) (d)
∴ Q=W 3 6 3 3
5 = W AB + WBC + WCA
Sol. (b)
WCA = 5 − W AB − WBC = 5 − 10 − 0
According to the question,
WCA = − 5J
p
∴ |WCA| = 5J
Hence, the magnitude of work done during the B
6p0
process C → A is 5J.

24. Five moles of Hydrogen gas initially at STP is 3p0


compressed adiabatically so that its A
temperature becomes 673 K. The increase in
internal energy of the gas is V
(R = 8 .3 J mol −1 K −1 , γ = 1.4 for diatomic gas) V0 5V0

[20 April 2019, Shift-II]


Now, the temperature at point A and B
(a) 80.5 kJ (b) 21.55 kJ
By, pV = nRT (from ideal gas equation)
(c) 41.50 kJ (d) 65.55 kJ
Temperature at point A,
Sol. (c) 3p V
TA = 0 0
Given, temperature, T2 = 673 K R
initial temperature T1 = 273 K Temperature at point B,
rate of heat flow, R = 8.3 J mol −1 K −1 30 p0 V0
TB =
γ = 1.4 for diatomic gas, moles of hydrogen gas, R
n= 5 So, the temperature difference, ∆T = TB − TA
108 AP EAMCET Chapterwise Physics

30 p0 V0 3p0 V0 Now, the efficiencies are,


∴ ∆T = −
R R 300
(a) When T1 = 500 K, T2 = 300 K ⇒η =1 − = 0.4
27p0 V0 500
or, ∆T =
R 350
(b) When T1 = 500 K, T2 = 350 K ⇒ η =1 − = 0.3
Now, due to the adiabatic process, change in 500
internal energy, (c) When T1 = 800 K, T2 = 400 K
3 400
∴ ∆ U = nCv ∆T or, ∆U = R ∆T …(i) ⇒ η =1− = 0.5
2 800
Work done by the gas undergoes the process (d) When T1 = 450K, T2 = 360 K ⇒
A → B.
∴W = Area under p-V graph
2 360
or W = 18 p0 V0 or W = R∆T …(ii) η =1− = 0.2
3 450
Now, by first law of thermodynamics, Hence, the correct code is matches to option (d).
Q = ∆U + W
27. A Carnot engine of efficiency 40%, takes
By putting the values from Eqs. (i) and (ii) to
heat from a source maintained at a
above Eq. we get
3 2 temperature of 500 K. It is desired to have an
∴ Q = R ∆T + R ∆T engine of efficiency 60%. Then, the source
2 3
13
temperature for the same sink temperature
or C ∆T = R ∆T must be [21 April 2019, Shift-II]
6
(a) 650 K (b) 750 K (c) 550 K (d) 850 K
(Q Q = nC ∆T, where n = 1 mole)
13 Sol. (b)
or C= R
6 % Efficiency of carnot engine,
 T 
So, the specific heat capacity in the process η = 1 − 2  × 100
13
A → B is C = R.  T1 
6 where, T2 = sink temperature
26. Match the temperatures of the source and and T1 = source temperature
sink (T1 and T2 respectively) of a Carnot heat For 40% efficiency of carnot engine, T1 = 500 K
40 
= 1 − 2  ⇒ 0.4 = 1 − 2
engine given in List-I with the corresponding T T

efficiencies given in List-II. [21 April 2019, Shift-I] 100  500  500
T2
List-I List-II
⇒ = 0.6 ⇒ T2 = 300 K
500
A. T1 = 500 K, T2 = 300 K (i) 0.2 For 60% efficiency, the source temperature of
Carnot engine for the same sink temperature,
B. T1 = 500 K, T2 = 350 K (ii) 0.3 60 T T 6
⇒ =1 − 2 ⇒ 2 =1 −
C. T1 = 800 K, T2 = 400 K (iii) 0.4 100 T1 T1 10
T2 4 10
D. T1 = 450 K, T2 = 360 K (iv) 0.5 ⇒ = ⇒ T1 = T2
T1 10 4
The correct match is 10
or T1 = × 300 = 750 K
A B C D A B C D 4
(a) (iii) (iv (ii) (i) (b) (iv) (iii) (ii) (i) 28. Which of the following statement is correct
(c) (iii) (i) (iv) (ii) (d) (iii) (ii) (iv) (i) regarding p-V graph? [22 April 2019, Shift-I]
Sol. (d) 1. Slope of p-V graph in an isothermal
Key Ideal Efficiency of a Carnot engine is given as, p
T
process is − .
η =1 − 2 V
T1 2. Slope of p-V graph in an adiabatic
where, T1 = surface temperature and T2 = sink p
temperature. process is − .
V
Thermodynamics 109
p
3. Slope of p-V graph in an isochoric process
γp
is − .
V 2
4. Slope of p-V graph in an isobaric process
is zero.
(a) 1, 3, 4 are correct
V1 V2
(b) 2, 3 are correct
(c) 1, 4 are correct 4. Slope of p-V graph in an isobaric process is
(d) 2, 3, 4 are correct zero. This option is correct.
Sol. (c) p
Statements 1 and 4 are correct. 1
1. Slope of p-V graph in an isothermal process
p 2
is − . This is correct.
V
p
V
T1 < T2 < T3
p-V curve is a straight line parallel to volume
axis, slope of p-V curve for an isobaric
dp
process = 0.
T3 dV
T2
T1 29. Three moles of an ideal monotomic gas
performs a cycle ABCDA as shown in the
V figure. The temperatures of the gas at the
O
For isothermal process, states A , B, C and D are 400 K, 800 K, 2400 K
dp p and 1200 K, respectively. The work done by
slope of p-V ′ curve =−
dV V the gas during this cycle is (R is universal gas
2. Slope of p - V group in an adiabatic process is constant) [22 April 2019, Shift-I]
p
− , which is wrong. P
V
Because slope of p-V graph in an adiabatic
γp B
process is − . C
V A
p D

I
(a) 1200 R (b) 3600 R (c) 2400 R (d) 2000 R
Sol. (c)
isochoric
According to the question,
p

adiabatic
V
O V1 V2

3. Slope of p-V graph in an isochoric process is B


γp C
− this option is wrong. A
V D
Because slope of p-V graph for isochoric T
process is
dp dV dp Given, number of moles of an ideal monoatomic
− = =∞

dV V 0 gas, n = 3,
V temperature of the gas of state A, TA = 400 K,
110 AP EAMCET Chapterwise Physics

temperature of the gas at state B, TB = 800 K, (p, V, T)


temperature of the gas at state C , TC = 2400 K, p
A
and temperature of the gas at state D , TD =1200 K,
Now, work done by the gas during this cycle is, Isochoric
W = nR∆T Adiabatic (pV1.5=constant)
(∴Work done in a isothermal process)
C
So, Wnet = W A→ B + WB→ C + WC→ D + WD→ A B p , 2V , T
isobaric 2√2 √2
Wnet = nR[(TB − TA) + (TC − TB) + (TD − TC) − (TD − TA)]
Wnet = 3R[0 + (2400 − 800) + 0 − (1200 − 400)] V
Wnet = 3R × [1600 − 800] p , V, T
Wnet = 3R × 800, Wnet = 2400 R 2√2 2√2
So, the work done by the gas during, As work done in adiabatic process,
A → B → C → D cycle is Wnet =2400 R.
R T −
T 

nR(T1 − T2)  2  (Taking, n = 1)
30. An insulated system contains 4 moles of an W= =
γ −1 (1.5 − 1)
ideal diatomic gas at temperature T. When a
heat Q is supplied to the gas, 2 moles of the V2 = 2V1
gas is dissociated into atoms and the T1 T
⇒ T2 = or T2 =
temperature remained constant.Then the 2 2
relation between Q and T is (R = universal 
W1 = RT 21 −
1 
 = 0.58 (Q r = i Given)
gas constant.) [22 April 2019, Shift-I]  2
(a) Q = RT (b) Q = 2 RT Work done in isobaric process,
(c) Q = 3 RT (d) Q = 4RT W2 = p∆V = nR∆T
⇒ W = nR
Sol. (a) T T   1 − 1 
−  = − RT  
Given, number of moles of an ideal diatomic gas 2 2 2  2 2 2
at the temperature, T = 4 = − 0.35
When heat Q is supplied to the gas, 2 mole of Now, heat given to system
the gas is dissociated into atoms and the
Q1 = nCp ∆T = R 
7 1 1 
temperature remains constant, therefore −  = − 1.23 J
2  2 2 2
heat supplied = change in its internal energy
Negative, Q1 shows that heat is given to the
i.e., Q = ∆u = (u f − ui)
system.
or Q = (internal energy of 4 moles of a
Efficiency of the cycle,
monatomic gas + internal energy of 2 moles of
net work done W1 + W2
diatomic gas) − (internal energy of 4 moles of a η= =
diatomic gas) heat input Q1
0.58 − 0.35
=  4 × RT + 2 × RT  −  4 × RT 
3 5 5 =
 2 2   2  1.23
= 6RT + 5RT − 10RT = RT 0.23
⇒ η= × 100 ≈ 18.6% ~
− 18%
1.23
31. A gas (γ = 15
. ) undergoes a cycle of adiabatic, Hence, the efficiency of cycle is 18%.
isobaric and isochoric processes in an order.
If the volume of the gas is doubled in the 32. Two heat engines X and Y of same efficiency
adiabatic process then the efficiency of the are connected in series in such a way that
cycle is approximately, [22 April 2019, Shift-II] the sink of X works as source of Y . X
receives heat at 900 K and rejects some heat
(a) 18% (b) 46.4% (c) 38.5% (d) 9.25%
to its sink at TK and in turn Y rejects heat to
Sol. (a) its sink at 400 K, then the temperature T is
According to the question, we drawn the
[23 April 2019, Shift-I]
following situation,
p p (a) 550 K (b) 600 K
In isochoric process, A = B
TA TB (c) 650 K (d) 700 K
Thermodynamics 111

Sol. (b) 34. A cylindrical vessel of uniform cross-section


According to the question, a series combination consisting of a gas of γ = 15
. is divided into
of two heat engines are shown in the figure two parts A and B using a piston. Initially
below, the piston is kept fixed such that part A has
T1 W1 T W2 T2 pressure p and volume 5 V and the part B has
pressure 8 p and volume V. If the piston is let
⊗ ⊗ free and the gas is allowed to undergo
X Y adiabatic process, then the final volume of
ηx ηy
the gas in part A is [22 April 2018, Shift-II]
Given, temperature of X heat engine, T1 = 900 K 8 10 13
(a) 3 V (b) V (c) V (d) V
and temperature of Y heat engine, T2 = 400 K 3 3 3
So, efficiency of x heat engine, Sol. (c)
T T
ηx = 1 − =1 − Initially
T1 900 A B
T 400 p, 5V 8p, V
Similarly, η y = 1 − 2 = 1 −
T T
Q Given, ηx = η y
T 400
1− =1 − Piston
900 T
T 2 = 900 × 400 Finally,
A x B
T = 600 K
pf pf
Hence, the correct option is (b). VA VB
33. Match the following lists.
List I List II For adiabatic expression in part A and B, we
have,
A. Zeroth law of I. Direction of flow of p ⋅ (5V)r = p f V Ar … (i)
thermodynamics heat
and 8p ⋅ (V)r = p f ⋅ VBr … (ii)
B. First law of II. Work done is zero Solving for V A with r = 1 ⋅ 5, we get
thermodynamics 10
VA = V
C. Free expansion of a gas III. Thermal equilibrium 3
D. Second law of IV. Law of conservation
Thermodynamics of energy
35. A diatomic ideal gas is used in Carnot’s
engine as working substance. During
[22 April 2018, Shift-I] adiabatic expansion of the cycle, if the
The correct answer is volume of the gas increases from V to 32 V,
then the efficiency of the engine is
A B C D A B C D
[22 April 2018, Shift-II]
(a) II IV III I (b) III IV II I
(c) III I II IV (d) I III IV II (a) 0.25 (b) 0.5 (c) 0.67 (d) 0.75

Sol. (b) Sol. (d)


(i) Zeroth law of thermodynamics states about p T1
B C
thermal equilibrium of different states in
contact. Adiabatic
expansion
(ii) First law is energy conservation law.
(iii) In free expansion of gases, work done is
always zero. A T2 D
(iv) Second law of thermodynamics discusses V
about flow of heat in thermal bodies.
112 AP EAMCET Chapterwise Physics

Given, volume at C = V and volume at D = 32V = Pressure due to weight of rider and cycle
For adiabatic expansion CD, + Atmospheric pressure initially inside tyre
γ −1 F 120 × 10 N
TC  VD  = + patm = + 1 × 105 = 6 × 105 2
TC VCγ − 1 = TD VDγ − 1 ⇒ =  A 24 × 10−4 m
TD  VC 
γ −1 7 Number of moles of air in the tube
= 
32V  −1
 = 325 pV 6 × 105 × 2 × 10−3
 V  n1 = =
2 RT RT
TC
⇒ = (32) 5 = 4 Volume of these moles at atmospheric pressure is
TD
nRT 6 × 105 × 2 × 10−3
Now, efficiency of Carnot’s cycle, V1 = = = 12 × 10−3 m3
T 1 3 patm 1 × 105
η = 1 − D = 1 − = = 0.75
TC 4 4 Initial volume of air inside tyre
75
V0 = × 2 × 10−3 = 1.5 × 10−3 m3
36. Freezing compartment of a refrigerator is at 100
0°C and room temperature is 27.3°C. Work So, to inflate the tube volume to be pumped in is
done by the refrigerator to freeze 1 g of V2 = V1 − V0 = (12 − 1.5) × 10−3 = 10.5 × 10−3 m3
water at 0°C is (Lice = 80 cal g − 1) Hence, number of strokes of pump required
[23 April 2018, Shift-I] V2 10.5 × 10−3
N= = = 21
(a) 336 J (b) 33.6 J (c) 3.36 J (d) 40 J Vpump 500 × 10−6
Sol. (b)
Coefficient of performance of refrigerator is 38. Assertion (A) A room can be cooled by
T2 opening the door of a refrigerator in it.
β=
T1 − T2 Reason (R) Heat always flows from a body
T2 = 0° C = 0 + 273 = 273K at higher temperature to a body at lower
T1 = 27.3° C ≈ 300K temperature. [23 April 2018, Shift-II]
273 (a) (A) and (R) are true and R is the correct
∴ β= ≈ 10 explanation of (A).
300 − 273
(b) (A), (R) are true and (R) is not the correct
Now, if Q2 is heat extracted and W is work explanation of (A).
performed them, (c) (A) is true but (R) is false
Q Q
β = 2 or W = 2 (d) (A) is false but (R) is true.
W β
Sol. (d)
where, Q2 = heat extracted from 1g of water to When door of refrigerator is opened, the room’s
make it ice heat is discarded in the room itself and heat
= mL1 = 80 cal = 80 × 4.2 J = 336 J that is created in working of refrigerator is also
So, W = 336 / 10 = 336
. J discarded in the room, making room hotter.
So, Assertion is false but Reason is true.
37. Tyre of a bicycle has volume 2 × 10 − 3 m 3.
Initially, the tube is filled 75% of its volume 39. A graph drawn between absolute
by air at atmospheric pressure 10 5 Nm − 2. temperature and volume of 3 moles of
When a rider is on the bicycle, the area of helium gas as shown in the figure. If 5 cal of
contact of tyre with road is 24 × 10 − 4 m 2. The heat is used in the process, then the work
done is [23 April 2018, Shift-II]
mass of rider with bicycle is 120 kg. If a
Y-axis
pump delivers a volume 500 cm 3 of air in
each stroke, then the number of strokes 30
required to inflate the tyre is (g = 10 ms − 2) V 20
(m3)
[23 April 2018, Shift-I] 10
(a) 10 (b) 11 (c) 21 (d) 20 X-axis
5 10 15
Sol. (c) T (K)
Pressure against which pump has to deliver air (a) 21.0 J (b) 8.4 J (c) 12.6 J (d) 6.2 J
Thermodynamics 113

Sol. (b) ⇒ When temperature of sink is reduced by 50 K,


100
In given graph, efficiency becomes %
V 3
V ∝T ⇒ = constant 100 1 1
T η= × =
Hence, process is a constant pressure process. 3 100 3
∆Q T − 50 1
So, ∆Q = nCp ∆T and ∆W = p∆V = nR∆T = nR ⋅ So, 1− 2 =
nCp T1 3
∆QR T2 50 1
=
2
= × ∆Q = 2 cal = 2 × 4.2J = 8.4 J 1− + = … (ii)
(5/ 2) R 5 T1 T1 3
Subtracting Eq (ii) from Eq. (i), we get
40. An ideal gas is found to obey pV 3/ 2 = 50 1 1 1
= − =
constant during an adiabatic process. If such T1 3 4 12
a gas initially at a temperature T is
adiabatically compressed to half of its initial T1 = 600 K
3
volume, then its final temperature is [23 April T2 = × 600 = 450 K
2018, Shift-II] 4
(a) 2 T (b) 2T (c) 2 2T (d) 4T 42. An ideal monoatomic gas is carried along the
Sol. (a) cycle ABCDA as shown in the figure. The
γ 3 total heat absorbed during this process is
Given, pV = constant , γ = [24 April 2018, Shift-I]
2
r−1
V  p
⇒ TV γ − 1 = constant ⇒ Tf = Ti  i 

 Vf  3p0
B C
3
−1
 2V  2
⇒ Tf = T  f 
 ⇒ Tf = T(2)1 / 2 ⇒ Tf = 2T
 Vf  p0
A D
1
41. A reversible Carnot heat engine converts th
4 O V0 2V0 V
of its input heat into work. When the
temperature of the sink is reduced by 50 K, (a) 10.5 p0 V0 (b) 7.5 p0 V0
1 (c) 2.5 p0 V0 (d) 1.5 p0 V0
its efficiency becomes 33 %. The initial
3 Sol. (a)
temperatures of the source and the sink Heat absorbed means heat is extracted from
respectively are [24 April 2018,Shift-I]
source, i.e.Q must be positive.
(a) 600 K, 550 K (b) 600 K, 450 K
This occurs along path
(c) 300 K, 150 K (d) 450 K, 350 K
A → B → C.
Sol. (b) ∴Heat added = ∆Q ABC = ∆U ABC + ∆W ABC
Q
Given work, W = = nCV (TC − TA) + 3p0 (2V0 − V0)
4 3 3
where, Q = input heat. = n R(TC − TA) + 3p0 V0 = (nRTC − nRTA) + 3p0 V0
2 2
W Q/ 4 1
Efficiency, η = = ⇒ η= 3
Q Q 4 = (pC VC − p AV A) + 3p0 V0
2
T2
Also, η =1 − 3
= (3p0 2V0 − p0 V0) + 3p0 V0
T1 2
where, T2 = temperature of sink 3 21
= × 5p0 V0 + 3p0 V0 = p0 V0 = 10.5p0 V0
and T1 = temperature of source. 2 2
T 1
1− 2 = … (i)
T1 4
11
Kinetic Theory of Gases
1. A gas volume 251 cm 3 at 20°C and pressure p A VA
nB × = nA ×
pB VB
78 cm of Hg. Find its volume at NTP TA TB
[17 Sep. 2020, Shift-I] NB p A VA N A pB VB
⇒ ⋅ = ×
(a) 420 cm (b) 440 cm (c) 240 cm 3 (d) 100 cm 3
3 3
M TA M TB
Sol. (*) NB pB VB TA
⇒ =
Volume of gas, V = 251 cm 3 NA p A V A TB
Temperature, T1 = 20° C = 273 + 20 = 293° K NB 3p A ⋅ 2V A ⋅ TA
=
p1 = 78 cm of Hg NA T
p A ⋅ VA ⋅ A
At NTP, 2
Temperature, T2 = 300 K N B 12 N 1
= ⇒ A = ⇒ N A : N B = 1 : 12
Pressure, p2 = 76 cm of Hg NA 1 N B 12
V2 = ?
3. The ratio of specific heats at constant
According to ideal gas equation,
pressure and at constant volume is γ. To find
p1 V1 p V
= 2 2 out the degree of freedom, the expression is
T1 T2
[17 Sep. 2020, Shift-II]
p1 V1 T2 78 × 251 × 300
⇒ V2 = = 2 γ+1
p2 T1 76 × 293 (a) f = (b) f =
γ −1 2
= 2638
. cm 3 2 1
(c) f = (d) f =
No option is matched. γ+1 γ+1
2. Two vessels A and B contain oxygen. The volume Sol. (a)
of B is twice that of A, the pressure of B is Cp
thrice that of A and the temperature of B is We know that, γ = …(i)
half of that of A. Then, find the ratio of CV
number of molecules of oxygen in vessels A f
Energy of N molecules of gas, U = RT
and B. [17 Sep. 2020, Shift-II] 2
(a) 1: 3 (b) 1 : 12 (c) 3 : 4 (d) 1 : 6 where, f is degree of freedom.
dU d f  fR
Sol. (b) ∴ CV = =  RT  =
dT dT  2  2
For vessels A and B,
fR ( f + 2) R
VB = 2V A ∴ Cp = CV + R = + R ⇒ Cp =
2 2
pB = 3p A
∴ From Eq. (i), we get
T
TB = A ( f + 2) R / 2 f + 2
2 γ= ⇒ γ=
fR / 2 f
If N A and N B be the number of molecules of
2
oxygens in vessels A and B respectively, then ⇒ fγ − f = 2 ⇒ f (γ − 1) = 2 ⇒ f =
according to ideal gas equation, γ −1
Kinetic Theory of Gases 115

4. The average kinetic energy of H 2 molecules at Sol. (b)


300 K is E. At the same temperature the Given, T1 = (273 + 20)K = 293K
average kinetic energy of O 2 molecule is p1 = 90 cm of Hg
[18 Sep. 2020, Shift-I] p2 = 75 cm of Hg
E E Since, volume of the gas is constant.
(a) E (b) (c) (d) 16 E
4 16 Hence, according to ideal gas equation,
p1 p
Sol. (a) = 2
T1 T2
Average kinetic energy of H 2 molecules at
T1 p2 293 × 75
300 K = E ⇒ T2 = =
Since, E ∝T p1 90
Hence, at same temperature average kinetic = 244.16 K = 244.16 – 273 = − 28.8°C
energy of O 2 molecule will be same as E.
8. A light container having a diatomic gas
5. The ratio of the adiabatic to isothermal enclosed within is moving with velocity v.
elasticities of a triatomic (non-linear) gas is Mass of the gas is M and number of moles is
[21 Sep. 2020, Shift-II] n. The kinetic energy of gas w.r.t. ground is
(a) 3 : 4 (b) 1 : 2 (c) 4 : 3 (d) 5 : 3 [22 Sep. 2020, Shift-I]
1 3 1
Sol. (c) (a) Mv 2 + nRT (b) Mv 2
2 2 2
For triatomic non-linear gas degree of freedom, 1 5 5
f =6 (c) Mv 2 + nRT (d) nRT
2 2 2
fR 6R
CV = = = 3R
2 2 Sol. (c)
∴ Cp = CV + R = 3R + R = 4R According to kinetic theory of gases,
∴ Ratio of adiabatic to isothermal elasticities of a Energy of diatomic gas having f number of
triatomic (non-linear) gas is degree of freedom is given as,
f
C
γ= p =
4R 4
= E1 = n RT [For n mole gas]
CV 3R 3 2
Since, degree of freedom of a diatomic gas,
6. A vessel contains mixture of hydrogen and f =5
oxygen gases in the ratio of their masses equal ∴ Kinetic energy of diatomic gas with respect to
to 1 : 5. The ratio of mean kinetic energies of centre of mass,
the two gasses is [21 Sep. 2020, Shift-II] f
E 2 = n RT
(a) 1 : 1 (b) 1 : 16 (c) 16 : 5 (d) 5 : 16 2
5
Sol. (a) E 2 = nRT
2
Given, M H 2 : M O2 = 1 : 5
Kinetic energy of gas with respect to ground
Mean kinetic energy per molecule,
3 = KE of centre of mass with respect to ground
E = K BT ⇒ E ∝ T + KE with respect to centre of mass
2 1 5
Hence, mean kinetic energy of gases, depends on = Mv2 + nRT
2 2
the temperature only.
Since, temperature of both gases is same. Hence, 9. The molecular motion ceases at
ratio of their mean kinetic energy will be 1 : 1. [22 Sep. 2020, Shift-II]
7. Pressure of a gas of constant volume at 20°C (a) 273 K (b) 273 °C (c) − 273 K (d) −273 °C
is 90 cm of Hg. At what temperature the Sol. (d)
pressure would change to 75 cm of Hg? According to kinetic theory of gases, average
[22 Sep. 2020, Shift-I] kinetic energy of gas molecules is
(a) 233.2°C (b) –28.8°C 3
K = RT i.e. K ∝ T
(c) –24.2°C (d) 28.8°C 2
116 AP EAMCET Chapterwise Physics

We know that, kinetic energy may not be negative. Q Degree of freedom of mixture
Its minimum value may be equal to zero. 2× 3+ 2× 5
f = =4
Hence, minimum possible temperature on the 2+ 2
earth is equal to 0 K at which molecular motion 2
starts ceasing. Q fmix = 1 + = 1.5
f
∴Minimum possible temperature
fmix RT
= 0 K = (0 − 273)°C = − 273°C Q Speed of sound =
M
10. The average translational kinetic energy of a 25 972
molecule in a gas becomes equal to 0.69 eV at 1.5 × ×
= 3 5
temperature about, 3 × 10−3
. × 10 −23 J K −1 ]
[Boltzmann’s constant = 138
= 900 = n × 100 m/s
[20 April 2019, Shift-I]
Hence, n =9
(a) 3370°C (b) 3388°C
R
(c) 5333°C (d) 5060°C 12. For a gas the value of = 0 .4 , so the gas is
CV
Sol. (c)
(R-universal gas constant)
Given, average translational kinetic energy
[21 April 2019, Shift-II]
= 0.69 eV = 0.69 × 1.6 × 10−19 V
(a) monoatomic (b) diatomic
As we know that, (c) triatomic (d) polyatomic
3
average translational kinetic energy = kT Sol. (b)
2
R
−19 3
0.69 × 1.6 × 10 = × 1.38 × 10 T −23 Given, for a gas the value of = 0.4
2 CV
0.69 × 1.6 × 10−19 × 2 we know that the relation between γ, R and CV
T= is
3 × 1.38 × 10−23
R
⇒ γ −1 = or γ − 1 = 0.4 ⇒ γ = 1.4
T = 5333K = 5333 − 273 = 5060°C CV
11. If the speed of sound in a mixture of 2 moles which is for a diatomic gas.
of Helium and 2 moles of Hydrogen at
972 13. Under standard conditions, the density of a
temperature K is n × 100 ms −1 , then the 1400
5 gas is kg - m −3 and the speed of sound
25 1089
value of n is (Take, R = J mol −1 K −1 ) propagation in it is 330 ms −1 , then the
3 number of degrees of freedom of the gas
[20 April 2019, Shift-II] molecules is [22 April 2019, Shift-I]
(a) 9 (b) 10 (c) 100 (d) 90 (a) 2 (b) 7 (c) 5 (d) 3
Sol. (a) Sol. (c)
972 25
Given, T = K, R = J mol −1 K −1 Given, density of gas, ρ =
1400
kg / m3
5 3 1089
Molecular mass of the mixture speed of sound, v = 330 m/s
n M + nH M H
= He He and under standard condition,
nHe + nH
Pressure of gas, p = 1 × 105 N / m2
Q nHe = 2moles, nH = 2 moles
If γ be the ratio of Cp and CV of a gas, then the
M He = 4, M H = 2 speed of sound in gas is given by
∴Molecular mass of the mixtures, γp γp
2× 4 + 2× 2 v= or = v2
M= ρ ρ
2+ 2
v2ρ 330 × 330 1400
8+ 4 γ= = ×
= = 3 gm/mole p 105 1089
4
γ =1.4
Kinetic Theory of Gases 117
Cp Q Molar specific heat of gas at constant pressure
So, γ= = 1.4
CV and constant volume are given by
Since, for diatomic gas, the volume of γ is 1.4. ∴ Cp ′ = mCp = m(620)
Hence, the degree of freedom for diatomic gas is and CV ′ = mCv = m(420)
equal to 5. As we know that,
14. The y-components of velocities of the Q Cp ′ − CV ′ = R
molecules of a gas are [Here, R = universal gas constant]
− 7 , − 6 , − 5 , − 4 , − 3 , − 2 , − 1, m[620 − 420] = R
0 , + 1, + 2 , + 3 , + 4 , + 5 , + 6 , + 7 ms − 1 then the ⇒ m=
R
…(i)
rms velocity is [22 April 2019, Shift-II] 200
56 28 Now, ideal gas equation,
(a) ms − 1 (b) ms − 1
3 3 pV = µRT
112 84 where, µ = number of moles of gas
(c) ms − 1 (d) ms − 1
3 3 or pV = RT [Q for 1 mole, µ = 1]
∴ pm = ρRT Q density, ρ = m 
Sol. (a)  V 
Given, Y-components of velocities are, R
− 7, − 6, − 5, − 4, − 3, − 2, − 1, 105 × = ρR(273) [From Eq. (i)]
0, + 1, + 2, + 3, + 4, + 5, + 6, + 7 ms− 1 200
As rms velocity, ρ = 1.85 Kg-m −3
Hence, the option (d) is correct.
v12 + v22 + K + vn2
vrms =
n 16. Three closed vessels A , B and C are at the
(−7) + (− 6) + K + 0 + K + 6 + 7
2 2 2 2 same temperature T and contain gases.
=
15 Vessel A contains only O 2, B contains only N 2
Since, (− x)2 = x 2
and C contains a mixture of equal quantities
of O 2 and N 2. If the rms speed of O 2 molecules
2(72 + 62 + 52 + 42 + 32 + 22 + 12 in vessel A is v1 and that of N 2 molecules in
⇒ vrms =
15 vessel B is v2 then the rms speed of O 2
2 × 140 56 molecules in vessel C is [23 April 2019, Shift-I]
⇒ vrms = = v1 + v 2 
15 3 (a)   (b) v1
 2 
56
Hence, the rms speed is m / s. v1
3 (c) (v1 v 2 ) (d)
2
15. The specific heat capacities of an ideal gas at Sol. (b)
the constant pressure and at constant volume
Key Idea Root mean square velocity of any gas is
are 620 Jkg −1K −1 and 420 Jkg −1K −1 given by
respectively. The density of the gas at STP is 3RT
approximately, [23 April 2019, Shift-I] vrms =
M
(a) 2.88 kgm −3 Where, M = molecular weight.
(b) 4.86 kgm −3
As, rms speed of any gas is depends on the
(c) 3.88 kgm−3 temperature of gas and molecular weight of gas.
. kgm−3
(d) 186
In a mixture of gases (say N 2 and O 2 ) at a
Sol. (d) constant temperature, rms speed is independent
Given, specific heat capacity of a gas at to the quantity (moles) of gas present in the gas
constant pressure, Cp = 620 J kg −1 K −1 mixture. So, in this problem the rms speed of O 2
in vessel C is v1 .
and specific heat capacity of gas at constant
volume, CV = 420 J kg −1 K −1 Hence, the correct option is (b).
118 AP EAMCET Chapterwise Physics

17. A thermally insulated vessel with nitrogen 19. The absolute temperature at which the rms
gas at 27°C is moving with a velocity of speed of a hydrogen molecule is equal to its
100 ms −1 . If the vessel is stopped suddenly, escape speed from the moon’s surface is
then the percentage change in the pressure (where, R is radius of moon is r, g is
of the gas is nearly acceleration due to gravity on Moon’s
surface, m is mass of hydrogen molecules
(assume entire loss in KE of the gas is given and k is Boltzmann constant)
as heat to gas and R = 8.3 Jmol −1K −1 ) [22 April 2018, Shift-II]
[22 April 2018, Shift-I]
mgR 2mgR
(a) 1.1 (b) 0.93 (c) 0.5 (d) 2.25 (a) (b)
2k k
Sol. (d) 3mgR 2 mgR
(c) (d)
Let there are n moles of N 2 gas in the cylinder. 2k 3k
Assuming all of K , E appears in form of heat, Sol. (d)
n Mv2  = nR∆T
1 f 3RT 3kT
2  2 vrms(hydrogen) = =
M m
Here, M = 28 g = 28 × 10−3 kg, f = 5 and vescape (moon) = 2gR
Increment in pressure due to change of
temperature is vrms = vescape
nR∆T 3kT 2mgR
Also, ∆p = ⇒ = 2gR ⇒ T =
V m 3k
 nR∆T  20. A diatomic gas consisting of rigid molecules
 
∆p  V  nR∆T is at a temperature of 87°C. If the moment of
So, = =
p  nRT  nRT inertia of the rotating diatomic rigid
 
 V  molecule is 2.76 × 10 − 39 gcm 2, then the rms
∆p nMv2 Mv2 angular speed of the molecule is (Boltzmann
⇒ = =
p fnRT fRT . × 10 − 23 JK − 1 )
constant = 138
So, percentage change in pressure is, [23 April 2018, Shift-I]
∆p 28 × 10−3 × 100 × 100 (a) 6 × 1012 rads − 1 (b) 3 × 1012 rads − 1
∴ × 100 = × 100 = 2.25%
p 5 × 8.3 × 300 (c) 6 × 1013 rads − 1 (d) 3 × 1013 rads − 1

18. For a molecule of an ideal gas, the number Sol. (a)


−3 Moment of inertia of diatomic molecule,
density is 2 2 × 10 cm and the mean free
8
2
10 −2 I = Mr 2
path is cm. The diameter of the gas 3
π 3I
molecule is [22 April 2018, Shift-I] So, radius, r = … (i)
−4 −4
2M
(a) 5 × 10 cm (b) 0.5 × 10 cm
rms speed of molecule,
(c) 2.5 × 10−4 cm (d) 4 × 10−4 cm
3kT
Sol. (a) v=
M
Mean free path, rms angular speed,
1
λ= v 3kT / M 2kT
2 π n d2 ω= = =
r 3I / 2M I
1 1× π
⇒ d2 = = 2 × 1.38 × 10− 23 × (87 + 273)
2π nλ 2 × π × 2 2 × 108 × 10− 2 =
1 1 2.76 × 10− 46
⇒ d2 = ⇒ d = × 10− 3 cm
4 × 106 2 = 360 × 1023
−4
⇒ d = 5 × 10 cm
= 36 × 1024 = 6 × 1012 rad / s
Kinetic Theory of Gases 119

21. The rms speed of oxygen molecule at a 22. The ratio of the speed of sound in a
certain temperature is 600 ms −1 . If the monatomic gas at 27°C and rms speed of the
temperature is doubled and oxygen molecule molecules of the same gas at a temperature
dissociates into atomic oxygen atoms, the of 127°C is [24 April 2018, Shift-I]
new rms speed is [23 April 2018, Shift-II] (a) 1 : 2 (b) 5 : 12
(a) 120 ms −1 (b) 150 ms −1 (c) 3 : 4 (d) 13 : 17
(c) 1200 ms −1 (d) 600 ms −1 Sol. (b)
Sol. (c) Speed of sound in gas
vrms =
3RT γ RT1
v= ...(i)
M M
Given, at temperature T1 , rms speed of molecules,
3RT1 3RT2
vrms = 600 = …(i) c= ...(ii)
M M
At temperature, T2 = 2πT1 , For monoatonic gas , r =
5
due to dissociation 3
3R(2T1 ) Also, T1 = 300 K, T2 = 400 K,
vrms = v = …(ii)
(M / 2) v γ. Τ1 5 / 3× 300
∴ Ratio, = =
Dividing Eq. (i) by Eq. (ii), we get c 3⋅ T2 3 × 400
v v 5
⇒ =2 =
600 c 12
⇒ v = 1200 ms −1
12
Oscillations
1. The variation of potential energy of a ⇒ 4 × 10−2 − 1 × 10−2 =
1
k × (2 × 10−2)2
harmonic oscillator is as shown in the figure. 2
Then, find the spring constant. ⇒ 3 × 10−2 = k × 2 × 10−4
[17 Sep. 2020, Shift-II] 3 × 10−2
⇒ k= = 1.5 × 10 2 = 150 Nm −1
U (J) 2 × 10−4
0.04
2. Three blocks of masses 700 g, 500 g and 400 g
suspended at the end of a spring as shown in the
figure, are in equilibrium. [18 Sep. 2020, Shift-II]

0.01

(0,0) y (mm)
20 mm
700 g
(a) 1 × 102 Nm − 1 (b) 150 Nm − 1
500 g
(c) 0.667 × 102 Nm − 1 (d) 3 × 102 Nm − 1
400 g
Sol. (b)
According to figure, when y = 20 mm
When the 700 g block is removed, the system
= 2 × 10−2 m
−2 has a period of oscillations of 3s. If both 700 g
then Umax = 0.04 J = 4 × 10 J
and 500 g blocks are removed, the period of
U (J)
oscillation becomes
0.04
12
(a) 1 s (b) 2 s (c) 3 s (d) s
5
Sol. (b)
The given situation is shown in the following
figure
0.01

(0,0) y (mm)
20 mm

When y = 0
then Umin = 0.01 J = 1 × 10−2 J
700 g
∴ The change in potential energy,
1 500 g
Umax − Umin = ky 2
2 400 g
Oscillations 121

When the block of 700 g is removed, then period Frequency of oscillation is given as
of oscillation is given as 1 k′
m f =
T ′ = 2π 2π M
k
∴ Time period of oscillation,
(700 + 500 + 400 − 700) × 10−3
= 2π 1 M
k T= = 2π
f k′
[where, k = spring constant]
M
T ′ = 2π
0.9 T = 2π [from Eq.(i)]
k k/2
1 /2
= 2π  
T ′ = 3s 2M M
But ⇒ T = 2π
0.9 0.9 k  2k 
∴ 3 = 2π ⇒ 9 = 4π2 ×
k k 4. The maximum force acting on a particle
⇒ k = 0.4 π 2 …(i) executing simple harmonic motion is 10 N.
When both blocks of 700 g and 500 g are The force on the particle when it is midway
removed, then period of oscillation is given as between mean and extreme positions will be
(700 + 500 + 400 − 700 − 500) × 10−3
T ′′ = 2π [21 Sep. 2020, Shift-I]
k
(a) 10 N (b) 12 N (c) 5 N (d) zero
0.4
= 2π [from Eq. (i)] Sol. (c)
0.4 π 2
Maximum force on the particle performing

= = 2s SHM, Fmax = 10 N
π
We know that, In SHM, when body is at
3. On a smooth inclined plane, a mass M is maximum displacement (amplitude a), then
attached between two massless springs of force on the particle is maximum.
force constant k each, as shown in the figure. ∴ Fmax = mass × acceleration = m ⋅ α max
The other ends of the springs are fixed to [∴α max = ω2 a]
firm supports. The period of oscillation of the ⇒ 10 = m ⋅ ω a ⇒ 10 = maω2
2
…(i)

Force on the particle when it is mid way  y = 
mass M is [18 Sep. 2020, Shift-II] a
 2
between mean and extreme position,
F = mω2 y = mω2 ⋅   Q y = a 
k a
 2  2
M maω2 10
k = = = 5N [from Eq. (i)]
2 2
5. A body executes simple harmonic motion
θ
with an amplitude A. At what displacement,
1/ 2 1/ 2
from the mean position, is the potential
(a) 2 π   (b) 2 π 
M 2M  energy of the body one fourth of its total

 2k   k  energy ? [21 Sep. 2020, Shift-II]
Mg sin θ 
1/ 2
A A 3A
(c) 2 π  (d) 2 π 
2 Mg  (a) (b) (c) (d) 3A
 
 2k   k  4 2 4

Sol. (a) Sol. (b)


According to diagram given in question, it is According to question,
1
clear that both springs are connected in series. In simple harmonic motion, potential energy =
∴Equivalent spring constant k′ will be 4
(total energy)
kk k⋅ k k
k′ = 1 2 = = …(i) 1 1 1 2 2 A2 A
k1 + k2 k + k 2 ⇒ mω y =
2 2
mω A ⇒ y =
2
⇒ y=
2 4  2  4 2
122 AP EAMCET Chapterwise Physics


6. Four pendulums A , B,C and D are hanged ⇒ = 2000 π
T
from the same elastic support as shown in
⇒ T = 0.001 s ⇒ T = 10−3 s
the figure. A and C are of the same length
Maximum velocity,
while B is smaller than A ,C while D is longer
10
than A. If A is given displacement, then at vmax = ω a = 2000 π × × 10−2
steady state [21 Sep. 2020, Shift-II]
π
= 2 × 102 = 200 ms −1

8. The kinetic energy, K of a body performing


simple harmonic motion varies with time t,
is indicated in graph [22 Sep. 2020, Shift-I]
B
K

C A (a)

t
T
D
K
(a) D will vibrate with max amplitude
(b) C will vibrate with max amplitude
(c) B will vibrate with max amplitude (b) t
T
(d) All four will oscillate with equal amplitude
Sol. (b)
According to given diagram, length of K
pendulum A and C is same. Hence, time period
 l (c)
 T = 2π  of pendulum A and C is also same.
 g
t
Therefore, they have same frequency of T
oscillation. Due to this, resonance takes place
and the pendulum C will vibrate with maximum K
amplitude.
(d) t
7. The displacement y ( in cm) in case of a T
simple harmonic wave is given by
10  πx 
y= sin 2000 πt −  . The period and
π  17  Sol. (a)
maximum velocity of the particles in the Kinetic energy of a body performing simple
medium will respectively be harmonic motion is given as
[22 Sep. 2020, Shift-I] 1
K = mv2 …(i)
(a) 10−3 s, 330 ms −1 (b) 10 −4 s, 20 ms −1 2
(c) 10 −3 s, 200 ms −1 (d) 10 −2 s, 2000 ms −1 where, v=
dy d
= ⋅ a sinωt [Q y = a sinωt]
dt dt
Sol. (c)
v = aω cosωt
Displacement equation in SHM is given as
π x ∴ From Eq. (i),we get
sin  2000 π t −
10
y=  cm 1 1
π  17  K = m(aω cosωt)2 = ma 2 ω2 cos2 ωt
2 2
Comparing with wave equation, 1 2  1 + cos 2ωt 
y = a sin (ωt − kx) K = ma ω 2
 …(ii)
2  2 
10 10
We get, a= cm = × 10−2 m The graph represented in option (a) is correct
π π
for expression of kinetic energy represented by
ω = 2000 π Eq. (ii)
Oscillations 123
A
9. The acceleration of a particle executing SHM The time taken by it to go from x = 0 to x = is
is [22 Sep. 2020, Shift-I] 2
T1 , then
(a) always zero A
(b) always constant = AsinωT1
2
(c) maximum at the extreme position
1 π
(d) maximum at the equilibrium position or = sinωT1 or sin = sinωT1
2 6
Sol. (c) π
or ωT1 = …(ii)
The acceleration of the particle executing SHM is 6
given as
Now, the time taken by the particle to go from
α = − ω2 y ⇒ α ∝ y A
Since, maximum value of displacement y from x = to x = A is T2 , then time taken from x = 0
2
mean position is equal to amplitude a.
to x = A is (T1 + T2). So, we may write using Eq.
Hence, acceleration of a particle executing SHM (i)
is maximum at extreme position.
A = Asin{ω( T1 + T2)}
10. The maximum velocity of a particle or 1 = sin{ω( T1 + T2 )}
performing simple harmonic motion is π
or sin = sin{ωT1 + ωT2 }
6.28 cm s −1 . If the length of its path is 8 cm, 2
then what is its period? [22 Sep. 2020, Shift-II] π
or ωT1 + ωT2 =
(a) 2s (b) 4s (c) 3s (d) 1s 2
π π
Sol. (b) or ωT2 + = [from Eq. (ii)]
6 2
In SHM,
π
vmax = 6.28 cm s −1 or ωT2 = …(iii)
3
Length of path = 8 cm
Length of path From Eqs. (ii) and (iii), we get
∴ Amplitude of particle, a = ωT1 π/6 1
2 = =
8 ωT2 π/3 2
= = 4 cm T1 1
2 ⇒ = ⇒ T1 < T2
∴ vmax = ω a T2 2
⇒ ω = vmax / a
12. A simple pendulum is placed inside a lift,
6.28
⇒ ω= rad s −1 which is moving with a uniform acceleration.
4 If the time periods of the pendulum while the
2 π 6.28 Q ω = 2 π  lift is moving upwards and downwards are in
⇒ =
T 4  T  the ratio 1:2, then the acceleration of the lift
4 × 2π 4 × 2 × 3.14 is (Acceleration due to gravity, g = 10 ms −2)
⇒ T= = = 4s
6.28 6.28 [20 April 2019, Shift-II]
11. A particle executes simple harmonic motion (a) 6 ms −2 (b) 0 ms −2 (c) 3 ms −2 (d) 2 ms −2
between x = − A and x = + A. If it takes a time Sol. (a)
T1 to go from x = 0 to x = A / 2 and T2 to go Given,
from x = A /2 to x = A. Then, time period of the pendulum while the lift is
[23 Sep. 2020, Shift-I] moving upwards and downward are in the ratio,
T1 : T2 = 1 : 2
(a) T1 < T2 (b) T1 > T2 (c) T1 = T2 (d) T1 = 2T2
Acceleration due to gravity, g = 10 m/s 2
Sol. (a) We know that,
Let say displacement-time equation of a particle If the lift is moving upward, then total
executing SHM starting from mean position is as time-period,
follow
l
x = Asinωt …(i) T1 = 2π …(i)
g+ a
124 AP EAMCET Chapterwise Physics

when the lift is moving downwards, then the Here, T = 8s (for a complete
total time period, cycle)
l 2π 2π π
T2 = 2π …(ii) Hence, ω = = = rad / s
g−a T 8 4
By dividing Eq. (i) to (ii), we get and acceleration in SHM,
T1 g−a α = − ω2 a sinω t
∴ = π
2
π 4
T2 g+ a α = −   × 1 × sin × 
 4  4 3
1 g−a
Now, = π2 3 π2 3
2 g+ a ⇒ α=− × =− cm s− 2
16 2 32
Square on the both sides, we get Hence, the correct option is (a).
 1  = g − a or g − a = 1
2
or   14. When a body is in SHM, then match the
 2 g+ a g+ a 4
following. [21 April 2019, Shift-II]
or 4 g − 4a = g + a or 3g = 5 a
3g 30
or a= ⇒ a= = 6 m/s 2 List-I List-II
5 5
A. Velocity is maximum I. At extreme
So, the acceleration of the lift is 6 m/s 2 . position
th
13. For a particle executing simple harmonic
KE is   of the
3 II. At mean position
B.
motion, the displacement-time (x-t) graph is  4
as shown in the figure. The acceleration of total energy
4
the particle at t = s is th

PE is  
C. 3 III. At half of the
3 [21 April 2019, Shift-I] of total amplitude
 4
x (cm)
energy
D. Acceleration is 3
1 maximum IV. At times of
2
0 the amplitude
4 8 12 t (s)
–1 The correct answer is
A B C D A B C D
(a) III I IV II (b) I III IV II
3 2 32 2 (c) II III IV I (d) II I IV III
(a) − π cm s − 2 (b) π cm s − 2
32 3 Sol. (c)
(c) +
3
π cm s − 2 (d) +
32
π cm s − 2 (A) At mean position, velocity is max = Aω
32 3 1
(B) In SHM, total Energy = kA2 …(i)
Sol. (a) 2
1
The displacement-time graph shown in the figure is and KE = K (A2 − x 2) …(ii)
a sine wave, so the equation of displacement, 2
x = 1sinωt 3
Q KE = TE
x (cm) 4
From Eqs. (i) and (ii), we get
kA − kx = TE =  kA2 
1 2 1 2 3 3 1
1 2 2 4 42 

0 ⇒ 4 A2 − 4 x 2 = 3A2
4 8 12 t (s) A
⇒ A2 = 4 x 2 ⇒ x = ±
–1 2
1 2 1
(C) PE = kx and TE = kA2
2 2
Oscillations 125

TE ⇒ kx 2 =  kA2  Velocity of particle, v = Amplitude × Oscillation


3 1 3 1
Q PE =
4 2 42  frequency
3 2 3 ∴ v = aω
⇒ x2 = A ⇒x = ± A
b −a
v = 
4 2 2v
 ω or ω = …(i)
(D) At extreme position, acceleration is maximum.  2  b −a
Hence, (A) → II, (B) → III, (C) → IV, (D) → (I) 2π
∴ Time period, T =
ω
15. The amplitude of a damped oscillator
Putting the value of ω from Eq. (i) to above
becomes half in one minute. The amplitude
formula,
1 2π b −a
after 3 minutes will be times the original. ⇒ T= × (b − a) ⇒ T = π
x 2v v
Then, x is [21 April 2019, Shift-II] So, the time period of a particle executing SHM
(a) 4 (b) 8 along a straight line from points A to B is,
(c) 6 (d) 12 b −a
T= π
Sol. (b) v
Amplitude of damped oscillator, 17. A particle is executing simple harmonic motion
A = A0 e − λt along a straight line PQ. At three points A, B
where, λ = constant, t = time and C on the line PQ, lying on one side of the
for, t = 1min mean position, the velocities of the particles
A0 are 8 ms − 1 , 7 ms − 1 and 4 ms − 1 , respectively. If
= A0 e − λ ⇒ e λ = 2
2
AB = BC = 1 m, the velocity of the particle at
for, t = 3min
mean position is [22 April 2019, Shift-II]
A = A0 (e − λ)3
(a) 9 ms − 1 (b) 47 ms − 1

A0 A
= 30 ⇒ X = 2 3 = 8 Q A0 = A0 
 2 3 (c) 65 ms − 1 (d) 10 ms − 1
λ 3
(e ) 2 X 
Sol. (c)
16. A particle executing SHM along a straight Given,
line has zero velocity at points A and B whose velocity of particle A, v A = 8 ms− 1
distance from O on the same line OAB are a velocity of particle B, vB = 7 ms− 1
and b, respectively. If the velocity at the mid
velocity of particle C, vC = 4 ms− 1
point between A and B is v, then its time
velocity of a particle in harmonic motion (SHM),
period is [22 April 2019, Shift-I]
π(b + a) b − a ⇒ v = ω a2 − x 2
(a) (b) π  
v  v  Square on the both sides, we get
b + a b − a v2 = a 2ω2 − x 2ω2
(c)   (d)  
 2v   2v  For particle A,
v2A = a 2ω2 − x 2ω2
Sol. (b)
⇒ 64 = a 2ω2 − x 2ω2 … (i)
According to the question,
For particle B,
v(velocity)
O B vB2 = a 2ω2 − (x + 1)2 ω2
A
(b – a ) ⇒ 49 = a 2ω2 − (x + 1)2 ω2 … (ii)
a
For particle C,
b
vC2 = a 2ω2 − (x + 2)2 ω2
∴ Amplitude
Distance travelled by the particles ⇒ 16 = a 2ω2 − (x + 2)2 ω2 … (iii)
= (A to B) By solving Eqs. (i), (ii) and (iii), we get
2
1
Amplitude of particles executing x = , a 2ω2 = 65 (Q v = aω )
3
simple harmonic motion (SHM) along a straight
b−a Hence, velocity of particle at the mean position,
line AB is (a) = .
2 v = aω = 65 ms− 1 .
126 AP EAMCET Chapterwise Physics

5
18. The potential energy of a simple harmonic Now, for th of oscillation from mean
oscillator of mass 2 kg at its mean position is 8
5 J. If its total energy is 9 J and amplitude is 5/8 oscillation from mean
1 cm, then its time period is
[22 April 2018, Shift-I] T/6 T/12 T/12
π π π π T T T T T
(a) s (b) s (c) s (d) s Time = + + + +
100 50 20 10 12 6 6 12 12
Sol. (a) 5
⇒ Time for th of oscillations
Given, total energy = 9J 8
7 7 × 3x 7
PE at mean position = 5J = T= = x
12 12 4
So, maximum KE = 9J − 5J = 4J
Now, in SHM 20. In the case of a simple pendulum executing
Maximum (at mean) KE = Maximum PE (at SHM at t = 0, the bob is not at the mean
extremes) position. The graph drawn between the
1 2 tension (T) in the string and time (t) is
∴ ka = 4J
2 [23 April 2018, Shift-I]
8 8 Y-axis
⇒ k = 2 = − 4 = 8 × 104 J / m 2
a 10
Now, time period
m 2 π (a) T
T = 2π = 2π × ⇒T = s
k 8 × 104 100
X-axis
19. A particle is executing SHM. The time taken t
th Y-axis
 3
for   of oscillation from extreme
8
positions is x. Then, the time taken for the
th (b)
 5 T
particle to complete   of oscillation from
8 X-axis
t
mean position is [22 April 2018, Shift-II]
Y-axis
5x 7x 21x 7x
(a) (b) (c) (d)
4 4 8 12
(c) T
Sol. (b)
We divide total distance 4A in 8 equal parts.
t=T/6 t=T/12 t=0 t=T/12 t=T/6 X-axis
t
2/8 1/8 x=0 1/8 2/8 Y-axis
3
So, for a displacement of th of an oscillation
8
from an extreme, (d) T
of oscillation
3/8 from extreme
X-axis
t
T/12 0 T/12 T/6
T T T 2+ 1 + 1 4 T Sol. (a)
Time = + + = T= T= As the bob is executing simple harmonic
6 12 12 12 12 3
T motion, whose bob is not at mean position
Given, = x or T = 3x graph (a) depicts the correct relationship
3
between tension in string and time.
Oscillations 127
YA
21. The displacement of a particle of mass 2 g So, k1 = spring constant for a rod is .
π L

executing SHM is given by y = 5 sin  4 t +  . If a rod and spring are connected, then it is a
 3 series combination.
Here, y is in metres and t is in seconds.
The kinetic energy of the particle, when k1=YA/L
T
t = is
4 [23 April 2018, Shift-II] k2=k
(a) 0.4 J (b) 0.5 J (c) 3 J (d) 0.3 J
m
Sol. (d)
π
Given, y = 5sin 4t +  So, (keq ) system =
k1 k2
=
kYA / L
=
k YA
 3 k1 + k2 k + YA kL + YA
2π π L
ω = 4 and so T = = s
4 2 m
T π So, T = 2π
t= = s keq
4 8
π m (kL + YA)
Velocity =
dy
= 20 cos  4t +  ⇒ T = 2π
dt  3 kYA
π 23. At t = 0 , a particle executing SHM with a
Velocity at t = s is
8 time period 3 s is in phase with another
π π
v = 20 cos  4 × +  = 20 cos150° particle executing SHM. The time period of
 8 3 the second particle is T (less than 3 s). If they
20 3 are again in the same phase for the third
= − 20 cos 30° = − ms −1
2 time after 45 s, then the value of T is .... .
KE of particle is [24 April 2018, Shift-I]
1 1 2 (a) 1 s (b) 1.5 s (c) 2 s (d) 2.5 s
K = mv2 = × × 100 × 3 = 0.3 J
2 2 1000 Sol. (d)
22. One end of a long metallic wire of length L, Given, initially both oscillating particles are in
area of cross-section A and Young’s modulus same phase. For keeping calculation simple, we
Y is tied to the ceiling. The other end is tied assume that they both are at mean positive
to a massless spring of force constant k and (x = 0) at t = 0.
a mass m is hung from the free end of the Now first particle is in same phase at given time
instances.
spring. If m is slightly pulled down and
released, then its time period of oscillation is t = 0s, 3s, 6s, 9s, 12s, 15s, … etc.
[23 April 2018, Shift-II] Now, we take time period of second particle
(from options) and select that value of T, which
m mYA gives same phase of both particles at t = 15s,
(a) 2 π (b) 2 π
k kL 30s, 45s … etc.
m (kA + YL) m (kL + YA) For second particle by option (a),
(c) 2 π (d) 2 π
kYA kYA t = 0s, 1s, 2s, 3s, 4s, 5s, 6s, … etc
By option (b),
Sol. (d)
t = 0s, 1.5s, 3s, 4.5s, 6s, … etc
For oscillating mass at end of a rod.
YA By option (c),
Restoring force = .x t = 0s, 2s, 4s, 6s, … etc
L
By option (d),
t = 0s, 25 . s, 15s, … etc
. s, 5s, 7.5s, 10s, 125
So, both particles are in same phase again at
t = 15s, 30s, 45s, … etc only, when time period of
m
second particle is 2.5s.
13
Waves
1. On producing the waves of frequency
1000 Hz in a Kundt’s tube, the total distance vpa
between 6 successive nodes is 85 cm. Then,
the speed of sound in the gas filled in the (b)
x
tube is [17 Sep. 2020, Shift-I]
(a) 330 ms −1 (b) 340 ms −1
(c) 350 ms −1 (d) 300 ms −1
Sol. (b) vpa
Frequency, f = 1000 Hz
Total distance between 6 successive nodes (c)
x
d = 85cm = 0.85 m
Since, distance between 6 successive nodes is
λ 5λ
equivalent to 2λ + =
2 2
vpa

(d)
x

2 4 6
1 3 5
Sol. (d)
λ λ λ/2 Plane wave is given as
y = a sin (ω t − kx)
5λ 0.85 × 2 at t = 0,
∴ d= = 0.85 ⇒ λ = = 0.34 m
2 5 y = a sin (ω × 0 − kx)
∴ Speed of sound in tube, ⇒ = a sin (− kx) ⇒ − a sin kx
v = f λ = 1000 × 0.34 = 340 ms −1 Particle velocity,
2. A plane wave y = a sin (ωt − kx) propagates dy d
vpa = = (− a sin kx)
dt dt
through a stretched string. The particle
x Q k =
2π 2π 
velocity versus x graph at t = 0 is = − ak cos kx = − ak cos

λ λ 
[17 Sep. 2020, Shift-I] 2πa 2 πx
=− cos …(i)
λ λ
vpa From Eq. (i),
−2πa − 2 πa
(a) When x = 0, vpa = ⋅ cos 0 =
x λ λ
λ −2πa π
When x = , vpa = ⋅ cos = 0
4 λ 2
Waves 129
λ − 2πa 2πa Path difference, ∆x = 25 m
When x = , vpa = ⋅ cos π =
2 λ λ Comparing the wave equation by
3λ −2 πa  −π y = A sin(ωt − kx), we get k = 0.01 π
When x = , vpa = cos  =0
4 λ  2  2π 2
⇒ = 0.01 π ⇒ λ =
−2 πa −2πa λ 0.01
When x = λ, vpa = ⋅ cos 2 π =
λ λ ⇒ λ = 2 × 102 m
Hence, correct graph is ∴ Phase difference,
vpa 2π 2π π
∆φ = × ∆x = × 25 =
λ 2 × 102 4
2πa/λ
5. Two waves of frequency f and amplitude a
λ/4 3λ/4 λ
λ/2 x superimpose with each other. The total
intensity is directly proportional to
–2πa/λ [18 Sep. 2020, Shift-I]
(a) a (b) 2a
(c) 2 a2 (d) 4 a2
3. Two tuning forks X and Y are of frequencies
Sol. (d)
280 Hz and 284 Hz. A third tuning fork Z is
of unknown frequency. When X and Z are When two waves of same frequency f and
having same amplitude a superimpose, then
sounded together certain beats are heard per
resultant amplitude,
second. When Y and Z are sounded together
A = a + a = 2a
beat frequency is found to be thrice as great.
Total intensity I of resultant wave is directly
The frequency of Z is [17 Sep. 2020, Shift-II]
proportional to square of amplitude.
(a) 282 Hz (b) 286 Hz (c) 280 Hz (d) 278 Hz
i.e., I ∝ A2 ⇒ I ∝ (2a)2 ⇒ I ∝ 4a 2
Sol. (d)
Given, frequency of tuning fork X and Y are,
6. On getting reflected at a surface, the
intensity of sound is found to be decreased
n X = 280 Hz ⇒ nY = 284 Hz
by 20%. If A be the amplitude of the incident
nZ = ?
sound waves, then the amplitude of reflected
When tuning forks X and Z are sounded sound waves is [18 Sep. 2020, Shift-I]
together, then beats produced is b (assume).
4 2 2 1
∴ nZ − n X = b …(i) (a) A (b) A (c) A (d) A
5 5 5 5
Again, when Y and Z are sounded together, then
nZ − nY = 3b …(ii) Sol. (b)
Dividing Eq. (i) by Eq. (ii), we have Intensity of sound wave is directly proportional
nZ − n X b n − 280 1 to the amplitude (A).
= ⇒ Z =
nZ − nY 3b nZ − 284 3 i.e., I ∝ A2
2
⇒ 3nZ − 840 = nz − 284  Iincident   Aincident 
  =  …(i)
⇒ 3nz − nZ = 840 − 284  Ireflected   Areflected 
⇒ 2nZ = 556 ⇒ nZ = 278 Hz Iincident = I
I 4
4. What is the phase difference between two Ireflected = I − 20% of I = I − = I
particles 25 m apart in a wave represented by 5 5
equation y = 0.03 sin(π [2t − 0.01 x ])s Aincident = A
travelling in a medium? [17 Sep. 2020, Shift-II] ∴From Eq. (i), we have
π π π I A2
(a) (b) (c) (d) π =
8 4 2 4I (Areflected)2
5
Sol. (b)
4 2 2
Equation of wave is given as ⇒ Areflected = A = A
5 5
y = 0.03sin (π[2t − 0.01 x])
130 AP EAMCET Chapterwise Physics

7. The distance between the successive node 1 T


f = …(i)
and anti-node is [18 Sep. 2020, Shift-II] 2l m
λ λ 3λ where, m = mass per unit length.
(a) λ (b) (c) (d)
2 4 4 M V ⋅ρ A⋅ l
= = = ⋅ ρ = Aρ
l l l
Sol. (c)
π d2 ρ
The distance between the successive node and ⇒ m= …(ii)
λ 4
anti-node is equal to , where λ is wavelength.
4 From Eqs. (i) and (ii), we get
A 1 T 1 4T 1 T
f = = =
2l π d 2 ρ/ 4 2l π d 2 ρ l π d 2 ρ
N N 1 T 1
N ⇒ f = ⇒ f ∝
ld πρ d
λ/4
f1 d 9.3 × 10−4 9.3 f 9
∴ = 2 = = ⇒ 2 =
A f2 d1 9 × 10−4 9 f1 9.3
8. A sound source is moving towards a ∴ Percentage change in frequency
stationary listener with 1/10th of the speed f − f1 f 
= 2 × 100 =  2 − 1 × 100
of sound. The ratio of apparent to real f1  f1 
frequency is [18 Sep. 2020, Shift-II]
=  − 1 × 100 = − 3.2%
9
2 2
(a)
10
(b)
11
(c)  
11
(d)  
9  9.3 
9 10  10   10 
10. When the area of cross-section of a stretched
Sol. (a) wire is halved and tension is doubled, the
Speed of sound source towards stationary speed of propagation of transverse waves
observer along it becomes k times the initial speed.
v Then, k
vs = [22 Sep 2020, Shift-II]
10
(a) 1 (b) 4 (c) 2 (d) 8
where, v is the speed of sound.
 v 
Sol. (c)
∴Apparent frequency, f ′ =  f Speed of transverse wave in stretched wire is
 v − vs  given as
 
 v  T
= f v=
v− v  m
 10  where, T = tension in the wire
10 f ′ 10 and m = mass per unit length of wire.
⇒ f′ = f ⇒ =
9 f 9
∴ v=
T Q m = M 
M/l  l 
9. A piano wire with a diameter of 0.90 mm is
replaced by another wire of diameter 0.93 mm Tl Tl
= = [Q M = V ⋅ ρ]
of the same material. If tension of wire is M V ⋅ρ
kept the same, then the percentage change Tl
in frequency of fundamental tone is = [Q V = A ⋅ l]
A⋅ l ⋅ρ
[22 Sep. 2020, Shift-II]
(b) −3 % (c) + 3.2 % (d) − 3.2 % T T
(a) + 3% v= ⇒ v∝
Aρ A
Sol. (d)
Given, diameter, d1 = 0.9 mm = 0.9 × 10−3 m v2 T2 A
⇒ = × 1 …(i)
= 9 × 10−4 m v1 T1 A2
d2 = 0.93mm = 9.3 × 10−4 m Given, A2 =
A1
, T2 = 2T1
Frequency of fundamental tone is given as 2
Waves 131

∴ From Eq. (i), we get Sol. (a)


v2 2T1 A Given that, source is stationary, vs = 0
= × 1 = 4
v1 T1 A1 Assume, velocity of observer vo and that of sound
2 in air is v and true frequency of sound is ν0 .
v2 = 2v1 ⇒ K = 2 Heard frequency, ν = 1% higher than true
frequency
11. A closed organ pipe and an open organ pipe ν = 1.01ν0
of same length produce 2 beats/s when they
From Doppler’s effect, if source is stationary and
are set into vibrations together in fundamental observer is moving towards source along a line
mode. The length of open pipe is now halved joining them. Then, expression of heard
and that of closed pipe is doubled. The frequency is given by
number of beats produced will be v + vo 
[23 Sep. 2020, Shift-I] ν =  ν
 v  0
(a) 7 (b) 4 (c) 8 (d) 2 ν v + vo v + vo
or = or 1.01 =
Sol. (a) ν0 v v
For a closed organ pipe, the frequency of or 1.01v = v + vo or 0.01v = vo
v
fundamental mode is νc = or
vo
= 0.01 or
vo
=
1
4Lc v v 100
where, v is the velocity of sound in air and Lc is
the length of the closed organ pipe.
13. An earthquake generates both transverse S
and longitudinal P waves in the earth with
For an open organ pipe, the frequency of
speeds 4.5 km s −1 and 8.0 km s −1 , respectively.
fundamental mode is
v A seismograph records that the first P-wave
νo = arrives 3.5 minutes earlier than the first
2Lo
S-wave. From the seismograph, the epicentre
where, Lo is the length of the open pipe. of the earthquake is located at a distance.
Q Lc = Lo (given) [20 April 2019, Shift-I]
∴ νo = 2νc …(i) (a) 1080 km (b) 2468 km (c) 2160 km (d) 4320 km
νo − νc = 2 …(ii)
Sol. (c)
Solving Eqs. (i) and (ii), we get
Let the distance of epicenter of earthquake from
νo = 4 Hz and νc = 2 Hz point of observation be d.
Now, length of open pipe is halved, then its
speed of S-wave, vS = 4.5 km −1 s
frequency of fundamental mode is
speed of P-wave, vP = 8 km −1 s
v  v 
ν0 = = 2  = 2νo = 2 × 4 = 8 Hz then, d = VP tP = VS tS
 Lo 
2   2Lo 
or 8 tP = 4.5 tS
 2
4.5
When the length of the closed pipe is doubled, tP = tS …(i)
8
its frequency of fundamental mode is The first P-wave arrives 3.5 min earlier than the
v 1 v  1 first S-wave.
vc' = =   = × 2 = 1 Hz
4(2Lo) 2  4Lo  2 Hence, tS − tP = 3. 5 × 60
Hence, number of beats produced per second tS − tP = 210 …(ii)
= νo − vc′ = 8 − 1 = 7 Hz From Eq. (i), we get
4.5
12. A source is stationary and the observer is in tS − tS = 210
8
motion along a line joining the source and the 8tS − 4.5tS
= 210
observer. If the frequency heard by the 8
observer is 1% higher than the true frequency, 3. 5 tS = 210 × 8
the ratio of velocity of the observer and that of 210 × 8
tS = = 480 s
sound in air is [23 Sep. 2020, Shift-I] 3. 5
(a) 1 : 100 (b) 2 : 100 (c) 3 : 100 (d) 1 : 10 Now, d = vS tS = 4.5 × 480 = 2160 km
132 AP EAMCET Chapterwise Physics

14. An observer moves towards a stationary ∴ For observer A, frequency of siren records by
th train A is given as,
1
source of sound with a speed of the speed  v + vA 
5 f A = fS  S 
 vS 
of sound. The wavelength and frequency of
the waves emitted by the source are λ and f  v + vA 
or 5.5 = 5 S 
respectively. The apparent frequency and  vS 
wavelength heard by the observer are v
or 1.1 = 1 + A
respectively, [20 April 2019, Shift-I] vS
(a) 1.2 f, λ (b) f, 1.2 λ
or v A = 01
. vS …(i)
(c) 0.8 f, 0.8 λ (d) 1.2 f, 1.2 λ
∴ For observer B, frequency of siren records by
Sol. (a) train B is given as,
When an observer moves towards on a stationary  v + vB 
source of sound, then apparent frequency heard fB = fS  S 
 vS 
by the observer increases. The apparent frequency
heard in this situation.  v + vB   v 
6 = 5 S  or 6 = 51 + B 
 v + v0   vS   vS 
f′ =   f, (As source is stationary)
 v − vs  ⇒
6
=1+
vB
Hence, vs = 0 5 vS
v + v0 
∴ f ′ =   f ⇒
6
–1 =
vB
 v  5 vS
v
Given, v0 = 6 – 5 vB
5 ⇒ =
5 vS
 v + v
  1 vB v
f′ =  5  f = 6 f = 1.2 f ⇒ = ⇒ B = 0.2
 v  5 5 vS vS
  or vB = 0.2 vS …(ii)
Motion of the observer does not affect the Now, from Eqs. (i) and (ii), we get
wavelength reaching the observer, hence vB 0.2vS v
wavelength remains λ . or = or B = 2
v A 0.1 vS vA
Hence, the apparent frequency and wavelength
heard by the observes are respectively ,1.2 f and λ. So, the ratio of the speed of train B to that of
train A is vB : v A = 2.
15. A siren placed at a railway platform is
emitting a sound of frequency 5 kHz. A 16. The speed of a transverse wave travelling in
passenger sitting in a moving train A records a wire of length 50 cm, cross-sectional area
the frequency of the siren as 5.5 kHz. 1 mm 2 and mass 5 g is 80 ms −1 . The Young’s
modulus of the material of the wire is
During his return journey by train B he 4 × 10 11 Nm −2. The extension in the length
records the frequency of the siren as 6 kHz.
of the wire is [20 April 2019, Shift-II]
The ratio of the speed of train B to that of
(a) 8 × 10−5 m (b) 8 × 10−4 m
train A is [20 April 2019, Shift-II]
(c) 16 × 10−5 m (d) 16 × 10−4 m
242 5 11
(a) (b) 2 (c) (d)
252 6 6 Sol. (a)
Given, length of the wire, l = 50 cm
Sol. (b)
= 5 × 10−2 m
Given, frequency of sound, fs = 5 kHz
Cross-sectional area of wire, A = 1 mm 2
frequency of siren records by moving train A, = 1 × 10−6 m 2
f A = 55
. kHz and mass of the wire, m = 5 g
frequency of siren records by moving train B, Speed of transverse wave, v = 80 m/s
fB = 6 kHz Now, speed of transverse wave given as
Now, let vs = speed of sound
Waves 133

T T T ν= 2 × 109 Hz ⇒ ν = 1000 2 MHz


∴ v= = ×l⇒ v= ×l
µ m m Hence, the correct option is (b).
T v2 m
⇒ v = ×l
2
⇒l = …(i) 18. At a temperature of 27°C, two identical
m T organ pipes produce notes of frequency 140
T
Hz. If the temperature of one pipe is raised
Now, Young’s modulus, Y = A to 57.75°C, then the number of beats
∆l
produced per second is [21 April 2019, Shift-I]
l
T (a) 7 (b) 5 (c) 3 (d) 9
or ∆l = ⋅l …(ii)
AY
Sol. (a)
From Eqs. (i) and (ii), we get
Here, n0 = 140 Hz, T0 = 27° C = 300 K and
v2 m T2 = 57.75° C = 330.75K
∴ ∆l =
AY The variation of speed of sound with
Putting the given values, we get temperature,
802 × 5 × 10−3
∆l = m vt
=
T
1 × 10−6 × 4 × 1011 v0 T0
or ∆l = 8 × 10−5 m
Let v0 = speed of sound at the room
So, the extension in the length of the wire is temperature.
∆l = 8 × 10−5 m.
330.75
vt = v0 = 1.05 v0
17. At a temperature of 314 K and a pressure of 300
100 kPa, the speed of sound in a gas is Hence, the beats produced in oragan pipe.
1380 ms − 1 . The radius of each gas molecule is Number of beats,
0.5Å. The frequency of sound at which the n = nt − n0 = 140 × 1.05 − 140 = 7
wavelength of sound wave in the gas becomes Q n = vt and n = v0 
 
equal to the mean free path of the gas  t
4L
0
4L 
molecules is So, the correct option is (a).
(Boltzmann constant = 1.38 × 10 − 23 JK − 1 .)
19. A source of sound S in the form of a block
[21 April 2019, Shift-I]
kept on a smooth horizontal surface is
(a) 1000 MHz (b) 1000 2 MHz connected to a spring, as shown in the
1000
(c) MHz (d) 500 MHz figure. If the spring oscillates with an
2
amplitude of 50 cm along horizontal
Sol. (b) between the wall and the observer O, the
Given, temperature, T = 314 K, maximum frequency heard by the observer
pressure, p = 100 kPa, = 1.0 × 105 Pa, speed of is 12.5% more than the minimum
sound, v = 1380 ms− 1 and diameter of gas frequency heard by him. If the mass of the
molecule, d = 10− 10 m or radius source of sound is 100 g, the force constant
1 1
r = × 10−10 m = Å of the spring is
2 2
kT (Speed of sound in air is 340 ms − 1 )
As, mean free path, λ = …(i)
2 πd 2 p [21 April 2019, Shift-I]
v Wall
and frequency, ν = … (ii)
λ
So, from Eqs. (i) and (ii), we get
2πd 2 p × v O
ν= S
kT
Now, putting the given values, we get (a) 40 Nm− 1 (b) 80 Nm− 1
. × 10− 20 × 10 5 × 1380
2 × 314 (c) 160 Nm− 1 (d) 320 Nm− 1
=
1.38 × 10− 23 × 314
134 AP EAMCET Chapterwise Physics

Sol. (c) If the 60% of tube is dipped vertically in water


Given, amplitude of spring, A = 50 × 10 m, −2 as shown in the figure,
maximum frequency heard by observer,
nmax = 1125
. nmin , mass of sound source, 40%
m = 100 g = 010
. kg and speed of sound,
v = 340 ms− 1 . 60%
As, the apparent frequency heard by observer
for moving source,
nv
n= 0
v − vs
Now, it is a closed organ pipe of length,
Hence, for nmax , vs = vsmax 40 2L
and for nmin , vs = − vsmax l′ = ×L=
n0 v 100 5
nmax = … (i)
v − vsmax ∴Fundamental frequency of air column is
V
and nmin =
n0 v
… (ii) f ′=
v + vsmax 4l ′
V
⇒ f′ =
By dividing Eqs. (i) by (ii), we get 
4 
2L
1125
. nmin v + vsmax  5
=
nmin v − vsmax 5 V 
⇒ f′ =  
⇒ 1125
. (v − vsmax ) = (v + vsmax ) 4  2L 
⇒ . ) = 2125
v(0125 . vsmax 5 5f
340 × 0125
. ⇒ f′ = × f or f ′ =
⇒ = vsmax 4 4
2125
.
⇒ vsmax = 20 ms− 1 … (iii) 21. A train approaching a railway crossing at a
Since, as we know for the spring mass system speed of 120 km/h sounds a whistle of
k frequency 576 Hz, when it is 288 m away
vsmax = Aω = A …(iv)
m from the crossing. The frequency heard by
From Eqs. (iii) and (iv), we get the observer standing on the road
v2 m perpendicular to the track from the crossing
k = smax2 at a distance of 384 m is
A
20 × 20 × 010. (Speed of sound in air = 340 ms −1 )
⇒ k= = 160 N m− 1
0.50 × 0.50 [21 April 2019, Shift-II]
Hence, the correct option is (c). (a) 632 Hz (b) 612 Hz
(c) 512 Hz (d) 472 Hz
20. A cylindrical tube open at both ends has a
fundamental frequency f in air. The tube is Sol. (b)
dipped vertically in water so that 60% of the According to the question,
tube is in water. Then, the fundamental Observer
frequency of air column is
[21 April 2019, Shift-II]
f 5f 384 m
(a) (b)
2 4
3f θ
(c) (d) 2f Train
4 Crossing
288 m
Sol. (b)
Let us assume the length of tube is L. When a sound source is moving towards a
stationary observer with speed u and the speed
It is an open organ pipe, so fundamental
V of sound is v, then frequency heard by observer,
frequency, f =
2L  v 
f′ =  f
 v − u
Waves 135

Given, frequency of whistle sound, f = 576 Hz Given,


speed of sound, v = 340m / s speed of train notes of frequency produced by the sources A
u =120km / hr and B is 680 Hz.
speed of observer, i. e., f A and fB = 680 Hz
5 Velocity of listener moves from A to B is constant = v,
u = 120 × cosθ
18 speed of sound, vS = 340 m/s,
5  288 
 and beats per second, n = 10
= 120 × ×
18  (384) + (288) 
2 2
Now, beats per second from point A to B is given as
5 3  v + u  v − u
u = 120 × × = 20 m/s n = fA s  − fB  s 
18 5  vs   vs 
Hence, frequency heard by observer,
340 + u   340 − u 
 340  10 = 680  − 680 
f′ =   × 576 = 612 Hz  340   340 
 340 − 20 
 340 + u   340 − u  
10 = 680  −
22. When the air column of a resonance tube is   340   340  
vibrated together with a tuning fork, 3 beats 340 + u − 340 + u 
are heard per second, either the temperature 1 = 68 
 340 
of the air column is 51°C or 16°C. The frequency
1 = 68
2u 
of the tuning fork is [22 April 2019, Shift-I] 
 340 
(a) 128 Hz (b) 98 Hz (c) 105 Hz (d) 256 Hz
340 340
Sol. (c) ⇒ 2u = =
68 68 × 2
Number of beats per second when the air
column of resonance tube, n = 3 u = 2.5 m/s
If n be the frequency of tuning fork, 24. A metal wire of length 80 cm, area of
Then , n ∝ T [T → temperature] cross-section 3 mm 2 and material density
n1
=
T1 3000 kg m − 3 is joined to another metal wire
n2 T2 of length 60 cm, area of cross-section 1 mm 2
At 51°C, T1 = 273 + 51 = 324 K and material density 9000 kg-m −3.
n1 = n + 3 { frequency of tuning fork increase at
the higher temperature} The free ends of the two wires are stretched
At16° C, T2 = 273 + 16 = 289 K between two rigid supports and a tension of
n2 = n − 3 [At lower temperature, frequency of 40 N is produced in the wires. The minimum
the tuning fork decreases.] frequency of the tuning fork which can
∴From Eq. (i), produce stationary waves with the joint of
n1 324 n + 3 18 the wires as a node is
= ⇒ = [22 April 2019, Shift-II]
n2 289 n − 3 17 200 400
(a) Hz (b) Hz
18n − 54 =17n + 51 3 3
⇒ 18n −17n = 54 + 51 ⇒ n = 105 Hz 500 700
(c) Hz (d) Hz
23. Two sources A and B are producing notes of 3 3
frequency 680 Hz. A listener moves from A to Sol. (c)
B with a constant velocity v. if the speed of Given, l1 = 80 cm = 80 × 10−2 m
sound in air is 340 ms −1 , the value of y so A1 = 3mm2 = 3 × 10−6 m2
that he hears 10 beats per second is ρ1 = 3000 kg-m−3
[22 April 2019, Shift-I] l 2 = 60 cm = 60 × 10−2 m
(a) 2.0 ms −1 (b) 2.5 ms −1 A2 = 1 mm2 = 10−6 m2
(c) 3.0 ms −1 (d) 3.5 ms −1 ρ2 = 9000 kg-m −3 ⇒ T = 40 N
Sol. (b)
136 AP EAMCET Chapterwise Physics

According to the question, 26. Match the following List I with List II.
for stationary wave to be formed, frequency of P
th harmonic due to Ist metal wise = frequency of [23 April 2019, Shift I]
Q th harmonic due to 2nd metal wire List I List II
1 T 1 T
P× = Q× (A) Transverse (i) Vibrations parallel to the
2l1 ρ1 A1 2l 2 ρ2 A2 wave direction of propagation
(B) Longitudinal (ii) Vibrations perpendicular to
P l1 ρ1 A1 80 × 10−2 3000 × 3 × 10−6
or = = wave the direction of propagation
Q l2 ρ2 A2 60 × 10−2 9000 × 10−6
(C) Beats (iii) Superposition of waves
P 4 travelling in the opposite
or = directions
Q 3
(D) Stationary (iv) Superposition of waves
Hence, p is 4th harmonic and Q is 3rd harmonic. waves travelling in same direction
So, the minimum frequency of the tuning fork,
E 0 produce stationary waves with given wire of The correct answer is
2 metals, frequency of tuning fork = frequency
A B C D A B C D
of Pth harmonic/Q th harmonic of 2 respective
wire. (a) (ii) (i) (iii) (iv) (b) (ii) (i) (iv) (iii)
(c) (iii) (iv) (i) (ii) (d) (iv) (i) (ii) (iii)
1 T
n= P×
2l1 ρ1 A1 Sol. (b)
A→ In transverse wave, vibrations are
1 40 perpendicular to the direction of propagation.
= 4×
2 × 80 × 10−2 3000 × 3 × 10−6 B→ In longitudinal wave, the vibrations are
500 parallel to the direction of propagation.
= Hz
3 C→ Beats are produced due to superposition of
the waves travelling in same direction.
25. A source producing sound of frequency
D→ Stationary waves are produced due to the
720 Hz is falling freely from the top of a superposition of waves, travelling in opposite
tower of height 20 m. The frequency of direction.
sound heard by an observer on the top of the Hence, the correct code is given by option (b).
tower when the source just reaches the
ground is 27. A police car moving at 22 ms −1 chases a
(Acceleration due to gravity, g = 10 ms − 2 and motor cyclist. The police man sounds horn at
speed of sound in air = 340 ms − 1 ) 176 Hz, while both of them move towards a
stationary siren of frequency 165 Hz. If the
[22 April 2019, Shift-II] number of beats heard by the motor cyclist
(a) 660 Hz (b) 680 Hz per second is zero, then the speed of
(c) 740 Hz (d) 760 Hz motorcycle is (Speed of sound in air
Sol. (b) = 330 ms −1 ) [23 April 2019, Shift-I]
Q Final velocity of source, vs2 = us2 + 2gh (a) 33 ms −1 (b) 22 ms −1
⇒ vs = 2gh (Q us = 0) (c) 44 ms −1 (d) 11 ms −1
or vs = 2 × 10 × 20 = 20 m / s Sol. (b)
According to the question, we draw the
[Given, h = 20 m, g = 10 m/s ] 2
following situation,
Frequency of sound heard by an observer, f2=165 Hz
 v  VS2=0
n′ = n 
 v + vs  f1=176 Hz
Given, n = 720 Hz, v = 340 ms− 1 vs 1 v0
 340 
⇒ n′ = 720  = 680 Hz
 340 + 20  A B wall
Waves 137

Given, velocity of car, vs 1 = 22 ms−1 Dividing Eq (i) by Eq. (ii), we get


speed of the sound in air, v = 330 ms −1 v2 M(a 2 + g2)1 / 2 66
= =
apparent frequency by cyclist from the police car, v1 Mg 60
v − v0 330 − v0
f1′ = f1 ⇒ f1′ = × 176 (a 2 + g2)1 / 2 121
v − vs 330 − 22 =
g 100
330 − v0 Squaring, we get
= × 176 2
308
⇒ a 2 + g2 = 
121 
 ⇒ a = 146 . 41 − 100
2
apparent frequency by cyclist from the stationary  100 
siren, ⇒ a 2 = 46. 41
v + v0 330 + v0 So, a = 6 . 8 ms −2
f2′ = f2 = × 165
v 330
Number of beats heard by the cyclist, 29. A reflector is moving with 20 ms −1 towards a
n = f1′ − f2′ = 0 (Given, n = 0) stationary source of sound. If the source is
f1′ = f2′ producing sound waves of 160 Hz, then the
176 165 wavelength of the reflected wave is
⇒ (330 − v0) = (330 + v0)
308 330 (speed of sound in air is 340 ms −1 )
⇒ v0 = 22 ms−1 [22 April 2018, Shift-I]
∴Hence, the speed of motorcycle is 22ms −1 . 17 17 17 17
(a) m (b) m (c) m (d) m
So, the correct option is (b). 8 11 9 16

28. A solid ball is suspended from the ceiling of a Sol. (c)


motor car through a light string. A transverse Wavelength of the reflected wave is
pulse travels at the speed 60 cm −1 on the  v − vs   v − vs  v
λ′ =   λ=  
string, when the car is at rest. When the car  v + vs   v + vs  ν
accelerates on a horizontal road, then speed 340 − 20 340 120 340
= × = ×
of the pulse is 66 cm −1 . The acceleration of 340 + 20 160 360 160
the car is nearly (g = 10 ms −2) λ′ =
17
m
[22 April 2018, Shift-I] 9
(a) 4.3 ms −2 (b) 2.9 ms −2 30. An object of density 2000 kg - m −3 is hung from
(c) 6.8 ms −2 (d) 5.5 ms −2
a thin light wire. The fundamental frequency
Sol. (c) of the transverse waves in the wire is 200 Hz.
When car is at rest, tension in string is T = mg. If the object is immersed in water such that
T = Mg = mv12 half of its volume is submerged, then the
T Mg fundamental frequency of the transverse
V1 = = ...(i) waves in the wire is [22 April 2018, Shift-II]
r µ
(a) 200 Hz (b) 173.2 Hz (c) 100 Hz (d) 141.4 Hz

T T Sol. (b)
Ma Fundamental frequency, initially
1 T 1 Vg(2000)
n1 = =
2l µ 2l µ

Mg Mg Fundamental frequency, finally

Vg 2000 −
(i) (ii) 1000 

1  2 
When car is accelerating tension, n2 =
2l µ
T= M(a 2 + g2)1 / 2
(here we applied loss of weight due to upthrust)
M(a 2 + g2)1 / 2 1 Vg × 1500
∴ v2 = =
µ 2l µ
138 AP EAMCET Chapterwise Physics

n1 2000 4 n′ T′ l
⇒ = = ⇒ = × … (i)
n2 1500 3 n T l′
n1 × 3 144T
⇒ n2 = = 100 3 Hz = 1732
. Hz Now, T ′ = T + 0. 44T =
2 100
x  100 − x 
and l′ = l − l= l
31. An observer and a source emitting sound of 100  100 
frequency 120 Hz are on the X -axis. The n′ 2
observer is stationary while the source of Also, =
n 1
sound is in motion given by the equation
2 144T l 12 100
x = 3sinωt (x is in metres and t is in = × = ×
1 100T  100 − x  l 10 100 − x
seconds). If the difference between the  
 100 
maximum and minimum frequencies of the
sound observed by the observers is 22 Hz, or (100 − x) = 60 or x = 40
then the value of ω is (speed of sound in air 33. A small source of sound vibrating at a
= 330 ms −1 ) [22 April 2018, Shift-II] frequency 500 Hz is rotated along a circle of
(a) 33 rad s −1 (b) 36 rad s −1 100
radius cm at a constant angular speed of
(c) 20 rad s −1 (d) 10 rad s −1 π
Sol. (d) 5 revolutions per second. The minimum and
Source, f=120Hz maximum frequency of the sound observed
Stationary
observer by a listener situation in the plane of the
circle is (speed of sound is 332 ms − 1 )
v0=0
[23 April 2018, Shift-I]
x=3 sin ωt
(a) 338.5 Hz, 612.5 Hz (b) 485.4 Hz, 535.6 Hz
Instantaneous speed of source is
dx (c) 435.3 Hz, 565.6 Hz (d) 485.4 Hz, 515.5 Hz
v= = 3ωsinωt
dt Sol. (d)
Difference between maximum and minimum The linear velocity of whistle, vs = rω
frequencies is 22 Hz. 100
vs = cm × 5 rev / s
 v   v  π
So, fmax − fmin = f   − f  = 22 1
 v − vs   v + vs  = m × 2π × 5 rev / s = 10 m / s
π
 v v 
⇒ f −  = 22 … (i) When whistle approaches the listener, then the
 v − vs v + vs  frequency of sound heard will be maximum and
Now here, f = 120 Hz, v = 330 ms− 1 , vs = 3ω minimum.
Substituting these values in Eq (i), we get  v  332
nmax = n  = 500 × = 515. 5 Hz
 330 330  − 1  v − vs  322
120  −  = 22 ⇒ ω = 10 s
 330 − 3ω 330 + 3ω   v  332
nmin = n  = 500 × = 4854
. Hz.
 v + vs  342
32. If the length of a stretched string is shortened
by x% and the tension is increased by 44%. 34. A progressive wave of frequency 500 Hz is
Then the ratio of the final and initial travelling with a velocity of 360 ms −1 . The
fundamental frequencies is 1 : 2, then the distance between the two points, having a
value of x is [23 April 2018, Shift-I] phase difference of 60 ° is ...........
(a) 20 (b) 30 (c) 40 (d) 60 [23 April 2018, Shift-II]
Sol. (c) (a) 1.2 m (b) 12 m (c) 0.12 m (d) 0.012 m
1 T Sol. (c)
Fundamental frequency for string, n =
2l m f = 500Hz, v = 360 ms −1
So, n∝ T /l λ= =
v 360
f 500
Waves 139

Now, a phase difference of 60° corresponds to a Sol. (c)


60
path difference of × λ. Frequency in stretched string,
360
n n T
So, distance between 2 particles is f = ⋅v =
2l 2l m
60 60 360
d= × λ= × = 012
. m where, l = length, T = tension and
360 360 500
m = mass per unit length
35. A source S emitting sound of frequency 288 πr 2 l ⋅ d
So, m= , where, d = density and
Hz is fixed on block B which is attached to l
the free end of a spring S2 and an observer O m = πr 2 d.
is on block A which is attached to the free Given, f A (first overtone, n= 2)
end of spring S1 as shown in the figure. The = fB (second overtone, n= 3)
blocks A and B are simultaneously displaced
2 T 3 T
towards each other through a distance of 0.5 ⇒ =
m and then left to oscillate. If the angular 2l A m A 2l B mB
velocity of each blocks is 40 rad s −1 , then the 1 3
⇒ =
maximum frequency observed by the l A πrA2 d 2l B πrB2 d
observer is (speed of sound in air is l A 2 rB 2 1 lA 1
340 ms −1 ) [23 April 2018, Shift-II] ⇒ = × = × ⇒ =
l B 3 rA 3 2 lB 3
O S
37. A open pipe of length l is vibrating in 3rd
overtone with maximum amplitude A. The
l
S1 S2 amplitude at a distance of from any open
16
A B
end is [24 April 2018, Shift-I]
(a) 288 Hz (b) 310 Hz
A 3A
(c) 324 Hz (d) 256 Hz (a) A (b) 0 (c) (d)
2 2
Sol. (c)
Frequency is maximum, when both O and ‘S’ are Sol. (c)
moving towards each other with maximum For open organ pipe, wavelength of nth overtone
speeds. 2l
λn = (n = 3)
Now, maximum speed of O = maximum speed of n+1
S [and these two occur together]. 2l l
λ= =
= vm= aω = 0.5 × 40 = 20 ms −1 4 2
So, maximum frequency As pipe is open, so antinode (maximum
v + vobserver   340 + 20  amplitude) will form at open ends.
= fmax = f  sound  = 288 ×  
 vsound − vsource   340 − 20  At l = 0, amplitude = A
l λ
= 324 Hz At or distance, amplitude, R = A cos φ
16 8
36. Two uniform stretched steel strings A and B where, φ = phase angle.
are vibrating under the same tension. The 2π λ π
φ= × = = 45°
first overtone of A is equal to the second λ 8 4
overtone of B. If the radius of A is twice that R = A cos 45°
of B, then the ratio of the lengths of the A
R=
strings is [24 April 2018, Shift-I] 2
(a) 2 : 3 (b) 1 : 2 (c) 1 : 3 (d) 1 : 4
14
Electric Charges and Fields
1. Four closed surfaces S1 , S2 , S3 and S4 together
with charges + q, − q and −2q are shown. a b c d
Through which one of the surfaces the net
flux is zero? [17 Sep. 2020, Shift-I]

(B) O θ
+q –q π/2 π 3π/2 2π 5π/2 3π
S2
S3
S1
–2q
S4

(a) (a) (b) (c) (c) (b) (d) (d)


(a) S1 (b) S 2 (c) S 3 (d) S 4
Sol. (c)
Sol. (b) Torque on electric dipole placed in uniform
According to given diagram, net charge enclosed electric field E is given as
through the surface S2 is zero. τ = pE sinθ …(i)
Hence, according to Gauss’s law, where, p = electric dipole moment
Charge inclosed θ = angle measured with respect to
Net flux through surface, S2 = and
ε0 direction of electric field.
0 From Eq. (i),
= =0
ε0 When θ = 0°⇒ τ = pEsin 0° = 0
π π
2. An electric dipole is situated in an electric When θ = ⇒ τ = pE sin = pE
2 2
field as shown in figure. The dipole and
electric field are both in the plane of paper. When θ = π ⇒ τ = pEsin π = 0
3π 3π
The dipole is rotated about an axis, When θ = ⇒ τ = pE sin = − pE
perpendicular to the paper at point A in 2 2
anti-clockwise direction. If the angle of When θ = 2π ⇒ τ = pE sin 2π = 0
rotation is measured with respect to the Hence, graph between τ and θ is given as
direction of the electric field, then the torque τ
for different values of the angle of rotation θ
+pE
is correctly represented by which graph
among a , b, c , d given in Fig (b)? λ 2π
[17 Sep. 2020, Shift-I] O x
λ/2 3π/2
A
–pE
–q +q b
(A)
Electric Charges and Fields 141

3. Three charges 4 q, Q and q are placed at


l
positions 0, and l respectively along a θ
2
straight line. If the resultant force on q is
zero, then Q is equal to [18 Sep. 2020, Shift-I] E
q
(a) − q (b) − 2q (c) − (d) 4qt
2
Sol. (a)
The given arrangement of charges is shown in
following figure, (a) q = 11 × 10− 8 C (b) T = 5.55 × 10− 3 N
4q Q q (c) q = 12 × 10− 9 C (d) T = 4.55 × 10− 3 N

A l/2 B l/2 C Sol. (b)


Resultant force on charge q is zero. Mass of cork ball, m = 1 g = 10−3 kg
4q ⋅ q KQq E = (3$i + 5$j) × 105 NC −1
i.e., K⋅ 2 + =0
l (l / 2) 2 θ = 37°
4q 4Q
⇒ + 2 =0
l2 l θ
q + Q=0
T Ey
Q= −q
1 2 3 4 −5 θ E
4. Five point charges , , , and nC are
π π π π π T sin θ Ex
located inside a pyramid. The total electric
flux through the surface of the pyramid is mg
[18 Sep. 2020, Shift-II] Force on cork ball due to electric field E
(a) 180 Nm 2 C − 1 (b) 90 Nm 2 C − 1 F = qE
(c) 55 Nm 2 C − 1 (d) 5 Nm 2 C − 1 According to given diagram,
Sol. (a) resolving all forces in two given direction,
According to Gauss’s law, total electric flux T sinθ = qE x …(i)
Total charge T cosθ + qE y = mg
through the surface of pyramid =
ε0 ⇒ T cosθ = mg − qE y …(ii)
 1 + 2 + 3 + 4 − 5  × 10−9 From Eqs. (i) and (ii), we have
  T sinθ
Σq  π π π π π  =
qE x
= =
ε0 ε0 T cosθ mg − qE y
5 5 × 4 × 10−9 20 × 10−9
= × 10−9 = = sin37°=0.6
πε0 4 πε0 4 πε0 6 3
= =
= 20 × 9 × 109 × 10−9 = 180 Nm 2 C −1 5
3 10 5

5. A charged cork ball having mass 1 g and


37º
charge q is suspended on a light string in a 4
uniform electric field as shown in figure. The
ball is in equilibrium at θ = 37 °, when value tanθ =
qE x
of electric field is E = (3 $i + 5 $j) × 10 5 NC −1 . mg − qE y
(Assume, T as tension in the string.) Which q × 3 × 105
⇒ tan 37° =
of the following options are correct? (Given, 10 −3
× 10 − q × 5 × 105
sin 37 ° = 0.60 and g = 10 ms −2) 3q × 105
3
[21 Sep. 2020, Shift-I] = −2
4 10 − 5q × 105
142 AP EAMCET Chapterwise Physics

⇒ 3 × 10−2 − 15q × 105 = 4 × 3q × 105 4 π ε0


⇒ λ = 2π ε0 r E = ⋅ rE
0.03 − 15 × 105 q = 12 × 105 q ⇒ q = 11 . × 10−8 C 2
1
From Eq. (i), we get = × 2 × 10−2 × 9 × 104
Tsin 37° = 1.1 × 10−8 × 3 × 105 2 × 9 × 109
T × 0.6 = 3.3 × 10−3 = 10−7 Cm −1 = 0.1 × 10−6 Cm −1
T=
3.3
× 10−3 = 5.5 × 10−3 N = 0.1 µ Cm −1
0.6
9. As shown in the figure below, a point charge
6. An electric dipole with dipole moment p is q moves from point P to a point S traversing a
placed with its axis at 30° to a uniform path PQRS in a uniform electric field E. The
electric field. The work done in rotating the electric field is directed along a direction
dipole to a position where its axis is parallel to X -axis. The coordinates of, P , Q, R
perpendicular to the field is [21 Sep. 2020, Shift-I] and S are (a , b, 0), (2 a , 0 , 0) (a , − b, 0) and
2 pE 3 pE (0 , 0 , 0), respectively. What is the work done by
(a) 2 pE (b) (c) (d) 0 the field in the process? [23 Sep. 2020, Shift-I]
3 2
Y
Sol. (c)
Given, for an electric dipole,
θ1 = 30° E
θ2 = 90° P
∴Work done in rotating the dipole from θ1 = 30°
to a position, where its axis is perpendicular
(θ2 = 90°) to electric field is given as S Q
X
W = pE (cosθ1 − cosθ2)
= pE(cos 30° − cos 90°)
 3  3 R
= pE  − 0 = pE
 2  2
(a) qEa (b) − qEa (c) 0 (d) qEb
7. Electric field lines of force of a positive point
Sol. (b)
charge are [21 Sep. 2020, Shift-II]
As we know that, work done by the uniform
(a) radially outward (b) radially inward electric field in moving a charge depends only
(c) parallel (d) converged on its final position and initial position.
Sol. (a) Displacement of charged particle in uniform
Electric field lines of forces of a positive point electric field , r = rS − rP
charge emerge radially outward and going to $ ) − (a$i + b$j + 0k
= (0$i + 0$j + 0k $)
infinity.
= − ai − bj
$ $
8. An infinite line charge produces a field of ∴Work done by electric field, W = ∫ Felec ⋅ dr
9 × 10 4 NC −1 at a distance of 2 cm. Its linear
charge density is [22 Sep. 2020, Shift-II]
Here, Felec = qE = qE $i (which is uniform)
(a) 0.1 µC m −1 (b) 0.2 µC m −1 W = Felec ⋅ ∫ dr = Felec ⋅ r
(c) 10 µC m −1 (d) 20 µC m −1
= (qE$i) ⋅ (− a$i − b$j)
Sol. (a) W = − qEa
Electric field produced by infinite line charge,
E = 9 × 104 NC −1 10. The number of electric lines that emerge
r = 2cm = 2 × 10−2 m from a finite charge +q is [23 Sep. 2020, Shift-I]
Linear charge density, λ = ? (a) infinite
We know that, (b) any finite number but not equal to zero
λ (c) proportional to the charge
E=
2 π ε0 r (d) zero
Electric Charges and Fields 143

Sol. (c) of friction is 0.2, then the acceleration of the


The number of electric field lines emerging from block is nearly, (Acceleration due to gravity,
any finite charge is any finite number and it is g = 10 ms −2) [20 April 2019, Shift-I]
proportional to the magnitude of charge, i.e.
higher value of charge will have more electric m
field lines associated to it.
q
11. Three charges of each magnitude
E=100 Vm–1
100 µC are placed at the corners A, B and C of
an equilateral triangle of side 4 m. If the h
charges at points A and C are positive and the
charge at point B is negative, then the 30°
magnitude of total force acting on the charge
at C and angle made by it with AC are (a) 1.3 ms −2 (b) 2.3 ms −2
[20 April 2019, Shift-I] (c) 3.3 ms −2 (d) 4.3 ms −2
(a) 5.625 N, 60° (b) 0.5625 N, 60° Sol. (b)
(c) 5.625 N, 30° (d) 0.5625 N, 30° According to the question, an inclined plane is
making an angle of 30° with the horizontal,
Sol. (a) placed in a uniform electric field of 100 nm −1 . It
According to the question, we can draw the can be such in the figure that a block of mass m
following diagram, is sliding down from rest at height h.
F F
3 0°=
60° os 0°
C Fnet N qE
c
o s3
gc
60°
m µm

F q
a
0 °
n3
si mg cos 30°
A B mg 30°
4m mg
30°
From the question, it clear that the charge on
the each corner is 100 µC. From the above diagram, the total force F acting
along inclined plane.
So, Fnet = F
kQ Q From fig,
= 12 2 ...(i) m gsin 30° − µ mg cos 30° − qE cos 30° = ma = F
r
Given, Q1 = Q2 = 100 µC =100 × 10–6 C, Given,
[1µC = 10−6 C] µ = 0.2 , m = 1 kg, , q = 0.01 C and h = 1 m
and r = 4m Putting these values, we get
Putting the given values in Eq. (i), we get 1 3 3
10 × – 0.2 × 10 × − 0.01 × 100 × =a
9 × 109 × (100 × 10−6)2 2 2 2
=
(4)2 a = 5 − 3 − 0.5 3 ≈ 2.3 ms −2
Fnet = 5625
. N, 60° Hence, the acceleration of the block is nearly,
2.3 ms −2 .
12. An inclined plane making an angle 30° with
the horizontal is placed in a uniform 13. Three infinitely long charged sheets are placed
as shown in the figure. The electric force
horizontal electric field of 100 Vm −1 as shown
acting on a charge −q placed at the point P is
in the figure. A small block of mass 1 kg and
(σ = surface charge density, ε 0 = permittivity
charge, 0.01C is allowed to slide down from
of the free space) [20 April 2019, Shift-II]
rest from a height, h = 1 m. If the coefficient
144 AP EAMCET Chapterwise Physics

at all or a charge of +10e or−6e but not a free


particle with a charge of, say 3.57e.
σ z=2a From above discussion, it is cleared that electric
charge is quantised, which means charge can be
–2σ z=a written as integral multiple of charge an
x electron but it cannot be written as integral
multiple of charge on half electron. It is because
–σ z=–a half of the charge of an electron does not exist.
Hence, assertion and reason both are correct
2σ $ 2σ $ 4σ $ 4σ $
(a) + k (b) − k (c) + k (d) − k and assertion is correct explanation.
ε0 ε0 ε0 ε0
15. The electric field intensity at a point on the
Sol. (b) axis of an electric dipole in air is 4 NC− 1 .
According to the question,
Then the electric field intensity at a point on
z
the equatorial line which is at a distance
equal to twice the distance on the axial line
σ z=2a and if the dipole is in a medium of dielectric
P
constant 4 is [21 April 2019, Shift-I]
–2σ z=a 1 1
(a) 1 NC − 1 (b) NC − 1 (c) 16 NC − 1 (d) NC − 1
x 8 16
Sol. (d)
–σ z=–a
As, electric field intensity on the axis,
2kP 2kP
Given, surface charge density = σ E axis = 3 ⇒ 4 = 3 for r >> a
r r
permittivity of the free space = ε0
(Q given, E axis = 4)
Now, the electric force acting on a charge −q at a kP
place the point P, ⇒ =2 …(i)
r3
σ $ 2σ $ σ $
∴ FP = − k− k− k Electric field intensity on equatorial line,
2ε0 2 ε0 2 ε0 k′ P
E eq = 3
4σ $ − 2σ $ r1
or =− k or Fp = k
2 ε0 ε0 k
where, r1 = 2r and k′ =
4
14. Assertion (A) Half of the charge of an kP
So, E eq =
electron does not exist. 4 × 8r 3
Reason (R) Electric charge is quantized. Now, from the Eq. (i), we get
[20 April 2019, Shift-II] 2 1
E eq = = NC− 1
(a) Both (A) and (R) are correct and (R) is the correct 4 × 8 16
explanation of (A ).
Hence, the correct option is (d).
(b) Both (A) and (B) are correct but (R) is not the
correct explanation of (A). 16. Two small spheres of each charge q, mass m
(c) (A) is correct but (R) is not correct. and material density d are suspended from a
(d) (A) is not correct but (R) is correct. fixed point with the help of inextensible
Sol. (a) light thread. When the spheres are in air, the
The charge on any body can be expressed as the angle between the threads is 90°. When the
integral multiple of basic unit of charge i.e., spheres are suspended in a liquid of density
charge on one electron. This phenomena is 2
called quantisation of electric charge. d, the angle between the threads is 60°. The
3
It can be written as q= ± ne, where n = 1,2,3..... value of dielectric constant of the liquid is
Charge is said to be quantised because it can
have only discrete values rather than any [21 April 2019, Shift-I]
arbitrary value, i.e., free particle has no charge (a) 6 3 (b) 2 5 (c) 5 3 (d) 7 2
Electric Charges and Fields 145

Sol. (a) The maximum value of v so that the electron


Key Idea At any instant, does not hit plate B is
T cosθ = mg and T sinθ = Fe ⇒ tanθ mg = Fe (Assume gravity free space, charge of
electron = 16 . × 10 −19 C and mass of electron
−31
= 9 .1 × 10 kg) [21 April 2019, Shift-II]
θ
T l B
T E
v
φ
Fe Fe 30º
A
Electron
mg mg
(a) 400 km s −1 (b) 3200 km s −1
(c) 800 km s −1 (d) 1600 km s −1
2
kq
Fe =
r2 Sol. (d)
r = 2l cos θ [from Fig.] . Vm−1
Given, electric field, E = 455
F
tan θ = e charge of electron, q e = 1.6 × 10−19 C
mg
mass of electron, me = 91 . × 10−31 Kg
So, for θ = 45° and φ = 45° According to the question,
2l Kq 2
r = 2 = 2 l ⇒ Fe = B
v 2l
kq 2
tan 45° = (Q tan 45° = 1) 4 cm
v cos 30º
2l mg E
v
2
kq 30º
mg = …(i)
2l A Electron
For, θ = 30° and φ = 60°
1 Maximum height in a projectile motion,
r = 2l × = l
2 v2 sin2 θ 4 v2 (sin 30)2
H= ⇒ = ...(i)
kq 2
k′ q 2 2a 100 2a
So , Fe′ = 2 ⇒ tan 30° = 2
l l − m′ g Q H = 4 cm = 4 m
 
 100 
As, the sphere is suspended in a liquid of density
2 Force in electric field, F = q e E
d, then the observed weight of the body,
3 qE
or me a = q e E or a = e ...(ii)
m′ = V  d −  =
2d m me
[Q m = V ⋅ d]
 3 3 From Eqs. (i) and (ii), we get
1 3k′ q 2 3 3k′ q 2 ∴
4
=
u2 me
or v =  4 × 8  qeE
= 2 ⇒ mg = …(ii)  
3 l mg l2 100 2(4)q e E  100  me
So, from Eq. (i) and (ii), we get 4 × 8 1.6 × 10−19 × 455
.
k 1 1 v= × = 1600 kms−1
3 3 k′ = ⇒ k′ = , K = 100 . × 10−31
91
2 4 π ε0 εr 4 π ε0
Hence, the maximum value of v so that the
⇒ εr = 6 3 electron does not hit plate B is 1600 kms −1 .
17. Two long parallel plates A and B are separated 18. Two identical charged spheres separated by a
by a distance of 4 cm with an electric field of distance repel each other with a force F. If
45.5 Vm −1 between the plates normally from 10% of electrons are transferred from one
plate A to plate B, as shown in the figure. An sphere of the other, then the force between
electron is projected from plate A with velocity them becomes [21 April 2019, Shift-II]
v at an angle of 30° with the surface of plate A. (a) F (b) 1.21 F (c) 0.99 F (d) 0.81 F
146 AP EAMCET Chapterwise Physics

Sol. (c) the plane of the figure at a large distance


According to the question, x (>> a) from O is
+q +q  1 
Let, = k
I II  4 π ∈0  [22 April 2019, Shift-I]
+Q –Q

r
+Q –Q
Force between these charged sphere,
+Q –Q
k(q)(q) kq 2
F= = 2
r2 r 4Qa 2Qa 8Qa
(a) k × (b) k × 3 (c) k × 3 (d) 0
If 10% of electrons are transferred from one x3 x x
sphere to the other then the charge on one
Sol. (a)
sphere become  q +  =
q 11q
and charge on
 10  10 According to the question,
other sphere become  q −  = .
q 9q
 –Q
10  10 +Q B K
Hence, force between them a (x >> a)
+Q –Q
k 
9 q   11 q 
  O
 10   10   kq 2   99 
F′ = =  2  
r2  r   100  +QA –Q

⇒ F′ = 0.99 F
Let OK = x due to AB, force at the point K is
19. Assertion (A) The work done by the only along AB
electrostatic force is zero when a point 2kQa
i.e., FK = 3
charge moves in a circular path around
(a 2 + x 2) 2
another charge.
∴There are three such FK′ s so the net resultant of
Reason (R) The dot product of force and these three,
displacement vectors gives work done. 4kQa 4kQa
[21 April 2019, Shift-II]
Fnet = 3
= 3

(a 2 + x 2) 2 x 3 1 + a  2
2
(a) Both (A) and (R) are correct and (R) is the correct  
explanation of (A).  x2 
(b) Both (A) and (R) are correct but (R) is not the 4kQa  a2 
correct explanation of (A). Fnet = ∴ 2 = 0 , (x > > a)
x3  x 
(c) (A) is correct but (R) is not correct.
(d) (A) is not correct but (R) is correct. 21. A ball of mass 1 g having a charge of 20 µC
Sol. (a) is tied to one end of a string of length 0.9 m
Q Work done, W = F⋅ s = Fs cosθ can rotate in a vertical plane in a uniform
work done is zero when force acts at right angle electric field 100 NC−1 directed upwards. The
to the direction of motion of charges hence, minimum horizontal velocity that must be
W = Fs cos 90° = 0. The work done by the given to the ball at the lowest position so
electrostatic force is zero when a point charge
moves in a circular path around another charge.
that it completes the vertical circle is
Since, the electrostatic force acts at right angle
(Let , g =10ms −2) [22 April 2019, Shift-I]
to the direction of motion of the charges. (a) 9 ms −1 (b) 18 ms −1
(c) 36 ms −1 (d) 6 ms −1
20. Six point charges each of the magnitude Q
are placed at the vertices of a regular Sol. (d)
1
hexagon of side a as shown in the figure. Given, mass of ball, m =1 g = kg
Electric field intensity on the line passing 103
through the centre O and perpendicular to charge on the ball, q = 20 µ C = 20 × 10−6 C,
Electric Charges and Fields 147

length of the string centre of one end, r = 0.9 m deflection angle, θ = 2°


electric field, E = 100 N/C Now, dipole moment p = magnitude of any one
and acceleration due to gravity, g = 10 m / s2 charge × separation distance between the two
Now, according to the question, charges
force exerted by the electric field due to charge p = q × 2d [Q 2 d = 1 m]
on ball is given as,
∴ p = 1 × 10− 6 × 1 = 10− 6 cm … (i)
∴Force = Charge on the ball × Electric field
∴Moment of inertia about the separation
F = qE ⇒ F = 20 × 10−6 × 100 distance is given as
F1 = 2 × 10−3 N …(i) ∴ I = 2 (md 2) = 2 × 1 × (1 / 2)2
Now, force due to the gravitation,
1 Moment of inertia, I = 1 / 2 ...(ii)
F2 = mg = 3 × 10 = 10 × 10−3 N …(ii) Time-period of oscillation is given as,
10
Net effective force on the ball, I
T = 2π
Feff = F2 − F1 pE
From Eqs. (ii) and (i), we get ∴Here,
Feff =10 × 10−3 − 2 × 10−3 ⇒ feff = 8 × 10−3 N I = moment of inertia
p = dipole moment
and E = electric field
r=0.9 m 1/ 2 1000 10
T = 2π = 2π = 2π ×
E 10− 6 × 20 × 103 20 × 2 2
r
T = 10π
mg Then, the time taken to come to equilibrium
1g, 20 µC
position again is
T 10 π
Now, effective gravitational acceleration at t= = = 2. 5 π s
lowest position is given as, 4 4
Feff = mgeff 23. In the following four cases, charged particles
Feff 8 × 10−3 are at equal distances from the origin.
geff = = = 8m / s2
m 10−3 Arranged them in the descending order of
The minimum horizontal velocity that mut be magnitude of the net electric field at the
given to the ball at the lowest position is, origin. [22 April 2019, Shift-II]
9
v = 5geff r ⇒ v = 5× 8 ×
10 Y Y
v = 4 × 9 = 36 ⇒ v = 6 m/s 5q 3q

22. A dipole has two charges + 1 µC and − 1 µC 1.


2q 0 –3q
X 2.
2q 0 –q X
and each of mass 1 kg. The separation
between the charges is 1 m. An electric field
20 × 10 3 Vm − 1 is applied on the dipole. If the Y Y
dipole is deflected through 2° from the
equilibrium position, then the time taken by 3. X 4. X
4q 0 –2q 3q –q
it to come to equilibrium position again is
[22 April 2019, Shift-II]
(a) 2.5 π s (b) 5 π s
(c) 15 π s (d) π s (a) 1, 2, 3, 4 (b) 2, 1, 3, 4
(c) 1, 3, 2, 4 (d) 4, 3, 2, 1
Sol. (a)
Sol. (c)
Given, mass of each charge, m = 1 kg
If electric due to charge | q | at origin is E then
separation distance between charges, 2 d = 1m
field due to charges| 2q | ; | 3q |,| 4q | and| 5q |
electric field, E = 20 × 103 V/m and are respectively 2E , 3E , 4E and 5E respectively.
148 AP EAMCET Chapterwise Physics

Y Y 20
Q′2 = Q2 + Q1 × = 1.86 + 372
. × 0.20
5q 100
⇒ Q′2 = 2⋅ 604 µC
1. X ⇒ X 80
2q 0 – 3q 0 2E 3E and Q′1 = Q1 × = 2 .976 µC
100
k k
5E Hence, F2 = 2 Q′1 ⋅ Q′2 = 2 2. 976 × 2.604 × 10−12
R R
Y Y
k
3q F2 = 2 7.7495
R
2q –q
2. X ⇒ X So, % increment in F2 ,
0 0 2E E
F2 − F1 k (7.7495 − 6.9192) × 10−12
× 100 = 2 × 100
3E F1 R k −12
× 6.9192 × 1 0
Y Y R2
= 12%
3. X ⇒ X
Hence, the correct option is (b).
4q 0 –2 q 4E 2E
25. ABC is a right angled triangle in which
AB = 3 cm, BC = 4 cm and right angle is at B.
Y Y Three charges +15µC, +12µC and − 20 µC are
placed respectively at A , B and C. The force
acting on the charge at B is [23 April 2019, Shift-I]
4. X ⇒ X (a) 1250 N (b) 3500 N (c) 1200 N (d) 2250 N
3q –q 3E E
Sol. (d)
According to the question, 3 charge particles are
So, the electric field, E1 = (5E)2 + (5E)2 = 5 2 E placed at the vertices of a right angle triangle
ABC as shown in the figure below,
E 2 = (3E)2 + (3E)2 = 3 2 E
C
E 3 = 4E + 2E = 6E
E 4 = 3E + E = 4E
4 FBC
Hence, descending order of magnitude of net FB 4
electric field is 1, 3, 2, 4.
90°
FAB
24. Two particles with charges + 3.72 µC and A 3 B
+186
. µC are some distance apart. If 20% of
the charge is transferred from first particle to Where, Q A = + 15µC , QB = 12µC
second particle then the electrostatic force and QC = − 20 µC
between them is [23 April 2019, Shift-I] kQ AQB
Now, the force, FAB = 2
(a) decreases by 12% (b) increases by 12% rAB
(c) increases by 4% (d) decreases by 4% k15 × 12 × 10−12
⇒ FAB = = k 20 × 10−8 N
Sol. (b) 9 × 10−4
Given, charge on the first particle, Q1 = + 372
. µC Similarly,
and charge on second particle, Q2 = 1.86 µC k × 20 × 12 × 10−12
FBC = = k15 × 10−8 N
Then the electrostatic force between charges, 16 × 10−4
kQ Q k
F1 = 12 2 = 2 (372 . × 1.86) 10−12 N Now, the resultant, FB = 2
FAB + FBC2 (Q θ = 90°)
R R
FB = 9 × 10 [ 20 + 15 ] = 2250 N
2 2
k
F1 = 2 (6.9192 × 10−12)
R Hence, the force acting on the charge at point B
If 20% of Q1 is given to Q2 , is 2250 N.
So, the correct option is (d).
Electric Charges and Fields 149

q 2 × 10− 6
26. Two points charges are kept in air with a ⇒ a= E= × 4.2 × 104 m / s2
separation between them. The force m 0.04
between them is F1 , if half of the space = 21
. m / s2 (downward)
between the charges is filled with a So, effective acceleration on bob,
dielectric constant 4 and the force between a e = a + g = 121. m / s2
1 In the absence of electric field,
them is F2. If rd of the space between the l l
3 T = 2π = 2π
g 10
charges is filled with dielectric of dielectric
F In the presence of electric field,
constant 9. Then 1 is
F2 l l
[22 April 2018, Shift-I] T ′ = 2π = 2π
27 16 81 100 ae 121.
(a) (b) (c) (d)
64 81 64 81 T 121. 11 10
= = ⇒ T′ = T
Sol. (d) T′ 10 10 11
44 10 44
When dielectric of thickness t is introduced Given, T= ⇒ T′ = × = 2s
in two charges at distance r, the effective 20 11 20
force between the charges is given by So, time taken in 15 oscillations
q1 q 2 = 2 × 15 = 30s
F=
4πε0 [r − t + t K ]2
28. The electric field due to a short electric dipole at
where, K = dielectric constant of medium a distance r on the axial line from its mid-point
In first case, t = r / 2and K = 4 is x times the electric field at a distance 2r on
q1 q 2 the equatorial line from the mid-point of
∴ F1 = 2

4 πε0  r − r / 2 + 4
r dipole. Then, the value of x is
 2  [22 April 2018, Shift-II]
q1 q 2 q1 q 2 (a) 16 (b) 9 (c) 25 (d) 36
= =
4 πε0 r 2 9 πε0 r
9 2
Sol. (a)
4
In second case, t = r / 3 and K = 9 E2=kp/x3
2

∴ F2 = q1 q 2 / 4 πε0  r − +
r r  q1 q 2
3 =
 3 3  4 πε  25 r 2
0 
 9
E1=k⋅2p/x3
4 πε0   r 2
25
F1 q1 q 2  9 100 On axis, field magnitude, E1 =
2kp
∴ = × =
F2 9 πε0 r 2 q1 q 2 81 r3
On equatorial axis, field magnitude
27. A simple pendulum with a bob of mass E2 =
kp
=
kp
40g and charge +2µC makes 20 oscillation (2r)3 8r 3
in So, E1 = 16E 2 ⇒ x = 16
44 s. A vertical electric field magnitude
4 .2 × 10 4 NC−1 pointing downward is 29. A point charge q is placed at origin. Let E A, E B
applied. The time taken by the pendulum and E C be the electric fields at three points A
to make 15 oscillation in the electric field (1, 2, 3), B (1, 1, − 1) and C (2, 2, 2) respectively
is (acceleration due to gravity = 10 ms −2) due to the charge q. Then, the relation between
[22 April 2018, Shift-I] them is
(a) 30 s (b) 60 s (c) 90 s (d) 15 s 1. E A ⊥ E B 2. E A || E C
Sol. (a) 3.|E B| = 4 |E C| 4.|E B| = 8 |E C|
Mass, m = 40 g = 0.04 kg, q = 2 × 10− 6 C [22 April 2018, Shift-II]
(a) 1 , 4 are correct (b) 2, 4 are correct
Now, ma = qE
(c) 1, 3 are correct (d) 2, 3 are correct
150 AP EAMCET Chapterwise Physics

Sol. (c) (a) 4 × 10− 5 N from C to A


In vector form, we write expressions of field at (b) 4 × 10− 5 N from C to B
A, B and C (c) 8 × 10− 5 N from C to A
 kq  $ (d) 8 × 10− 5 N from C to B
EA =  2 (i + 2$j + 3k $)
2 3/ 2 
(1 + 2 + 3 ) 
2
Sol. (a)
kq Initially charge on A and B is q
EB = 2 $)
⋅ ($i + $j − k
(1 + 12 + (− 1)2)3 / 2
A d B
kq $)
EC = ⋅ (2$i + 2$j + 2k kq 2
(2 + 2 + 22)3 / 2
2 2
Force, F= = 4 × 10− 5 N
d2
kq $ $)
So, EA = (i + 2$j + 3k Now, A is touched by C, then;
143 / 2
Charge on C = q / 2
kq $)
EB = 3 / 2 ($i + $j − k Charge on A = q / 2
3
So, force on C = FA + FB
kq $)
EC = 3 / 2 (2$i + 2$j + 2k kq / 2⋅ q / 2 $ kq ⋅ q / 2 $
12 = rAC + rBC
(d / 2)2 (d / 2)2
As, E A ⊥ EB = 0 and EB = 4| EC |.
kq 2 / 4 kq 2 / 2
= − 2 [as r$ AC = − r$BC ]
30. An electric dipole consists of two particle each d2 / 4 d /4
of mass 1 kg separated by 1 m carrying kq 2 kq 2
charges 1 µC and − 1µC respectively. It is in = 2
(1 − 2) = − 2 = − 4 × 10− 5 N
d d
equilibrium in a uniform electric field of
So, force is of same magnitude but in opposite
2 × 10 4 Vm −1 . If it is deflected by a small direction, i.e from C to A as suggested by minus
angle 2°, minimum time taken by it to come sign.
back again to the mean position is (in seconds)
32. Four positive point charges + q are kept at
[22 April 2018, Shift-II]
the four corners of a square of side l. The net
(a) 2.5 π (b) 2 π
(c) 5 π (d) 4 π
electric field at the mid-point of any one side
of the square is [23 April 2018, Shift-I]
Sol. (a)
 1 
Torque on dipole, when it is deflected by a small  Take, = k
angle θ is τ = pE sinθ = pEθ { for small angle,  4 πε 0 
sinθ = θ} 4kq 16kq 8kq kq
But τ = Iα (a) (b) (c) (d)
l2 5 5 l2 5 l2 l2
where, I = mr 2 + mr 2 = 2mr 2
So, angular acceleration of dipole is Sol. (b)
pEθ The force at midpoint of any side of a square
α= let’s say at point P which is midpoint of side AB
I
Time to align back with field is  using θ = αt 2 
1 is due to charges at corners A, B, C and D
 2  FP = FAP + FBP + FCP + FDP
2θ 2θI 2I
t= = = = 50 ≈ 2.5π s A l B
α pEθ PE +q θ
+q
l/2
31. Two equally charged metal spheres A and B
−5
repel each other with a force of 4 × 10 N. P l
Another identical uncharged sphere C is l/2
touched to A and then placed at the θ
+q +q
D l C
mid- point of the line joining the spheres A
and B. The net electric force on the sphere C is AP = DP = l
[23 April 2018, Shift-I] BP = CP = l 2 + (l / 2)2 = 5l 2 / 4
Electric Charges and Fields 151

Electric field due to charges at A and D will be Sol. (b)


equal in magnitude and opposite in direction, so Charge enclosed, q = 8 C
will cancel each other. q
Electric flux, φ =
Similarly, the vertical components of electric εm
field due to charges at B and C will also cancel εm = Kε0
each other, but the horizontal components of Dielectric constant, K = 4
these will add up to give resultant electric field. 8 2
φ= ⇒ φ=
kq 4kq 2 16 kq 4ε0 ε0
E = 2⋅ ⋅ cosθ = 2 × 2 × =
2
( 5l / 4) 2
5l 5 5 5 l2
36. Two charged particles each of mass 9.8 g and
33. A proton and an α-particle start from rest in charges +20 µC and − 20 µC are attached to
a uniform electric field. The ratio of times the two ends of a massless and rigid uniform
taken by them to travel the same distance in non-conducting rod of length 50 cm. This
the field is [23 April 2018, Shift-II] arrangement is held in a uniform electric
(a) 5 : 2 (b) 3 : 1 (c) 2 : 1 (d)1 : 2 field of 12.1 NC −1 , such that the rod makes a
very small angle with the field direction. If
Sol. (d)
the rod is set free, the minimum time needed
qE 2eE
Acceleration of α-particle, a r = = for the rod to become parallel to the direction
m 4
of the electric field is ............... seconds.
Acceleration of photon,
[24 April 2018, Shift-I]
eE
ap = (a) 5 (b) 8 (c) 12 (d) 17
1
They travel same distances in time t1 and t2 . Sol. (a)
1  2eE  2 1  eE  2
⇒   t1 =   t2 +q
2 4  2 1 
+
θ
t22 1 t 1 E= 12.1 N/C
⇒ = ⇒ 2 = –q –
t12 2 t1 2
34. Two charged balls moving in the same
direction with same velocity v are placed in This arrangement of electric dipole in electric
an electric field. After some time, one ball field will undergo SHM, when the rod is set free.
v
moves with velocity at an angle of 60 ° with Time period of this SHM is given by
2 I
T = 2π
the initial direction and the other ball moves PE
at right angles to the initial direction with a where, I = moment of inertia of dipole,
velocity v’. Then, the value of v’ is
p = electric dipole moment and E = electric field.
[23 April 2018, Shift-II]
I = 2mr 2
v v v 50
(a) (b) (c) (d) v mass, m= 9.8 ×10−3 kg, r = ×10−2
2 3 2 2
2
I = 2× 9.8 ×10 ×  ×10−2 

Sol. (*) −3 50
 2 
Data in question is given insufficient.
No information about masses of balls is given. . ×10−4 k- gm2
I =1225
Also, it is not given weather the balls are p = q(2l) = 20 ×10−6 × 50 ×10−2 = 10−5 C - m
identical or not. E =121. N/ C
35. Flux coming out from a positive charge of 8 C, . ×10−4
1225
T = 2π = 2× 314
. × 3181
.
placed in a medium of dielectric constant 4 is 10−5 ×121
.
[24 April 2018, Shift-I] T =19.98 ≅ 20 s
1 2
(a) (b) (c) 8ε0 (d) 32 ε0 But time taken by the rod to become parallel will
2 ε0 ε0 be T/4, so required time is 5s.
15
Electrostatic Potential
and Capacitance
V (V)
1. If a unit positive charge is taken from one
point to another over an equipotential 6
surface, then [17 Sep. 2020, Shift-I]
4
(a) work is done on the charge C
(b) work is done by the charge 2 A B
(c) work done is constant
(d) no work is done x (m)
1 2 3 4
Sol. (d)
Potential difference between any two points on i.e., V = constant
equipotential surface is zero. ∴ Electric field at x = 2 m (at point C)
i.e., V1 − V2 = 0 − dV
E= =0 [Q V = constant]
∴ Work done to move a unit positive charge dx
from one point another, 3. Three capacitors of capacitances C1 = 2 µF,
W = q(V1 − V2) = q[0] = 0 C 2 = 3 µF and C 3 = 5 µF are connected in
2. The variation of electric potential with series. A potential difference of 155 V is
distance from a fixed point is shown in applied across the combination. Choose the
figure. What is the value of electric field at correct option. [17 Sep. 2020, Shift-II]
x = 2 m? [17 Sep. 2020, Shift-I] (a) Least potential difference is across C 3 .
V (volt) Equivalent capacitance of combination is
 30  µF.
 
6  31
The voltage across C1 is 75 V.
4
(b) Least potential difference is across C1 .
Equivalent capacitance of combination is
2
 30  µF.
 
 51
x (metre)
0 1 2 3 4 The voltage across C 2 is 50 V.
(c) Least potential difference is across C1 .
(a) 0 (b) 3 (c) 2 (d) 6 Equivalent capacitance of combination is
Sol. (a)  30  µF.
 
According to given diagram, potential is  31
constant in region AB. The voltage across C 3 is 30 V.
Electrostatic Potential and Capacitance 153

(d) Least potential difference is across C 2 . Sol. (d)


Equivalent capacitance of combination is Electric potential on the surface of sphere,
 30  µF. V = 200 V
 
 31 Radius, R = 5 cm = 5 × 10−2 m
The voltage across C1 is 50 V. Charge, q = 5C
Sol. (a)
The given situation is shown in the following
O B A
figure.
C1=2µF C2=3µF C3=5µF 10 cm
15 cm

Electric potential on the surface of sphere,


Q
V = 9 × 109
R
Q 10−8
155 V 200 = 9 × 109 ⋅ −2
⇒ Q= C
Equivalent capacitance is given as 5 × 10 9
1 1 1 1 1 1 1 31 Work done to move the charge from point A to
= + + = + + =
C C1 C2 C3 2 3 5 30 point B is given as
30 W = V AB ⋅ q
⇒ C= µF
31  Q Q
= (VB − V A)q =  9 × 109 ⋅ − 9 × 109 ⋅  5
In series combination of capacitances, charge  rB r A
flowing through each capacitor is same which is  10−8 10−8 
given by = 9 × 109  −  5
 9 × 10 −1
9 × 15 × 10−2 
q = CV
= 45 × 109 × 10−8  
1 1
=
30
× 155 = 150 µC = 1.5 × 10−4 C −
 9 × 10−1 9 × 15 × 10−2 
31
= 450 × 10 −
100 
∴Potential difference across C1 , = 50 
1 50
q 1.5 × 10−4 9  15   15 
V1 = = = 0.75 × 102
C1 2 × 10−6 = 166.66 J = 166.7 J
⇒ V1 = 75 V 5. The capacitance of a spherical condenser is
Potential difference across C2 , 1 µF. If the spacing between the two spheres
q 1.5 × 10−4 is 1 mm, the radius of the outer sphere is
V2 = = = 0.5 × 102
C2 3 × 10−6 [18 Sep. 2020, Shift-II]
⇒ V2 = 50 V (a) 30 cm (b) 6 m (c) 5 cm (d) 3 m
Potential difference across C3 ,
Sol. (d)
q 1.5 × 10−4
V3 = = = 0.3 × 102 Given, capacitance of spherical capacitor,
C3 5 × 10−6
C = 1µF = 10−6 F
⇒ V3 = 30 V Spacing between two spheres of spherical
Hence, potential difference across C3 is least as capacitor, r2 − r1 = 1 mm = 10−3 m
30 V. rr
∴ C = 4 πε0 1 2
r2 − r1
4. The electric potential on the surface of a charged
1 (r − 10−3)r2
spherical conductor of radius 5 cm is 200 V. ⇒ 10−6 = × 2
Work done in moving a charge of + 5 C from a 9 × 10 9
10−3
−3
point A to another point B situated at distances ⇒ 9 = r2 − 10 r2 ⇒ 9000 = 1000r22 − r2
2

of 15 cm and 10 cm respectively from the ⇒ 1000r22 − r2 − 9000 = 0


centre of the sphere is [18 Sep. 2020, Shift-II] −(−1) ± (−1)2 − 4 × 1000 × (−9000)
(a) 16.7 J (b) 22.3 J ⇒ r2 =
2 × 1000
(c) 88.8 J (d) 166.7 J
154 AP EAMCET Chapterwise Physics

1± 36 × 106 1 ± 6000 Sol. (c)


= =
2000 2000 When a capacitor is connected to a batter (DC
1 + 6000 source), then it is impossible to flow a steady
= = 3. 0005 − 3 m
~
2000 current but a transient current flows through
the capacitors plates, i.e. current flow for
6. A 100 V battery is connected across the sometime and finally it decreases to zero.
series combination of the two capacitors of 4
µF and 8µF. The energy stored in the series
8. Taking earth to be a metallic sphere, its
capacity will approximately be
combination is [21 Sep. 2020, Shift-I]
−2 −2
[21 Sep. 2020, Shift-II]
(a) 0.75 × 10 J (b) 1.33 × 10 J
(a) 6.4 × 10 F
6
(b) 700 F (c) 700 µF (d) 700 pF
(c) 0.5 J (d) 1 J
Sol. (b) Sol. (c)
Radius of earth, R = 6400 km = 6.4 × 106 m
Potential difference across terminals of battery,
V = 100 V Taking earth to be a metallic sphere, its
capacitance is given as
C1=4µF C2=8µF 1
C = 4 πε0 R = × 6.4 × 106
9 × 109
= 0.7111 × 10−3 F = 7111 . × 10−6 F
= 711 µF ~ − 700 µF

100 V 9. Electric potential at a point distant 0.5 m


from a spherical conductor of radius 0.2 m
Equivalent capacitance of series combination is charged to +1 nC is [21 Sep. 2020, Shift-II]
given as
(a) + 9 V (b) − 9 V (c) + 18 V (d) −18 V
1 1 1 1 1 2+ 1
= + = + =
C C1 C2 4 8 8 Sol. (c)
1 3 8 Charge on spherical conductor, q = +1nC
⇒ = ⇒ C = µF
C 8 3 = 10−9 C
Same charge will be flow in series combination Radius, R = 0.2 m
which is given as Distance of point from spherical conductor,
8
q = CV = × 10−6 × 100 r = 0.5 m
3 1 q
∴Electric potential, V = ⋅
8
= × 10−4 C 4 πε0 r
3 10−9 9
1 q2 = 9 × 109 × = = 18 V
∴Energy stored, E = 0.5 0.5
2 C
2 10. When a 5 C charge is kept in a uniform
 8 × 10−4 
  electric field, a force of 5000 N acts on it.
1   4
= × 3 = × 10−2 J Find the potential difference between two
2 8 −6 3
× 10 points in that field, separated by a distance
3
of 1 cm. [22 Sep. 2020, Shift-I]
⇒ E = 1.33 × 10−2 J
(a) 10 V (b) 250 V (c) 1000 V (d) 2500 V
7. When a capacitor is connected to a battery Sol. (a)
[21 Sep. 2020, Shift-I] Charge, q = 5 C
(a) an alternating current flow in the circuit Force, F = 5000 N, d = 1 cm = 10−2 m
(b) no current flow in the circuit ∴Potential difference between the given points,
(c) a current flow for sometime and finally it F  F
decreases to zero V=E×d = ×d Q E = q 
q  
(d) current keeps on increasing and reaches
5000 −2
maximum after sometime = × 10 = 10 V
5
Electrostatic Potential and Capacitance 155

11. The electric potential at the surface of an 14. The electric potential at a point on the axis of
atomic nucleus (Z = 50) of radius 9 × 10 −15 m is an electric dipole depends on the distance r of
[22 Sep. 2020, Shift-I] the point from the dipole as
(a) 4 × 106 V (b) 8 × 106 V [22 Sep. 2020, Shift-II]
(c) 4 × 10−6 V (d) 8 × 10−6 V (a) ∝ r −1 (b) ∝ r −2 (c) ∝ r (d) ∝ r −3
Sol. (b) Sol. (b)
Atomic number, Z = 50 Electric potential at point on the axial position of
Radius, r = 9 × 10−15 m electric dipole is given as
1 p
Electric potential at the surface of atomic V= ⋅
nucleus, 4 πε0 r 2
1 q 1 Ze 1
V= ⋅ = ⋅ [Q q = Ze] i.e. V ∝ 2 ⇒ V ∝ r −2
4 πε0 r 4 π ε0 r r
50 × 1.6 × 10−19
= 9 × 109 × = 8 × 106 V 15. When two identical capacitors are charged
9 × 10−15
individually to different potentials and then
12. The capacity of parallel plate condenser is connected in parallel, after disconnecting
5 µF. When a glass plate is placed between from the source [23 Sep. 2020, Shift-I]
the plates of the condenser, its potential (a) net charge = sum of initial charges
difference reduces to 1/8 of the original value. (b) net potential difference ≠ sum of individual initial
The magnitude of relative dielectric constant potential difference
(c) net energy stored < sum of individual initial
of glass is [22 Sep. 2020, Shift-II]
energy
(a) 4 (b) 6 (d) All of the above
(c) 7 (d) 8
Sol. (d)
Sol. (d)
Let say capacitance of two identical capacitors be
Given, C = 5 µF = 5 × 10−6 F C and they are charged through a potential V1
Initial potential = V and V2 , respectively.
When glass plate of dielectric constant K is V V
+ 1– + 2–
introduced between the plates of capacitor, then
potential difference is ,
V C C
V′ =
8 Q1=CV1 Q2=CV2
V V
∴ K = = After connecting in parallel,
V′ V / 8 C
+ –
⇒ K =8

13. When a charge of 20 C is taken from one


point to another separated by a distance of + –
0.2 m, work of 2 J is required to be done. C
What is the potential difference between the Net potential difference,
two points? [22 Sep. 2020, Shift-II] C V + C2 V2 CV1 + CV2
VC = 1 1 = (Q C1 = C2 = C)
(a) 2 × 10−2 V (b) 4 × 10−4 V C1 + C2 C+ C
(c) 8 V (d) 0.1 V V + V2
= 1
Sol. (d) 2
Charge, q = 20 C Which is not equal to the sum of individual
d = 0.2 m initial potential difference, i.e. VC ≠ V1 + V2 .
W = 2J For an isolated system, charge remains always
W 2 conserved,
Potential difference, V = = = 0.1 V i.e. sum of initial charge = sum of final charge
q 20
156 AP EAMCET Chapterwise Physics

Since, potential difference across each capacitors 17. A capacitor is made of a flat plate of area A
are different and connected in parallel, so and a second plate of stair-like structure as
charge redistribution will take place from higher
shown in the figure. The area of each stair is
potential capacitor to lower potential capacitor.
As we know that if charge flow, then there is
A
and the height is d. The capacitance of the
some loss of energy in form of heat. 3
i.e., U f < U i arrangement is [20 April 2019, Shift-I]
A/3
16. An imaginary equilateral triangle ABC of
side length 2 m is placed in a uniform A/3 d
electric field E = 10 NC −1 as shown. Then, A/3 d
V A − VB [23 Sep. 2020, Shift-I]
d

A ε0 A 6 ε0 A 3 ε0 A 11ε0 A
(a) (b) (c) (d)
3d 11d d 18 d
Sol. (d)
According to the question,
2m

2m

E
A/3

A/3 d
A/3 d

B C d
2m

A
(a) –5 V (b) + 5V (c) –10 V (d) + 10 V Given, area of each stair =
3
Sol. (c) and height of each stair = d
Given figure,
∴ Capacitance of parallel plate capacitor is given as
A
ε0 A
C=
d
2m

First capacitance of capacitor,


2m

E=10 NC–1
ε A
C1 = 0
3d
60º second capacitance of capacitor,
B 2m C 60º ε A ε A
r C2 = 0 = 0
B 3(2d) 6 d
B
and third capacitance of capacitor,
As we know, V A − VB = − ∫ E ⋅ dr ε A ε A
C3 = 0 = 0
A 3(3d) 9 d
Here, E is constant. So, we may write above Equivalent capacitance of capacitor,
expression as Ceq = C1 + C2 + C3
V A − VB = − E ⋅ r ⇒ V A − VB = − Er cosθ ε A ε A ε A
Ceq = 0 + 0 + 0
where, r is distance between points A and B, 3d 6d 9d
θ is angle between E and displacement vector r (6ε0 + 3ε0 + 2ε0) A 11 ε0 A
from point A to point B. Ceq = =
18 d 18 d
∴ V A − VB = −10 × 2 × cos 60°(θ = 60° from figure)
1 Hence, the capacitance of the arrangement is
= −10 × 2 × = −10 V 11ε0 A
2 .
18 d
Electrostatic Potential and Capacitance 157

18. The radius of a soap bubble is r and the or − ∫ (x$i − 2y$j + zk


$ )(dx$i + dy$j + dzk
$)
0 2 4
surface tension of the soap solution is S. The
electric potential to which the soap bubble be
=− ∫ x dx + 2∫ y dy − ∫ z dz
2 1 0
raised by charging it so that the pressure 0 4
 – x2   z2 
inside the bubble becomes equal to the ⇒  + [y 2 ]2 –  
pressure outside the bubble is  2 2  2 0
(ε 0= permittivity of the free space)
–1 2 1
[20 April 2019, Shift-I] ⇒ (0 − 22) + (22 – 12) – (42 – 02)
2 2
Sr Sr 4Sr 8Sr
(a) (b) (c) (d) 1 1
8 ε0 4 ε0 ε0 ε0 ⇒ – (–4) + (4 – 1) – (16) = + 2 + 3 − 8
2 2
Sol. (d) ∆ V = − 3 V or ∆ V = 3 V
Q Pressure due to surface tension inside the soap 20. Three point charges of 3 µC, 4 µC, and 5 µC
4S
bubble pi = …(i) are arranged at the three corners of a right
r
angled triangle ABC as shown in the figure.
Electrostatic pressure outside the soap bubble,
The work done in moving the charges at A
σ2
p0 = …(ii) and C, so that the three charges are located at
2 ε0 the three corners of an equilateral triangle of
where, σ = surface charge density and side 3 cm is [20 April 2019, Shift-II]
ε0 = permittivity of the free space 4µ C
kQ
Q Electric potential, V = A
r
k(σ A)
⇒ V= (Q Q = σ. A)
r
Q Area of sphere, A = 4 πr 2 4cm

1  σ 4 πr 2  σ r  1 
So, V =   = Q k = 
4 π ε0  r  ε0  4 π ε0  5µC
B C
ε0 V 3µC 3 cm
or σ=
r (a) 0.3 J (b) 1.1 J (c) 2.2 J (d) 3.3 J
According to the question, Sol. (d)
pi = p0
According to question,
From Eqs. (i) and (ii), we get
q1=4µC
 ε0 V 
2

  A
4S σ 2
4S  r  8S r
= ⇒ = ⇒V =
r 2 ε0 r 2 ε0 ε0
8Sr 4 cm
Hence, the pressure outside the bubble is .
ε0

19. The potential difference between two points


A(2 , 1, 0) m and B(0 , 2 , 4) m in an electric field B q3=5µC
q2=9µc 3cm C
(x$i − 2 y$j + zk$ ) Vm −1 is [20 April 2019, Shift-II]
(a) 2 V (b) 3 V (c) 1 V (d) 6 V In a right angle triangle, in ∆ABC
AC 2 = AB2 + BC 2
Sol. (b)
$ ) Vm −1 ⇒ AC = AB2 + BC 2 = (4 × 10−2)2 + (3 × 10−2)2
Given, electric field E = (x $i − 2 y$i + zk
⇒ AC = 5 × 10−2 m
Now, electric potential difference between two
points A(2, 1, 0) and B(0, 2, 4) is given as Initial electric potential energy of three charges,
kq q k q1 q 3 k q2 q3
∴ ∆V = − ∫ E ⋅ dr U= 1 2 + +
AB AC BC
158 AP EAMCET Chapterwise Physics

k(4 × 3 × 10−12) k(4 × 5 × 10−12) k(3 × 5 × 10−12) 22. In a parallel plate capacitor the separation
= + +
4 × 10−2 5 × 10−2 3 × 10−2 between plates is 3 x. This separation is filled
= k[3 × 10−10 + 4 × 10−10 + 5 × 10−10 ] by two layers of dielectrics, in which one
= 9 × 109 × 12 × 10−10 = 108 × 10−1 = 10.8 J layer has thickness x and dielectric constant
q1=4µC 3 k, the other layer is of thickness 2 x and
A
dielectric constant 5 k. If the plates of the
capacitor are connected to a battery, then the
3cm 3cm
ratio of potential difference across the
dielectric layers is [21 April 2019, Shift-I]
1 4 3 5
(a) (b) (c) (d)
2 3 5 6
B 3 cm C
q2=3µC q3=5µC Sol. (d)
Key Idea Capacitance of a parallel plate capacitor is
When three charges located of an equilateral ε A
triangle of side 3 cm, the final potential energy give by relation, C = εr . 0
of three charges system, d
kq q kq q kq q where, εr = dielectric constant, A = area of parallel
= 1 2 + 2 3 + 1 3 plate and d = distance between the plate.
AB BC AC
k(4 × 3 × 10−12) k(3 × 5 × 10−12) k(4 × 5 × 10−12) Here, d0 = 3x, d1 = x, ε1 = 3k,d2 = 2x and ε2 = 5k
= + +
3 × 10−2 3 × 10−2 3 × 10−2 The capacitor is shown in the figure below,
C1 C2
9 × 109
= [12 × 10−10 + 15 × 10−10 + 20 × 10−10 ]= 14.1 J
3
Hence, the work done in moving charges at
points A and C, W = (141 . − 10.8) = 33
. J. 3k 5k

21. The potential difference (V A − VB) in the


arrangement shown in the figure is
x
(q = 1 µ C, x = 2 cm, y = 3 cm)
3x
[21 April 2019, Shift-I]
y ε A ε A
x x Now, C1 = 3 k 0 and C2 = 5k 0
+q A B –q x 2x
(a) 5.4 × 105 V (b) 2.7 × 105 V Since, m in series combination of capacitor,
stored charge in each capacitor is same.
(c) 5.4 × 102 V (d) 2.7 × 102 V
Q Q
Sol. (a) So, V1 = and V2 =
C1 C2
An arrangement is shown in the figure below, θx θ2x
2cm 3cm 2cm ⇒ V1 = and V2 =
3kε0 A 5kε0 A
+q=1µC A B –q=–1µC θx
Here, potential at a point A, V1 3kε0 A 5 5
The ratio, = = =
V A = V A+ q + V A− q V2 θ2x 3× 2 6
5kε0 A
9 × 109 × 10− 6 9 × 109 × 10− 6
= 9 × 105  − 
1 1
= −
2 × 10− 2 5 × 10− 2  2 5 Hence, option (d) is correct.

Similarly, potential at a point B, 23. Three point charges of 2 mC each are kept at
VB = VB + q + VB − q = 9 × 105  − 
1 1 the vertices of an equilateral triangle of side
 5 2 50 cm. If the system is supplied energy at the
rate of 2 kW, the time taken to move one of the
Hence, V A − VB = 9 × 105  − − + 
1 1 1 1
 2 5 5 2 charges to the mid point of the line joining
⇒ V A − VB = 54 . × 105 V the other two charges is [21 April 2019, Shift-II]
Hence, the correct option is (a). (a) 18 s (b) 36 s (c) 72 s (d) 144 s
Electrostatic Potential and Capacitance 159

Sol. (c) Due to one charge, potential on hexagon’s centre


According to the question,
1 q
2×10–3C P is given as, potential, V = .
4 π ε0 r
2 m kq
Potential, V =
0 =1

1/2
r
10 0 m

 2

m
1
 k = 4 πε , k = 9 × 10 C / N-m 
9 2
5

Q
 0 
9 × 109 × 10 × 10−6
2×10–2C 2×10–3C V= = 9 × 2 × 10 × 10
3 2

1/2 m 5 × 10−2
3 V =18 × 105 N - m2 / C2 …(i)
Energy of system of three charges, U =  1 2 
kq q
 r  Due to all six charges, total potential at the
k(2 × 10−3 × 2 × 10−3) centre of hexagonal is Vtotal = 6 × (V1 )
or U= × 3 = 24 k × 10−6 J Now, from Eq. (i), we get
(1 / 2)
Vtotal = 6 × 18 × 105 ⇒ Vtotal = 108
. × 107 V
When one charge move to the mid point of the
line joining the other two charges. 25. The potential in an electric field varies as
Energy of the system, V = (x 2 − y 2). The electric lines of the force in
k (2 × 2 × 10−6) k(2 × 2 × 10−6) X -Y plane are [22 April 2019, Shift-I]
U′ = × 2+ Y-axis
(1 / 4) 1/ 2
−6
U ′ = 40 k × 10 T
Net energy, ∆U = U ′ − U = (40 − 24) k × 10−6 (a) X-axis
or ∆U = W = 16 × 9 × 103 J
Given that energy supplied to the system at the
rate of 2 kW. Y-axis
So, W = 2 kW × t ⇒16 × 9 × 103 = 2 × 103 × t
⇒ t = 72s
Hence, time taken to move one of the charge to (b) X-axis
the mid point of the line joining the other two
charge is 72 s.
Y-axis
24. A regular hexagon of side 5 cm has a charge
10 µC at each of its vertices. The potential at
the centre of hexagon is [22 April 2019, Shift-I] (c) X-axis
(a) 0 V (b) 18 × 105 V
. × 107 V
(c) 108 . × 105 V
(d) 108
Y-axis
Sol. (c)
Given,
length of sides of hexagon, r = 5 cm = 5 × 10−2 m (d) X-axis
charge on each vertices of hexagon, q =10 µC
= 10 × 10−6 C
Now, according to the question,
Sol. (c)
10 µC 10 µC The potential in an electric field varies as,
5cm V = (x 2 − y 2)
5cm ∴ Electric field,
10 µC 10 µC
P dv d v $
E = −  $i + j [Q E = ∆ V]
 dx dy 
10 µC
10 µC
160 AP EAMCET Chapterwise Physics

d d 
= −  (x 2 − y 2)$i + (x 2 − y 2)$j 27. A charge 5C is placed at the centre of shell of
 dx dy  radius, r = 3 m and having charges 5 C. The
E = − [2x$i − 2y$j] ⇒ E = − 2x$i + 2y$j r
potential at a point distance from the
Expression of the electric field is linear equation 2
in two variables, i.e. straight lines in X-Y plane centre of the shell will be [22 April 2019, Shift-II]
and slope having 45°, 35° etc. (a) − 9 × 109 V (b) 30 × 109 V
Hence, the option (c), represents correct graph. (c) 45 × 109 V (d) − 15 × 109 V
26. A capacitance of 2 µF is required in an Sol. (c)
electrical circuit across a potential of 1.0 kV. Given,
A large number of 1 µF capacitors are charge on the surface of the shell, q = 5C
available which can withstand a potential radius of shell, r = 3 m
difference not more than 300 V. The Now, the electric potential at a point at a distance
minimum number of capacitors required to r from the centre of the shell is given as,
achieve this is [22 April 2019, Shift-II]
∴ V=
1

q
(a) 24 (b) 32 (c) 8 (d) 16 4 πε0 r
Sol. (b) …(i)
Here, the required capacitance = 2µF, and 5  1 
V1 = 9 × 109 × Q = k = 9.0 × 109 
potential difference, V = 1.0 kV = 1000 V 3  4 πε0 
According to required the capacitance, we draw V1 = 15 × 109 V
the following circuit, ∴Electric potential at r / 2 due to charge at the
C1 C1 C1 centre, from Eqs. (i), we get
5 Q r = 3 
V2 = 9 × 109 × 
∴ r / 2 = 3 / 2
C1 C1 C1
m rows (3 / 2)
C1 C1 C1 V2 = 30 × 109 V
∴Total potential,
n colums ∴ V = V1 + V2 = (15 × 109 + 30 × 109)
1000 V V = (15 + 30) × 109
V = 45 × 109 V
Given, capacitance of each capacitor, C1 = 1µF,
potential difference, V1 = 300 V 28. A spherical capacitor has outer sphere of
Now, from above the circuit potential difference radius 5 cm and inner sphere of radius 2 cm.
across each capacitor, When the inner sphere is earthed, its
V1 =
1000
= 300 ⇒ n = 333
.
capacity is C1 and when the outer sphere is
n C
earthed its capacity is C 2. Then 1 is
So, for maximum bearable capacitance, we take C2
n = 4. Hence, capacitance of each row in which
all 4 capacitors are in series, [23 April 2019, Shift-I]
1 1 1 1 1 4 1 5 2 7 3
= + + + = ⇒ C′ = µF (a) (b) (c) (d)
C 1 1 1 1 1 4 2 5 3 7
1 m
The total capacitance of m rows, m × = µF Sol. (a)
4 4 Key Idea If a spherical capacitor has outer radius
Since, it is given that the total capacitance of R2 and inner radius R1 . Then the capacitance,
circuit is 2µF. (a) when outer shell is earthed,
m R1 R2
So, 2 µF = µF ⇒ m = 8 C = 4 πε0
4 R2 − R1
Thus, the minimum number of capacitor,
(b) When inner shell is earthed,
= m × n = 8 × 4 = 32
C ′ = C + 4 πε0 R2
Hence, the correct option is (b).
Electrostatic Potential and Capacitance 161

Given, radius of outer sphere of capacitor, 30. A parallel plate capacitor has a capacity
R1 = 2 cm and Radius of inner sphere of capacitor, 80 × 10 −6 F, when air is present between its
R2 = 5 cm plates. The space between the plates is filled
 RR  2 × 5  with a dielectric slab of dielectric constant 20.
∴ C1 = 4 πε0  1 2 + R2  = 4 πε0  + 5
 R2 − R1  5 − 2  The capacitor is now connected to a battery
25 of 30V by wires. The dielectric slab is then
C1 = 4 πε0 Farad removed. Then, the charge passing through
3
the wire is [22 April 2018, Shift-I]
R1 R2 2× 5 10
and C2 = 4 πε0 = 4 πε0 = 4 πε0 (a) 12 × 10−3 C (b) 25.3 × 10−3 C
R2 − R1 5− 2 3
(c) 120 × 10−3 C (d) 45.6 × 10−3 C
25
C1 25 5 Sol. (d)
Hence, = 3 = = Cair = 80µF
C2 10 10 2
3 Cdielectric = εr Cair = 1600 µF
Hence, the correct option is (a). Charge stored in the presence of air,
q air = Cair × V = 80 × 30 × 10− 6 = 2400 µC
29. The charge on 4 µF capacitor, in the given Charge stored in presence of dielectric medium,
circuit is [23 April 2019, Shift-I] q d = Cdielectric × V = 1600 × 30 × 10− 6 = 48000 µC
1µF When dielectric is removed, effective charge
4µF remained is
q = q d − q air
= (48 − 2.4) × 10− 3 C
5µF
q = 456 . × 10− 3 C

31. Three uncharged capacitors of capacities C1 ,


3µF
C 2 and C 3 are connected as shown in the
figure. A, B and C are at potentials V1 , V2 and
10 V V3, respectively, then the potential at O is
(a) 24 µC (b) 100 µC (c) 2.4 µC (d) 30 µC [22 April 2018, Shift-I]
Sol. (a) A
The circuit can be redrawn as,
C1
4µF 6µF
A B O
C2 C3

B C
3µF
C V + C 2 V2 + C 3 V3 C V+ C 2 V2 − C 3 V3
(a) 1 1 (b) 1 1
C1 + C 2 + C 3 C1 + C 2 + C 3
+ – C1 V1 − C 2 V2 − C 3 V3
10V (c) (d) zero
C1 + C 2 + C 3
4×6
Ceq = = 2.4 µF
4+ 6 Sol. (a)
As we know, potential drop across parallel Total charge in the capacitors (accumulated at
branch AB is V AB = 10 V point O)
So, charge Q = Ceq V = 2.4 × 10 × 10−6 C 15 Ω
A
Q = 24 µC
Since, in a series capacitor branch, there is equal R1
storage of charge in each capacitor.
B
So, 4µF capacitor store a charge of 24 µC.
Hence, the correct option is (a). Qtotal = C1 V1 + C2 V2 + C3 V3
162 AP EAMCET Chapterwise Physics

Total capacitance of the combination, 33. Four capacitors marked with capacitances
Ctotal = C1 + C2 + C3 (w.r.t. point O) and breakdown voltages are connected as
So, potential at O, shown in the figure. The maximum emf of
Q C V + C2 V2 + C3 V3 the source, so that no capacitor breaks down
V = total = 1 1
Ctotal C1 + C2 + C3 is [23 April 2018, Shift-I]

32. One plate of a parallel plate capacitor is 5µF 4µF


connected to a spring as shown in the figure.
The area of each plate of the capacitor is A 1kV 2 kV
and the distance between the plates is d, + –
when the battery is not connected and the V
2µF 3 µF
spring is unstretched. After connecting the
battery, in the steady state the distance
2 kV 1kV
between the plates is 0.75 d, then the force
constant of the spring is [22 April 2018, Shift-II] (a) 10.5 kV
Spring (b) 5.25 kV
Capacitor (c) 2.25 kV
(d) 1.25 kV
Sol. (c)
V Ceq =20/9
5 µF 4µ F 1

Battery 1kV 2kV Ceq =6/5


2
3 ε0 V 2 A 8 ε0 V 2 A + –
(a) (b)
8 d3 3 d3
9 ε0 V 2 A 32 ε0 V 2 A 2 µF 3µ F
(c) (d) + –
32 d 3 9 d3
1kV 2kV V
Sol. (d)
In equilibrium, force between plates of capacitor Resultant capacitance of series combination 1,
q2 1 1 9
= spring force ⇒ = kx = + =
2ε0 A 5 4 20
where, x = extension in spring. Now before 20
Ceq 1 = = 2.25 µF
charging, when spring is unstretched (x = 0) 9
3
distance of plates is d and after charging it is d. Resultant capacitance of series combination 2,
4
3 1 1 1 5
So, x=d− d= d = + =
4 4 3 2 6
q2 C2V 2 6
Hence, k = = Ceq 2 = = 1 ⋅ 2µF
2ε0 A ⋅ x 2ε0 Ax 5
 ε20 A2  20
⋅ V2 So, charge on upper branch is = V and charge
 (3/ 4 d)  32  ε V 2 A  9
= =
1  9  d 3 
0
6
on lower branch is V.
 2ε0 A ⋅ d  5
 4 
Electrostatic Potential and Capacitance 163

Capacitor Voltage drop Given Maximum 36. In the circuit shown in figure, if the point R
across it breakdown value of emf is earthed and point P is given a potential of
voltage it can bear +1800 V, then charges on C 2 and C 3 are
V = q /C 1 kV 9 respectively [23 April 2018, Shift-II]
=
C1 20 1 4 1 kV ( 4 / 9)V 4
= × = V C2
9 5 9 = 2 ⋅ 25 kV
2kV 18 4µF
20 1 5 = C1
C2 = × = V 2 kV ( 5 / 9)V 5 +1800V
9 4 9
= 3 .6 kV P Q R
3 µF C3
2 kV 10
6 1 3 =
C3 = × = V 2 kV ( 3 / 5)V 3
5 2 5 2µF
= 3.33 kV
(a) 2.4 × 10−3 C ; 12
. × 10−3 C
1 kV 5
6 1 2 = (b) 1.6 × 10 C ; 0.8 × 10−3 C
−3
C4 = × = V 1 kV (2 / 5) 2
5 3 5 . × 10−3 C ; 16
(c) 32 . × 10−3 C
= 2 .5 kV
(d) 4.8 × 10−3 C; 2 .4 × 10−3 C
So, smallest value is 2.25 kV. Sol. (a)
34. A Van de Graaff generator has a spherical Ceq of system = (C2 parallel C3) series C1
metal shell as an electrode which is at a 1 1 
=1  +  = 2µF
potential 15 × 10 6 V. If the dielectric strength  3 (4 + 2) 
of the surrounding medium is 5 × 10 7 Vm − 1 , So, charge taken from source
then the diameter of the shell is = q eq = Ceq ∆V = 1800 × 2 × 10−6 C = 3600 µC
[23 April 2018, Shift-I] Potential droop across
(a) 30 cm (b) 15 cm (c) 60 cm (d) 120 cm q 3600 × 10−6
C1 = C 1 = = 1200 V
Sol. (c) CC 1 3 × 10−6
Potential difference, V = 15 × 106 V So, potential drop across combination of 4µF and
Dielectric strength of medium = 5 × 107 V / m 2µF capacitors
Electric field intensity, E = 5 × 107 V / m = 1800 − 1200 = 600 V
Minimum radius of spherical shell required for Hence,
the purpose is given by q 2 = C2 VQR = 4 × 10−6 × 600 = 2. 4 × 10−3 C
V 15 × 106 and q 3 = C3 VQR = 2 × 10−6 × 600 = 1.2 × 10−3 C
r= = = 0.3 m
E 5 × 107
37. Four capacitors of capacitances 2 µF , 3µF , 4µF
Hence diameter, d = 2r = 0.6 m = 60 cm
and xµF are connected to a battery of emf 6 V
35. Electric field vector in a region is given by and of negligible internal resistance, as
E = (3 i$ + 4 y$j)Vm −1 . The potential at the origin shown in the figure. If the ratio of the
3
is zero. Then, the potential at a point charges on x µF and 4µF capacitances is ,
8
(2, 1) m is [23 April 2018, Shift-II]
then the value of x is [24 April 2018, Shift-I]
(a) 7 V (b) 8 V (c) −8 V (d) −7 V xµF
Sol. (c)
− ∂Vx $ − ∂V y $ 3µ F
As, E= i+ j
∂x ∂y 2µF
− ∂Vx − ∂V y
= 3 and = 4y
∂x ∂y 4µF
⇒ Vx = − 3x and V y = − 2y 2

So, V = potential function = − (3x + 2y 2)


Potential at point (2, 1) is 6V
V = − (3 × 2 + 2 × 12) = − 8V (a) 2 (b) 5 (c) 3 (d) 8
164 AP EAMCET Chapterwise Physics

Sol. (b) Sol. (c)


As x and z are in parallel Potential of capacitor before inserting insulator,
x µF +σ –σ
+ –
3 µF (x+2) µF + –
+ –
+ –
3 µF 2 µF + –
4 µF 4 µF d
σd
V=
ε0
6V 6V where, σ = surface charge density,
d = distance between plates
⇒ Let charges on 2µF, 3µF, 4µF and xµF are q 2 ,
q 3 , q 4 and q x . and ε0 = permittivity of vacuum.
q 4 = C4 V = 4µF × 6 = 24µC σd
Given, =100
⇒ As, given
qx 3 3
= ⇒ q x = 24 × = 9µC ε0
q4 8 8
Now, potential after inserting 2mm of insulator.
Potential difference on xµF Let its dielectric constant is k.
V1 = x = V = 
q 9 q V1 = E1 ⋅ 2 + E 2 (d − 2)
x x C σ σ
V1 = 2 + (d − 2)
9 ε0 k ε0
Same volt will be across 2µF too.
x σ
⇒ V1 =  2 + d − 2
 
Now, remaining  6 −  volt potential difference
9 k 
ε0
 x
will drop across 3µF. To maintain potential difference, same distance
is increased by 1.6mm
As 3µF and (x + 2µF ) are in series, so charge on σ
them will be same So, extra potential difference, V2 = (1.6)
ε0
q 3 = q( x + 2)
Clearly, V1 + V2 = V
 9
3 6 −  = (x + 2)
9
σ 2  σ σd
 x x  + d − 2 + 1.6 =
ε0  k  ε0 ε0
6 x − 9 = 3x + 6 ⇒ 3x =15 ⇒ x = 5
σd 2σ σ σ
⇒ − − (d − 2) = 1.6
38. The plates of a parallel plate capacitor are ε0 ε0 k ε0 ε0
charged upto 100 V. A 2 mm thick insulator 2σ 2σ σ 2
⇒ − = 1.6 ⇒ 2− =1.6
sheet is inserted between the plates. Then to ε0 ε0 k ε0 k
maintain the same potential difference, the
⇒ 2k − 2=1.6k
distance between the plates is increased by
2
1.6 mm. The dielectric constant of the ⇒ 0.4k = 2 ⇒ k =
0.4
insulator is [24 April 2018, Shift-I]
⇒ k= 5
(a) 6 (b) 8 (c) 5 (d) 4
16
Current Electricity
1. A resistor of resistance R is connected  l − l2 
r= 1  560 − 500  10
between the terminals of a cell of emf E and  R = 
 l2   500 
internal resistance r. If I is the current
60 6
through the circuit. The terminal potential = × 10 = = 1.2 Ω
difference of the cell is given by 500 5
[17 Sep. 2020, Shift-I]
3. In a balanced meter bridge, the segment of
ER E−R wire opposite to a known resistance of 70 Ω
(a) IR (b) E − Ir (c) (d)
R+ r r is 70 cm. The unknown resistance is
Sol. (b) [18 Sep. 2020, Shift-I]
Cell of emf E and internal resistance r is (a) 30 Ω (b) 60 Ω
connected with resistor of resistance R as shown (c) 90 Ω (d) 15 Ω
in the figure.
Sol. (a)
R
In balanced meter bridge known resistance,
S = 70Ω, l = 70 cm
I ∴Unknown resistance,
100 − l  70(100 − 70)
R = S  = = 30Ω
E r  l  70
Cell
When cell is discharging, then terminal
4. Resistance of a tungsten wire at 150°C is
potential difference across the cell is given as 133 Ω. Its temperature coefficient of
V = E − Ir
resistance is 0.0045 °C − 1 . The resistance of
where, I = current drawn by cells. this wire at 500 °C is [18 Sep. 2020, Shift-I]
(a) 180 Ω (b) 225 Ω
2. In a potentiometer experiment the balancing (c) 258 Ω (d) 317 Ω
length with a cell is 560 cm. When an
external resistance of 10 Ω is connected in Sol. (c)
parallel to the cell, then the balancing length Resistance of tungsten wire at 150°C is given as
changes by 60 cm. Find the internal R150 = 133 Ω
resistance of the cell. [17 Sep. 2020, Shift-I] ⇒ R0 (1 + αt) = 133 [Q Rt = R0 (1 + αt)]
(a) 1 Ω (b) 2 Ω (c) 1.2 Ω (d) 2.1 Ω ⇒ R0 (1 + 150α) = 133 …(i)
Similarly, resistance of wire at 500°C is given as
Sol. (c)
R500 = R0 (1 + 500α)
Given, l1 = 560 cm, R = 10 Ω ,
R0 (1 + 500α) = R500 …(ii)
l 2 = l1 − 60
From Eqs. (i) and (ii), we get
⇒ l 2 = 560 − 60 = 500 cm
R0 (1 + 150α) 133
∴Internal resistance of cell is given by =
R0 (1 + 500α) R500
166 AP EAMCET Chapterwise Physics

( + 500α) 1331
1331 ( + 500 × 0.0045)
⇒ R500 = = 6. A copper wire of radius 0.1 mm and
1 + 150α 1 + 150 × 0.0045 resistance 2 kΩ is connected across a power
[given, α = 0.0045° C −1 ] supply of 40 V. The number of electrons
43225
. transferred per second between the supply
= = 258.06 Ω ~
− 258 Ω
1.675 and the wire at one end is [21 Sep. 2020, Shift-I]
(a) 2.00 × 1016 (b) 1.25 × 1017
5. The V-I graph for a conductor at temperature
(c) 2.85 × 1017 (d) 3.25 × 1016
T1 and T2 are as shown in the figure. T2 − T1 is
proportional to [18 Sep. 2020, Shift-II] Sol. (b)
Radius of copper wire, r = 0.1 mm
V
T2 = 1 × 10−4 m
Resistance, R = 2k Ω = 2 × 103 Ω
Power supply, V = 40 V
T1 Current flowing through the wire,
V 40
θ θ I= = = 2 × 10−2 A
9 0° – R 2 × 103
θ I ∴Charge flowing per second
q = It = 2 × 10−2 × 1 = 2 × 10−2 C
(a) cos 2θ (b) sin 2θ
(c) cot 2θ (d) tan 2θ ∴Number of electrons transferred,
q 2 × 10−2
Sol. (c) n= = = 1.25 × 1017 electrons
e 1.6 × 10−19
V-I graph for conductor at temperature T1 and T2
is shown in the figure. 7. The wire of potentiometer has resistance 4Ω
V and length 1 m. It is connected to a cell of
T2 emf 2V and internal resistance 1Ω. The
current flowing through the potentiometer
wire is [21 Sep. 2020, Shift-I]
(a) 0.1 A (b) 0.2 A
T1
(c) 0.4 A (d) 0.8 A
θ
–θ Sol. (c)
90 °
θ Resistance of potentiometer wire, R = 4Ω
I
Emf of cell, E = 2 V
We know that resistance of a conductor is Internal resistance, r = 1Ω
directly proportional to its temperature.
Current flowing through the potentiometer wire,
Hence, R1 ∝ T1 ⇒ tanθ ∝ T1 E 2 2
I= = = = 0.4 A
⇒ tanθ = KT1 …(i) R+ r 4+1 5
and R2 ∝ T2 ⇒ tan(90°−θ) ∝ T2
⇒ cotθ ∝ T2 ⇒ cotθ = KT2 …(ii) 8. In given figure, an ammeter reads 5A and
∴ From Eqs. (i) and (ii), we get voltmeter reads 40 V. The actual value of
KT2 − KT1 = cot θ − tanθ resistance R is [22 Sep. 2020, Shift-I]
cosθ sinθ
K (T2 − T1 ) = − R
sinθ cosθ A
cos2 θ − sin2 θ
⇒ T2 − T1 =
K sinθ cosθ
cos 2θ 2cot 2θ
⇒ T2 − T1 = ⇒ T2 − T1 = V
1 K
sin 2θ
2 (a) 8 Ω (b) greater than 8 Ω
⇒ T2 − T1 ∝ cot 2θ (c) less than 8 Ω (d) 200 Ω
Current Electricity 167

Sol. (b) Sol. (d)


Current in ammeter, I = 5 A The given circuit diagram is
5A I1 R
A I=1.5 A
P
I2 RA=2Ω RB=4Ω

V 3Ω
We know that, voltmeter has a very high
resistance. Hence, current flowing through RD=6Ω RC=12Ω
voltmeter is very low.
Q
∴ 5 = I1 + I 2 ⇒ I1 = 5 − I 2
As, I 2 > 0
∴ I1 < 5 ⇒
40
<5 Q I = 40  Equivalent resistance between point P and Q is
 R 
1
R given as
⇒ 8< R ⇒ R> 8 Ω 1 1 1 1
= + +
RPQ R A + RD 3 RB + RC
9. The external diameter of 5m long hollow
1 1 1 1 1 1
tube is 0.1 m and thickness of its wall is =+ + = + +
2 + 6 3 4 + 12 8 3 16
0.005 m. If ρ = 1.7 × 10 −8 Ω-m, then its
1 25
resistance will be [22 Sep. 2020, Shift-I] ⇒ =
RPQ 48
(a) 5.7 × 10−5 Ω (b) 2.7 × 10−5 Ω
(c) 2 × 10−5 Ω (d) 5 × 10−5 Ω ⇒ RPQ =
48

25
Sol. (a)
∴ Potential difference between the points P and
Given, length, l = 5 m
Q is given as
External diameter, d1 = 01 . m 48
d 01
. VPQ = I ⋅ RPQ = 1.5 × = 2.88 V
∴External radius, r1 = 1 = = 0.05 m 25
2 2
Thickness, t = 0.005 m 11. Two wires of equal diameters, lengths l1 , l2
∴Internal radius, r2 = r1 − t and having resistivities S1 , S2 respectively are
= 0.05 − 0.005 = 0.045 m joined in series. The equivalent resistivity of
∴Area of cross-section of hollow tube, the combination is [23 Sep. 2020, Shift-I]
A = π(r12 − r22) = 3.14 [(0.05) 2 − (0.045) 2 ] S1 l1 + S 2 l2 S1 l2 + S 2 l1
(a) (b)
= 3.14 × 4.75 × 10 −4 = 14.915 ×10−4 l1 + l2 l1 − l2
= 1.49 ×10−3 m 2 S1 l2 + S 2 l1 S1 l1 − S 2 l2
l 5 (c) (d)
∴Resistance, R = ρ ⋅ = 1.7 × 10−8 × l1 + l2 l1 − l2
A 1.49 × 10−3
= 5.7 × 10−5 Ω Sol. (a)
According to question,
10. Potential difference between the points P and R1 R2 Req
Q in the circuit shown is [22 Sep. 2020, Shift-I]
(l1, S1) (l2, S2) (l, S)
P i=1.5 A
In series combination of resistance,
RA=2Ω RB=4Ω Req = R1 + R2
Sl S1 l1 S2 l 2
3Ω = +
A A1 A2
RD=6Ω RC=12Ω Here, A1 = A2 = A and l = l1 + l 2
Q S(l1 + l 2) = S1 l1 + S2 l 2
S l + S2 l 2
Equivalent resistivity, S = 1 1
(a) 4.5 V (b) 1.2 V (c) 2.4 V (d) 2.88 V l1 + l 2
168 AP EAMCET Chapterwise Physics

12. The colour code for a resistance of 22 Ω ± 5% First case when, C = 100 Ω
is [23 Sep. 2020, Shift-I] Now, By balanced wheat stone bridge,
(a) brown-brown-black-gold R A 100
= …(i)
(b) red-red-brown-silver RB R
(c) red-red-black-gold Second case when, C =121 Ω
(d) red-red-orange-silver RB 121
= …(ii)
Sol. (c) RA R
The colour code for a resistance is From Eqs. (i) and (ii), we get
R = 22 Ω ± 5% ∴ R = 100 × 121
We may write it as, R = 22 × 100 ± 5%Ω R = 110 Ω
In carbon colour coding resistors have two bands
for the resistance value, one multiplier and one 14. In the circuit shown, if the current through
tolerance 3band. In multiplier band, from the left 1
the resistor R is A, the value of R is
first colour shows first significant number, 5
second colour shows second significant number [20 April 2019, Shift-II]
and third colour is for multiplier of 10. By using
the colour code chart, we find that 2 stands for
red and 0 for black. 5% tolerance means resistor
has golden band. 5V, 2Ω R 2V, 1Ω
So, colour coding for resistor from left should be
red-red-black-gold
13. Four resistors A , B, C and D form a Wheatstone
bridge as shown in the figure. The bridge is (a) 2 Ω (b) 3Ω (c) 5Ω (d) 1Ω
balanced when C = 100 Ω. If A and B are
Sol. (d)
interchanged, the bridge balances for C = 121 Ω.
According to the question,
The value of D is [20 April 2019, Shift-II]
1
A
A B 5
V1 = 5 V R V2 = 2 V
G R1 = 2 Ω R2 = 1 Ω

C D

Equivalent circuit of above circuit shown below,


+ –
E
(a) 10 Ω (b) 100 Ω (c) 110 Ω (d) 120 Ω
E
Sol. (c)
According to the question, balanced wheat R
stone bridge, can be shows as follows
r

A B
V1 V
G − 2
R1 R2
Now, equivalent emf, =
1 1
C R +
R1 R2
5 2

1
∴ e= 2 1 = V
1 1 3
+ – +
E 2 1
Current Electricity 169

∴Internal resistance, Sol. (c)


R ⋅R 2×1 2
r= 1 2 = = Ω Key Idea Heat transfer through conduction wall is
R1 + R2 2+ 1 3 given by mathematical expression,
1 dQ (T − T0)
e 1 = kA 1
∴Current, I = or = 3 dt x
r+ R 5 2+ R dQ
where, = power transferred through the wall and
3 dt
1
=
1/ 3

1
=
3 x = thickness of the wall
5 2 + 3R 5 3(2 + 3R) Here, side of cube, a = 60 cm. Hence, area A = 6a ,
3 Q Total area = 6 (area of one side of the cube)
⇒ 6 + 9R = 15 ⇒ 9R = 15 – 6 thickness x = 01 . cm, thermal conductivity,
⇒ 9R = 9 or R = 1Ω k = 4 × 10− 4 cal s− 1 cm− 1 ° C− 1 , temperature
difference, (T1 − T0) = 1000°C and potential of DC
15. Assertion (A) When a wire of aluminium source V = 400 V. Hence, the power
and another wire of silicon are heated from kA × 4184
. × (T1 − T0) k6a 2 × 4184
. × 103
room temperature to 80°C, the conductivity P= =
x x
of aluminium increases and that of silicon 4 × 10− 4 × 6 × (60)2 × 103 × 4184.
decreases. P= J
01
.
Reason (R) Aluminium has positive
P = 361.49 W
temperature coefficient of resistivity and
Given, voltage supply of DC = 400 V
silicon has negative temperature coefficient
of resistivity. [21 April 2019, Shift-I]
Hence, power generated in a resistance,
V2 V2
(a) Both (A) and (R) are correct and (R) is the correct P= = 361.49 ⇒ R =
explanation of (A). R P
400 × 400
(b) Both (A) and (R) are correct but (R) is not the R= = 0.4426 Ω
correct explanation of (A). 361.49 × 103
(c) (A) is correct but (R) is not correct. Hence, the correct option is (c).
(d) (A) is not correct but (R) is correct.
17. In the given circuit, the electric currents
Sol. (d) through 15Ω and 6 Ω, respectively are :
Key Idea In nature, metals have positive temperature [21 April 2019, Shift-II]
coefficient and semiconductors have negative
temperature coefficient. 15 Ω 6V
Aluminium is a metal, so its conductivity
decreases with increase in temperature, due to
the excessive collision of electron. Whereas, Si is
a semiconductor therefore its conductivity 6Ω 12 Ω
increases with increase in temperature. All metals
have positive temperature coefficient of resistivity
and semiconductor has negative temperature 9V
coefficient of resistivity. Hence, assertion (A) is (a) 0 A, 0.5 A (b) 0 A, 1 A
incorrect but reason (R) is correct.
(c) 0.5 A, 1 A (d) 1 A, 0 A
16. The walls of a closed cubical box of edge 60 cm Sol. (a)
are made of material of thickness 1 mm and The electric circuit as shown below,
thermal conductivity, 4 × 10 − 4 cal s − 1cm − 1 ° C− 1 . 6V
15 Ω
The interior of the box is maintained 1000°C
above the outside temperature by a heater I2 II
placed inside the box and connected across
400 V DC supply. The resistance of the heater 6Ω I1 12 Ω
is [21 April 2019, Shift-I] I
(a) 4.41 Ω . Ω
(b) 441
(c) 0.441 Ω (d) 441 Ω 9V
170 AP EAMCET Chapterwise Physics

Applying KVL in loop (I), Applying KVL in loop (I),


⇒ −9 + 6I1 + 12(I1 + I 2) = 0 −6 + 3(I1 − I 2) + 2.8I1 = 0
⇒ 18I1 + 12I 2 = 9 ⇒ 58. I1 − 3I 2 = 6 …(i)
⇒ 6I1 + 4I 2 = 3 …(i) Applying KVL in loop (II),
Applying KVL in loop (II), 2I 2 + 3(I 2 − I1 ) = 0 …(ii)
⇒ −6 + 15I 2 + 12(I1 + I 2) = 0 From Eqs. (i) and (ii), we get
⇒ 12I1 + 27I 2 = 6 .  I 2  − 3I 2 = 6
5
⇒ 58
⇒ 4I1 + 9I 2 = 2 …(ii) 3 
From Eqs. (i) and (ii), we get ⇒ 6.66I 2 = 6 ⇒ I 2 = 0.9A
I1 = 0.5A, I 2 = 0 Hence, current through 2 Ω resistor is 0.9A.
Hence, current through15Ω is 0 but through 19. The length of a potentiometer wires is l. A
6Ω is 0.5 A.
 l
cell of emf E is balanced at a length   from
18. In the given circuit, the current through 2Ω  3
resistor is [21 April 2019, Shift-II] positive end of the wire. If the length of the
2Ω  l
wire is increased by   , the distance at
 2
which the same cell gives the balancing
3Ω point is (Cell in the primary is ideal and no
4Ω
series resistance is present in the primary
2µF circuit.) [22 April 2019, Shift-I]
2l l l 4l
(a) (b) (c) (d)
3 2 6 3
6V 2.8 Ω Sol. (b)
According to question, the figure is as
1 1
(a) 9 A (b) 0.9 A (c) A (d) A K1
9 0.9 E

Sol. (b)
The electric circuit as shown below,
l/3
2Ω V

3Ω If K be the potential gradient of the


potentiometer wire, then emf of the cell which
2µ F 4Ω gives balancing length l / 3 is given by
l
E = K. …(i)
3
2.8 Ω
…(ii) Q K = 
6V V l V V
Where, E= . =
At the steady state, the capacitor behaves as l 3 3  l 
open circuit. Therefore, simple circuit will be When, length of potentiometer wire is increased
2Ω by l / 2, then new length,
l 3l
II I2 l1 = l + =
2 2
3Ω ∴New potential gradient,
I1 V 2V
I K′ = ⇒ K′ =
3l / 2 3l
If l′ be the new balancing length,
6V 2.8 Ω then, E = K ′ l′ …(iii)
Current Electricity 171
2V (a) 25 paisa (b) 50 paisa
or E= l′
3l (c) 20 paisa (d) 10 paisa
2V
.l′ =
V
⇒ l′ =
l From Eq. (iii), E = V  Sol. (d)
3l 3 2  3  Here, electric energy cost in ` = 25 paisa / kWh,
n mass of water, m = 4.6 Kg
20. n identical resistance are taken in which initial temperature of water, Ti = 25° C and
2
resistors are joined in series in the left gap final temperature of water,
n and Tf = boiling point of water = 100° C
and the remaining resistances are joined in As heat used by water to boil,
2
Q = ms(Tf − Ti)
parallel in the right gap of a metre bridge.
So, Q = 4.6 × 4.2 × 103 × (100 − 25)
Balancing length in cm is [22 April 2019, Shift-I]
(Q Specific heat of water, S = 4.2 × 103 J kg − 1 ° C)
n2 n2
(a) 100 ⋅ (b) 100 ⋅ ⇒ Q = 1449 × 103 J
n + 4
2
n +1 2
∴ Number of units of electric energy,
1 1 1449 × 103
(c) 400 ⋅ 2 (d) 400 ⋅ 2 N=
Q
=
n + 4 n +1 1 kWh . × 106
36
Sol. (a) (Q1 kWh = 36
. × 106 J)
Meter bridge is shown in the figure below, ⇒ N = 0.40 unit
R S Cost of electric energy in ` = ` × N = 25 × 0.40
= 10 Paisa
Therefore, the cost of heating is 10 paisa.
Left gap Right gap
22. A 500 W heater is designed to operate at
200 V potential difference. If it is connected
l 100–l across 160 V line, the heat it will produce in
n 20 minutes is [22 April 2019, Shift-II]
When resistances are joined in series in left
2 (a) 384 kJ (b) 483 kJ
gap each of resistance R1 , then equivalent (c) 843 kJ (d) 348 kJ
resistance in left gap. Sol. (a)
R R R n Rn
R = 1 + 1 + 1 + K . times = 1 Case I Resistance of heater when it operates at
2 2 2 2 2
n 200 V and power, P1 = 500 W
When resistors are joined in parallel in right V2 V2
2 Q Power, P = ⇒ R1 = 1
gap, then the equivalent resistance in right gap. R P1
1 1
= +
1
+
1 n
+ .... times =
n (200)2 200 × 200
R1 = =
S R1 R1 R1 2 2R1 500 500
2R R1 = 80 Ω
⇒ S= 1
n Case II Now, it connected across 160 V line the
If l be the balancing length in the meter bridge heat it will produce in 20 minute is
wire, then V2 = 160 V
R1 n
V 2 (160)2
R
=
l
⇒ 2 =
l ∴ P2 = 2 =
S 100 − l 2R1 100 − l R R
n (160)2 160 × 160
= = = 2 × 160 (Q R = 80 Ω)
n2 l 100 n2 80 80
= ⇒ l= 2
4 100 − l n +4 P2 = 320 W
∴Now, heat produce is 20 minutes
21. Electrical energy costs 25 paisa per kilowatt dQ = P2 × time = 320 × 20 × 60
hour. Assuming that no energy is wasted, the
dQ = 384 kJ
cost of heating 4.6 kg of water from 25 °C to
the boiling point is [22 April 2019, Shift-II]
172 AP EAMCET Chapterwise Physics

23. A cell of emf ε and internal resistance r is 24. In a meter-bridge if the left and right gaps
connected across a variable load resistance R. are connected with 2 Ω and 3 Ω resistances,
The graph drawn between its terminal voltage respectively then the bridge is balanced. The
and resistance R is [23 April 2019, Shift-I] resistance to be connected with 3Ω resistance
to get the balancing point at midpoint of the
bridge wire is [23 April 2019, Shift-I]
(a) V (b) V
(a) 3 Ω in series (b) 3 Ω in parallel
(c) 6 Ω in series (d) 6 Ω in parallel
R R Sol. (d)
Key Idea In a meter-bridge, if the balancing point is
at the midpoint of the bridge wire, then it shows the
resistance gaps have equal resistance.
(c) V (d) V Given, R1 = 2Ω
and R2 = 3Ω
R
For meter-bridge,
R
R1 l1
=
Sol. (a) R2 L − l1
According to the question, circuit diagram of a Where, l1 is balancing length,
cell is given below, R1 l
= 1 (Q l1 = L − l1 )
+R– R2 l1
V
⇒ R1 = R2 ⇒ 2 Ω = 3 Ω | | x Ω
I
Here, x is the parallel balancing resistance to 3Ω
resistor.
r ε 3x
Hence, = 2 ⇒ x = 6Ω
As,
V
r=E−V
Ir = E − V ⇒ 3+ x
R Hence, the correct option is (d).
⇒ V 1 +  = E ⇒ V =
r E
.R
 R  R+ r 25. The equivalent resistance between A and B is
6 Ω. The value of R1 is [22 April 2018, Shift-I]
General equation of a straight line passing
through the origin, 15Ω

y = mx
E
Here, y = V , m = = slope of the line
R+ r R1 2Ω 3Ω
and x = R
As this line is passing through the origin and B×
has a slope which reduces as R increases. Hence,
the graph is (a) 20 Ω (b) 10 Ω (c) 5 Ω (d) 25 Ω
V Sol. (b)
Here 2Ω and 3Ω resistances are short circuited,
so circuit can be reduced as
15R1
R AB = =6
15 + R1
⇒ 15R1 − 6R1 = 15 × 6
O R ⇒ 9R1 = 90 ⇒ R1 = 10 Ω
Current Electricity 173

26. A battery of emf 10 V is connected to a 27. Two cells P and Q each of emf 2.16 V are
uniform wire AB of 1m length and having a connected in series with a resistor of 19.6 Ω.
resistance of 10 Ω in series with a 10 Ω An ideal voltmeter reads 2V, when
resistor as shown in the figure. Two cells of connected across the cell P and 1.92 V when
emf 2V and 3V having internal resistance 2 Ω connected across the cell Q. The ratio of the
and 3 Ω, respectively are connected as shown internal resistances of the cell P and Q is
in the figure. If the galvanometer shows null
[22 April 2018, Shift-II]
deflection at point J on the wire, then the
(a) 1 : 2 (b) 2 : 3 (c) 3 : 4 (d) 1 : 3
distance of point J from the point B is.
[22 April 2018, Shift-I] Sol. (b)
P r1 Q r2
10V
+ – 10Ω + – + –
2V 1.92V

R=19.6
J
A B
G
2V 2Ω Emf of each cell = 216
. V
Emf of circuit = 4.32 V
4.32
2V 2Ω Current in circuit, I =
r1 + r2 + 19.6
(a) 48 cm (b) 50 cm
Now, using V = E − I ⋅ r for both cells, we have
(c) 52 cm (d) 54 cm
4.32r1
For P, 2 = 216
. −
r1 + r2 + 19.6
Sol. (c)
4.32r2
Given r = 10Ω, R = 10Ω, E = 10 V and L = 100 cm. and For Q,1.92 = 216
. −
r1 + r2 + 19.6
Now, voltage drop on the wire,
So, we have,
E
E′ = ×R 4.32r1
(r + R) 0.16 = … (i)
r1 + r2 + 19.6
10
⇒ E′ = × 10 = 5 R 4.32r2
20 and, 0.24 = … (ii)
r1 + r2 + 19.6
So, potential gradient on wire,
5 From Eqs. (i) and (ii), we get
x = E ′/ L = V / cm … (i) r1 : r2 = 2 : 3
100
Effective emf of combination in secondary 28. A resistor has bands with colours orange,
circuit, green, silver and gold. Then, the resistance of
V ε ε
= 1 + 2 the resistor is [22 April 2018, Shift-II]
reff r1 r2
(a) (350 ± 5) m Ω (b) (350 ± 17.5) m Ω
1 1 1 5 (c) (35 ± 5%) m Ω (d) (250 ± 5%) m Ω
⇒ Here, = + =
reff 2 3 6
Sol. (b)
6 12
⇒ V = (1 + 1) = Green Silver
5 5 Orange Gold
Now, at balancing point (from point A) is
12 100
l=V/x= ×
5 5
l = 48 cm
Given, orange = 3,
So, length from point B is
Green = 5
100 − 48 = 52 cm
Silver = 10− 2
174 AP EAMCET Chapterwise Physics

E1 5 5
Gold (tolerance) = ± 5% Current, i = = = = 0.01 A
R + R1 50 + 450 500
From colour code, resistance value is
[35 × 10− 2 ± 5%] So, V = iR = .01 × 50 = 0.5 V
emf of primary cell,
= 350 × 10− 3 ± 5% ohms
V 0.5 × 4.50
E= x = = 0.225 V
= 350 m - Ω ± 17. 5 m - Ω l 10
29. A DC source with internal resistance R0 is 31. The bulb which glows with maximum
connected to three identical resistors each of intensity in the given circuit is
resistance R as shown in the figure. If the [23 April 2018, Shift-II]
thermal power generated in the circuit is 2Ω 4Ω
highest, then [23 April 2018, Shift-I]

3Ω
R R 5Ω
R
6Ω
Bulb

20V

(a) 4 Ω bulb (b) 2 Ω bulb


(a) R = 2 R 0 (b) R = 3R 0 (c) 3 Ω bulb (d) 6 Ω bulb
R
(c) R = 0 (d) R = R 0 Sol. (a)
3
Given circuit is
Sol. (b) VA VB
For maximum power, R0 = Req of circuit.
2Ω 4Ω
R
3Ω
A R R A R R
B B A R B 6Ω 5Ω

+ –
Given, resistances are in parallel combination, 20 V
the equivalent value is As, resistance of portion B is higher.
1 1 1 1 1 3 ∴A greater potential drop across B occurs.
= = + + =
Req R0 R R R R Now, in section B,
R
⇒ R0 = or R = 3R 0 .
Power, P =
V2
3
R
30. In a potentiometer, a wire of length 10 m So, least resistance gives a higher power output.
having resistance 50 Ω is used. A battery of ∴ 4Ω bulb glows with maximum intensity.
5 V and a resistor of 450 Ω are connected in
series to the wire. If an unknown battery of 32. In the circuit shown in figure, power developed
emf E balances the potentiometer at 450 cm, across 1 Ω, 2 Ω and 3 Ω resistances are in
then the value of E is [23 April 2018, Shift-I] the ratio. [23 April 2018, Shift-II]

(a) 0.225 V (b) 1.25 V (c) 2.25 V (d) 0.0225 V 1Ω


Sol. (a) 3Ω
Given length of wire, l = 10 m, resistance of
i 2Ω
wire, R = 50 Ω, emf of battery, E1 = 5V,
balancing length, x = 450 cm = 4 ⋅ 5 m
series resistor, R1 = 450 Ω (a) 1 : 2 : 3 (b) 4 : 2 : 27 (c) 6 : 4 : 9 (d) 2 : 1 : 27
Current Electricity 175

Sol. (b) Sol. (c)


Current distribution is as shown in figure If resistance R1 , R2 , R3 , R4 and R6 from
2i/3 1Ω Wheatstone bridge, then current will
independent of resistance R6 .
i 3Ω
R1 R
i For this, = 3
i/3 2Ω R2 R4

∴Powers are ⇒ R1 R4 = R2 R3
4 2 2 34. A fuse wire of radius 0.2 mm blows off with
P1 = I12 R1 = i , P2 = I 22 R2 = i 2
9 9 a current of 5 A. The fuse wire of same
27 2 material but of radius 0.3 mm will blow off
and P3 = I 32 R3 = 3i 2 = i . ⇒ P1 : P2 : P3 : : 4 : 2 : 27
9 with a current of [24 April 2018 Shift-I]

33. In the given circuit, current I is independent 15 5 3


(a) A (b) A
of the resistance R6. Then [24 April 2018, Shift-I] 2 2
27
R5 (c) 5 A (d) 5 A
8

I R1 R3 Sol. (c)
If I = current rating of fuse wire
and r = radius of fuse wire, then relation, I 2 ∝ r 3
R6 3/ 2
R2 R4 I 2  r2 
=  ,
I1  r1 

(a) R1 R 2 R 5 = R 3 R 4 R 6 given r1 = 0.2mm,I1 = 5A


(b)
1
+
1
=
1
+
1 r2 = 0.3 mm, I 2 =?
R5 R 6 R1 + R 2 R 3 + R 4 3/ 2
I 2 = 5×  
0.3 27
=5 A
(c) R1 R 4 = R 2 R 3  0.2 8
(d) R1 R 3 = R 2 R 4
17
Magnetic Effects of Current
1. A current I a flows in a circular arc of radius 2. A proton moving with a velocity 2.5 × 10 7 m/s,
r subtending an angle θ as shown in the enters a magnetic field of intensity 2.5 T
figure. Find the magnetic field at the centre making an angle 30 ° with the magnetic field.
O of the circle. [17 Sep. 2020, Shift-I] The force on the proton is [18 Sep. 2020, Shift-I]
(a) 3 × 10− 12 N (b) 5 × 10− 12 N
A B
(c) 6 × 10− 12 N (d) 9 × 10− 12 N
Sol. (b)
Velocity of proton, v = 25
. × 107 m/s
I θ I
Magnetic field, B = 2.5 T ⇒θ = 30°
Magnetic force on proton in magnetic field is
given as F = Bqvsinθ
= 2.5 × 1.6 × 10−19 × 2.5 × 107 sin 30°
µ Iθ 2µ I.sin θ
(a) 0 (b) 0 1
4 πr 4 πr = 6.25 × 1.6 × 10−12 × = 5 × 10−12 N
2
2µ 0 I sin θ 2µ 0 I sin θ
(c) (d)
2r 4r 3. A horizontal overhead powerline at a height
Sol. (a) of 5 m from the ground and carries a current
Magnetic field at the centre due to current of 150 A from East to West. The magnetic
carrying circular loop is given as field directly below it on ground is
µ I (a) 6 × 10− 6 T, towards South [18 Sep. 2020, Shift-I]
BC = 0 …(i)
2r (b) 6 × 10− 6 T, towards West
(c) 7 × 10− 6 T, towards East
A B
(d) 8 × 10− 7 T, towards North
Sol. (a)
Magnetic field below the wire on the ground is
I Oθ
given as
I=150 A N

5m W E
Magnetic field due to current carrying circular
arc making an angle θ at the centre, is given as S
θ θ µ0 I Ground
B= ⋅ BC = ⋅ [From Eq. (i)]
2π 2 π 2r µ0 I 150
B= ⋅ = 2 × 10−7 × [Q µ 0 = 4 π × 10−7 TmA−1 ]
µ Iθ 2π r 5
B= 0
4 πr = 6 × 10−6 T (toward South)
Magnetic Effects of Current 177

4. A charge q moving in a circle of radius r metre Now, dividing Eq. (i) by Eq. (ii), we get
B1 µ 0 I / 16R 1
makes n rev/s. Magnetic field at the centre of ∴ = =
B2 µ 0 I / 2R 8
the circle is [18 Sep. 2020, Shift-II]
2 πq 2 πq
(a) × 10− 7 NA −1 m − 1 (b) × 10− 7 NA −1 m − 1 6. Choose the wrong statement to complete: An
nr r ideal solenoid has [21 Sep. 2020, Shift-I]
2 πnq 2 πq
(c) × 10− 7 NA −1 m − 1 (d) NA −1 m − 1 (a) the turns widely separated.
r r
(b) the turns closely wound.
Sol. (c) (c) the length is very much greater than the radius.
Current associated due to the movement of (d) the magnetic field inside almost uniform.
charge q in circular path of radius r is given as Sol. (a)
I= =
q q Q T = 1  An ideal solenoid has length much greater than
T (1 / n)  n  its radius and the turns are closely wound on it,
= qn such a way that it produce uniform magnetic
field inside it.

7. When a helium nucleus covers a circle of radius


q 0.8 m in 2 s, find the value of magnetic field B
r
at the centre of the circle. [21 Sep. 2020, Shift-II]
10−19
(a) T (b) µ 0 × 10−19 T
µ0
2 × 10−19
(c) 2 µ 0 × 10−19 T (d) T
µ I µ0
∴ Magnetic field at the centre, B = 0
2r
µ 0 ⋅ qn (4 π × 10−7)qn
Sol. (b)
= = Charge on helium nucleus,
2r 2r
2πnq q = +2e = 2 × 1.6 × 10−19 C = 32
. × 10−19 C
= × 10 NA −1 m −1
−7
Radius of helium nucleus, r = 0.8 m
r
Time period, T = 2s
5. If B1 is the magnetic field induction at a point
∴ Current associated with helium nucleus,
on the axis of a circular coil of radius R q 32 . × 10−19
situated at a distance R 3 and B2 is the I= = = 1.6 × 10−19 A
T 2
magnetic field at the centre of the coil, then ∴ Magnetic field at the centre of the loop,
B µ I µ × 1.6 × 10−19
the ratio of 1 is equal to B= 0 = 0
B2 [18 Sep. 2020, Shift-II] 2r 2 × 0.8
(a)
1
(b)
1
(c)
1
(d)
1 = µ 0 × 10−19 T
3 8 4 2
8. A proton enters a magnetic field of flux
Sol. (b) density 1.5 Wb m −2 with a velocity of
According to question, 2 × 10 7 ms −1 at an angle of 30° with the field.
µ 0 IR2
B1 = The force on the proton will be
2 (R2 + r 2)3 / 2 [21 Sep. 2020, Shift-II]
Here, r=R 3 (a) 2.4 × 10−12 N (b) 24 × 10−12 N
µ 0 IR2 µ 0 IR2 µ IR2 (c) 0.24 × 10−12 N (d) 0.024 × 10−12 N
∴ B1 = = = 0 3
2 [R + (R 3) ]
2 2 3/ 2
2 [4R ]2 3/ 2
16R Sol. (a)
µ 0I Magnetic flux density, B = 1.5 Wbm −2
B1 = …(i)
16R Velocity of proton, v = 2 × 107 ms −1
µ I θ = 30°
and B2 = 0 …(ii)
2R
Charge on proton, q = 1.6 × 10−19 C
[magnetic field at centre]
178 AP EAMCET Chapterwise Physics
∴ Force on the proton, Net magnetic field at O,
F = Bqvsinθ B = B due to straight wire PQ (upward) + B due to
= 1.5 × 1.6 × 10−19 × 2 × 107 × sin 30° straight wire CD (upward) + B due to circular wire
1 (downward)
= 4.8 × 10−12 × = 2. 4 × 10−12 N µ I µ I µ I
2 =− 0 ⋅ − 0 ⋅ + 0
4π r 4π r 2r
9. When a helium nucleus makes a full rotation µ0 I µ I µ I  − 1 + 1
=− + 0 = 0
of a circle of radius 0.8 m in 2.5 s, then the 2πr 2r 2r  π 
value of magnetic field B at the centre of the
µ 0 I  −1 + π  µ 0 I (π − 1)
circle will be [22 Sep. 2020, Shift-I] = = [downward]
2r  π  2 πr
(a) 4 π × 10−25 T (b) 2 π × 10−26 T
(c) 4 π × 10−26 T (d) 2 π × 10−25 T 11. Figure shows the cross-sectional view of the
Sol. (c) hollow cylindrical conductor with inner radius
Charge on helium nucleus, R and outer radius 2R, carrying uniformly
q = 2e = 2 × 1.6 × 10−19 = 3.2 × 10−19 C distributed current i along its axis. The
r = 0.8 m, T = 2.5 s magnetic induction at point P at a distance
Current associated due to one rotation of 3 R/ 2 from the axis of the cylinder will be
helium nucleus, [22 Sep. 2020, Shift-II]
q 32 . × 10−19
I= = = 1.28 × 10−19 A
T 2.5
∴Magnetic field at the centre of the circle
formed by helium nucleus, R
µ I 4 π × 10−7 × 1.28 × 10−19 3R/2
B= 0 = 2R
2r 2 × 0.8 P
= 3.2 π × 10−26 T
which is nearest to 4 π × 10−26 T. 5µ 0i
(a) zero (b)
72 πR
10. An infinitely long straight conductor is bent 7µ 0 i 5µ 0i
into shape as shown in figure. It carries a (c) (d)
18 πR 36 πR
current I A and the radius of circular loop is
r m. The magnetic induction at the centre of Sol. (d)
the circular loop is [22 Sep. 2020, Shift-II] The given situation is shown in the following figure,

r
I O
O 3R/2
R

µ 0 I ( π − 1) µ 0 I( π + 1) P
(a) (b) 2R
2 πr 2 πr
µ I(2 π − 1) µ I(2 π + 1)
(c) 0 (d) 0
2 πr 2 πr We know that, magnetic field induction at point P
is given as
Sol. (a)
µ 2i  OP 2 − R2 
The given situation is shown in the figure, B= 0 ⋅  
4 π OP  (2R)2 − R2 
µ 0 2i  (3R / 2)2 − R2  µ 0 i  (9R2 l 4) − R2 
r = ⋅  =
I O 4 π 3R / 2  4R2 − R2  3πR  3R2 

r µ 0 i  5R2  µ 0 i 5 5µ 0 i
= = × =
3πR 12R2  3 πR 12 36 π R
P I Q C D
Magnetic Effects of Current 179

12. Magnetic field dB to a current element at any Given, the ratio of heats generated through the
shunt and galvanometer is 7 : 5,
point on its axis is [23 Sep. 2020, Shift-I]
and resistance of galvanometer, R g = 112 Ω
(a) minimum
Heat generated through shunt (ps) 7
(b) zero ∴ =
(c) varies between minimum and maximum Heat generated through galvanometer (p g) 5
(d) depends on the position of the point V2
Sol. (b) Rs 7  V2 V2 
∴ = Q pS = and p g = 
Magnetic field dB due to a current element at V2 5  RS Rg 
any point is given by Rg
µ Idl 5 5
dB = 0 2 sinα [From Biot-Savart’s law] Rs = × R g ⇒ Rs = × 112 = 80Ω
4π r 7 7
At a point on its axis, Hence, the resistance of shunt = 80Ω
α = 0 ∴ dB = 0
15. Assertion (A)When proton and a neutron
13. An electron having charge 1.6 × 10 −19 C and enter into a transverse magnetic field with
mass 9 .1 × 10 −31 kg is moving with speed equal speeds, then they trace a circular paths
of equal radii.
4 × 10 6 ms −1 in a magnetic field 2 × 10 −1 T in a
Reason (R) In a transverse magnetic field
circular orbit. The force acting on electron the period of revolution of a charged particle
and the radius of the circular orbit is in a circular path is directly proportional to
[23 Sep. 2020, Shift-I] the mass of the particle. [20 April 2019, Shift-I]
(a) 12.8 × 10−13 N, 1.1 × 10−4 m (a) Both (A) and (R) are correct and (R) is the correct
(b) 1.28 × 10−14 N, 1.1 × 10−3 m explanation of (A).
(c) 1.28 × 10−13 N, 1.1 × 10−3 m (b) Both (A) and (R) are correct but (R) is not the
(d) 1.28 × 10−13 N, 1.1 × 10−4 m correct explanation of (A).
(c) (A) is correct but (R) is not correct.
Sol. (d) (d) (A) is not correct but (R) is correct.
Given that,
Sol. (d)
q = 1.6 × 10−19 C
m = 9.1 × 10−31 kg (A) When a proton and a neutron enter into a
transverse magnetic field with equal speeds,
v = 4 × 106 m/s
then they trace circular paths of equal radii.
B = 2 × 10−1 T
This statement is not correct because neutron
Magnetic force on electron, F = qvB will not move in a circular path as there is no
= 1.6 × 10−19 × 4 × 106 × 2 × 10−1 charge on neutron.
F = 1.28 × 10−13 N So, (A) is not correct but (R) is correct.
Radius of circular orbit, Hence, option (d) is correct.
mv 91. × 10−31 × 4 × 106 1 th
r= = 16. If only of the main current is to be
qB 1.6 × 10−19 × 2 × 10−1 51
= 1 .1 × 10−4 m passed through a galvanometer then the
1th
14. The ratio of heats generated through shunt shunt required is R1 and if only of the
and galvanometer is 7 : 5 when they are 11
connected to make an ammeter. If the main voltage is to be developed across the
resistance of the galvanometer is 112 Ω then galvanometer, then the resistance required
the resistance of the shunt is R
R2. Then 2
[20 April 2019, Shift-I] R1 [20 April 2019, Shift-I]
(a) 80 Ω (b) 8 Ω (c) 15.6 Ω (d) 1.56 Ω 1 50 500
(a) (b) (c) (d) 500
Sol. (a) 500 9 3
180 AP EAMCET Chapterwise Physics

Sol. (d) According to diagram in the question, magnetic


According to the question, field at the centre C will be sum of the magnetic
field due to straight part B1 and magnetic field
Case I
G due to circular part B2 .
i ig i Q From Maxwell’s right hand thumb rule
direction of both magnetic field at the point C
will be same and above the plane of paper.
(i–ig)
µ i µ i µ i
So,BC = B1 + B2 = 0 + 0 = 0 (1 + π)
R1 4 πr 4r 4 πr
1 Hence, the magnetic field at the centre C of the
If only th of the main current i is to be passed µ
51 arrangement shown in figure is, 0 i(1 + π).
through galvanometer G then the shunt 4 πr
required is
18. To measured a magnetic field between the
Main current, i = 51 magnetic poles of a loud speaker, a small coil
ig =1 having 30 turns and 2.5 cm 2 area is placed
G G G
∴ R1 = ⇒ R1 = = perpendicular to the field and removed
i − ig 51 − 1 50 immediately. If the total charge flown
G G through the coil is 7 .5 × 10 −3C and the total
⇒ R1 = = …(i)
51 − 1 50 resistance of wire and galvanometer is 0.3 Ω,
Case II then the magnitude of the magnetic field is
[20 April 2019, Shift-I]
G
i ig i (a) 0.03 T (b) 0.3 T (c) 3 T (d) 3 × 102 T
Sol. (b)
(i–ig)
Given,
R1 number of turns in the coil, N = 30 turns
1 area of the coil, A = 2.5 cm 2 = 2. 5 × 10−4 m 2
If only th of the main voltage is developed
11 total charge flowing through the coil,
across the G then the resistance required, R2 . QNet = 7.5 × 10−3 C
R2 = G(VG − 1) and total resistance of wire and galvanometer,
R2 = G(11 − 1) = 10 G …(ii) R = 0.3 Ω
Now, from Eqn. (i) and (ii), we get We know that,
flux in the coil
R2 10G R net charge, Qnet =
= = 500 ∴ 2 = 500 net Resistnace
R1 G R1
φ
50 ∴ Qnet = …(i)
R
17. The magnetic field at the centre C of the We know that, flux φ = magnetic field × area
arrangement shown in figure is × number of turns in the coil
[20 April 2019, Shift-I]
or φ = BNA …(ii)
i ∞ Now, from Eqs. (i) and (ii), we get
BNA
r ∴ QNet =
C R
Putting the given values, we get
r
B × 30 × (2.5 × 10– 4 )
7.5 × 10−3 =
µ0 i µ 0i 0.3
(a) (1 + π ) (b) (1 + π )
2 πr 4 πr −3 B × 7.5 × 10−3
7.5 × 10 = ⇒ B = 0. 3 T
µ i µ i 0.3
(c) 0 (1 + π ) (d) 0 (1 + π )
πr r Hence, the magnitude of the magnetic field,
Sol. (b) B = 0.3 T.
Magnetic Effects of Current 181

19. An electron accelerated through a potential 21. A galvanometer of resistance G Ω , is shunted


difference V, passes through a uniform by a resistance S Ω. To keep the main current
transverse magnetic field and experiences a in the circuit unchanged, the resistance to be
force F. If the accelerating potential is connected in series with the galvanometer is
increased to 2V, the electron in the same [21 April 2019, Shift-I]
magnetic field will experience a force. G2 S S2 SG
[20 April 2019, Shift-II] (a) (b) (c) (d)
S+G S+G S+G S+G
F
(a) F (b) (c) 2 F (d) 2F
2 Sol. (a)
Sol. (c) Given, galvanometer resistance, RG = G and
Q Magnetic force, F = q(v × B) shunt resistance, RS = S,
= qv Bsinθ Let the series resistance is R, so the net
For uniform transverse magnetic field, θ = 90° resistance of circuit after connected R,
G× S
 2qV  R0 = R + … (i)
So, F = qvB = q  B G+ R
 me 
Since, it is given that the current should not
when electron accelerated through a potential change. So, it should be G,
difference, R0 = G … (ii)
 2eV  From Eq. (i) and (ii), we get
F = e  B …(i)
 me  G× S
G= R+
G+ S
When electron accelerated through a potential
GS G2
difference 2V. ⇒ R= G− ⇒ R=
 2e(2V)  G+ S G+ S
F′ = e   B
 me  Hence, the correct option is (a).

From Eqs. (i), we get F′ = 2F 22. A proton and an α-particle are


simultaneously projected in opposite
20. A coil in the shape of an equilateral triangle direction into a region of uniform magnetic
of side 2 cm is suspended from a vertex such field of 2 mT perpendicular to the direction of
that it hangs in a vertical plane between the the field. After some time it is found that the
poles of a permanent magnet producing a velocity of proton has changed in direction by
horizontal magnetic field of 100 × 10 −3 T. The 90°. Then at this time, the angle between the
magnetic field is parallel to the plane of the velocity vectors of proton and α-particle is
coil. For the moment of couple acting on the [21 April 2019, Shift-I]
coil to be 2 3 × 10 −5Nm, the current to be (a) 60° (b) 90° (c) 45° (d) 180°
passed through the coil is [20 April 2019, Shift-II] Sol. (c)
(a) 0.5 A (b) 1 A Key Idea In a circular motion, a body changes its
(c) 2 A (d) 4 A direction by 90° in one-fourth of its time period.
Sol. (c) Given, magnetic field, B = 2mT
Given, magnetic field, B = 100 × 10−3 T Let Tp be the time-period of revolution of proton
Torque, τ = 2 3 × 10−5 Nm in the magnetic field.
2πmp
Now, torque τ = MB = IAB Tp = …(i)
eB
where, M = magnetic dipole moment,
mα ~− 4mp and q α = 2q p
B = magnetic field
I = current to be passed through the coil. and Tα be the time-period of revolution of
α-particle in the magnetic field,
Putting the given values in above expression,
2πmp 2π(4mp)
2 3 × 10−5 = I ⋅ 3 × 10−4 × 100 × 10−3 or I = 2 A Tα = = …(ii)
qB (2e)B
182 AP EAMCET Chapterwise Physics

Now, the ratio of time-period of proton and Sol. (a)


α-particle, by dividing Eq. (i) and (ii),
Magnetic field at centre of a circular coil of n
Tp 1
= ⇒ Tα = 2Tp turns,
Tα 2 µ i
B= 0 n
Hence, the time-period of α-particle is double of 2r
the proton, i.e. if proton covers 90° of angle
If wire is stretched to double it’s length,
from its starting, then α-particle will cover 45°
of the angle. ⇒ 2(2πrn) = 2πr ′ n′
∴ Hence, the correct option is (c).
2r
n′ = n =
2r Q r ′ = r
or n  
r′ (r / 3)  3
23. A bar magnet placed in a uniform magnetic n′ = 6n
field making an angle θ with the field
For same field at the centre,
experiences a torque. If the angle made by the
µ 0 in µ 0 i ′ n′
magnet with the field is doubled, the torque ⇒ =
2r 2r ′
experienced by the magnet increases by
in i ′(6n) i
41.4%. The initial angle made by the magnet ⇒ = ⇒ i′ =
with the magnetic field is [21 April 2019, Shift-I] r (r / 3) 18
(a) 60° (b) 30° (c) 90° (d) 45° Q volume remains same after double coil’s
length,
Sol. (d) A
⇒ l × A = 2l × A′ ⇒ A′ =
As we know, torque in a magnetic field, 2
τ = MBsinθ ρl
Resistance, R=
if θ = θ1 , then τ1 = MBsinθ1 … (i) A
Similarly, if θ = θ2 then ρ (2l) 4ρl
after double coil’s length, R′ = = = 4R
τ 2 = MBsinθ2 = MBsin 2θ1 (Q Given, θ2 = 2θ1 ) (A / 2) A
41.4
Hence, emf E ′ = i ′ R′ =   (4R) = E
Given τ 2 = τ1 + τ1 × = 1.414 τ1 = 2τ1 i 2
(Q E = iR)
100  18  9
⇒ 2 τ1 = MBsin 2θ1 … (ii)
25. The magnetic field normal to the plane of a
From Eqs. (i) and (ii), we get
coil of N turns and radius r which carries a
1 sinθ1
= current i is measured on the axis of the coil at
2 sin 2θ1 a distance h from the centre of the coil. This is
⇒ sin 2θ1 = 2sinθ1 … (iii) smaller than the field at the centre by the
As we know that sin 2 θ = 2sinθ cosθ fraction, [22 April 2019, Shift-I]
Hence, 2sinθ1 cosθ1 = 2sinθ1 3 h2 2 h2 3 r2 2 r2
(a) ⋅ (b) ⋅ (c) ⋅ (d) ⋅
1 2 r2 3 r2 2 h2 3 h2
2cosθ = 2 ⇒ cosθ = ⇒θ = 45°
2 Sol. (a)
Hence, the correct option is (d). The magnetic field normal to the plane of a coil
of N turns and radius r having current i on the
24. A circular coil connected to a battery of emf axis at a distance h from the centre is given by
E produced a certain magnetic induction µ 0N i r 2
Baxis =
field at its centre. The coil is unwound, 3

stretched to double its length rewound into 2(r 2 + h2) 2


3
1 rd  h2 

2
a coil of of the original radius and µ 0 Nir 1 + 2 
2
3 µ 0N i r 2  r 
connected to a battery of emf E to produce = 3
=
2r 3
same field at the centre. Then, E is  h2  2
1 + 2 
2 3/ 2
2(r )
[21 April 2019, Shift-II]  r 
2E 3E 9E E [From Binomial theorem, (1 + x)n = 1 + nx]
(a) (b) (c) (d)
9 7 4 6
Magnetic Effects of Current 183

µ 0N i  3h2  v mg cosθ m cot θ


Baxis = 1 − 2  ...(i) t= = =
2r  2r  gsinθ qB. gsinθ qB
0.02 cot 45°
Magnetic field on the centre of a circular current t= = 5s [Q cot 45° = 1]
carrying coil is given by 4 × 10−3 × 1
µ Ni
Bcentre = 0 …(ii) 27. A wire of length 44 cm carrying a current of
2r 2 A is bent and the two ends are joined. This
From Eqs. (i) and (ii), we get shape is placed in a uniform magnetic field of
 3h2  50 mT. If the magnetic field is in North-South
Baxis = Bcentre 1 − 2 
 2h  direction, then the maximum torque acting
on the shape is [22 April 2019, Shift-II]
3h2
Baxis = Bcentre − . Bcentre …(iii) . × 10− 3 Nm
(a) 154 . × 10− 3 Nm
(b) 077
2r 2
(c) 3.08 × 10− 3 Nm (d) Zero
From Eq. (iii), it is clear that the magnetic field
on the axis is smaller than the field at the centre Sol. (a)
3h2 Given, length of the wire, L = 44 cm
by a fraction 2 .
2r current flowing through the wire, I = 2A
and uniform magnetic field,
26. A small block of mass 20 g and charge 4 mC B = 50 mT = 50 × 10− 3 T
is released on a long smooth inclined plane of
Now, the area of the loop will be maximum
inclination angle of 45°, A uniform horizontal
when it is made in the form of a circle.
magnetic field of 1 T is acting parallel to the
∴Circumference of circle = 2πr
surface, as shown in the figure. The time from
the start when the block loses contact with Q Circumference of circle = length of the wire (L)
the surface of the plane is [22 April 2019, Shift-I] or 2πr = 44
44
q r= = 7 cm
B m 2π
∴Area of loop, A = πr 2
A = π × 49 × 10− 4 cm 2
Now, the maximum torque acting on the shape,
θ
τmax = BiA
(a) 2 s (b) 3 s (c) 5 s (d) 6 s = 50 × 10− 3 × 2 × 49 π × 10− 4
Sol. (c) τmax = 1.54 × 10− 3 Nm

Given inclination of plane, 28. A toroid has a non-ferromagnetic core of


F inner radius 20.5 cm and outer radius
q 21.5 cm, around which 4200 turns of a wire
B θ m are wound. If the current in the wire is 10 A,
sin
mg the magnetic field inside the core of the toroid
mg mg cosθ is (µ 0 = 4 π × 10 − 7 Hm − 1) [22 April 2019, Shift-II]
θ = 45° (a) 20 mT (b) 40 mT (c) 20 π mT (d) 40 π mT
Mass of block, m = 20 g = 0.02 kg Sol. (b)
charge on the block, q = 4 mC = 4 × 10−3 C Given, inner radius of non-ferromagnetic core,
Magnetic field, B = 1 T ri = 20.5 cm
Magnetic force on the charge particles, F = Bqv outer radius of non-ferromagnetic core,
r0 = 21.5 cm
Particle will leave the inclined plane, when
r + r0 42
mg cosθ mean radius of toroid, r = i = = 21 cm
F = mg cosθ ⇒ Bqv = mg cosθ ⇒ v = 2 2
qB
number of turns of wire, N = 4200
Time taken to reached at the velocity v is given by
and current in wire, I = 10 A
v = 0 + gsinθ t [Q u = 0, a = gsinθ]
µ 0 = 4 π × 10−7 Hm− 1
184 AP EAMCET Chapterwise Physics

∴Circumference of core = length of the wire Sol. (b)


2πr = l Given, radius of coil, R = 10 × 10−2 m, number of
Then, number of turns per unit length, turns in coil, N = 100 turns, current through coil,
n n I = 0.5 A, magnetic field, B = 2 T and change in
N= =
l 2πr angle of rotating coil, θ = 0° to180°.
Now, magnetic field inside the core of the toroid Work done in turning a loop from angle θ1 to θ2 .
is magnetic field, B = µ 0 NI W = MB (cosθ1 − cosθ2)
= 4 π × 10− 7 ×
n
× 10 ⇒ W = NIAB [cos 0° − cos 180° ]
2πr ⇒ W = 100 × 0.5 × π × 100 × 10−4 × 2 [1 − (−1)]
n Hence, W = 2π J
= 2 × 10− 7 × × 10
r ∴The correct option is (b).
(Q Outer radius of core r = 21.5 cm)
4200 31. Two infinitely long wires carry currents 4A
B = 2 × 10− 7 × × 10 and 3A placed along X -axis and Y -axis
21
= 40 × 10−3 = 40 × 10− 3 T
respectively. Magnetic field at a point P(0, 0,
d) m will be …… T. [22 April 2018, Shift-I]
B = 40 mT
4µ 0 3µ 0 7µ 0 5µ 0
(a) (b) (c) (d)
29. Magnetic field at the centre of a circular loop 2 πd 2 πd 2 πd 2 πd
of area A is B. Then the magnetic moment of Sol. (d)
the loop is (µ 0-permeability of the free
space) [23 April 2019, Shift-I] Magnetic field at point P due to I x
2 µ I
(a)
BA
(b)
BA A
(c)
BA A
(d)
2 BA A Bx = 0 x (− Y-direction)
µ0π µ0 µ0π µ0 π 2πd
µ ⋅4 $
or Bx = − 0 j
Sol. (d) 2πd
As we know, the magnetic field due to a current Similarly, magnetic field at point P due to I y,
carrying circular loop at the centre,
µ i
B= 0 …(i) Iy=3A
2R
So, the magnetic moment of the Loop,
Ix=4A
M = iA P d
From Eq. (i), we get (0, 0, d) By
2BR
M= .A
µ0
A B
Q A = πR 2 ⇒ R = Bx
π
µ 0I y
2BA A By = (X-direction)
Hence, M= 2πd
µ0 π
µ ⋅3
⇒ B y = 0 a $i
Hence, the correct option is (d). 2πd
30. A circular coil of radius 10 cm with 100 ∴Resultant magnetic field,
µ
turns carrying a current of 0.5 A lies in a B = Bx2 + B2y = 0 42 + 32
magnetic field of 2 T such that the normal 2πd
5µ 0
drawn to the plane of the coil makes an B= (at − 45° from X-axis)
angles θ with the direction of the field. Work 2πd
done in rotating the coil to change the angle 32. Two moving coil galvanometer, X and Y have
θ from 0° to 180° is [23 April 2019, Shift-I] coils with resistance10 Ω and14 Ω cross-sectional
(a) π J (b) 2 π J areas 4 .8 × 10 −3 m 2 and 2.4 × 10 −3 m 2, number of
(c) 4π J (d) 8π J turns 30 and 45 respectively. They are placed in
Magnetic Effects of Current 185

magnetic field of 0.25 T and 0.50 T respectively. µNIr 2


Baxis = 54 × 10− 6 =
Then, the ratio of their current sensitivities 2(r + x 2)3 / 2
2

and the ratio of their voltage sensitivities are Here, r = 3 cm = 3 × 10− 2 m


respectively [22 April 2018 Shift-I]
and x = 4 cm = 4 × 10− 2 m.
(a) 2 : 3, 14 : 15 (b) 5 : 7, 2 : 1 From above relation, we get
(c) 2 :13, 1 : 2 (d) 14 : 15, 2 : 9
54 × 10− 2 × 2 × (25 × 10− 4 )3 / 2
µNI =
Sol. (a) (3 × 10− 2)2
There is no information regarding spring µNI
constant. So, Bcentre = = 250 µT
2r
So, let’s assume their spring constants are same.
N BA 35. A long straight wire Q i
Current sensitivity, I S = P
k carrying electric 45°
d
IS1 N1 B1 A1 k2 current ‘i is bent at
So, = its mid-point to form
IS2 N 2 B2 A2 k1
an angle of 45° as
30 × 0.25 × 4.8 × 10− 3  1 
= ×  shown in the figure. Magnetic field at a point
. × 10− 3
45 × 0.5 × 24 1  P at a distance d from the point Q of bending
IS1 is [23 April 2018, Shift-I]
= 2/ 3
IS2 µ 0i µ 0i
(a) [ 2 − 1] (b) [ 2 − 1]
Voltage sensitivity, VS = NBA / Rk 4 πd 2 πd
µ i µ i
V NB AkR 2  R  2 14 14 (c) 0 (d) 0
So, S 1 = 1 1 1 2 2 = ×  2  = × = 4 πd 2 πd
VS 2 N 2 B2 A2 k1 R1 3  R1  3 10 15
Sol. (a)
33. A beam of protons enters a uniform magnetic Magnetic field at a point i
Q
field of 0.314 T with a velocity 4 × 10 5 ms −1 in from a current carrying P d
45º
a direction making an angle 60° with the conductor making angle
direction of the magnetic field.The path of θ1 and θ2 with the ends
the beam is (mass of proton = 16 . × 10 −27 kg) of wire is given using
Bio-Savart’s law as
[22 April 2018 Shift-II]
µ I
(a) a circle of radius 0.2 m B = 0 (sinθ1 + sinθ2)
4 πr
(b) a straight line
µ 0I
(c) a helix with a pitch 4 cm = [sin 90° + sin135° ]
4 π  
d
(d) a helix with a pitch 4 mm
 2
Sol. (c)
2µ 0 I  1  µ 0I
As, v is not perpendicular to B, so path of = 1 −  = ( 2 − 1)
particle is helix. 4 πd  2 4 πd
2πm
Pitch of helical path = v cosθ × 36. A current carrying square loop is placed near
Bq a straight infinitely long current carrying
1 2 × π × 1.6 × 10− 27 wire as shown in the figure. The torque
= 4 × 10 × ×
5
= 4 cm
2 0.314 × 1.6 × 10− 19 acting on the loop is [23 April 2018, Shift-I]
i2
34. The magnetic field due to a current carrying
loop of radius 3 cm at a point on its axis at a
distance of 4 cm from its centre of 54 µ T. il
l
Then, the value of the magnetic field at the
centre of the loop is [22 April 2018 Shift-II] i2
(a) 250 µT (b) 150 µT (c) 75 µT (d) 125 µT a
b
Sol.(a)
186 AP EAMCET Chapterwise Physics

µ 0 i1 i 2 l µ0 i1 i 2 l
(a) (b) 39. A straight conductor of length 32 cm carries a
2 π ab 2 π a(a + b ) current of 30 A. Magnetic induction at a point
µ i i l(b − a) in air at a perpendicular distance of 12 cm from
(c) 0 1 2 (d) 0
2π ab the mid-point of the conductor is
Sol. (d) [23 April 2018, Shift-II]
The forces acting on the sides of the square due (a) 0.2 G (b) 0.3 G (c) 0.4 G (d) 0.5 G
to current i, all lie in the planes of the coil. Sol. (c)
Therefore, there is no net torque.
µ 0I
β= (sinθ1 + sinθ2)
37. The energies required to set up in a cube of 4 πr
side 10 cm
(i) a uniform electric field of 10 7 Vm − 1 and 16 cm 20 cm
(ii) a uniform magnetic field of 0.25 Wbm − 2 i=30A θ1
12 cm θ2
are respectively about
(µ 0 = 4 π × 10 − 7 Hm − 1 , ε 0 = 8 .9 × 10 − 12 Fm − 1 ) 16 cm

[23 April 2018, Shift-I]


(a) 0.445 J, 25 J (b) 4.45 J, 2.5 J 10−7 × 30  16  −5
= 2 ×  = 4 × 10 T
(c) 44.5 J, 25 J (d) 0.44 J, 2.5 J 12 × 10−2  20 
Sol. (a) = 0.4 × 10−4 T = 0.4 G
1 1 B2
Energy densities are uE = ε0 E 2 ⇒ uB = 40. A circular loop and an infinitely long straight
2 2 µ0
conductor carry equal currents, as shown in
So, energy required to setup a uniform electric the figure. The net magnetic field at the centre
field in cube of side 10 cm is of the loop is B1 , when the current in the loop
1
U E = uE × Volume of cube = ε0 E 2 × l 3 is clockwise and B2 when the current in the
2 loop is anti-clockwise. Then B1 / B2 is
1 − 12
= × 8.9 × 10 × (10 ) × (0.1)3
7 2
[24 April 2018, Shift-I]
2
= 4.45 × 10− 12 + 14 − 3 = 4.45 × 10− 1 = 0.445 J
and energy required to setup a uniform
i B
magnetic field in cube of side 10 cm is
U B = uB × Volume of cube
B2 l 3 0.25 × 0.25 × (01 . )3
= = −7
= 25 J i
2µ 0 4 π × 10 × 2
15 13 13 17
(a) (b) (c) (d)
38. Two long straight parallel conductors are 19 15 17 19
carrying currents i1 and i2 in the same
direction. Work done per unit length, when
Sol. (*)
the distance between them is doubled is When current is clockwise, then net magnetic
[23 April 2018, Shift-II] field,
µ i µ i µ i
µ0 µ0 B1 = 0 − 0 ⇒ B1 = 0 1 − 
1
(a) 2 × i1 i 2 (b) i1 i 2 ln[2 ]
2π 2π 2r 2πr 2r  π
µ0 Similarly, when current is anti-clockwise
(c) i1 i 2 ln[4] (d) 0
2π µ i
B2 = 0 1 + 
1
Sol. (b) 2r  π
π − 1 22
µ 0 i1 i 2
2d
2d −1
Work done = ∫d F. dx = ∫ 2πx ⋅ dx Now,
B1
= π = 7
B
⇒ 1 =
15
d
B2 π + 1 22 B 29
µ 0 i1 i 2 µ ii +1 2
= [ln x]2d d = 0 1 2 (ln 2) π 7
2π 2π
18
Magnetism and Matter
1. Unit of pole strength of a magnet is of coil makes 90° with the direction of magnetic
field.
[17 Sep. 2020, Shift-I]
∴ θ = 90°
(a) Am −1 (b) Am 2 (c) Am −2 (d) Am
∴ τ = nIAB sin 90° = nIAB
Sol. (d)
Dipole moment of a magnet, 3. If the angles of dip at two places are 30° and
M = IA ⇒ 2ml = IA [m = pole strength] 45° respectively, then the ratio of horizontal
IA components of earth’s magnetic field at the
⇒ m= two places will be [17 Sep. 2020, Shift-II]
2l
Ampere × metre2 (a) 3 : 2 (b) 1 : 2 (c) 1 : 3 (d) 1 : 2
Unit of pole strength (m) =
metre Sol. (a)
Am2 If Be be the earth’s net magnetic field, then the
= = Am
m ratio of horizontal components of earth’s
magnetic field at the two places is given as
2. Given figure shows the north and south H1 B cosδ1 cos δ1
poles of a permanent magnet in which a coil = e =
H2 Be cos δ 2 cos δ 2
of n turns of cross-sectional area A is
cos 30°
resting, such that when a current I is passed = [Q δ1 = 30° , δ 2 = 45° ]
through the coil, the plane of the coil makes cos 45°
and angle θ with respect to direction of =
3/ 2
=
3
magnetic field B. If the plane of magnetic 1/ 2 2
field and the coil are horizontal and vertical ∴ H1 : H2 = 3: 2
respectively, the torque on the coil will be
[17 Sep. 2020, Shift-II] 4. The deflection produced in a tangent
galvanometer, whose coil has a resistance of
N S 9Ω is 30°. The potential difference across the
coil is 4.5 V. If the number of turns in the
Coil coil is 10, the radius of the coil is (Given,
(a) nI AB cos θ BH = 3.14 × 10 − 5 T) [17 Sep. 2020, Shift-II]
(b) nI AB sinθ (a) 2 3 × 10− 2 m (b) 10 3 × 10− 2 m
(c) nI AB (c) 6 × 10− 2 m (d) 3.5 × 10− 2 m
(d) None of the above, since the magnetic field is radial
Sol. (b)
Sol. (c) For tangent galvanometer,
According to question, torque acting on the coil
Deflection, θ = 30°
in magnetic field B is given as
Resistance of coil, R = 9 Ω
τ = nIAB sinθ
Potential difference across coil, V = 4.5 V
When the plane of magnetic field and the coil are
horizontal and vertical respectively, then normal Number of turns in the coil, N = 10
BH = 3.14 × 10−5 T
188 AP EAMCET Chapterwise Physics

Current flowing through the coil, Sol. (d)


V 4.5
I= = = 0.5 A The given situations of two bar magnets is
R 9 shown in the following figure.
We know that, current flowing through the
galvanometer is given as S
2r BH tanθ
I=
µ0 N
l M
µ0 N I 4 π × 10−7 × 10 × 0.5
⇒ r= =
2 BH tanθ 2 × 3.14 × 10−5 × tan 30°
N
= 10 3 × 10−2 m l
S N
5. A magnet of magnetic moment M is rotated
through 360° in a magnetic field H, the work Magnetic dipole moment of each bar magnet
done will be [18 Sep. 2020, Shift-I] M1 = M 2 = M = ml
(a) MH (b) 2MH (c) 2π MH (d) 0 M1 and M 2 are perpendicular to each other as
Sol. (d) shown in the figure.
Work done to rotate magnetic dipole in M2
magnetic field from angle θ1 to θ2 is given as
W = MB(cosθ1 − cosθ2) M1
Here, θ1 = 0° and θ2 = 360°
∴ W = MB(cos 0° − cos 360°) = MB(1 − 1) = 0
∴Net magnetic moment of the system,
6. A straight wire carrying current i is turned M= M12 + M 22 + 2M1 M 2 cos 90°
into a circular loop. If the magnitude of
magnetic moment associated with it in MKS = M12 + M 22 = M2 + M2
units is M, the length of wire will be = M 2 = ml 2 = 2 ml
[18 Sep. 2020, Shift-II]
4πM 4πi Mπ
8. A magnetic needle lying parallel to a magnetic
(a) 4πiM (b) (c) (d) field requires W units of work to turn it
i M 4i
through 60°. The torque required to maintain
Sol. (b) the needle in this position will be
Suppose, l be the length of wire. [21 Sep. 2020, Shift-II]
l 3
Hence, l = 2πr ⇒ r = (a) 3W (b) W (c) W (d) 2W
2π 2
∴ Magnetic dipole moment, Sol. (a)
M = iA
2
According to the question, work done required
l2
⇒ = iπ ⋅   to rotate magnetic needle from θ1 = 0° to
l
M = i ⋅ πr 2
= iπ ⋅ 2
 2π  4π θ2 = 60°.
il 2 4 Mπ 4πM W = MB(cosθ1 − cosθ2)
⇒ M= ⇒ l2 = ⇒ l=
4π i i = MB(cos 0°− cos 60°)
= MB1 − 
1
7. Two identical thin bar magnets, each of  2
length l and pole strength m, are placed at MB
right angle to each other with north pole of W= ⇒ MB = 2W …(i)
2
one touching south pole of the other. The
3
magnetic moment of the system is ∴Torque,τ = MBsin 60° = MB
2
[21 Sep. 2020, Shift-I] 3
= 2W × [from Eq. (i)]
(a) 0.5 l m (b) l m 2
(c) 2 l m (d) 2 l m = W 3= 3W
Magnetism and Matter 189

9. If the dipole moment of a short bar magnet The value of the torque acting on the magnet
is 1.25 A-m 2, find the magnetic field on its is [22 Sep. 2020, Shift-II]
axis at a distance of 0.5 m from the centre of (a) 2 π × 10−7 Nm (b) 2 π × 10−5 Nm
the magnet. [22 Sep. 2020, Shift-I] (c) 0.5 Nm (d) 0.5 × 102 Nm
(a) 1.0 × 10−4 NA −1 m−1 (b) 4.0 × 10−2 NA −1 m−1 Sol. (a)
(c) 2.0 × 10−6 NA −1 m−1 (d) 6.64 × 10−8 NA −1 m−1 Given, length of bar magnet,
Sol. (c) l = 10 cm = 10−1 m
Dipole moment, Pole strength, m = 10−3 A-m
M = 1.25 A-m 2 B = 4 π × 10−3 T
r = 0.5 m θ = 30°
Magnetic field on the axial position of bar ∴ Magnetic dipole moment,
magnet is given as M = ml = 10−3 × 10−1 = 10−4 A-m 2
µ 2M 2 × 1.25 ∴Torque, τ = MB sinθ
B = 0 × 3 = 10−7 ×
4π r (0.5)3 = 10−4 × 4 π × 10−3 × sin 30°
1.25 1
= 10−7 × 2 × = 2 × 10−6 NA −1 m −1 = 4 π × 10−7 × = 2 π × 10−7 N-m
0.125 2

10. A circular parallel plate capacitor of radius R 12. A thin rod of length L is magnetised and has
and spacing d is being charged by a constant magnetic moment M. The rod is then bent in
current I D. Find the magnetic field between a semicircular arc. The magnetic moment in
the plates at a distance r from the axis, the new shape is [23 Sep. 2020, Shift-I]
where r > R. [22 Sep. 2020, Shift-I]
(a)
M
(b)
M
(c)
M
(d)
2M
µ I r µ I µ I L π 2π π
(a) 0 D 2 (b) 0 D (c) 0 D (d) Zero
2 πR 2 πR 2 πr Sol. (d)
Sol. (c) The magnetic dipole moment of a magnetic
The given situation is shown in the following dipole is given by
figure. M = m× L …(i)
ID
where, m is pole strength and L is dipole length.
R According to question,
m m
r>R ID d L
P
Now, rod is bent in form of a semi-circular arc,
r r

R +Q R
m –m
ID
M ′ = m × (2R)
= m × 2 ×   Q πR = L or R = L 
L
Magnetic field at point P at a distance r (> R) due  π  π 
to current I D is given as
2
µ I Q B = µ 0 ⋅ I  = (mL)
B= 0⋅ D π
2π r  2π r 
2
M′ = M [from Eq. (i)]
11. A bar magnet of length 10 cm and having π
the pole strength equal to 10 −3 A-m is kept in
13. A metal rod is subjected to cycles of
a magnetic field having magnetic induction
magnetisation at the rate of 42 Hz. Density
B equal to 4 π × 10 −3 T. It makes an angle of of the metal is 6 × 10 3 kg m −3 and its specific
30° with the direction of magnetic induction.
heat capacity is 0 .1 × 10 −3 cal kg −1 °C −1 . If the
190 AP EAMCET Chapterwise Physics

area of its B-H loop corresponds to energy Now, distance of magnet A from needle,
density of 10 −2 Jm −3, then the rise in its d1 = 30 − 6 = 24 cm
temperature in one minute is and distance of magnet B from needle, d2 = 6 cm
[20 April 2019, Shift-II] ∴Magnetic field on a axial line is given as,
(a) 5°C (b) 10°C (c) 15°C (d) 20°C µ 2M
B= 0 3
4π d
Sol. (b)
Hence, the ratio of the magnetic moment for
Energy of area of B-H loop, A and B
∆Q = ms (∆θ) 3
M1 M M1  d1 
⇒ 10−2 × 42 × 60 = 6 × 103 × 01. × 10−3 × 4.2 × ∆θ = 32 ⇒ = 
d13 d2 M 2  d2 
10−2 × 42 × 60
⇒ ∆θ = ∴ d1 = 24 cm, d2 = 6 cm
. × 10−3 × 4.2
6 × 103 × 01 3
⇒ ∆θ = 10° C M1  24 
= 
M2  6 
14. The magnetic susceptibility of a paramagnetic M1
substance at −173 ° C is 15 . × 10 −2. To have the = (4)3 = 64
M2
susceptibility 0 .5 × 10 −2, the change in So, the ratio of the magnetic moments of A and
temperature in °C is [21 April 2019, Shift-II] B is 64 : 1.
(a) 100 (b) 180 (c) 200 (d) 220 16. The resultant magnetic moment of three
Sol. (c) magnetic dipoles, each of the magnetic
Given, magnetic susceptibility of a moment M shown in the arrangement is
paramagnetic substance, χ m1 = 1.5 × 10−2 [23 April 2019, Shift-I]
T1 = − 173°C = (−173 + 273) = 100 K
(a) 2 M (b) ( 2 + 1) M
χ m2 = 0.5 × 10−2 ,
(c) ( 2 − 1) M (d) M
According to the Curie’s law,
1 χ T Sol. (b)
χ m ∝ or m1 = 2 Three magnetic moments of magnetic dipole,
T χ m2 T1
are shown in the figure below as,
χ 
⇒ T2 = T1  m1  M M
 χ m2 
 1.5 × 10−2  90°
⇒ T2 = 100  = 300 K
 0.5 × 10−2 
T2 = (300 − 273)
T2 = 27° C
Hence, change in temperature, ∆T = T2 − T1 M
∆T = 27 − (−173) = 200°C

15. Two short bar magnets A and B are arranged Here, M1 = M 2A + M B2 + 2M A M B cos 90°
co-axially. The distance between their ⇒ M1 = M2 + M2 + 0
centres is 30 cm. A compass needle placed on
their axis at a distance of 6 cm from B shows ⇒ M1 = M 2
no deflection. The ratio of the magnetic Now, the resultant, magnetic moment,
moments of A and B is [22 April 2019, Shift-II] Mresultant = M1 + M C
(a) 16 : 1 (b) 1 : 16
(Q M1 and M C are in the same
(c) 64 : 1 (d) 1 : 64
direction)
Sol. (c) ⇒ Mresultant = M 2 + M
Given,
⇒ Mresultant = (1 + 2) M
distance between the magnets, d = 30 cm
compass needle distance from magnet, B = 6 cm ∴correct option is (b).
Magnetism and Matter 191

17. Two short bar magnets each of magnetic Sol. (c)


moment of 9 Am 2 are placed such that one is Given, BH / BV = 1 / 3
at x = −3 cm and the other at y = −3 cm. If where, BH and BV are respectively, the horizontal
their magnetic moments are directed along and vertical components of earth’s magnetic
positive and negative X -directions field
respectively, then the resultant magnetic 3 BH
tanδ = BV / BH = = 3
field at the origin is [22 April 2018, Shift-I] BH
(a) 100T (b) 10T (c) 0.1T (d) 0.001T
δ = tan− 1 3 = 60°
Sol. (c) Angle of dip, δ = 60°
At origin, both magnetic fields will be in same
direction. 20. A sample of a paramagnetic salt containing
Magnetic field due to M1 (axial point)
3 × 10 24 atomic dipoles each of dipole
µ 2M 2× 9 moment 2 × 10 −23 A-m 2 is subjected to a
B1 = 0 × 3 = 10− 7 ×
4π r 27 × 10− 6 uniform magnetic field of 880 mT and cooled
2 to a temperature of 3.5 K. The degree of
B1 = × 10− 1 T
3 magnetic saturation achieved is 10%. If the
sample is subjected to a magnetic field of
Magnetic field due to M 2 (equatorial point)
990 mT and cooled to a temperature of 2.1 K,
µ M 9 1
B2 = 0 × 3 = 10− 7 × = × 10− 1 T then the total dipole moment of the sample
4π r 27 × 10− 6 3 is [23 April 2018, Shift-II]
As both B1 and B2 points in same direction, so (a) 11.25 A-m 2 (b) 23.5 A-m 2
effective field, B =  +  × 10− 1 T
1 2 (c) 15 A-m 2 (d) 75 A-m 2
 3 3
Sol. (a)
⇒ B = 10− 1 T = 0 .1 T Initially total dipole moment of sample
18. A short bar magnet of magnetic moment 10
= 3 × 1024 × 2 × 10−23 × = 6 JT −1
0.21 A-m 2 is placed with its axis perpendicular 100
to the direction of the horizontal component From Curie’s law,
of the earth’s magnetic field. The distance of B
m ∝ , we get
the point on the axis of the magnet from the T
centre of the magnet where the resultant B1 B
= 2
magnetic field is inclined at 45° with the T1 T2
horizontal component of the earth’s field 990 35 .
or B2 = 6 × × = 11.25 A-m 2
direction is (horizontal component of the 880 21 .
earth’s magnetic field = 4 .2 × 10 −5 T)
21. In the magnetic meridian of a certain place,
[22 April 2018, Shift-II]
the vertical component of the earth’s
(a) 12 cm (b) 20 cm (c) 5 cm (d) 10 cm magnetic field is 0.3464 G and the dip angle
Sol. (d) is 30°. The horizontal component of the
Resultant field is inclined at 45° to BH , so earth’s magnetic field at this location is
B = BH tanθ [24 April 2018, Shift-I]
µ M M × 10− 7 (a) 0.4 G (b) 0.6 G (c) 0.7 G (d) 0.8 G
⇒ 0 ⋅ 3 = BH ⇒ r 3 = ⇒ r = 10 cm
4π r BH Sol. (b)
BV BV 0.3464
19. At a certain place, the horizontal component tanδ = ⇒ BH = =
BH tanδ (1 / 3)
of earth’s magnetic field is 1/ 3 times the
vertical component. The angle of dip at that = 0.3464 × 1.73 = 0.6 G
place is [23 April 2018, Shift-I]
(a) 30° (b) 45° (c) 60° (d) 90°
19
Electromagnetic
Induction
1. The energy stored in a coil of inductance L Sol. (a)
carrying a steady current i is in the form Lenz’s law actually is that induced current
[17 Sep. 2020, Shift-I] always tends to oppose the cause which
(a) magnetic produces it. So in order to do work against
opposing force, we have to put extra effort. This
(b) electrical
extra work leads to the periodic change in
(c) both magnetic and electrical magnetic flux, hence more current is induced.
(d) heat Thus, the extra effort is just transformed into
Sol. (a) electrical energy which is law of conservation of
The energy stored in a coil of inductance L, energy. Therefore, Lenz’s law is a consequence
carrying a steady current i is given as of the law of conservation of energy.
1 4. When a rectangular coil is rotated in a
E = LI 2
2 uniform magnetic field about an axis passing
This energy is stored in the coil in the form of through its centre and perpendicular to the
magnetic energy. field, the emf induced in the coil varies
2. The current in a coil changes from 3 A to 1 A [17 Sep. 2020, Shift-II]
in 0.1 s in a coil of self-inductance 8 mH. The (a) linearly (b) exponentially
emf induced in the coil is [17 Sep. 2020, Shift-I] (c) sinusoidally (d) laterally
(a) 16V . × 10−2 V
(b) 16 Sol. (c)
(c) 16 × 10−2 V (d) 2 V When a rectangular coil is rotated in a uniform
Sol. (c) magnetic field about an axis passing through its
Change in currents in the coil centre and perpendicular to field, then magnetic
flux associated with coil is given as
∆I = (3 − 1) A = 2A
φ = BA cosθ = BA cosω t …(i)
Time interval, ∆t = 0.1 s
dφ d
Self induction, L = 8 mH = 8 × 10−3 H ∴ Induced emf, e = − =− BA cosω t
dt dt
∴ Induced emf in the coil, [From Eq. (i)]
dI ∆I 2
e=L =L = 8 × 10−3 × = 16 × 10−2 V = + BAω sinω t
dt ∆t 0.1
⇒ e = e 0 sin ω t …(ii)
3. Lenz’s law is a consequence of the law of where, e 0 is the peak value of induced emf.
conservation of [17 Sep. 2020, Shift-II] i.e. e 0 = BAω
(a) energy (b) charge From Eq. (ii) it is clear that, induced emf varies
(c) linear momentum (d) electric flux sinusoidally.
Electromagnetic Induction 193

5. The energy stored in a 50 mH inductor Sol. (c)


carrying a current of 4 A is [18 Sep. 2020, Shift-I] Since, coil of inductance L is divided into four
(a) 0.4 J (b) 4.0 J (c) 0.8 J (d) 0.04 J equal parts. Hence, inductance of each part is
L
Sol. (a) same and equal to .
4
Given, L = 50 mH = 5 × 10−2 H, I = 4A i.e., L1 = L2 = L3 = L4 =
L
∴ Energy stored in the inductor is given as 4
1 1 Since, L1 ,L2 , L3 and L4 are connected in parallel.
E = LI 2 = × 5 × 10−2 × 42 = 0.4 J
2 2 Hence, effective inductance L ′ of the
combination is given as
6. The coefficient of mutual inductance between 1 1 1 1 1
the primary and the secondary coil of a = + + +
L ′ L1 L2 L3 L4
transformer is 0.2 H. When the current in the
primary changes by 5 As − 1 , then the induced 1 1 1 1
= + + +
emf in the secondary will be L/4 L/4 L/4 L/4
4 4 4 4
[18 Sep. 2020, Shift-I] = + + +
L L L L
(a) 5 V (b) 1 V (c) 25 V (d) 10 V
1 16 L
⇒ = ⇒ L′ =
Sol. (b) L′ L 16
Given, coefficient of mutual induction,
M = 0.2 H 9. The magnitude of induced emf is directly
The rate of change in current in primary coil. proportional to the rate of change of
dI magnetic flux linked with the coil. This
= 5 As−1 statement is known as [21 Sep. 2020, Shift-I]
dt
∴Induced emf in the secondary coil, (a) Ohm’s law (b) Lenz’s law
dI (c) Faraday’s law (d) Ampere’s law
e = M = 0.2 × 5 = 1 V
dt Sol. (c)
Faraday’s law
7. What is the phase difference between the
flux linked with a coil rotating in a magnetic 10. A current I = 10 A is passed through the part
field and the induced emf produced in it? of a circuit shown in the figure. What will be
[18 Sep. 2020, Shift-II] the potential difference between A and B when
π π I is decreased at constant rate of 10 2 As −1 at
(a) 0 (b) (c) (d) π
4 2 the beginning ? [21 Sep. 2020, Shift-II]
Sol. (c) 12 V
Magnetic flux linked with coil rotating in A B
magnetic field B is given as 2Ω 5 mH
φ = BA cosωt
− dφ d (a) − 7.5 V (b) 3.5 V (c) − 3.5 V (d) 4 V
∴Induced emf , e = = − BA cosωt
dt dt Sol. (a)
= + BAωsinωt The given circuit diagram is shown below,
π
⇒ e = BAω cos ωt −  12 V
 2 A B
Hence, phase difference between flux linked 2Ω 5mH
with coil and induced emf is π / 2.
Given, I = 10A
8. A coil of inductance L is divided into four ∴
dI
= 102 As−1
equal parts and all the parts are connected in dt
parallel. The effective inductance of the Induced emf across inductor coil,
combination is [18 Sep. 2020, Shift-II] dI
e = L = 5 × 10−3 × 102 ⇒ e = 0.5 V
L L L dt
(a) (b) (c) (d) 4L
4 8 16
194 AP EAMCET Chapterwise Physics

Applying Kirchhoff’s voltage law between point 13. Lenz’s law is consequence of the law of
A and B.
conservation of [22 Sep. 2020, Shift-II]
V AB + 2 × 10 − 12 − 0.5 = 0
(a) charge (b) momentum
⇒ V AB + 20 − 12.5 = 0 (c) mass (d) energy
⇒ V AB + 7.5 = 0
Sol. (d)
⇒ V AB = −7.5 V
Lenz’s law is consequence of the law of
11. The Lenz’s law gives [22 Sep. 2020, Shift-I] conservation of energy.
(a) the direction of induced current
14. A rod of length 1.0 m is rotated in a plane
(b) the quantity of charge flowing
perpendicular to a uniform magnetic field of
(c) the magnitude of the induced emf
induction 0.25 T with a frequency of 12 rev/s.
(d) motional emf in the circuit
The induced emf across the ends to the rod
Sol. (a) is [22 Sep. 2020, Shift-II]
According to Lenz’s law, induced current (a) 18.89 V (b) 3 V (c) 15 V (d) 9.42 V
produces such a direction that it opposes the
cause by which it produces. Hence, Lenz’s law Sol. (d)
gives the direction of induced current. Length of rod, l = 1.0 m
B = 0.25 T, f = 12 rev/s
12. Consider the situation shown in the figure.
B ω l 2 0.25 × 2 πf
The wire AB is sliding on the fixed rails with ∴ Induced emf, e = = × 12
a constant velocity. If the wire AB is replaced 2 2
by semicircular wire, the magnitude of the = 0.25 πf = 0.25 π × 12
induced current will [22 Sep. 2020, Shift-I] = 3 π = 9.42 V

A
15. The magnetic flux through a circuit of
resistance R changes by an amount ∆φ in
time ∆t. Then the total quantity of electric
charge Q, which is passing during this time
through any point of the circuit is given by
[23 Sep. 2020, Shift-I]
∆φ ∆φ
(a) Q = (b) Q = ×R
B ∆t ∆t
∆φ ∆φ
(c) Q = − + R (d) Q =
(a) increase ∆t R
(b) remain the same
Sol. (d)
(c) decrease
As we know, induced emf,
(d) increase or decrease depending whether the
∆φ
semicircle bulges towards the resistance or e= ....(i)
away from it ∆t
and induced current,
Sol. (b) e
Magnitude of induced current in the wire AB is i= …(ii)
R
given as
Bvl sinθ If the total electric charge Q has been flown
I= during ∆t time through any point of the circuit,
R then we may write
Here, θ = 90°
Q Q e
Bvl i= or = [from Eq. (ii)]
∴ I= ∆t ∆t R
R
Q
When wire AB is replaced by semicircular wire, then or e= ×R …(iii)
∆t
neither l nor θ are changed. B, v and R are already
constant. Hence, induced current will remains Equating both Eqs. (i) and (iii), we get
∆φ Q ∆φ
same. = × R or Q =
∆t ∆t R
Electromagnetic Induction 195

16. A coil of area 10 m 2 is placed in a uniform radius of wire become r / 2, so its length will be 4l
−1 −1 and its resistance will become 16R.
magnetic field of 0.3 NA m , with its plane
The number of turns is doubled in a coil, so its
perpendicular to the field. The coil rotates at radius should be doubled to accomodate the
a uniform rate to complete the revolution in length of wire. The area of coil will become 4 times.
8s. Find the average emf in the coil during ⇒ V2 = 8V1 …(i)
intervals when the coil rotates from
Now, current induced in the coil is P,
[23 Sep. 2020, Shift-I]
V2
(i) 0° to 90° position, ∴ P1 = 1
R
(ii) 90° to 180° position, V2
and power dissipated, P2 = 2 …(ii)
(iii) 180° to 270° position and 16R
(iv) 270° to 360° position. From Eqs. (i) and (ii), we get
(a) 3/2 V, 3/2 V, –3/2 V, –3/2 V (8V1 )2 64V12
P2 = ⇒ P2 =
(b) 3/2 V, –3/2 V, 3/2 V, –3/2 V 16R 16R
(c) 0 V, 0 V, 0 V, 0 V 4V 2
∴ P2 = 1
(d) –3/2 V, –3/2 V, 3/2 V, 3/2 V R
V 2 4V 2
Sol. (a) Then the ratio, P1 : P2 = 1 : 1 or P1 : P2 = 1 : 4
R R
Given that, area, A = 10 m2
Magnetic field, B = 0.3 NA −1 m −1 18. A metal rod AB of length 50 cm is moving at
T = 8s a velocity 8 ms − 1 in a magnetic field of 2T. If
When plane is perpendicular to field, then flux the field is at 60° with the plane of motion as
passing through it, φ = BA cos 0° = 0.3 × 10 × 1 = 3 shown in the figure, then the potentials V A
Wb and VB are related by [21 April 2019, Shift-I]
When plane is parallel to field, then flux passing
through it, φ = BA cos 90° = 0
For 0° to 90° position, average emf, × × × ×
T
∆φ φ90 ° − φ0 ° 3− 0 3 × × ×
εavg = = = = V ×
∆t T/4 2 2 × × 60º × ×
For 90° to 180° position, × × ×
φ − φ90 ° 0− 3 3 A B → 8 ms–1
εavg = − 180 ° =− = V × × × ×
T/4 2 2
(a) VA − VB = 8 V (b) VA − VB = 4V
Negative sign has been taken because flux
(c) VB − VA = 8V (d) VB − VA = 4V
associated to plane is decreasing.
Similarly, for 180° to 270° position, Sol. (b)
φ − φ180 ° 3 − 0 −3 A metal rod AB shown in the figure moves with
εavg = − 270 ° =− = V
T/4 2 2 speed 8 ms− 1 .

For 270° to 360° position, Magnetic field


φ − φ270 ° 0− 3 3 60º
εavg = + 360 ° =+ =− V
T/4 2 2
A B
17. A coil is placed in a time varying magnetic
field. The power dissipated due to current
induced in the coil is P1 . If the number of
turns is doubled and radius of the wire is As the emf induced in the metal rod,
halved, the power dissipated is P2. Then P1 : P2 e AB = V A − VB = l ⋅ (v × B)
is [20 April 2019, Shift-II] But, V A − VB = lvBsin(90° − 60°)
(a) 1 : 4 (b) 3 : 2 (c) 2 : 1 (d) 4 : 1 V A − VB = 50 × 10− 2 × 8 × 2 × sin 30°
Sol. (a) 1
= 8 × = 4V
According to the question, 2
Hence, the correct option is (b).
196 AP EAMCET Chapterwise Physics

19. A coil of mean area 500 cm 2 and having 1000 21. A circular coil of area 0.1 m 2 having 200 turns
turns is held with its plane perpendicular to is placed in a magnetic field of 40 T. The
a uniform field of 0.4 G. If the coil is turned plane of the coil makes 30° with the field. If
1 the field is removed for 0.1 s then the
through 180° in second, then the average
10 induced emf in the coil is [22 April 2019, Shift-II]
induced emf is (Take, 1 G = 10 −4 T) (a) 4000 V (b) 4000 3 V
(c) 2000 V (d) 2000 3 V
[21 April 2019, Shift-II]
(a) 0.04 V (b) 0.4 V (c) 4 V (d) 40 V Sol. (b)
Given, area of circular coil, A = 01
. m2
Sol. (a) number of turns in the coil, N = 200 turns
Given, area of coil A = 500 cm2 = 500 × 10−4 m2 and magnetic field, B = 40 T
number of turns, N =100, magnetic field, The angle between plane of the coil and
B = 0.4G= 0.4 × 10−4 T magnetic field, θ = 30°
1
∆t = Time taken = = 01
. sec, coil turned, θ = 0 ∴Flux associated with plane coil in uniform
10 magnetic field is given as,
to θ = 180°.
∴ φ1 = NBS cosθ (Q θ = 30°)
Change in flux
S = Area of surface
= θ f − θi = NBA cos180° − NBA cos 0° = −2NBA
φ1 = 200 × 40 × 01. × cos 30°
We know induced emf
θ = angle between the direction of magnetic
∆θ 2NBA 2 × 1000 × 0.4 × 10−4 × 0.05
E av = − = = field and normal to the surface.
∆t ∆t 01
. 3
= 0.04 V = 200 × 40 × 01. ×
2
20. Two concentric coplanar circular conducting φ1 = 400 3
loops have radii R and r (R >> r). Their mutual When, field removed for 0.1 sec then the flux,
inductance is proportional to φ2 = 0
[22 April 2019, Shift-I] Net flux, dφ = φ1 − φ2 = 400 3 and dt = 01
. sec
r R r2 R2 Now, induced emf in the coil when field is
(a) (b) (c) (d)
R r R r removed for 0.1 sec is
Sol. (c) dφ 400 3
∴ E= = or E = 4000 3 V
Let us consider a coil having N1 turns and dt 01
.
radius r is surrounded by another coil having N 2 22. A long solenoid with 2000 turns per meter
turns and of radius R as shown in the figure has a small loop of radius 3 cm placed inside
below,
the solenoid normal to its axis. If the current
N2 i through the solenoid increases steadily from
R
π2
2 . A to 5.5 A in
15 s, the induced emf in the
N1 100
r loop is [23 April 2019, Shift-I]
1 (a) 0.144 mV (b) 0.288 mV
(c) 0.072 mV (d) 0.316 mV
Magnetic field due to coil 2 at the centre,
µ NI Sol. (b)
B2 = 0 2 Key idea Magnetic field inside a solenoid of infinite
2R
length is given by expression
Magnetic flux through the coil, B = µ 0 ni
N µ N I. πr 2 where, n = number of turns per unit length.
φB 1 = N1 B2 A = 1 0 2 …(i)
2R Given, number of turns in solenoid, n = 2000,
But, φB 1 = MI …(ii) current through solenoid, i i = 1.5 A and i f = 55. A
From Eqs. (i) and (ii), we get So, ∆B = µ 0 n (i f − i i)
N1µ 0 N2 I. πr 2 N µ N . πr 2 r2 Putting the given values,
MI = ⇒M= 1 0 2 ⇒M∝
2R 2R R = 4 π × 10−7 × 2000 (55
. − 1.5)
Electromagnetic Induction 197

⇒ ∆B = 4 π × 10−7 × 2000 × 4 Sol. (d)


Now, emf induced in a loop of radius 3 cm, NφB
Self-inductance of a coil =
dφ ∆BA I
e=− = cosθ µ 0I
dt ∆t N × πR 2
2R πµ 0
4 π × 10−7 × 2000 × 4 × π × (3 × 10−2)2 (cos 0°) = = ⋅ NR
⇒e = I 2
π 2 / 100 L2 N 2
So, =
(Q θ = 0°) L1 N1
⇒ E = 0.288 mV N2 500
L2 = L1 × = 108 × = 90 mH
Hence, the correct option is (b). N1 600

23. A conducting rod PQ of P 26. Two circular loops of diameters 0.6 cm and
40 cm are kept coaxially with a separation
length 1 m is moving
of 15 cm between their centres. If a
with a uniform speed 2 qA
current 2 A flows through the smaller
ms −1 in a uniform qB loop, then the flux linked with the bigger
magnetic field of 4 T loop is (approximately) [24 April 2018, Shift-I]
which is directed into (a) 9 × 10−11 Wb (b) 0.9 × 10−11 Wb
Q
the paper. A capacitor of . × 10−11 Wb
(c) 18 (d) 2.7 × 10−11 Wb
capacity 10 µF is connected as shown in the
Sol. (*)
figure. Then, the charge on the plates of the
(None of the option is matching)
capacitor are [22 April 2018, Shift-I]
Magnetic field intensity due to small loop at
(a) q A = + 80 µC, q B = − 80 µC location of larger loop is
(b) q A = − 80 µC, q B = + 80 µC 2
µ 0 Ir 2 4 π ×10−7 × 2× (0.3×10−2) 36 π
(c) q A = + 125
. µC, q B = 1.25 µC B1 = = = 3 ×10−9 T
2x 3 2× (15×10−2)3 15
(d) q A = − 125
. µC, q B = + 1.25 µC
Flux linked with larger loop is
Sol. (a) 86 π
Charge on each plate should be φ2 = B1 A2 = 3 ×10−19 × π × (20 ×10−2)2
15
q = CV
36 π 2
where, V = vBl = × 4 ×10−11 = 0.42 × 10−11 Wb
153
⇒ q = C(vBl) = 10 × 10− 6 × 2 × 4 × 1
q = 80 µC 27. A current carrying circular loop is
So, charges on plates are ± 80 µC . perpendicular to a magnetic field of
induction 10 −4 T. If the radius of the loop
24. The length of a wire required to make a starts shrinking at a uniform rate of 2
solenoid of length l and self-induction L is mms−1 , then the emf induced in the loop
[22 April 2018, Shift-II] at the instant, when its radius is 20 cm
4 πL l Ll 2 πLl µ 0 Ll will be [24 April 2018, Shift-I]
(a) (b) (c) (d)
µ0 4 πµ 0 µ0 4π (a) 0.02 πµV (b) 0.08 πµV
(c) 0.03 πµV (d) 0.05 πµV
Sol. (a)
Length of wire in solenoid = 2πr × N Sol. (b)
 µ N2 A Given, B = 10−4 T, r = 20 cm = 20 × 10−2 m
Ll ∴ L = 0  4 πLl dr 2mm m
= 2πr  l  = =− = −2 × 10−3
µ 0 πr 2   µ0 dt s s
 A = πr
2
 dφ d
Induced emf, ε = − ⇒ ε = − (B ⋅ πr 2)
25. A coil of wire of radius r has 600 turns and self dt dt
= − Bπ  2r ⋅ 

inductance of 108 mH. The self inductance of a dr 2 dr
ε = − Bπ ⋅
coil with same radius and 500 turns is dt  dt 
[23 April 2018, Shift-II] ε = 10− 4 × π × 2 × 20 × 10− 2 × 2 × 10−3
(a) 80 mH (b) 75 mH (c) 108 mH (d) 90 mH ε = 8 π × 10− 8 = 0.08 π(µV)
20
Alternating Current
1. Why 220 V AC is more dangerous than 220 V Power factor = cos φ
DC? [17 Sep. 2020, Shift-I] R R 3
= = = = 0.6
(a) The DC attracts Z 5R / 3 5
(b) Peak voltage for AC is much larger
(c) The body offers less resistance to AC 3. A transformer works on the principle of
(d) Due to some other reason [18 Sep. 2020, Shift-I]
Sol. (b) (a) self-induction
Rms value of AC, Vrms = 220 V (b) electrical inertia
Peak value of AC, V0 = 2 Vrms = 2 × 220 = 311 V (c) magnetic effect of electric current
(d) mutual induction
VDC = 220 V
Since, peak value of 220 V AC is 311 V which is Sol. (d)
much more than 220 DC. Hence, 220 V AC is Transformer works on the principle of mutual
more dangerous than 220 V DC. induction.

2. In a series L - C - R circuit, the inductive 4. When a pure resistor is connected to an AC


reactance is twice the resistance and the source, the phase difference between the
capacitive reactance is 1/3rd of the inductive voltage and the current through the resistor is
reactance. The power factor of the circuit is [18 Sep. 2020, Shift-II]
(a) 90° (b) 180° (c) 45° (d) 0°
[17 Sep. 2020, Shift-II]
(a) 1.5 (b) 1.15 (c) 0.6 (d) 0.5 Sol. (d)
In purely resistive circuit, alternating voltage
Sol. (c) and alternating current, both are in same phase,
In L-C-R circuit, hence phase difference is zero.
XL
X L = 2R and X C =
3 5. Find the current through the primary coil (P)
Impedance of L-C-R circuit is given as of the transformer shown below.
Z= R 2 + (X L − X C) 2 [21 Sep. 2020, Shift-I]
2
R2 +  X L − L 
X
=
 3 
2 23 V
R2 +  L  =
2X 4 X L2 230 V P S (115 Ω)
= R2 +
 3  9
4 16 2
= R2 + (2R)2 = R2 + R
9 9
25R2 5 (a) 0.08 A (b) 0.04 A (c) 0.02 A (d) 0.01 A
= ⇒ Z= R
9 3
Alternating Current 199

Sol. (c) Sol. (a)


The given situation is shown in the figure For a transformer, it is given that, N P = 100,
VP = 100V, VS = 2000V
115Ω We know that,
NS V
230 V P S = S
23 V NP VP
VS N P 2000 × 100
⇒ NS = = = 2000
VP 100
For the given transformer,
Vp = 230 V 8. A tuned circuit of a transistor oscillator unit
VS = 23 V has an inductance of 5 mH and a capacitance
RS = 115 Ω of 5 pF. The natural frequency of the
Currrent in secondary coil, oscillator is [21 Sep. 2020, Shift-II]
V 23 (a) 100 kHz (b) 1 GHz
IS = S = = 0.2 A
RS 115 (c) 10 MHz (d) 1 MHz
We know that, in a transformer Sol. (d)
VS I For tuned circuit of a transistor oscillator unit,
= P
VP IS L = 5 mH = 5 × 10−3 H
VS I S 23 × 0.2 C = 5 pF = 5 × 10−12 F
⇒ IP = = = 0.02 A
VP 230 ∴ Natural frequency of tuned oscillator,
1 1
f = =
6. The coils of a step-down transformer have 2π LC 2π 5 × 10−3 × 5 × 10−12
500 and 5000 turns. In the primary coil an 1
AC current of 4A at 2200 V is sent. The value =
2π 2.5 × 10−14
of the current and potential difference in the
secondary coil is [21 Sep. 2020, Shift-II] 107 10
= = × 106 Hz
(a) 20 A, 220 V (b) 0.4 A, 22000 V 2π 2.5 9.93
(c) 40 A, 220 V (d) 40 A, 22000 V = 1.006 × 106 Hz
Sol. (c) ≈ 1 × 106 Hz ≈ 1 MHz
For a step-down transformer,
9. In a series, L-C-R circuit [22 Sep. 2020, Shift-I]
N P = 5000, N S = 500
(a) the voltage leads the current if X L < XC
I P = 4A, VP = 2200V
(b) the voltage leads the current if X L > XC
For a transformer, we know that
(c) the voltage and current are in phase
NS V I
= S = P (d) the current leads the voltage if X L > XC
NP VP IS
Sol. (b)
NS VS
Using, = Phasor diagram of L-C-R series circuit is shown in
NP VP the following figure,
N S VP 500 × 2200
⇒ VS = = = 220 V
NP 5000 VL
V I
Again, using S = P
VP IS
I P × VP 4 × 2200
IS = = = 40 A V (when VL>VC)
VS 220 VL–VC
φ
7. The primary of a transformer has 100 turns
and operates at 100 V-200 W. The number of
turns in the secondary, if the output voltage
VC
is 2000 V, then [21 Sep. 2020, Shift-II]
(a) 2000 (b) 200 (c) 100 (d) 500
200 AP EAMCET Chapterwise Physics

For phasor diagram, it is clear that if VL > VC , 12. When an inductor of inductance 6 / π H, a
then voltage V leads φ angle from current.
 V − VC 
capacitor of capacitance 50 / π µF and resistor
−1  X − XC 
where, φ = tan−1  L  = tan  L  of resistance R are connected in series with
 VR   R 
an AC supply of rms voltage 220 V and
Hence, voltage leads the current with angle φ, frequency 50 Hz, the rms current through
when the circuit is 440 mA. Match the inductive
VL > VC ⇒ IX L > IX C ⇒ X L > X C reactance, X L the capacitive reactance, X C
the resistance R and the impedance Z of the
10. In an L-C-R circuit, the capacitance is
circuit given in List-I with the corresponding
changed from C to 4C. For the same resonant
values given in List-II. [20 April 2019, Shift-I]
frequency, the inductance should be changed
from L to [22 Sep. 2020, Shift-II] List- I List- II
L L
(a) 2L (b) (c) (d) 4L (A) XL (i) 200 Ω
2 4
Sol. (c) (B) XC (ii) 300 Ω
In L-C-R circuit, (C) R (iii) 500 Ω
1
Resonant frequency, f0 = …(i) (D) Z (iv) 600 Ω
2π LC
When, C ′ = 4C, then resonant frequency is given A B C D A B C D
as (a) (iv) (ii) (i) (iii) (b) (iv) (iii) (i) (ii)
1 1 1 (c) (iv) (i) (ii) (iii) (d) (i) (iv) (iii) (ii)
f ′0 = = =
2 π L ′ C ′ 2π L ′ ⋅ 4C 4π L ′ C Sol. (c)
6
According to question, Given, inductance of inductor, L = H,
f0 = f ′0 π
50
1 1 capacitance of capacitor, C = µF
= π
2π LC 4 π L ′ C supply voltage, Vrms = 220 V,
1 1 1 1 supply frequency, f = 50 Hz
⇒ = ⇒ =
LC 2 L ′ C LC 4L ′ C and supply current, Irms = 440 mA
L Now, (A) inductive reactance, X L = ω L
⇒ 4L ′ = L ⇒ L ′ =
4 X L = 2πf × L (Qω = 2πf)
11. In an AC circuit, the current flowing is 6
= 2π × 50 × = 600Ω
 π π
I = 5 sin 100 t −  A and the potential X L = 600 Ω
 2 1 1
(B) Capacitance reactance, X C = =
difference is e = 200 sin (100 t) V. The power ωC 2πfC
consumption is equal to [23 Sep. 2020, Shift-I] π × 106
(a) 1000 W (b) 40 W (c) 20 W (d) 0 XC =
2π × 50 × 50
Sol. (d) 106
π ⇒ XC = ⇒
Given that, current, I = 5sin100 t −  A 2500 × 2
 2 X C = 200Ω
Potential difference, e = 200sin(100t) V (C) Resistance, R
Phase difference between current and potential ∴ Impedance of LCR circuit,
π
difference, φ = Z 2 = R 2 + (X L − X C) 2
2
Power consumption, P = VrmsIrms cos φ Putting the given values, we get
π (500)2 = R2 + (600 − 200)2
=   
5 200 
 cos 250000 = R2 + (400)2
 2  2  2
R2 = 250000 − 160000
P=0 Q cos π = 0
  R = 90000 ⇒ R = 300 Ω
2
 2 
Alternating Current 201

1
36 ⋅ + 16 × + 48 (0 − 0)
(D) Impedance, T T
=
V
Z = rms =
220
× 103 ⇒ Z = 500 Ω T  2 2 
Irms 440 or E avg = 18 + 8 = 26 V
∴ Erms = E avg = 26 V
13. In an oscillating LC circuit, the maximum
charge on the capacitor is Q. The charge on So, the rms value of emf is 26 V.
the capacitor when the energy is stored
15. In the given electrical circuit, if the switch S
equally between the electric and magnetic
is closed then the maximum energy stored in
fields is [20 April 2019, Shift-I]
Q Q Q
the inductors is : [21 April 2019, Shift-I]
(a) (b) (c) Q (d) 1 F, 4C
2 3 2
Sol. (d)
Given, in an oscillating LC circuit,
maximum charge on the capacitor = Q
6H
We know that,
total energy stored in the capacitor,
1 Q2 S
U= 2 F, 2C
2C (a) 3 J (b) 9 J (c) 12 J (d) 6 J
∴Energy is equally distributed in a electric field
and magnetic field, so the energy stored in the
Sol. (a)
capacitor is half of maximum. In figure below, if switch S is open then total
1 energy stored in the capacitors, is suppose E 0 .
∴ (energy stored in C, in electric field)
2 1F, 4C
= energy stored in C, in magnetic field
1 1 Q2 1 Q′2
∴ × = ×
2 2C 2 C
1 Q2 1 Q′2 2Q2 Q
or = × or Q′2 = or Q′ = 6H
4 C 2 C 4 2
Hence, the charge on the capacitor when the S
energy is started equally between the electric 2F, 2C
Q
and magnetic field is . Energy in1F capacitor,
2 2
E1 = CV 2 = C   =
1 1 Q 1 Q2 1 42
= × = 8J
14. If the emf of an AC source is given by 2 2  C 2 C 2 1
6 sin ωt + 4 sin 2ωt V, then the rms value of 1 Q2 22
the emf is [20 April 2019, Shift-II] Similarly, E 2 = = = 1J
2 C 2× 2
(a) 10 V (b) 26 V (c) 32 V (d) 20 V So, the total energy, E 0 = E1 + E 2 = 8 + 1 = 9J
Sol. (b) Now, switch’s is closed then the common
Given, emf of an AC source, potential of Capacitors,
E = 6 sinωt + 4sin 2ωt C V + C2 V2 1 × 4 + 2 × 1 6
Vcommon = 1 1 = = = 2V
Now, E 2 = (6 sinωt + 4sin 2ωt)2 C1 + C2 1+ 2 3
∴E 2 = 36sin2 2 ωt + 16sin2 2 ωt + 48 sinωt ⋅ sin 2ωt Hence, now the new arrangement of energy,
ωt 3ωt 
= 36sin2 ωt + 16sin2 2ωt + 48  cos − cos
1 1
 E1 = C1 Vcommon
2
= ×1 × 4 = 2J
 2 2  2 2
Now, the average value of E 2 , 1 1
and E 2 = C2 Vcommon
2
= × 2× 4 = 4 J
T
1 2 2
∴ E avg = ∫ [E 2 dt]
T0 Now, from conservation of energy,
1 T cos ωt cos 3ωt  
= ∫ 36sin 2 ωt + 16sin 2 2ωt + 48 
Energy stored in the inductor,
−  − E 0 − (E1 + E 2) = 9 − (2 + 4) = 3 J
T 0   2 2   EL ~
Hence, the inductor has 3 J of energy.
202 AP EAMCET Chapterwise Physics

16. An emf of 15 V is applied to a circuit Sol. (c)


containing 5 H inductance and 10 Ω Given situation following the figure below,
resistance. The ratio of the currents at time, L R
t = ∞ and t = 1 s is [21 April 2019, Shift-II]
e e2
(a) (b)
e2 − 1 e −1
e e2
(c) (d) 2
1 − e2 e −1 12 V, 50 Hz

Sol. (d) Given for AC circuit, Vrms = 12 V,


According to the question, The circuit as shown frequency , f = 50 Hz,
below, current, I = 0.5 A,
π
5H 10 Ω phase difference, φ =
3
∴Impedance of the AC circuit,
I (t) V 12
Z = rms = ⇒ Z = 24 Ω
I 0.5
Power factor,
15 V R π R
⇒ cos φ = ⇒ cos =
The current in the circuit, Z 3 24
t = I(∞) − [I(∞) − I(0)]e − t / τ
I() …(i) 1 R
⇒ =
L 5 1
Q Time constant, τ = = = sec 2 24
R 10 2 ⇒ R =12 Ω
at t → ∞, inductor behaves as short circuit
hence circuit will be 18. A coil has an inductance 0.7 H and it is
10 Ω joined in series with a resistance of 220 Ω.
When AC of 220 V, 50 Hz is applied to it,
I (∞) then wattless component in the circuit is
[22 April 2019, Shift-II]
(a) 5 A (b) 0.5 A (c) 0.7 A (d) 7 A
15 V Sol. (b)
15
So, I(∞) = = 1.5A …(ii) Given, inductance, L = 0.7 H, resistance
10 R = 220 Ω,
at t → 0, inductor behaves as open circuit AC potential, V0 = 220 V and frequency,
so, no current flow in the circuit, I(0) = 0 f = 50 Hz
So from Eq. (i), According to the question, we draw the
−t
following circuit,
t = 1.5 − (1.5 − 0)e (1 / 2) = 1.5(1 − e −2 t)
⇒ I()
at 1 = 1.5(1 − e −2)
t = 1sec, ⇒ I() …(iii)
0.7 H 220 Ω
From Eq. (ii) and (iii), we get
I
I(∞) 1.5 e2
⇒ = −2
= 2
I()
1 1.5(1 − e ) e − 1
220 V 50 Hz
17. When an inductor L and a resistor R in series
Since, in a series circuit, the current is same for
are connected across a 12 V, 50 Hz supply, a
all series connected elements.
current of 0.5 A flows in the circuit. The V 220
current differs in the phase from applied So, I 0 = = [Q ω = 2 πf]
voltage by π /3 radian. Then the value of R is
Z R + (ωL)2
2

[22 April 2019, Shift-I] 220


⇒ I0 =
(a) 10 Ω (b) 3 Ω (c) 12 Ω (d) 15 Ω (220)2 + (2π × 50 × 0.7)2
Alternating Current 203
220 220
= = = 0.7A 20. For the AC circuit shown below, phase
48400 + 48312.04 311 difference between emf and current is
As current in an AC circuit is given as, π
radian as shown in the graph. If the
I 0 = I R + jI L = I cos φ + jI sin φ 4
where, I L = I sin φ is wattless current impedance of the circuit is 1414 Ω,
ωL Q sin φ = ωL 
So, I L = 0.7 ×   then the values of P and Q are
Z  Z  [22 April 2018, Shift-I]
70 π
= 0.7 × = 0.495~− 0.5 A
311 P Q
Hence, the wattless component of current is 0.5 A.

19. In the given circuit, the angular frequency


of the voltage source is 70 × 10 3 rad s −1 . The
circuit effectively behaves like, E=E0 sin (100t)
[23 April 2019, Shift-I]
Y-axis
10µF 1µF 10Ω i/E

i E
X-axis
π
4 t

AC source
(a) purely resistive circuit (a) 1 k Ω, 10 µF (b) 1 k Ω, 1 µF
(b) series RL circuit (c) 1 k Ω, 10 mH (d) 1 k Ω, 1 mH
(c) series RC circuit Sol. (a)
(d) series LC circuit with R = 0 In the shown figure, current is ahead of voltage,
Sol. (c) so its a RC circuit, so P is a resistor and Q is a
Given, L = 10 µH, capacitor.
C = 1 µF, R = 10 Ω and angular frequency, Now, in RC circuit,
ω = 70 × 103 rad s−1 Z= R2 + X C2
Now, the impedance of series LCR circuit,
For simple solution,
2

R +  ωL −
1  Z = 1414 = 1000 × 1.414
Z= 2
 Given
 ωC 
= 1000 2 = 1000 1 + 1

= (1000)2 + (1000)2
Z = 10 2 +  70 × 10 +3 × 10 × 10 −6 − 
1
 70 × 10 +3 × 1 × 10 −6  ⇒ X C = 1000 R = 1000 Ω
1 1
2
XC = ⇒C =
Z = 100 +  0.7 −
100  Now,
⇒ ωC ωX C
 7 
1
⇒ C= = 10 µF
⇒ Z = 100 + (− 1358
. )2 100 × 1000
Here, negative sign shows that the circuit
properties are more likely to be capacitive than 21. An inductor and a resistor are connected in
the inductive i.e., series to an AC source. The current in circuit
⇒ XC > XL is 500 mA, if the applied AC voltage is 8 2 V
So, the circuit behaves like series RC circuit. 175
at a frequency of Hz and the current in
Hence, the correct option is (c). π
the circuit is 400 mA, if the same AC voltage
204 AP EAMCET Chapterwise Physics

225 (a) Only C


at a frequency of Hz is applied. The
π (b) L and R in series
values of the inductance and the resistance (c) C and R in series or L, C and R in series
are respectively [22 April 2018, Shift-II] (d) Only R

(a) 60 mH, 71 Ω (b) 60 mH, 71 Ω Sol. (c)


π
(c) 60 mH, 71 Ω (d) 60 mH, 71 Ω As current leads by , so circuit must be more
4
Sol. (d) capacitive them inductive. Hence, it is either a
V V C-R combination or L-C-R combination.
For an L-R circuit, I = =
Z R + L2ω2
2
24. A resistor and an inductor are connected in
2

=  
V series to an AC source of voltage
⇒ R + Lω
2 2 2

I 150 sin(100 πt + π) volt. If the current in the


Here, I1 = 500 × 10− 3 A  2π 
circuit is 5 sin 100 πt +  ampere, then the
ω1 =
175
× 2π
rad
= 350
rad  3
π s s average power dissipated and the resistance
V1 =8 2
2
of the resistor are respectively
 8 2  [24 April 2018, Shift-I]
⇒ R2 + L2 (350)2 =  −3

 500 × 10  (a) 187.5 W, 30 Ω (b) 187.5 W, 15 Ω
⇒ R2 + L2 (350)2 = 512 … (i) (c) 375 W, 30 Ω (d) 375 W, 15 Ω
and R2 + L2 (550)2 = 800 … (ii) Sol. (b)
Solving, we get R = 71 Ω and L = 60 mH
Given, peak voltage, V0 = 150 V
22. The rms value of emf is given by Peak current, I0 = 5
E = (8 sin ωt + 6 cos ωt) volt is Phase difference between voltage and current,
[23 April 2018, Shift-I] 2π π
φ= π− = = 60°
(a) 5 2 V (b) 7 2 V 3 3
(c) 10 V (d) 10 2 V Now, average power in R-L circuit,
Pav = Vrms ⋅ I rms ⋅ cos φ
Sol. (a)
V I 150 5 1
E = (8sinωt + 6 cosωt) = 0 ⋅ 0 cos 60° = × ×
2 2 2 2 2
= (A cosθsinωt − Asinθ cosωt)
where, A cosθ = 8 and Asinθ = 6. Pav = 187. 5 W
V 150
A2 (cos2 θ + sin2 θ) = 82 + 62 = 100 Impendence, Z = 0 =
I0 5
A = 10
A 10 5 × 2
rms value, Erms = = = =5 2
2 2 2 Z=30π
XL
23. In the AC circuit shown, E = E 0 sin(ωt + φ)
60°
 π
and i = i0 sin ωt + φ +  . Then, the box R
 4
contains [23 April 2018, Shift-II] Z = 30 Ω
R
BOX = cos φ
Z
R 1
= cos 60° ⇒ R = 30 ×
30 2
R = 15 Ω

AC source
21
Electromagnetic Waves
1. Microwaves are used in [17 Sep. 2020, Shift-I] Sol. (c)
(a) TV (b) radio transmission When a charge moves with acceleration, then it
(c) Radar (d) atmospheric research produces a time varying electric and magnetic
field. Hence, electromagnetic waves are
Sol. (c) produced by accelerated or deaccelerated charges
Microwaves have short wavelength, hence only.
energy dispersion during long distance
communication is very low. Thus, they are used 5. If a source is transmitting electromagnetic
in radar system. wave of frequency 8.2 × 10 6 Hz, then
wavelength of the electromagnetic waves
2. Radiations of intensity 0.5 Wm − 2 are striking
transmitted from the source will be
a metal plate, the pressure on the plate is
[21 Sep. 2020, Shift-I]
[17 Sep. 2020, Shift-II]
(a) 36.5 m (b) 40.5 m
(a) 0.166 × 10−8 Nm −2 (b) 0.332 × 10−8 Nm −2
(c) 42.3 m (d) 50.9 m
(c) 0.111 × 10−8 Nm −2 (d) 0.083 × 10−8 Nm −2
Sol. (a)
Sol. (a)
Frequency of electromagnetic wave,
Intensity of radiation,
ν = 8.2 × 106 Hz
I = 0.5 Wm −2 Wavelength, λ = ?
Pressure on the plate due to radiation is given by We know that, velocity of electromagnetic wave
I 0.5 is given as
p= = = 0.166 × 10−8 Nm −2 c
c 3 × 108 c = νλ ⇒ λ =
ν
3. Practically ozone layer absorbs radiations of 3 × 108
= = 36.5 m
wavelength [18 Sep. 2020, Shift-I] 8.2 × 106
(a) less than 3 × 10− 7 m (b) greater than 3 × 10−7 m
(c) equal to 3 × 10− 7 m (d) All of these 6. Pick out the longest wavelength from the
following types of radiations.
Sol. (a)
[21 Sep. 2020, Shift-II]
Practically ozone layer absorbs ultraviolet
(a) Visible blue light (b) Ultraviolet light
radiations coming from sun and wavelength of
ultraviolet radiation become less than 3 × 10−7 m. (c) X-rays (d) Visible red light
Sol. (d)
4. Electromagnetic waves are produced by
Ultraviolet light is electromagnetic radiation
[18 Sep. 2020, Shift-II]
with a wavelength shorter than that of visible
(a) charges at rest only light, but longer than X–rays.
(b) changes in uniform motion only ∴ λred > λblue > λ UV > λ X − ray
(c) accelerated or decelerated charges only
Hence, visible red light has longest wavelength.
(d) an uncharged stable particle at rest
206 AP EAMCET Chapterwise Physics

7. Light is an electromagnetic wave. Its speed in 10. A lamp delivers a luminous flux of 100 W to
vacuum is given by the expression an absorber of area 1 cm 2. The force due to
[22 Sep. 2020, Shift-II] radiation pressure is [20 April 2019, Shift-II]
µ0 ε0 1 (a) 3.3 × 10−4 N (b) 16.5 × 10−7 N
(a) µ 0 ε0 (b) (c) (d) (c) 3.3 × 10−6 N (d) 3.3 × 10−7 N
ε0 µ0 µ 0 ε0
Sol. (d)
Sol. (d) Q Force = Rate of change of linear momentum
Speed of light (EM wave) in vacuum or air is dp d  v  1 dv
given as F= =   =
dt dt  c  c dt
1
c= where, ν = electromagnetic wave energy.
µ 0 ε0 I 100
or F= = . × 10−8 N
= 333
where, µ 0 = permeability of free space c 3 × 108
and ε0 = permittivity of free space.
[Q I = 100 watt, given]
8. For television broadcasting, the frequency or . × 10−7 N
F = 33
employed is normally in range
11. Which of the following is/are the
[23 Sep. 2020, Shift-I] property/properties of a monochromatic
(a) 30-300 MHz (b) 30-300 GHz electromagnetic wave propagating in the
(c) 30-300 kHz (d) 30-300 Hz free space? [21 April 2019, Shift-I]
Sol. (a) 1. Electric and magnetic fields will have a phase
π
Generally, 30 to 300 MHz frequency is used for difference .
2
the television broadcasting signals.
2. The energy of the wave is distributed equally
This range of frequency is called VHF (Very
between electric and magnetic fields.
Higher Frequency) band.
3. The pressure exerted by the wave is the
9. An electromagnetic wave of frequency product of its speed and energy density.
1 × 10 14 Hz is propagating along z-axis. The 4. The speed of the wave is equal to the ratio of
the magnetic field to the electric field.
amplitude of electric field is 4 Vm −1 , then
(a) 1 and 3 (b) Only 2 (c) 2 and 3 (d) Only 4
energy density of the electric field will be
(Permittivity of free space Sol. (b)
= 8 .8 × 10 −12 C 2 N −1 m −2) [20 April 2019, Shift-I] The energy of electromagnetic wave is equally
−13 −3 −13 −3 distributed between electric and magnetic
(a) 35.2 × 10 Jm (b) 70.4 × 10 Jm
field. Some more properties of electromagnetic
(c) 70.4 × 10−12 Jm −3 . × 10−12 Jm −3
(d) 352 waves are as given below,
Sol. (c) (i) Electric and magnetic field will have zero
Given, phase difference.
electromagnetic wave frequency, fm = 1.0 × 1014 Hz (ii) The pressure exerted by electromagnetic wave
amplitude of the electric field, E 0 = 4 Vm−1 is the ratio of energy density to the speed of
permittivity of free space, ε0 = 8.8 × 10−12 C 2 N −1 m−2 light.
The value of energy density (energy/volume) is (iii) The speed of wave is equal to the ratio of
given by electric field to the magnetic field.
1 Hence, the correct option is (b).
∴ u = ε0 E 02
2 12. The amplitude of the electric field in a
Putting the given values, we get parallel beam of plane electromagnetic
1 1 waves of intensity 53.1 W m −1 is
= × 8.8 × 10−12 × (4)2 J/m 3 = × 8.8 × 16 × 10−12
2 2 (Permittivity of free space = 8 .85 × 10 −12
∴ u = 70.4 × 10−12 Jm −3 C2N −1 m −2) [21 April 2019, Shift-II]
Hence, the energy density of the electric field will (a) 400 NC −1 (b) 50 NC −1
be 70. 4 × 10−12 Jm −3 . (c) 100 NC −1 (d) 200 NC −1
Electromagnetic Waves 207

Sol. (d) But, by Faraday’s law of electromagnetic


induction, (EMI)
Given, dφB
intensity of electromagnetic wave ,I = 53.1 Wm−2 ε =−
dt
Permittivity of free space, From Eqs. (i) and (ii), we get
ε0 = 8.85 × 10−12 C2N −1 m−2 dφB
1 ∫ E. dl = − dt
Intensity of electromagnetic wave, I = ε0 E 02 c
2 D → IV, According to the Maxwell, when electric
where, E 0 = amplitude of the electric field. field and hence electric flux charges with time,
then an additional current comes into existence to
2I 2 × 531. the conduction current called displacement
E0 = =
∈0 c 8.85 × 10−12 × 3 × 108 current. Therefore, the total current across the
loop,
E 0 = 200 NC−1
I = Ic + Id
13. Match the following List-I with List-II. Hence, the modified form of the Ampere’s law is
[22 April 2019, Shift-I] ∫ B. dl=µ 0 [Ic + Id ]
List-I List-II 14. A point source of electromagnetic radiation
A. ∫ EdA (i) 0 has an average power output of 960 W. The
dφ peak value of the electric field at a distance
B. ∫ BdA (ii) − B
400 cm from the source is [22 April 2019, Shift-I]
dt
(a) 60 Vm−1 (b) 120 Vm−1 (c) 30 Vm−1 (d) 180 Vm−1
C. ∫ Edl (iii) Q
∈0 Sol. (a)
Given, average power output,
D. ∫ Bdl (iv) µ 0 ( i c + i d )
P = 960 W
Codes Distance, r = 400 cm = 4 m
A B C D
Intensity of EM waves is given by
(a) (iii) (ii) (i) (iv) P 1 P
(b) (iv) (i) (iii) (ii) I= = ε0 E 02 c ⇒ E 02 =
4 πr 2 2 2πr 2 ε0 c
(c) (iii) (i) (ii) (iv)
(d) (iii) (i) (iv) (ii) P
∴ E=
Sol. (c) 2πr 2 ε0 c
A → III According to Gauss’s law, total electric =
960
flux passing through a closed surface is equal to . × 42 × 8.85 × 10−12 × 3 × 108
2 × 314
1
times the total charge enclosed within the = 0.36 × 104 = 0.6 × 102 = 60 Vm−1
ε0
Q 15. A plane electromagnetic wave propagating in
surface. i.e., φ = ∫ E. dA =
ε0 a non-magnetic dielectric medium is given by
B → I, According to gauss’ law of magnetism, E = E 0 [ 4 × 10 − 7 x − 50 t], where x is in metre
total magnetic flux passing through a bar and t is in second. If the relative permeability
magnet around its enclosed surface is zero of the medium, µ r = 1 then the dielectric
because the magnetic lines of forces emerging constant of the medium is [22 April 2019, Shift-II]
from North-pole is equal to magnetic lines of (a) 2.42 (b) 5.76 (c) 8.26 (d) 4.84
force entering into South direction.
Sol. (b)
i.e., ∫ B. dA = 0 Here, equation of the electromagnetic wave,
C → II, The electromotive force in a wire is the E = E 0 [4 × 10− 7 x − 50t] … (i)
line integral, General equation of electromagnetic wave
E = ∫ E. dl …(i) E = E 0 [kx − ωt] … (ii)
So, by the compairing Eqs. (i) and (ii), we get
k = 4 × 10− 7 and ω = 50
208 AP EAMCET Chapterwise Physics

2π Q k = 2π  A dV Q E = V 
Hence, wavelength, λ = m   ⇒ I d = ε0  
4 × 10− 7  λ d dt  d
50 Putting the given values, we get
and frequency, f = Hz (Q ω = 2πf)
2π 8.85 × 10−12 × π × (4 × 10−4 ) × 5 × 106
⇒ Id = A
∴Speed of light, v = f λ 1 × 10−4
50 2π ⇒ I d = 556 ⋅ 28 × 10−6 A
⇒ v= × = 12. 5 × 107 m / s
2π 4 × 10− 7 = 0.556 mA
As, we know the speed of light, Hence, the correct option is (c).
1
c= 17. In a plane electromagnetic wave, the electric
µ 0 ε0
field oscillates with a frequency 2 × 10 10 s −1
v µ 0 ε0
⇒ = and amplitude 40 Vm −1 , then the energy
c µ med εmed
density due to electric field is
⇒ µ r εr =
c2  µ ε
Q µ r = med and εr = med (ε 0 = 8.85 × 10 −12 Fm −1 ) [22 April 2018, Shift-I]
v2  µ0 ε0
 . × 10−9 Jm−3
(a) 152 (b) 2.54 × 10−19 Jm−3
c2
(c) 3.54 × 10−9 Jm−3 (d) 4.56 × 10−9 Jm−3
⇒ εr = (Q µ r = 1)
v2 Sol. (c)
2
 3 × 108  Energy density due to electric field is
⇒ εr =  7
= 576
.
 12. 5 × 10  1
U = ε0 E 2
2
Hence, the dielectric constant of the medium
E 40
is 5.76. Here, E= 0 =
2 2
16. A parallel plate capacitor consists of two 1 40 × 40
∴ U = × 8.85 × 10− 12 ×
circular plates each of radius 2 cm, separated 2 2
by a distance of 0.1 mm. If the potential . × 10− 9 J / m 3
= 354
difference across the plates is varying at the
rate of 5 × 10 6 Vs −1 , then the value of 18. An electromagnetic wave of frequency
displacement current is [23 April 2019, Shift-I] 2 MHz propagates from vacuum to a
non-magnetic medium of relative
(a) 5.56 A (b) 5.56 mA
(c) 0.556 mA (d) 2.28 mA
permittivity 9. Then its’ wavelength
[22 April 2018, Shift-II]
Sol. (c) (a) increases by 100 m (b) increases by 50 m
According to the question, a parallel plate (c) decreases by 50 mm (d) decreases by 100 m
capacitor is shown in the figure below,
Sol. (d)
In vacuum wavelength,
E c 3 × 108
⊕ λ0 = = = 1.5 × 102 = 150 m
f0 2 × 106
2cm
In medium, speed of wave is
c c
v= ≈
0.1 mm εrµ r εr
(as medium is non-magnetic)
Given, radius = 2cm, distance between plates, c c
dV ∴ v= = = 1 × 108 ms-1
d = 0.1 mm and = 5 × 106 Vs−1 9 3
dt
Frequency remaining same, wavelength in
Displacement current in the capacitor, v 108
dφ dE medium is λ = = = 50 m
I d = ε0 = ε0 A f0 2 × 106
dt dt
dE ∴Wavelength decreases by 100 m.
I d = ε0 A
dt
Electromagnetic Waves 209

19. An electromagnetic radiation has an energy Also, k=
14.4 keV. To which region of the λ
2π 2π
electromagnetic spectrum it belongs? or λ= = = 6.67 × 10−3 m
[23 April 2018, Shift-I] k 300 π
(a) Infrared (b) Visible 21. An electromagnetic wave of frequency 45 MHz
(c) Ultraviolet (d) X-ray travels in free space along X -axis. At some
Sol. (d) point and at some instant, the electric field has
Given radiation has 14.4 keV energy, so it is a maximum value of 750 NC −1 along Y -axis.
within the energy range for X-rays which is The magnetic field at this position and time is
between 100 eV to 100 keV. [24 April 2018, Shift-I]
The wavelength of given radiation, (a) 2.5 × 10−6 $j (b) 5 × 10−6 k$ T
hc 6.64 × 10− 34 × 3 × 108 (c) 2.5 × 10−6 k$ T (d) 2.5 × 10−6 $i T
λ= joule = volt
E 14.4 × 103 × 1.6 × 10− 19
Sol. (c)
= 0.86 Å
X- rays have wavelength range between 01 . Å to Relation between electric and magnetic field in
1 Å. electromagnetic wave
E
20. The oscillating electric field of an B=
c
electromagnetic wave is given by E = 750 N / C
E y = 30 sin(2 × 10 11 t + 300 πx)Vm−1 . Then, the
Speed of light,
value of wavelength of the electromagnetic c = 3 × 108 m / s
wave is [23 April 2018, Shift-II] 750
B=
(a) 5.67 × 10−3 m (b) 6.67 × 10−3 m 3 × 108
. × 10−3 m
(c) 667 (d) 7.66 × 10−3 m
B = 2. 5 × 10− 6 T
Sol. (b) As, B must be perpendicular to both c and E,
From equation of wave, i.e.it is along Z-axis.
k = wave number = 300π B = 2. 5 × 10− 6 k$ T
22
Ray Optics and
Optical Instruments
1. The lower half of a vessel of depth 2d cm is 2. A ray of light travels from a medium of
filled with a liquid of refractive index µ 1 and refractive index 2 to another medium of
the upper half with a liquid of refractive refractive index 2. Total internal reflection
index µ 2. The apparent depth of the vessel takes place when the angle of incidence is
seen perpendicularly is [17 Sep. 2020, Shift-I] [17 Sep. 2020, Shift-II]
 µµ   1 1 (a) 30° (b) 45° (c) 60° (d) 80°
(a) d  1 2  (b) d  + 
 µ1 + µ 2   µ1 µ 2  Sol. (b)
 1 1  1  Refractive index of first medium, µ 1 = 2
(c) 2d  +  (d) 2d  
 µ1 µ 2   µ 1µ 2  Refractive index of second medium, µ 2 = 2
Sol. (b) If i C be the angle of incidence in the case of
If immiscible liquids of refractive indices µ 1 and total internal reflection, then
µ 2 are filled in a vessel and their real depths are 1 µ1 1
2
µ1 = ⇒ =
d1 and d2 as shown below sin i C µ 2 sin i C
2 1 1
⇒ = ⇒ 2=
2 sin i C sin i C
d cm µ2 1
⇒ sin i C = ⇒ sin i C = sin 45° ⇒ i C = 45°
2

3. A light wave has a frequency of 4 × 1014 Hz


d cm µ1 and a wavelength of 5 × 10 − 7 m in a medium.
The refractive index of the medium is
[18 Sep. 2020, Shift-I]
Then, apparent depth of the vessel seen (a) 1.5 (b) 1.33 (c) 1.0 (d) 0.66
perpendicularly is given as Sol. (a)
d d
dapp = 1 + 2 Frequency of light wave, ν = 4 × 1014 Hz
µ1 µ2
Wavelength, λ m = 5 × 10−7 m
Here, d1 = d2 = d Wavelength of light wave in air
d d
∴ dapp = + Velocity of light in air 3 × 108
µ1 µ2 λ air = =
Frequency of light 4 × 1014
1 1  −7
= 7.5 × 10 m
=d + 
 µ1 µ2 
Ray Optics and Optical Instruments 211

∴ Refractive index of medium is given as Hence, image appears to be raised by


λ air 7.5 × 10−7 0.08
m = 0.04 m
µ= = = 1.5
λmedium 5 × 10−7 2
Hence, travelling microscope must be moved
4. A prism (µ = 1.5) has the refracting angle of upward by 0.04 m.
30°. The deviation of a monochromatic ray
6. A ray of light is incident normally on a plane
incident normally on its one surface will be
mirror. The angle of reflection will be
(given, sin 48 °36 ′ = 0.75) [18 Sep. 2020, Shift-II]
[22 Sep. 2020, Shift-II]
(a) 18° 36′ (b) 22 ° 38′ (c) 18° (d) 22 °1′
(a) 0° (b) 90°
Sol. (a) (c) will not be reflected (d) 60°
The given situation is shown in the following Sol. (a)
figure.
When a ray of light incident normally on a
A
plane mirror, then angle of incidence , i = 0°
90º 90–30=60º Normal
90º
e
i=30º
δ

C B Hence, angle of reflection, r = i = 0°.


For the surface AB,
1 sin i 1 sin 30°
7. The focal length of a spherical mirror made
= ⇒ = of steel is 150 cm. If the temperature of the
µ sin e µ sin e
mirror increases by 200 K, its focal length
1
⇒ sin e = µ sin 30° = 1. 5 × = 0. 75 become (coefficient of linear expansion of
2 steel α = 12 × 10 −6°C −1 .) [20 April 2019, Shift-I]
= sin 48° 36′ [∴sin 48° 36′ = 0. 75]
(a) 186.3 cm (b) 153.6 cm
⇒ e = 48° 36′ (c) 150.036 cm (d) 150.36 cm
From figure angle of deviation,
Sol. (d)
δ = e − i = 48° 36′−30° = 18° 36′
Given,
5. A travelling microscope is focused on an ink focal length of spherical mirror, f = 150 cm
dot marked on a piece of paper. When a coefficient of linear expansion of steel,
glass slab (µ = 1.5) of thickness 0.12 m is α = 12 × 10−6 ° C −1
placed on the ink dot. The travelling As we know that,
microscope has to be moved ……… in order ∆R
to restore the focus. [22 Sep. 2020, Shift-I] coefficient of linear expansion, α =
R × ∆T
(a) 0.04 m, downwards (b) 0.04 m, upwards ∆R
(c) 0.06 m, downwards (d) 0.06 m, upwards = α∆T
R
Sol. (b) ∆f Q f = R 
= α ∆T  
We know that, f  2
Real depth
Refractive index, µ = ∆f = f α ∆T
Apparent depth
f′ − f = f α ∆ T
Here, µ = 1.5 and real depth = thickness of glass where, f = initial focal length of the mirror and
slab = 0.12 m
f ′ = final focal length of mirror
012
.
∴ µ= f ′ = f + f α ∆T ⇒ f ′ = f (1 + α ∆T)
Apparent depth
f ′ = 150(1 + 12 × 10−6 × 200) (Q∆T = 200 K)
0.12
Apparent depth = = 0.08 m f ′ = 150.36 cm
1.5
212 AP EAMCET Chapterwise Physics

8. An object is placed 0.l m infront of a convex Sol. (c)


lens of focal length 20 cm made of a material According to the question,
of refractive index 1.5. The surface of the lens
away from the object is silvered. If the radius
7 cm
of curvature of the silvered surface is 22 cm,
then the distance of the final image from the
silvered surface is [20 April 2019, Shift-I]
10 cm
(a) 10 cm (b) 11 cm (c) 12 cm (d) 13 cm
Sol. (b)
Given, focal length of convex lens, f = 20 cm
object distance, u = − 0.1 m = − 10 cm the apparent distance of object from the mirror,
refractive index of material, µ = 1.5 distance of image from the mirror, d
radius of curvature of silvered surface, R = 22cm = 7 cm + apparent depth
and focal length of concave mirror, Real depth
Q Apparent depth =
−R µ
fm = = − 11 cm
2 10
= (Qµ =1.33, given)
Therefore, the power of the mirror, 1.33
1 1 10
Pm = − ⇒ Pm = D ...(i) Hence, d = 7 cm + = 14.5 cm
fm 11 1.33
Further, focal length of the lens is 20 cm. 10. A girl of height 150 cm with her eye level at
1
So, the power of lens, Pl = D ...(ii) 140 cm stands in front of plane mirror of
20 height 75 cm fixed to a wall. The lower edge
When the light after passing through the lens of the mirror is at a height of 85 cm above
will be reflected back by concave mirror through
her feet level. The height of her image the girl
convex lens again then , image will formed.
can see in the mirror is [21 April 2019, Shift-I]
Now, the power,
(a) 130 cm (b) 140 cm (c) 120 cm (d) 150 cm
P = power of mirror + power of lens
+ power of lens Sol. (c)
P = Pm + Pl + Pl ...(iii) Ray diagram of a girl standing in front of a
From Eqs. (i), (ii) and (iii), we get plane mirror is given below,

+ 2   =
1 1 1 1 1 21
P= + + ⇒ P= M2
11 20 20 11  20  110
The focal length of equivalent mirror, P 75 cm
Eye 0 I1 (x+10) cm
110 θ
f =− cm
21 x
150 cm

M1
Now, the object in front of a convex lens,
140 cm I2
1 1 1 −21 1 1
= + ⇒ = − ⇒ v = −11 cm 85 cm
f v u 110 v 10
Hence, the distance of final image from the
silvered surface is 11cm. a a
9. An object is fixed at the bottom of a vessel Applying the property of triangle and from
and water is filled in the vessel upto a height similar triangles OPM1 and OI1 I 2 ,
of 10 cm. A plane mirror is placed at a height PM1 II
tanθ = = 1 2
of 7 cm from the surface of water in such a OP OI1
way that its reflecting surface faces the water. Since, PM1 = 140 − 85 = 55 cm
The distance of the image from the mirror is Let OP = a and OI1 = 2 a
(Refractive index of water, n = 133
. ) 55 x
[20 April 2019, Shift-II] tan θ = = ⇒ x = 55 × 2 = 110 cm
a 2a
(a) 7.5 cm (b) 7 cm (c) 14.5 cm (d) 21.8 cm
Ray Optics and Optical Instruments 213

So, the maximum height seen by the girl, Sol. (d)


H = 110 + 10 = 120 cm According to the question,
Since, the height above her eyes, has not effected
the mirror height, because it is just in front of the Object
mirror. Hence, the correct option is (c).
f 2f
11. A small object is enclosed in a transparent
2f f v'= Screen
solid sphere of radius 8 cm. The object is 5 0 cm
situated at 2 cm from the centre of the
sphere. If its image appears to be at 3.2 cm u 25
from the nearest side, then the refractive v=75 cm
index of the material of the sphere is
[21 April 2019, Shift-II] Given, focal length, f = 25 cm
(a) 1.62 (b) 1.45 and distance between image of an object and
screen,
(c) 1.55 (d) 1.50
v = 75 cm
Sol. (d) Now, By lens formula,
According to question, 1 1 1
= −
f v u
µ 3.2
8 cm [Q Because screen is moved closer to the lens.]
0 1 1 1
m = −
6c u f v
2 cm 1 1 1 75 × 25 75
= − ⇒ u= = cm
u 25 75 50 2
When the screen Shift-upto by 25 cm, then the
If µ 1 and µ 2 are refractive indices of first and screen will be at 2f.
second medium with respect to air, then ∴For to get sharp image, object has to be at 2f.
µ 2 µ1  µ 2 − µ1  So, the distances is v − u = f .
− = 
v u  R  50 −
75 25
= = 12.5 cm
 at 2 f 
 = 
Given, refractive index of material of sphere, , 2 2  v 50 cm
µ 1 = µ , refractive index of air, µ 2 = 1, radius of This is the distance through which the object
sphere, R = 8 cm, u = − 6 cm, shifted from the lens.
v = − 32
. cm
1 µ 1−µ µ 1 1−µ 13. In a spherical glass marble of radius 6 cm, a
⇒ − = ⇒ − =
−32
. −6 8 6 32 . 8 small air bubble is formed at 1 cm from the
µ µ 1 10 7µ 14 centre of the marble. The apparent position
⇒ + = + ⇒ =
6 8 8 32 24 32 of the air bubble from the nearest point on
⇒ µ = 1.50 the surface of the marble is about,

12. A thin converging lens of focal length 25 cm (refractive index of glass is 1.5.)
forms a sharp image of an object on a screen [22 April 2019, Shift-II]
placed at a distance of 75 cm from the lens. (a) 3.3 cm (b) 4.6 cm (c) 5.4 cm (d) 7.0 cm
Later the screen is moved closer to the lens Sol. (a)
by a distance 25 cm. The distance through
Refractive index of glass,
which the object is to be shifted so that its Real distance (d1 )
image on the screen is sharp again is µ=
Apparent distance (d2)
[22 April 2019, Shift-I]
(a) 50 cm towards the lens Given, d1 = (6 − 1) cm = 5 cm
5
(b) 50 cm away from the lens or 1.5 = (Qµ = 1.5, given)
(c) 12.5 cm towards the lens d2
(d) 12.5 cm away from the lens or d2 = 333
. cm
214 AP EAMCET Chapterwise Physics

sin i sin 0°
14. When an object is moved along the principle So, = 2 ⇒ = 2
axis of a concave mirror placed in air, the sin r1 sin r1
image coincides with the object if the object is sin r1 = 0 or r1 = 0
50 cm from the mirror. If the mirror is placed sin r2 1
Now, = ⇒ sin e = 2sin r2
at a depth of 20 cm in a transparent medium, sin e µ
the image coincides with the object when the But, r1 + r2 = 60°
object is 40 cm from the mirror. The refractive So, sin e = 2 sin 60° = 2× 3 / 2> 1
index of the liquid is [23 April 2019, Shift-I]
5 4 3 5 So, light is incidenting at more than critical
(a) (b) (c) (d) angle and totally internally reflected.
4 3 2 3
∴Deviation angle = (i + e) − (r1 + r2)
Sol. (c) = 0 − 60° = 60° (in
According to the question, an object is placed on magnitude)
the principle axis of a concave mirror is shown
in the figure below, 16. As shown in the figure, a parallel beam of
light incidents on the upper part of a prism
µ of angle 1.8° and material of refractive index
1.5. The light emerging out from the prism
30 cm 20 cm falls on a concave mirror of radius of
50 cm curvature 40 cm. This distance of the point
from the principal axis of the mirror where
the light rays are focussed after reflection
from the mirror is [22 April 2018 Shift-II]
Since, it is given that, an object at 50 cm from
1.8°
pole is act like, it placed at 40 cm from the pole
when mirror is placed inside a transparent R=40cm
medium.
⇒ vreal = 50 cm and vapparent = 40 cm
Now, distance outside the transparent medium,
Dreal = vreal − thickness of transparent medium, Prism
Dreal = 50 cm − 20 cm = 30 cm Concave mirror
Similarly, Dapparent = (40 − 20) cm = 20 cm
(a) 4.76 cm (b) 1.57 mm
Hence, the refractive index of the liquid, (c) 3.14 mm (d) 6.28 mm
D 30 3
µ = actual = =
D apparent 20 2 Sol. (b)
Parallel beam is deviated by prism by angle,
∴Hence, the correct option is (c).
δ = (µ − 1) A = (1. 5 − 1) × 1. 8° = 0. 9°
15. A light ray incidents normally on one surface π
= 0. 9° × radian
of an equilateral prism. The angle of deviation 180°
of the light ray is (refractive index of the This beam from prism is also parallel and is
material of the prism = 2) converged at a point in focal plane by concave
[22 April 2018, Shift-I] mirror.
(a) 60° (b) 30° (c) 0° (d) 120°
Sol. (a) δ Note:
Angle of
Given, i = 0° δ incidence
x over
mirror=δ

r2 R 20 π
Distance, x = F × δ = ×δ= × 0. 9° ×
2 2 180°
Ray Optics and Optical Instruments 215

= 0.157 cm = 1. 57 mm For, objective,


u = − 2 cm, f = 1.5 m
17. A lens forms real and virtual images of an
1 1 1
object, when the object is at u1 and u2 By − = , we have,
v u f
distances respectively. If the size of the
1 1 1
virtual image is double that of the real = − v = 6 cm
image, then the focal length of the lens is v 1.5 2
(take, the magnification of the real image For eyepiece,
as m) [23 April 2018, Shift-I] v = − 25 cm, f = 6.25 cm
u1 + u 2  u1 − u 2  1 1 1
(a)   m (b)   2m
Using − = , we get
v u f
 2   3 
1 1 1
u − u2  u1 + u 2  = −
(c)  1  3m (d)   2m u −25 6.25
 2   3 
u = − 5 cm
Sol. (c) 5
1 1 1
Lens maker formula, − =
v u f
Case 1 Real image v and f are positive, u is
negative, so 6
1 1 1 u u
+ = ⇒ 1 +1= 1
v1 u1 f v1 f So, distance between two lenses = 11 cm.
Since, magnification for real image, 19. Magnifying power of an astronomical
− v1 telescope for normal adjustment is 10 and
m=
u1 length of the telescope is 110 cm.
So, −
1
+1=
u1
… (i)
Magnifying power of the same telescope,
m f when the image is formed at the near point
Case 2 Virtual image, as image is formed is [24 April 2018, Shift-I]
infront of lens both u and v are negative, so (a) 14 (b) 18 (c) 23 (d) 26
1 1 1 − u2 u
− − = or −1 = 2 Sol. (a)
v2 u2 f v2 f Magnifying power in normal adjustment,
Given, size of virtual image = 2 × size of real where fo and fe are focal lengths of objective
image and eyepiece.
− u2 f
So, −
1
+1= … (ii) m = o = 10
2m f fe

Adding Eqs. (i) and (ii), we get fo = 10 fe


Given, tube length, fo + fe = 110
1 1 u u − 3 u1 − u2
− − + 1 − 1 = 1 − 2 or = 10 fe + fe = 110
m 2m f f 2m f
fe = 10 cm and fo = 100 cm
(u1 − u2) 3m
or f = Now, magnifying power, when image formed at
2
near point
m = o 1 + e 
18. In a compound microscope, the focal lengths f f
of two lenses are 1.5 cm and 6.25 cm. An fe  D
object is placed at 2 cm from the objective 100 1 + 10  = 10 × 35
m=  
and the final image is formed at 25 cm from 10  25 25
the eye lens. The distance between the two
⇒ m = 14
lenses is ........ (in cm). [23 April 2018, Shift-II]
(a) 6 (b) 7.75 (c) 9.25 (d) 11
Sol. (d)
23
Wave Optics
3λ λ
(a) cos θ = (b) cos θ =
1. The limit of resolution of an oil immersion 2d 4d
objective microscope of numerical aperture λ 4λ
(c) sec θ − cos θ = (d) sec θ − cos θ =
0.8 for light of wavelength 0.6 µ m is d d
[17 Sep. 2020, Shift-I] Sol. (b)
15
. 3 5 7 According to figure, point P and point Q are at
(a) µm (b) µm (c) µm (d) µm
8 8 8 8 same phase.
Sol. (b) According to figure,
Numerical aperture, NA = 0.8 O R
Wavelength, λ = 0.6 µm
∴ Limit of resolution θθ
λ 0.6 0.3 3 d
= = = = µm Q
θ
2 NA 2 × 0.8 0.8 8
A (90° –2θ) P
2. Light from a source travels out with a
velocity c. If the source moves away from the
observer with a velocity v, then the relative B
velocity of light with respect to observer is
[17 Sep. 2020, Shift-II] In ∆POR,
PR d
(a) c (b) c + v (c) c − v (d) c + v
2 2 cosθ = =
OP OP
Sol. (a) d
⇒ OP = …(i)
The speed of light in vacuum is the same for cosθ
any inertial reference as c (3 × 108 m/s). This is In ∆QOP,
true no matter how fast a light source is moving OQ
sin(90° − 2θ) =
relative to an observer. OP
OQ
3. PQ represents a wavefront and AO and BP, ⇒ cos 2θ =
OP
the corresponding two rays. Find the
condition on Q for constructive interference ⇒ OQ = OP cos 2θ …(ii)
at P between ray BP and reflected ray OP. ∴Path difference,
[18 Sep. 2020, Shift-I] ∆ = OP + OQ = OP + OP cos 2θ
O = OP(1 + cos 2θ) [Q1 + cos 2θ = 2cos2 θ]
d
= ⋅ 2cos2 θ [from Eq. (i)]
θ cosθ
Q d = 2d cosθ
λ
But path difference is .
A P 2
λ λ
∴ 2d cosθ = ⇒ cosθ =
2 4d
B
Wave Optics 217

4. A Fraunhofer diffraction pattern due to a 7. Two-point white dots are 2mm apart on a
narrow slit is obtained on a screen placed at black paper. They are viewed by eye of pupil
a distance D from the slit whose slit width is diameter 3 mm. What is the maximum
a. The distance of first secondary maximum distance at which these dots can be resolved
from the central maximum is by the eye? (λ = 500 nm) [22 Sep. 2020, Shift-I]
[18 Sep. 2020, Shift-II] (a) 5 m (b) 1 m (c) 6 m (d) 10 m
3Dλ 3Dλ 2 Dλ 2Dλ
(a) (b) (c) (d) Sol. (d)
a 2a 3a a
Distance between two point white dots,
Sol. (b) x = 2 mm = 2 × 10−3 m
In Fraunhofer diffraction pattern, the direction Diameter of pupil, d = 3 mm = 3 × 10−3 m
of secondary maximum is given as
and λ = 500 nm = 5 × 10−7 m
λ λ
θ = (2n + 1) = (2 × 1 + 1) If D be the maximum distance at which these
2a 2a two point dots can be resolve, then
3λ x 1.22 λ
⇒ θ= =
2a D d
∴Distance of first secondary maximum from the xd 2 × 10−3 × 3 × 10−3
central maximum is given by ⇒ D= =
3λ 3Dλ 1.22 λ 1.22 × 5 × 10−7
x = θD = ⋅D= D = 9.8 ~− 10 m
2a 2a

5. The central fringe in the interference pattern 8. Conditions of diffraction is


obtained in Young’s double slit experiment [22 Sep. 2020, Shift-II]
will be a dark fringe when the phase a a a a
(a) ≠ 1 (b) > > 1 (c) < < 1 (d) ≤1
difference between the waves from the two λ λ λ λ
slits is [21 Sep. 2020, Shift-I] Sol. (d)
π π For diffraction, the width a of slit must be less
(a) zero (b) (c) π (d)
2 3 than or comparable to wavelength of light used.
a
Sol. (c) i.e., ≤1
When the central fringe in the interference λ
pattern in YDSE be a dark fringe, then path 9. A mixture of yellow light of wavelength 580 nm
λ
difference, will be . and blue light of wavelength 450 nm is
2
incident normally on an air film of thickness
λ
i.e., ∆x = 2.9 × 10 −4 mm. The colour of reflected light is
2
[23 Sep. 2020, Shift-I]
∴Phase difference,
2π (a) red (b) blue
∆φ = × ∆x (c) violet (d) yellow
λ
2π λ Sol. (b)
= × = π
λ 2 When a light wave of wavelength λ falls at an
angle of incidence i on a film of refractive index
6. Which of the following generates a plane µ and thickness t, then the condition for
wavefront ? [21 Sep. 2020, Shift-II] constructive interference in the reflected system
(a) Point source is
(b) Extended source λ
2µt cos r = (2m + 1) , m = 0, 1, 2, ....
(c) Monochromatic source 2
(d) All light sources For normal incidence, i = r = 0
Sol. (b) λ
∴ 2µt cos 0 = (2m + 1)
An extended source produce plane wavefront. A 2
point source produce spherical wavefront. λ
⇒ 2µt = (2m + 1)
2
218 AP EAMCET Chapterwise Physics

λ 12 × π 9 × 10−6
⇒ 2t = (2m + 1) [Q µ = 1 for air] or λ= × m
2 180 5
4t 4 × 2. 9 × 10−7 or λ = 3768 Å
⇒ λ= = m
2m + 1 2m + 1 So, the wave length of incident light is
[Q t = 2. 9 × 10−4 mm = 2. 9 × 10−7 m] λ = 3768 Å.
11.6 × 10−7
⇒ λ= m 12. Unpolarised light from air incidents on the
2m + 1 surface of a transparent medium of
For m = 0, λ 0 = 11.6 × 10−7 m = 1160 nm refractive index 1.414 such that the reflected
11.6 × 10−7 light is completely polarised. Match the
For m = 1, λ1 = = 386 . 67 nm
2 ×1 + 1 angles given in List-I with the corresponding
11.6 × 10−7 values given in List-II. [21 April 2019, Shift-I]
For m = 2, λ 2 = = 232 nm
2× 2+ 1
Hence, wavelength of blue light of 450 nm is List-I List-II
closest to λ1 = 386. 67 nm. A. Angle of reflection (i)  2
2 sin− 1  
Hence, colour of reflected light is blue.  3

10. In a Young’s double slit experiment, if the B. Angle of refraction (ii)  2 −1  1 


sin− 1   − sin  
slit separation is twice the wavelength of  3  3
light used, then the maximum number of
interference maxima is [20 April 2019, Shift-I] C. Angle between incident (iii) sin− 1  1 
 
and completely  3
(a) 0 (b) 3 (c) 5 (d) 7 polarised light
Sol. (c) D. Angle of deviation of the (iv) cos − 1  1 
For possible interference maxima on the screen  
incident ray  3
the condition is
d sinθ = nλ …(i) The correct match is
Here, d = 2λ (Given) Codes
2λ sinθ = nλ A B C D A B C D
2sinθ = n (a) (ii) (iii) (i) (iv) (b) (ii) (iii) (iv) (i)
The maximum value of sinθ is 1. (c) (iv) (i) (iii) (ii) (d) (iv) (iii) (i) (ii)
Hence, n = 2 × 1 = 2 Sol. (d)
Thus the Eq. (i) must be satisfied by 5 integer Figure showing the ray diagram of refraction at
values i.e. − 2, − 1, 0, 1, 2 . a plane surface.
Hence, the maximum number of possible Given, i p = angle of polarisation
interference maxima is 5.
φ Polarised ray
11. The angular deviation of 5th order dark
fringe is 12° in a single slit experiment. If the ip θ
width of the slit is 9 µm then the wavelength
of the incident light is [20 April 2019, Shift-II] µ=√2
(a) 4862 Å (b) 5892 Å r Angle of
Angle of δ deviation
(c) 6022 Å (d) 3768 Å refraction
Sol. (d)
Given, angular deviation of 5th order dark
fringe, θ = 12°, 2
So, i p = tan− 1 µ ⇒ i p = tan− 1 2 = sin− 1
Width of the slit, d = 9 µm 3
λ From Snell’s law,
Now, angular deviation of nth fringe = n ⋅
d sin i p = µ sin r
π λ sin i p
∴ 12 × = 5× Here, µ = 1.414 = 2 ⇒ sin r =
180 9 × 10−6 µ
Wave Optics 219

1 2 Sol. (a)
sin r =
2 3 Given,
1 refractive index of glass plate, µ = 3
r = sin− 1
3
Polarising angle θ of incident light is given as,
angle of refraction
µ = tanθ
Now, the angle of deviation,
or 3 = tanθ or θ = tan−1 ( 3)
δ = ip − r
∴angle of incident, θ = 60° [Q tan 60 = 3]
2 1
δ = sin− 1 − sin− 1
3 3 refractive index is given as,
sin i
Angle of reflection, µ=
sin r
θ = 90° − r ⇒ r = 90° − θ
1 1 or 3sin r = sin 60° [Q i = θ = 60° ]
sin−1 = 90° − θ ⇒ = sin (90° − θ)
3 3 3 1
or 3sin r = or sin r =
1 1 2 2
cosθ = ⇒ θ = cos−1
3 3 −1  1 
or r = sin   ⇒ r = 30°
Angle between incident and completely  2
polarised light is given by Thus, the angle of refraction of the ray is 30°.
φ = ip + θ
15. In a Young’s double slit experiment, the two
+ cos− 1   = 2 sin− 1
2 1 2
φ = sin− 1 slits are separated by 0.5 cm and the screen
3  3 3
Hence, the correct option is (d).
is at 0.5 m from the slits. If 20000 bright
fringes are counted per meter on the screen,
13. A Young’s double slit experimental setup is then the wavelength of light used is
immersed in water of refractive index 1.33. [22 April 2019, Shift-II]
It has slit separation 1 mm and the distance (a) 5000 Å (b) 5890 Å (c) 6000 Å (d) 5460 Å
between slits and screen is 1.33 m. If the
Sol. (a)
wavelength of incident light on slits is Dλ
6300 Å, then the fringe width on the screen Q Fringe width, β = ,
d
is [21 April 2019, Shift-II]
where, D = distance between screen and slits,
(a) 6.3 mm (b) 0.63 mm
d = distance between two slits and λ =
(c) 0.63 m (d) 6.3 m βd
wavelength of light, λ =
Sol. (b) D
When Young’s double slit experiment setup is 1
Given, d = 0.5 × 10− 2 m, β = , D = 0. 5 m
λD 20000
immersed in water then fringe width, y = . −2
µd 0.5 × 10 1
∴λ = = × 10− 6 or λ = 5000 Å.
Given, wavelength of incident light on slits, 20000 × 0.5 2
λ = 6300 Å = 6300 × 10−10 m
distance between slit and screen, D = 1.33 m,
16. In a Young’s double slit experiment, light of
and refractive index of water,µ = 1.33 wavelength 5900 Å is used. When the slits
Hence, the fringe width on the screen,
are 2 mm apart, the fringe width is 1.2 mm.
6300 × 10−10 × 1.33 If the slit separation is increased to one and
y= half times the previous value, then the fringe
1.33 × 1 × 10−3
width will be [23 April 2019, Shift-I]
y = 6.3 × 10−4 m = 0.63 mm
(a) 0.9 mm (b) 0.8 mm (c) 1.8 mm (d) 1.6 mm
14. A ray of light is incident on the surface of a Sol. (b)
glass plate of refractive index 3 at the In a Young’s double slit experiment, wavelength
polarising angle. The angle of refraction of of light, λ = 5900 Å and distance between the
the ray is [22 April 2019, Shift-I] slits, d1 = d = 2 × 10−3 m,
(a) 30° (b) 45° (c) 60° (d) 37°
220 AP EAMCET Chapterwise Physics

3
Fringe width, β1 = 1 .2 × 10−3 m and d2 = d Sol. (c)
2 Minimum separation for a microscope, so that
λD images of 2-near by objects are just resolved is
As, the fringe width, β =
d given by
λ D λ D 1. 22 fλ 1. 22 × 5 cm × 5500 Å
So, β1 = 1 1 and β 2 = 2 2 dmin = =
d1 d2 D 8 mm
β d 1. 22 × 5 × 10− 2 × 5500 × 10− 10
Given, λ1 = λ 2 and D1 = D2 ⇒ 1 = 2 =
β 2 d1 8 × 10− 3
Putting the given values, we get = 41. 9 × 10− 7 m = 4.19 × 10− 6 m
3 = 4. 2 µm
d
1.2 × 10−3
⇒ = 2 ⇒ β 2 = 0.8 mm
β2 d 19. Two point sources S1 and S2 separated by a
Hence, the correct option is (b). distance 10 µm emit light waves of
wavelength 4 µm in phase. A circular wire of
17. Two polaroids are placed in the path of radius 40 µm is placed around the sources as
unpolarised light beam of intensity I 0 such shown in figure, then [23 April 2018, Shift-I]
that no light is emitted from the second
polaroid. If a third polaroid whose (O is the centre of the circle and OS1 = OS2)
polarisation axis makes an angle θ with that
B
of the first polaroid is placed between the Wire
polaroids, then intensity of light emerging
from the last polaroid is [22 April 2018, Shift-I] S1 S2

(a)  0  sin2 2θ (b)  0  sin2 2θ


I I A C
 8  4
(c)  0  cos 2 θ
I
(d) I0 cos 2 θ
2 D
(a) points A and B are dark and points C and D are
Sol. (a)
bright
When unpolarised light passes through first
(b) points A and B are bright and point C and D are
polariser, it becomes polarised and its intensity
dark
becomes half.
(c) points A and C are dark and points B and D are
∴After first polariser, intensity I1 = I 0 / 2 bright
I (d) points A and C are bright and points B and D are
After second polariser, intensity I 2 = 0 cos2 θ
2 dark
(Malus law)
Sol. (c)
After third polariser, intensity
I Path difference at point B,
I 3 = 0 cos2 θ cos2 (90° − θ)
2 ∆x B = S1 B − S2 B = 0
(because this is at 90° angle from first polariser)
I I I B
⇒ I 3 = 0 cos2 θsin2 θ = 0 (2sinθ cosθ)2 = 0 sin2 2θ
2 8 2
40 µm
18. A microscope has an objective of aperture
8 mm and focal length of 5 cm. The S1 10 µm S2
minimum separation between two objects to
be just resolved by the microscope is
(wavelength of light used = 5500 Å)
[22 April 2018, Shift-II] and at point D,
(a) 2.2 µ m (b) 3.4 µ m ∆x D = S1 D − S2 D = 0
(c) 4.2 µ m (d) 3.6 µ m Phase difference,
Wave Optics 221


φ= × (0) = 0 λD
λ Also, β=
d
So, intensity at points B and D Combining these, we get
I = Imax cos2 (φ / 2) = Imax
 πx 
So, points B and D are bright. I = I 0 cos2  
 β 
Path difference at point A,
∆x A = 35µm [40 − 5] 21. In Young’s double slit experiment, the two
But, figure shows ∆x A = ∆x C is maximum for slits are illuminated by a light beam
given circle. When path difference is maximum, consisting of wavelengths 4200 Å and 5040 Å.
intensity will be minimum. If the distance between the slits is 2.4 mm
So, points A and C are dark. and the distance between the slits and the
screen is 200 cm, the minimum distance
20. In Young’s double slit experiment of central
from the central bright fringe to the point
fringe is I 0 and fringe width is β. If a point is
where the bright fringes due to both the
at a distance x from the central fringe, then
wavelengths coincide is [24 April 2018, Shift-I]
the intensity at that point is
[23 April 2018, Shift-II] (a) 0.7 mm (b) 1.4 mm
(c) 2.1 mm (d) 2.8 mm
 πx   x
(a) I0 cos 2   (b) I0 cos 2   Sol. (c)
 β  β
 πx  πβ  Let n1 fringes of λ1 = 4200 Å and n2 fringes of
(d) I0 cos 2 
I0
(c) cos 2   
4  β   x  λ 2 = 5040 Å wavelength are formed in a fixed
distance on screen.
Sol. (a) So, nλ = constant or n1 λ1 = n2 λ 2
Intensity at distance x from central maximum is
For minimum distance to be coincide,
given by
φ n1 = n + 1, n2 = n
I = 4I 0 cos2 (n + 1)4200 = n(5040)
2
2πd 4200n + 4200 = 5040n
where, φ = sinθ ⇒ n= 5
λ
nλ D
Required distance, x = 1 1
d
S1 x Given, D = 200 cm = 2m, d = 2⋅ 4 × 10− 3 m,
d θ n1 = 5 + 1 = 6, λ1 = 4200 Å
6 × 4200 × 10− 10 × 2
S2 x=
D 2. 4 × 10− 3
x = 2.1 × 10− 3 m = 2.1 mm
24
Dual Nature of Radiation
and Matter
1. If ultraviolet radiation of 6.2 eV falls of an 6.63 × 10−34 × 3 × 108
V0 = − 1.24
aluminium surface, then kinetic energy of 1.6 × 10−19 × 4.36 × 10−7
the fastest emitted electron is − 1.60 eV
= 2.85 − 1.24 = 1.61 eV ~
(work-function = 4.2 eV) [17 Sep. 2020, Shift-I]
3. Which of the following statements is correct
. × 10−19 J
(a) 32 . × 10−21 J
(b) 32 regarding photoelectric effect?
(c) 7 × 10−25 J (d) 9 × 10−31 J
[18 Sep. 2020, Shift-II]
Sol. (a) 1. The electrons are emitted if emitting
Energy of ultraviolet radiation, surface is at high temperature.
E = 6.2 eV 2. Photoemission occurs if wavelength is
Work-function, φ = 4.2 eV less than a critical value.
∴ According to Einstein’s photoelectric 3. The KE of photoelectrons is proportional
equation, kinetic energy of the fastest emitted
to the square of the amplitude of incident
electron,
1
radiation.
2
m vmax = E − φ = 6.2 − 4.2 = 2 eV 4. The photoelectric current is proportional
2
= 2 × 1.6 × 10−19 J to the frequency of incident radiation.
1 2 (a) 1 (b) 2 (c) 3 (d) 4
⇒ mvmax = 3.2 × 10−19 J
2 Sol. (b)
2. The retarding potential necessary to stop the When the wavelength of incident radiation on
emission of photoelectrons, when a target the metal surface is less than threshold
material of work function 1.24 eV is irradiated (critical) wavelength, then photoelectric emission
takes place.
with light of wavelength 4 .36 × 10 − 7 m is
[18 Sep. 2020, Shift-I] 4. If the wavelength of a photon is 4000 Å,
(a) 4.08 eV (b) 2.84 eV then its energy will be [21 Sep. 2020, Shift-I]
(c) 1.60 eV (d) 0.36 eV (a) 4.95 × 10− 19 J (b) 5.95 × 10− 19 J
Sol. (c) (c) 3.95 × 10− 19 J (d) 6.95 × 10− 19 J
Given, work function, φ0 = 1.24 eV Sol. (a)
= 1.24 × 1.6 × 10−19 J Wavelengh of photon, λ = 4000Å = 4 × 10−7 m
−7
Wavelength, λ = 4.36 × 10 m Energy of photon, E =
hc
According to Einstein’s photoelectric equation, λ
hc 6.62 × 10−34 × 3 × 108
eV0 = − φ0 =
λ 4 × 10−7
−34
6.63 × 10 × 3 × 108
⇒ eV0 = − 1.24 × 1.6 × 10−19 = 4.96 × 10−19 J ~
− 4.95 × 10−19 J
4.36 × 10−7
Dual Nature of Radiation and Matter 223

5. The maximum velocity of an electron Monochromatic light


emitted by light of wavelength λ incident on
the surface of a metal of work function φ is
[h = Planck’s constant, m = mass of electron E C
and c = speed of light] [21 Sep. 2020, Shift-II]
2(hc + λφ) 2(hc − λφ)
(a) (b) 5V S2
mλ m
2(hc − λφ) 2(hλ − φ)
(c) (d) S1 5V
mλ m
(a) 5233.3 Å (b) 4133.3 Å
Sol. (c) (c) 4166.7 Å (d) 5336.7 Å
According to Einstein’s photoelectric equation,
maximum kinetic energy of emitted electron is Sol. (b)
hc 1 2 hc − λφ Let threshold frequency of emitter plate = v0
K max = −φ ⇒ mvmax =
λ 2 λ Energy of photon in first case is E.
2(hc − λφ) 2(hc − λφ) When switch S1 is closed and switch S2 is open,
⇒ vmax =
2
⇒ vmax =
mλ mλ So, E = hν0 + (5 + 1) eV …(i)
For second case, when switch S1 is open and
6. A metal surface is illuminated by a light of switch S2 is closed and frequency of incident
given intensity and frequency to cause light is doubled.
photoemission. If the intensity of then, 2E = hv0 + (20 − 5) eV …(ii)
illumination is reduced to one-fourth of its From Eqs. (i) and (ii), we get
original value, then the maximum kinetic ⇒ 2(hν0 + 6eV) = hν0 + 15eV
energy of the emitted photoelectrons would ⇒ 2hν0 + 12eV = hν0 + 15eV
become [22 Sep. 2020, Shift-I]
⇒ hν0 = 3eV
(a) unchanged 3 × 1.6 × 10−19
(b) half of the original value ⇒ ν0 = = 7.25 × 1014 Hz
6.62 × 10−34
(c) twice of the original value
c 3 × 108
(d) four times of the original value Q λ0 = =
ν0 7.25 × 1014
Sol. (a)
λ 0 = 41333
. Å
In photoelectric emission, maximum kinetic
energy of emitted photoelectrons does not Hence, the threshold wavelength of the emitter
depend upon the intensity of incident radiation. plate is 4133.3 Å.
It depends upon the frequency of incident 8. An electron of charge e and mass m moving
radiation. Hence, when intensity of incident
radiation is reduced to one-fourth, then
with an initial velocity v0$i is subjected to all
maximum kinetic energy of emitted electric field E $j. The de-Broglie wavelength
0
photoelectrons remains unchanged. of the electron at a time t is
7. In a photoelectric experiment, a (Initial de-Broglie wavelength of the electron
monochromatic light is incident on the = λ 0) [20 April 2019, Shift-II]
emitter plate E, as shown in the figure. e 2 E02t 2
When switch S1 is closed and switch S2 is (a) λ 0 (b) λ 0 1 +
m2 v 02
open, the photoelectrons strike the collector
λ0 λ0
plate C with a maximum kinetic energy of (c) (d)
e 2 E02t 2  e 2 E02t 2 
1 eV. If switch S1 is open and switch S2 is 1+ 1 + 
closed and the frequency of the incident m2 v 02  mv 02 
light is doubled the photoelectrons strike the
Sol. (c)
collector plate with a maximum kinetic
Q de-Broglie relation of a charged particles,
energy of 20 eV. The threshold wavelength of
h
the emitter plate is [20 April 2019, Shift-I] λ=
mv
224 AP EAMCET Chapterwise Physics

Velocity of charged particle at time t, 10. An α-particle moves in a circular path of


2

v = v0 $i + t j or |v| = v02 +  0 t 
eE 0 $ eE radius 1 cm in a uniform magnetic field of
m  m  0.125 T. The de-Broglie wavelength
h associated with the α-particle is
Hence, λ = [21 April 2019, Shift-II]
2
m v02 +  0 t 
eE
. × 10−12 m
(a) 165 (b) 3.3 × 10−12 m
 m 
(c) 4.95 × 10−12 m (d) 6.6 × 10−12 m
h λ0
or λ= or λ = Sol. (a)
e 2 E 02 t 2 e 2 E 02 t 2
mv0 1 + 2 2
1+ Given magnetic field , B = 0125 . T,
m v0 m2 v02
Radius of the circular path, R = 1 cm = 10−2 m
 h 
Q λ 0 =  We know radius of circular path in a uniform
 mv 0 magnetic field,
mv
9. The maximum kinetic energy of a R= ⇒ mv = qBR
qB
photoelectron liberated from the surface of
lithium with work function 2.35 eV by h
and de -Broglie wavelength, λ =
electromagnetic radiation whose electric mv
component varies with time as : [Q q = q α , For α- particle]
E = a[1 + cos(2 πf1 t)] cos2 πf 2t (where a is a or λ =
h
constant) is ( f1 = 3.6 × 10 15 Hz, q α BR
and f 2 = 1.2 × 10 15 Hz and Planck’s constant 6.6 × 10−34
λ=
2 × 1.6 × 10−19 × 0125
. × 1 × 10−2
h = 6 .6 × 10 − 34 Js) [21 April 2019, Shift-I]
[Q q α = + 2C = 2 × 1.6 × 10−19 ]
(a) 2.64 eV (b) 7.55 eV −12
(c) 12.52 eV (d) 17.45 eV ⇒ λ = 1.65 × 10 m
Hence, de-Broglie wavelength associated with
Sol. (d) α-particle is1.65 × 10−12 m .
Here, work function, W0 = 2.35 eV and the
electric component of electromagnetic radiation 11. All electrons ejected from a metallic surface
E = a[1 + cos(2πf1 t)] cos(2πf2 t) by incident light of wavelength 400 nm
⇒ E = [a cos(2πf2 t) + a cos(2πf1 t) cos(2πf2 t)] travelled 1 m in the direction of uniform
Q cos A cos B = 1 [cos(A + B) − cos(A − B) electric field of 2 NC−1 and came to rest. The
  work function of the surface is
 2 
a [22 April 2019, Shift-I]
⇒ E = a cos(2πf2 t) + cos 2π( f1 + f2)t
2 (a) 1.1 eV (b) 2.2 eV
a
− cos 2π( f1 − f2)t (c) 3.1 eV (d) 5.1 eV
2 Sol. (a)
So, the electric component has 3 sub-components
Given, wavelength of incident radiation,
with frequencies are,
λ = 400 nm = 4 × 10−7 m
f2 , ( f1 + f2) and ( f1 − f2)
electric field, E = 2 N/C and distance, s =1 m
So, for maximum kinetic energy of
photoelectron, we take photon of maximum ∴Energy of the incident light,
frequency. Hence, hc 6.6 × 10−34 × 3 × 108
E= =
hν λ 4 × 10−7
Emax = max
e 4.95 × 1019
= 4.95 × 10−19 J =
6.6 × 10− 34 × (36 . × 1015 + 1.2 × 1015) 1.6 × 10−19
=
1.6 × 10− 19 J = 3.09 ~− 3.1 eV
= 19.8 eV If a be the retardation of emitted electrons in
Hence, the maximum kinetic energy, the electric field,
KEmax = Emax − W0 = 19.8 − 2. 35 = 17.45 eV then, a=
qE
Hence, the correct option is (d). m
Dual Nature of Radiation and Matter 225

initial speed u of emitted electron is calculated Sol. (a)


as
Given, light of wavelength, λ = 488 nm and
2qE
v2 = u2 − 2as ⇒ 0 = u2 − 2a × 1 ⇒ u2 = 2a = stopping potential, of photoelectrons, V = 0.38 V
m As, energy of photon
∴Maximum kinetic energy of the electron, hc 1250
1 2qE E= = eV
K max = mu2 = m ⋅ = qE = 1.6 × 10−19 × 2 λ 488
2 m So, E = 2.56 eV
K max = 32. × 10−19 J = 2 eV Q Maximum kinetic energy of a photo-electron
∴Work function of the surface, KEmax = eV0
W0 = E − K max = 3.1 − 2 = 1.1 eV Where, V0 = stopping potential
12. All electrons ejected from a metal surface by ∴ KEmax = eV0 = 0.38 eV
the incident light of wavelength 200 nm can From Einstein’s photoelectric equation,
be stopped before travelling 1 m in the E = KEmax + W0
direction of uniform electric field of 4 NC− 1 . ⇒ W0 = E − KEmax
The work function of the metal surface is = 2 .56 − 0.38 = 2 .18 ≈ 2 .16 eV
[22 April 2019, Shift-II] Hence, the work function of cathode material is
2.16 eV.
(a) 2 eV (b) 2.2 eV
(c) 4 eV (d) 6.2 eV So, the correct option is (a).

Sol. (b) 14. The figures shows the variation of photo-


Given, wavelength, λ = 200 nm , current i with anode potential V for three
electric field, E = 4 NC− 1 and distance, d = 1 m differential radiations. Let I a , I b and I c be the
Since, electron is stopped in distance d in the intensities and f a , f b and f c be the frequencies
direction of electric field E, then the stopping for the curves a , b and c respectively. Then
potential is given as, [22 April 2018, Shift-II]
Vs = E ⋅ d = 4 × 1 = 4V i
Energy of photon,
c
1250
E= eV b
λnm
1250 a
⇒ E= = 6.2 eV V
200
Einstein’s photoelectric equation, (a) fa = fb and Ia ≠ Ib (b) fa = fc and Ia = Ic
(c) fa = fb and Ia = Ib (d) fb = fc and Ib = Ic
E = W0 + KE
⇒ W0 = E − eV Sol. (a)
Stopping potentials for b and a are same
because KEmax = stopping potential
∴ fa = fb
⇒ W0 = 6.2 − 4 = 2. 2 eV
Hence, the work function of metal surface is and saturation current for b and a are different
2.2 eV. ∴ Ib ≠ Ia

13. Light of wavelength 488 nm produced by an 15. An α-particle and a proton are accelerated
Agron laser is used in the photoelectric from rest by the same potential, then the
effect. When light from this spectral line is ratio of their de-Broglie wavelength is
incident on the cathode, the stopping [23 April 2018, Shift-I]
potential of the photoelectrons is 0.38 V. The (a) 2 2 : 1 (b) 1 : 2 2 (c) 1 : 2 (d) 2 : 1
work function of the cathode material is Sol. (b)
[23 April 2019, Shift-I] de-Broglie’s wavelength,
(a) 2.16 eV (b) 216 eV h h h
(c) 21.6 eV (d) 0.216 eV λ= = =
p 2mE 2meV
226 AP EAMCET Chapterwise Physics

The particles are at same potential, so 17. When light of frequency ν incidents on two
λα h / 2mα e α V metallic plates A and B, photo electrons are
=
λp h / 2mp e p V emitted. If the work function of A is more
Mass of α-particle = 4 times mass of proton. than that of B, the correct curve of the
Charge of α-particle = 2 times charge of proton. following curves drawn between stopping
λα mp e p 1
potential V and incident frequency ν is
So, = = [24 April 2018, Shift-I]
λp 4mp 2e p 2 2
V V
16. Photons of wavelength λ emitted by a source B
A B
of power P incident on a photo cell. If the A
(a) (b)
current produced in the cell is I, then the
percentage of incident photons which
produce current in the photo cell is (where,
ν ν
h is Planck’s constant and c is the speed of
light in vacuum) [23 April 2018, Shift-II] V V
100ePc 100ePλ B
(a) (b) B A
Ihλ Ihc
(c) (d)
100Ihλ 100Ihc A
(c) (d)
ePc ePλ
Sol. (d) ν ν
In photoemission, only one electron can
produce one photon. Sol. (d)
So, if only x % of incident photons can produce Work function is proportional to y-intercept on
electrons, then V versus ν graph. In case (d), line of A cuts larger
intercept.
n 100 × I × hc
I= =
t ePλ
25
Atoms
1. Frank and Hertz experiment proves that Similarly, for n = 3,
−136 . 2
[17 Sep. 2020, Shift-I] E3 = z …(ii)
(a) light moves in the form of waves as well as 9
particles Given, ∆E = E 3 − E 2 = 47.2
(b) the electron does not radiate energy, while −136 . 2  −136 . z2 
⇒ z −  = 47.2
moving in an orbit 9  4 
(c) the energy states of an atom are quantised 5 2
(d) the whole of the positive charge of the atom is ⇒ 136 . × z = 47.2
36
concentrated in the nucleus
⇒ z 2 = 24.98
Sol. (c) ⇒ z2 ~− 25 ⇒ z = 5
Frank an Hertz’s experiment demonstrated the
existence of excited states in mercury atoms. It 3. The second line of Balmer series has
confirms the prediction of quantum theory that wavelength 4861 Å. The wavelength of the
electrons occupy only discrete, quantised energy first line of Balmer series is
states. [18 Sep. 2020, Shift-II]
2. A hydrogen like atom has one electron (a) 1216 Å (b) 6563 Å
revolving round a stationary nucleus. If the (c) 4340 Å (d) 4101 Å
energy required to excite the electron from Sol. (b)
the 2nd orbital to 3rd orbit is 47.2 eV, find Wavelength of the second line of Balmer series,
the atomic number of the given atom. λ 2 = 4861 Å
1
= R 2 − 2 
[18 Sep. 2020, Shift-I] 1 1
λ2 2 n 
(a) 3 (b) 4 (c) 5 (d) 6
Sol. (c) For second line, n = 4
= R 2 − 2  ]
1 1 1
The energy of nth orbit is given as ∴
λ2 2 4 
− RhCz 2
En =
= R − 
n2 1 1 1
4861  4 16 
For hydrogen atom, z = 1 and n = 1, hence
− RhC ⋅12 1 3R
E1 = = − RhC ⇒ =
12 4861 16
16
We know that, RhC = 136 . eV ⇒ R= …(i)
3 × 4861
∴ E1 = − 136. eV
Wavelength of first line (n = 3) of Balmer series
For n = 2, is given as
− RhCz 2 −136 . z2
= R 2 − 2  = R 
E2 = = 1 1 1 5
2
2 4 λ1 2 3   36 
−136. z2 16 5
⇒ E2 = …(i) = × [From Eq. (i)]
4 3 × 4861 36
228 AP EAMCET Chapterwise Physics

3 × 4861 × 36 C
⇒ λ1 = = 6562. 35 Å
16 × 5 λ1
− 6563Å
~
B
4. In Bohr’s theory the potential of an electron λ2 λ3
Kr 2
at a position is , where K is a constant. A
2
λ1 λ 2
Then, the quantised energy of the electron in (a) λ 3 = λ1 + λ 2 (b) λ 3 =
nth orbit is [22 Sep. 2020, Shift-I] λ1 + λ 2
nhK nh K (c) λ1 + λ 2 + λ 3 = 0 (d) λ23 = λ21 + λ22
(a) (b)
2 πm 2π m Sol. (b)
nhm nh m The given situation is shown in the following
(c) (d) diagram,
2 πK 2π K
Sol. (b) EC C
Given, potential energy of electron, λ1
K r2
U= EB B
2
λ2 λ3
dU d Kr 2
∴ F= = = Kr
dr dr 2 EA A
mv2 mv2 Energy levels are related as
But F= ⇒ Kr =
r r E A < EB < EC
K According to energy level diagram,
⇒ v=r …(i)
m E C − E A = (E C − E B) + (E B − E A)
According to Bohr’s postulates, hc hc hc
⇒ = +
nh nh λ 3 λ1 λ2
L = mvr = ⇒ mr ⋅ v = 1 1 1
2π 2π ⇒ = +
λ 3 λ1 λ2
K nh
⇒ mr ⋅ r = [from Eq. (i)] 1 λ + λ1
m 2π ⇒ = 2
λ3 λ1 λ 2
nh
⇒ r2 Km = λ λ
2π ⇒ λ3 = 1 2
λ1 + λ 2
nh
⇒ r2 = …(ii)
2 π Km 6. WK Roentgen discovered [23 Sep. 2020, Shift-I]
We know that, energy of electron in nth orbit, (a) short radio waves
E n = Kr 2 (b) X-rays
nh (c) electrons
= K⋅ [from Eq. (ii)]
2π K m (d) laws of electromagnetic induction

nh K Sol. (b)
= X- rays were discovered by scientist W K
2π m
Roentgen in November 8, 1895.
5. Energy levels A , B, C of a certain atom
7. Minimum excitation potential of Bohr’s first
correspond to increasing values of energy orbit of H-atom is [23 Sep. 2020, Shift-I]
that is, E A < E B < E C . If λ 1 , λ 2 , λ 3 are the
(a) 3.6 V (b) 10.2 V
wavelengths of radiations corresponding to
(c) 13.6 V (d) 3.4 V
the transitions C to B, B to A and C to A
respectively, as shown in figure. Which of Sol. (b)
the following statement is correct? As we know, excitation potential
excitation energy
[22 Sep. 2020, Shift-II] =
e
Atoms 229

Minimum excitation energy will be difference 9. Match the following List-I with List-II in
between ground level energy (n = 1) and first connection with Bohr’s atomic model.
excited state (n = 2).
[20 April 2019, Shift-II]
For hydrogen atom, energy in ground state,
E1 = −136 . eV List I List II
Energy in first excited state,
(A) Speed of revolution (i) 1 2 π Ze 2
13. 6 13. 6
E2 = − 2 = − = −34. eV of electron 4 π ∈0 nh
2 4
Energy required for excitation, ∆E = E 2 − E1 (B) Kinetic energy (ii)
2
 1  2 π 2 me 4 Z 2
− 
= −34. − (−136
.)  4 π ∈0  n2 h2
= 10.2 eV
(C) Total energy (iii)  1  2 π 2 me 4 Z 2
2
∆E 10.2
∴ Excitation potential, = = eV = 10.2 V  
e e  4 π ∈0  n2 h2

8. In a system, a particle A of mass m and (D) Frequency (iv)  1 


2
4 π 2 Z 2e 4 m
charge −2q is moving in the nearest orbit  
 4 π ∈0  n3 h3
around a very heavy particle B having charge
+q. Assuming Bohr’s model of the atom to be The correct answer is
applicable to this system, the orbital angular
A B C D A B C D
velocity of the particle A is
(a) i iii ii iv (b) ii iv iii i
[20 April 2019, Shift-I]
(c) iii i iv ii (d) iii i ii iv
2 π m2q 2 3 π m3q 2
(a) (b) 3 2 Sol. (a)
ε0 h4 ε0 h
1 2 π Ze 2
2 π mq 4 5 π m2q 3 (A) Speed of revolution of electron =
(c) 2 3 (d) 4 πε0 nh
ε0 h ε30 h2 2
 1  2π 2 me 4 Z 2
Sol. (c) (B) Kinetic energy =  
 4 πε0  n2 h2
In a system, a particle A moving around a particle B 2
in a circular path then applied the centripetal  1  2 π 2 me 4 Z 2
(C) Total Energy = −  
force directed and center of the circular path.  4 πε0  n2 h2
So, the electric force, Fe = centripetal force, Fc 2
 1  4 π2 Z2 e 4 m
kq1 q 2
= mrω2 ...(i) (D) Frequency =  
r2  4 πε0  n3 h3
1 Hence, A → i; B → iii; C → ii; D → iv.
Given, q1 = 2q, q 2 = q and k =
4 πε0
Putting these values in Eq. (i), we get
10. Magnetic moment due to the motion of the
1 q (2q) electron in nth energy state of hydrogen atom
= mrω2 …(ii) is proportional to ……… [21 April 2019, Shift-I]
4 πε0 r 2
(a) n− 2 (b) n
According to the Bohr’s model, (c) n2 (d) n3
nh
mvr = Sol. (b)

Magnetic moment of a moving electron, which
…(iii) Q ω = 
nh v
⇒ mr 2ω = shown in the figure below,
2π  r 
From Eqs. (ii) and (iii), we get
2πmq 4
ω= 2 3 [Q n=1]
ε0 h e–
R
Hence, the orbital angular velocity of the
2π mq 4
particle A is 2 3 .
ε0 h
230 AP EAMCET Chapterwise Physics

e eω Q ω = 2π  Sol. (b)
Current, i = =  
T 2π  T  h
We know that momentum of a photon, p =
eω 2 λ
As, magnetic moment, M = iA = πR h h
2π ⇒ mv = ⇒v = ..(i) [Q p = mv ]
ev 2 Q ω = v  λ mλ
⇒ M= R  
2R  R Given, hydrogen atom emits a photon during a
evR eL transition from n = 4 to n = 2 ,
⇒ M= = According to Bohr’s formula for H-atom, the
2 2m
wavelength of the emitted photon is given by
nh
where, L = mvr and L = 1 1 1
2π ∴ = R 2 − 2 
enh λ  n1 n2 
So, M= ⇒ M ∝n
2πm where, R = Rydberg’s constant = 1.097 × 107 m−1
= 1.097 × 107  2 − 2 
Q e , h, m are constant of electron, 1 1 1

Hence, the correct option is (b). λ 2 4 
⇒ λ = 4.86 × 10−7 m
11. In a hydrogen atom, an electron of mass
Q Mass of a proton = 1.6 × 10−27 kg
9 .1 × 10 −31 kg revolves about a proton in
Now, from Eq. (i), we get
circular orbit of radius 0.53 Å. The radial
Hence, recoil speed of photon,
acceleration and angular velocity of electron
6.62 × 10−34
are respectively, [21 April 2019, Shift-II] v= = 0.8513 m/s
1.6 × 10−27 × 4.86 × 10−7
(a) 9 × 1022 ms −2 , 41
. × 1016 s −1
. × 1016 ms −2 , 9 × 1022 s −1
(b) 41 or v ≈ 0.814 ms−1
(c) 9 × 1016 ms −2 , 41
. × 1022 s −1
13. Speed of electron in its 1st Bohr’s orbit is given
. × 10 ms , 9 × 1016 s −1
(d) 41 22 −2
by 2.18 × 10 6 ms −1 . If the time period of electron
Sol. (a) in n th orbit is measured as 4.10 femto second,
Given, mass of electron, me = 9.1 × 10−31 kg and the value of n is [22 April 2019, Shift-I]
radius of circular orbit, r = 0.53 × 10−10 m
(a) 1 (b) 2 (c) 3 (d) 4
Q An electron revolves about a proton in circular
orbit. So, centripetel force = electrostatic force Sol. (c)
me v2 kq1 q 2 Given, speed of the electron in 1st Bohr’s orbit,
=
r r2 v1 = 2.18 × 106 m/s
me v r = kq1 q 2
2 Time period of nth orbit,
Q v = rω . f-s = 41
Tn = 410 . × 10−15 s
∴ me r(rω)2 = kq1 q 2 Radius of Bohr’s first orbit, r1 = 0.53 × 10−10 m
∴Orbital period of electron in Bohr’s first orbit,
kq 2 kq 2
⇒ ω2 = ⇒ω = . × 0.53 × 10−10
2πr1 2 × 314
me r 3 me r 3 T1 = = =1.52 × 10−16 s
ν1 . × 106
218
∴angular velocity of electron,
Again, Time period of n th orbit is given by
9 × 109 × (1.6 × 10−19)2 . × 10−15
ω= T
Tn = n3 T1 ⇒ n3 = n =
41
= 26.98
. × 10−31 × (0.53 × 10−10)3
91 T1 1.52 × 10−16
ω = 41. × 1016 s−1 n = 27 ⇒ n = (27) ⇒ n = 3
3 1 /3

Hence, the radial acceleration of electron


= mrω2 14. A hydrogen atom emits a photon of
36
= 0.53 × 10−10 × (41 . × 1016)2 = 9 × 1022 ms−2 wavelength when it is jumped from its
35R
12. When a hydrogen atom emits a photon nth excited state to the ground state. Then
during a transition from n = 4 to n = 2, its the quantum number n is
recoil speed is about, [21 April 2019, Shift-II]
(R is Rydberg constant.) [22 April 2019, Shift-II]
(a) 4.28 ms −1 (b) 0.814 ms −1
(a) 8 (b) 7 (c) 5 (d) 6
(c) 2.07 ms −1 (d) 0.407 ms −1
Atoms 231

Sol. (d) 16. Photons of frequencies equal to the


Key Idea When a electron jump from nth state to frequencies of H β and H ∞ lines of hydrogen
ground state, then the lyman-series of hydrogen incident on a photosensitive plate, whose
spectrum will appear. threshold frequency is equal to the
36 frequency of H α line of hydrogen. The ratio
Given wavelength, λ nth =
35R of the maximum kinetic energies of the
As wavelength in lyman-series, emitted electrons is [22 April 2018, Shift-I]
(a) 7 : 16 (b) 3 : 4
= R1 − 2 
1 1
(Q n = 2, 3, 4) (c) 8 : 27 (d) 5 : 36
λ Ly  n 
n2 Sol. (a)
⇒ λ Ly = Energy of photons of
R(n2 − 1)
H α line = ∆E(3 → 2)
Where, λ Ly is the wavelength of the photon,
. ×5
.  −  =
1 1 136
when a electron jumps from nth state to ground = 136 = 1.89 eV
state.  4 9 36
Hence, λ Ly = λ nth Energy of photons of H β line = ∆E(4 → 2)
12 × 136
.  −  =
n2 36 1 1 .
⇒ = ⇒ n2 = 36 ⇒ n = 6th state. = 136 = 2. 55 eV
R(n2 − 1) 35 R  4 16  64
Energy of photons of H ∞ line = ∆E(∞ → 2)
15. If the first excitation potential of a .
136
= = 34
. eV
hypothetical hydrogen like atom is 15 V, 4
then the third excitation potential of the Ratio of kinetic energy of emitted photons
atom is [23 April 2019, Shift-1]
4 . − 1.89 0.66 0.7 7
255
(a) 13.6 V (b) V = = ≈ ≈
75 34. − 1.89 1.51 1.6 16
15 75
(c) V (d) V
16 4 17. Hydrogen atom is in its nth energy state. If
de-Broglie wavelength of the electron is λ,
Sol. (d)
then [22 April 2018, Shift-I]
Let total energy of hydrogen like atom at ground 1 1
state = − E (a) λ ∝ (b) λ ∝ (c) λ ∝ n2 (d) λ ∝ n
n2 n
So, 1st excitation
Energy = E 2 nd − E1 st Sol. (d)
−E Angular momentum of electron in nth orbit of
15eV =  2  −  2 
E
 2  1  hydrogen is
nh
(Q First excitation potential is given) L = mvr =

3E
⇒ = 15eV h mv 1
4 de-Broglie wavelength, λ = ⇒ =
mv h λ
15 × 4
⇒ E= eV = 20 eV r n 2πr
3 ⇒ = ⇒λ =
λ 2π n
Now, the third excitation energy
Now, r ∝ n2 (Bohr radius, r = 0.529 × n2 Å)
E 3 = E 4 th − E1 st
⇒ λ∝n
−E
=  2  −  2 
E
 4  1  18. A stationary hydrogen atom undergoes a
15 E transition from n = 5 to n = 4. Recoil speed of
E3 = + the atom is (R = Rydberg constant, h =
16
Planck’s constant and m = mass of the
Now, putting the value of E, we get
proton).
15 75
E3 = × 20 = eV [22 April 2018, Shift-II]
16 4
Rh 9m 9Rh 7 Rh
Hence, the correct option is (d). (a) (b) (c) (d)
m 400 Rh 400 m 400
232 AP EAMCET Chapterwise Physics

Sol. (c) state and ground state respectively, then the


value of n is [23 April 2018, Shift-II]
For transition 5 → 4,
2(λ 2 − λ1 ) 2 λ 2 − λ1
= hR − 
h 1 1 (a) (b)
Momentum of photon, p =
λ  16 25 2 λ 2 − λ1 2(λ 2 − λ1 )

hR  − 
1 1 4λ 2 − λ1 4(λ 2 − λ1 )
p  16 25 9hR (c) (d)
and recoil speed, v = = = 4(λ 2 − λ1 ) (4λ 2 − λ1 )
m m 400 m
Sol. (d)
19. The difference between the radii of nth and n
(n + 1)th orbits of hydrogen atom is equal to
the radius of (n − 1)th orbit of hydrogen. The λ1

angular momentum of the electron in the nth 2


orbit is ______ (h is Planck’s constant). λ2
[23 April 2018, Shift-I] 1
h 2h 3h 4h
(a) (b) (c) (d) We have,
π π π π hc hc
λ1 = and λ 2 =
Sol. (b) ∆E n→ 2 ∆E n→ 1
Radius of nth orbit in an atom,
λ1  − 2  = λ 2  − 2 
136
. 136
. 136
. 136
.
n2 h2 ⇒
rn =  4 n   1 n 
4π 2 mZe 2
4(λ 2 − λ1 )
So, rn ∝ n2 ⇒ n=
4λ 2 − λ1
Given in question, difference between radii of
(n + 1)th and nth orbit = radius of nth orbit. 21. The approximate value of principal quantum
⇒ (n + 1)2 − n2 = (n − 1)2 number for a circular orbit of hydrogen atom
n2 − 4n = 0 or n = 4 of radius 530 nm is [24 April 2018, Shift-I]
According to Bohr’s postulate, angular (a) 26 (b) 100 (c) 200 (d) 21
h Sol. (b)
momentum is an integral multiple of .
2π Radius of electron of hydrogen in nth orbit,
So, for 4th orbit, rn = n2 × 0. 53 Å
nh 4h 2h 530 nm = n2 × 0. 053 Å
L= = =
2π 2π π ∴ 530 × 10−9 m = n2 × 0. 53 × 10−10 m
5300
20. If λ 1 and λ 2 are the wavelength of the ⇒ n2 =
0. 53
photons emitted, when electrons in the n th
orbit of hydrogen atom fall to first excited ⇒ n2 = 104
n = 100
26
Nuclei
6 C→ 5 B + β + X ,X
1. In the nuclear reaction, 11 11
4. In the Uranium radioactive series, the initial
stands for [17 Sep. 2020, Shift-I] nucleus is 238 206
92 U and final nucleus is 82 Pb.

(a) a neutron (b) an electron When the Uranium nucleus decays to lead,
(c) a neutrino (d) an anti-neutrino the number of α-particles emitted is ..........
and the number of β-particles emitted is …… .
Sol. (c)
[18 Sep. 2020, Shift-II]
Nuclear reaction is given as
(a) 6, 8 (b) 8, 6 (c) 16, 6 (d) 32, 2
6 C→ 5 B + β + X
11 11
…(i)
When positron (β) is emitted from a nuclei, then Sol. (b)
atomic number decreases by one unit, while the According to question,
mass number remains the same. Initial nuclei = 238
92 U
Hence, Eq. (i) is written as Final nuclei = 206
82 Pb

6 C → 5 B + 1 β + ν (neutrino)
11 11 0
Change in atomic number = 92 − 82 = 10
Hence, X is a neutrino ν. When an α- particle emits from a nuclei, then
its atomic number is decreased by 2 units and
2. Weak nuclear force always operates between atomic mass is decreased by 4 units. When
[17 Sep. 2020, Shift-I] aβ-particle emits from a nuclei, then atomic
(a) electrons and neutrino number of parent nuclei is increased by 1 unit
whereas its atomic mass remains same.
(b) heavier elementary particles
(c) charged particles Change in atomic mass = 238 − 206 = 32
(d) all the objects in the universe Since, atomic mass changes only due to
emission α-particle, hence number of emitted
Sol. (b) α-particle
Weak nuclear force always operates between 32
= =8
heaviour elementary particles. 4
Number of emitted β-particles = total change in
3. A radioactive sample has an activity A in air. atomic number due to emission of α-particles −
If the sample is kept inside water, then its total change in atomic number
activity A′ [17 Sep. 2020, Shift-II] = 8 × 2 − 10 = 6
(a) becomes less than A
(b) becomes more than A 5. The half-life of a radioactive sample is T. The
(c) is same as A fraction of the initial mass of the sample
(d) will be less than or equal to A depending on the that decays in an interval T/2 is
density of water [21 Sep. 2020, Shift-II]
Sol. (c) 1 ( 2 − 1) ( 2 + 1)
(a) (b) 2 (c) (d)
Activity of a radioactive sample does not depend 2 2 2
on the type of medium in which it is kept. Hence, Sol. (a)
if the radioactive sample is kept inside water, Fraction remains after n half-lives is given as
then its activity (A′) remains same as in air. n t/T
=   =  
N 1 1
i.e., A′ = A [where, T is half-life]
N 0  2  2
234 AP EAMCET Chapterwise Physics
t
Let say lead has mass number A and atomic
=  
N 1 T
⇒ number Z after emission of 6α and 4β-particles.
N 0  2 Nuclear reaction for given situation is given as
T
90 Th → Z Pb + 62 He + 41 β
232 A 4 0
Given, t=
2 Since, atomic number of both sides are equal hence
T/2 1
90 = Z + 6 × 2 + 4 × (−1)
=   =   =
N 1 T 1 2 1

N 0  2  2 2 90 = Z + 12 − 4
or Z = 82
6. The binding energy of a nucleus is equivalent Similarly, comparing the mass number of both
to [22 Sep. 2020, Shift-I] sides, 232 = A + 6 × 4 + 4 × 0
(a) the mass of nucleus 232 = A + 24
(b) the mass of proton A = 208
(c) the mass of neutron
(d) the mass defect of nucleus 9. In a nuclear reactor the activity of a
radioactive substance is 2000/s. If the mean
Sol. (d)
life of the products is 50 minutes, then in the
Binding energy is the energy associated with
steady power generation, the number of
the strong force that holds the nucleons
together in the nucleus.
radio nuclides is [20 April 2019, Shift-I]

The mass of a nucleus is less than the mass of (a) 12 × 105 (b) 60 × 105
the individual nucleons that make up that (c) 90 × 105 (d) 15 × 105
nucleus. The mass difference (mass defect ∆m) Sol. (b)
between the two is equivalent to the binding Given,
energy of the nucleus.
nuclear reactor the activity of a radioactive
7. If 75% of a radioactive sample disintegrates substance,
dN
in 16 days, the half-life of the radioactive = 2000/s
sample is ……… days. [22 Sep. 2020, Shift-II] dt
(a) 6 (b) 4 (c) 8 (d) 12 and mean-life of the products,
τ = 50 min = 50 × 60 sec
Sol. (c)
Now, the mean-life of the radioactive substance
If N 0 be the initial amount of radioactive
sample, then after t = 16 day, remaining amount, is inversely proportional to disintegration
N = N 0 − 75% of N 0 constant λ i.e.,
75 N 1 1 1
= N0 − N0 = 0 τ = ⇒λ = = per second
100 4 λ τ 50 × 60
Since, we know that, ∴ The rate of decay is proportional to the
n n
number of radio nuclides is given as
N = N 0   = N 0  
1 N0 1

 2 4  2 dN
∝N
n 2 n dt
1 1 
⇒   =  
1 1
⇒ =  dN 1
4  2  2  2 = λN ⇒ 2000 = ×N
t 16 dt 50 × 60
⇒ n= 2 ⇒ =2 ⇒ =2
T1 / 2 T1 / 2 Where, λ is a disintegration constant.
16 Putting the given values, we get
⇒ T1 / 2 = = 8 days N = 2000 × 50 × 60 ⇒ N = 60 × 105
2
Hence, the number of nuclides is 60 × 105 .
8. 90Th emits 6 α and 4 β-particles and gets
232

10. Half-life of a radioactive substance is


converted into a lead. The mass number and
18 minutes. The time interval between its
atomic number of lead is [23 Sep. 2020, Shift-I]
20% decay and 80% decay in minutes is
(a) 208, 82 (b) 82, 208 (c) 210, 82 (d) 210, 84 [20 April 2019, Shift-II]
Sol. (a) (a) 6 (b) 9 (c) 18 (d) 36
Nuclei 235
R
Sol. (d) So, R = − λN ⇒ = − λ = constant
After n half-life, the numbers of atom left N
undecayed is given by, where, λ = decay constant
n R
So, the graph versus t is as,
N = N 0  
1
N
 2
t
y
Q n=
t  1  T1 / 2
or N = N 0  
T1  2
2 R =λ
N
For 20% decay of radioactive substance,
20
N = N0 − N 0 = 0.8 N 0
100
t
or  1  1 / 18
0.8 N 0 = N 0   …(i) t
x
 2 '

For 80% decay of radioactive substance, So, the correct graph shown in option (d).
t
N 0 = 0.2 N 0 or 0.2 N 0 = N 0  
80 1 2 / 18 12. If the binding energy of N14 is 7.5 MeV per
N = N0 −
100
…(ii)  2 nucleons and that of N15 is 7.7 MeV per
t1 − t2 nucleon, then the energy is required to
From Eqs. (i) and (ii), we get ⇒ 4 =  
1 18
remove a neutron from N15 is
 2
[21 April 2019, Shift-II]
Taking log on the both sides, we get (a) 5.25 MeV (b) 0.2 MeV
t1 − t2
 t2 − t1 
  (c) 10.5 MeV (d) 0.4 MeV
log 4 = log  
1 18
⇒ log 22 = log 2 18 
 2 Sol. (c)
t − t1 Given, binding energy per nucleon of
⇒ 2= 2 ⇒ t2 − t1 = 36 min
18 N14 = 7.5 MeV/nucleon
Binding energy per nucleon of
11. The rate of disintegration of a radioactive N15 = 7.7 MeV/nucleon
sample is R and the number of atoms present
∴ Total BE of N14 = 7.5 × 14 MeV
R
at any time t is N. When is taken along Total BE of N15 = 7.7 × 15 MeV
N
Then, the energy required to remove a neutron
Y -axis and t is taken along X -axis, the from N15 ,
correct graphs is [21 April 2019, Shift-I]
= BEtotal N15 − BEtotalN14
Y Y = (7.7 × 15 − 7.5 × 14) MeV = 10.5 MeV

13. A radioactive substance of half life 138.6


(a) R (b) R
N N days is placed in a box. After n days only
20% of the substance is present then the
t
X
t
X value of n is [ln (5) =161
. ] [22 April 2019, Shift-I]
Y Y (a) 693 (b) 138.6
(c) 277.2 (d) 322
(c) R (d) R
N N Sol. (d)
Half life of a radioactive substance,
t
X
t
X n1 / 2 = 138.6 days
If N 0 be the initial amount of radioactive
Sol. (d) substance, then remaining amount after n days is
given by
Law of radioactive decay is given as,
20 N
dN N = 20% of N 0 = × N0 = 0
= − λN 100 5
dt
dN By radioative decay’s law,
where, = rate of disintegration
dt
236 AP EAMCET Chapterwise Physics
n n
Sol. (b)
N = N 0   1 / 2 ⇒ 0 = N 0  
1 n N 1 138. 6
 2 5  2 Given, energy released, E 0 = 188 MeV per nucleus
n fission and mass, m = 100 gm
1 1  138. 6
As number of nucleus in 100 gm of 92 U235
⇒ = 
5  2 100
N= × NA
Taking log on the both sides, we get M
n
where, N A = Avogadro number and
ln = ln  ln 
1 1 138. 6 1 n 1
⇒ ln = M = molecular mass
5  2 5 138.6  2
100
n ⇒ N= × 6.022 × 1023
ln 5 = ln 2 235
138.6
= 2.56 × 1023 nucleus
ln 5 Q ln 5 = 1. 61
⇒ n = 138.6 ×  ln 2 = 0.693 Energy released in fission of one nucleus
ln 2  
E 0′ = E 0 = 188 × 106 ×1.6 × 10−19 J
1.61
n = 138.6 × ⇒ n = 322 days = 300.8 × 10−13 J
0.693
So, Energy released in N-nucleus fission,
14. Assertion (A) Fragments produced in the E = E 0′ N = 2⋅ 561023 × 300.8 × 10−13 J
fission of 92U 235 are radioactive. ⇒ E = 7.71 × 1012 J
Hence, the correct option is (b).
Reason (R) The fragments in the fission of
235
U have a proton to neutron ratio of 2.5. 16. If 200 MeV of energy is released in the
235
[22 April 2019, Shift-II] fission of one nucleus of 92 U, then the
(a) Both (A) and (R) are correct and (R) is the correct number of nuclei that must undergo fission
explanation of (A).
to release an energy of 1000 J is
(b) Both (A) and (R) are correct but (R) is not the
[22 April 2018, Shift-I]
correct explanation of (A).
(c) (A) is correct but (R) is not correct. (a) 3125
. × 1013 (b) 6.25 × 1013
(c) 12.5 × 1013 (d) 3125
. × 1014
(d) (A) is not correct but (R) is correct.
Sol. (c) Sol. (a)
E total 1000J
Key Idea Number of nuclei = =
Nuclear fission of 92 U235 is expressed as E1 200MeV
235
+ 0 n1 → 56Ba141 + 36Kr 92 + 3n1 + Q 1000 J
92 U =
As, fission by product of 92 U235 are 56 Ba141 and 200 × 106 × 1.6 × 10− 19 J
52
36 Kr , which both are slightly radioactive. 1014
⇒ N=
32.
The ratio of proton to neutron of Ba141 .
56
⇒ N = 3125
. × 1013
56
X Ba = = 0.65
(141 − 56) 17. The half life of U against α-decay is
238
92

Similarly, the ratio for 36 Kr .92


13.86 × 10 s. The activity of 1 g sample of
16

36 36 238
X Kr = = = 0.64 92 U is [22 April 2018, Shift-II]
92 − 36 56
(a) 1.26 × 104 s −1 (b) 1.26 × 10−4 s −1
Hence, the assertion (A) is correct but reason (c) 12 .6 × 104 s −1 (d) 12 .6 × 10−4 s −1
(R) is not correct.
Sol. (a)
15. The energy released when one nucleus of Activity of a sample is
U 235 undergoes fission is 188 MeV. The dN 0. 6931
92
R= = | − λ N |= λN = ×N
energy released when 100 g of 92U 235 dt T1 / 2
undergoes fission is [23 April 2019, Shift-I]
=
0. 6931
× n × NA =
0. 6931 m
× × NA
(a) 3.55 × 1012 J (b) 7.71 × 1012 J T1 / 2 T1 / 2 M
(c) 3.55 × 1013 J (d) 7.71 × 1013 J With values, we get R = 1. 26 × 104 s− 1
Nuclei 237
N 0 6.25
18. The maximum potential energy due to Here, N = (100 − 9375
. )× = × N0
electrostatic repulsion between two 100 100
n 4 4
hydrogen nuclei is nearly (radius of the ∴  1  = 625 =  5  =  1 
     
nucleus  2 10000  10   2
 1  or n = 4.
= 11
. fermi)  = 9 × 10 9 Nm 2C− 2
 4 πε 0  20. A radioactive element X converts into
[23 April 2018, Shift-I] another stable element Y . Half life of X is
(a) 0.65 MeV (b) 2.09 MeV 2 hours. Initially only X is present. After a
(c) 3.31 MeV (d) 0.92 MeV time t, if the ratio of atoms of X to Y is 1 : 4 ,
Sol. (a) then the value of t is [24 April 2018, Shift-I]
kq1 q 2 (a) 2 hours
Potential energy due to two charges = . (b) 4 hours
d
(c) between 4 hours and 6 hours
For hydrogen atom, q1 = q 2 = 1. 6 × 10− 19 C
(d) 6 hours
9 × 109 × (1.6 × 10− 19)2
U= [∴ d = 2ε] Sol. (c)
2.2 × 10− 15
2hours
10.47 × 10− 4 X → Y Initially at t = 0, amount of X in
= 10.47 × 10− 14 J = eV
1.6 × 10− 19 sample = 5X
= 6.5 × 105 eV = 0.65 MeV After time ‘t’ amount of X remained in sample
=X
19. The number of half lives elapsed before N 1  T
t

93.75% of a radioactive sample has decayed From, = 


N 0  2
is [23 April 2018, Shift-II]
t t
X 1  2
=   ⇒   =
(a) 6 (b) 4 (c) 2 (d) 8 1 2 1
We have,
Sol. (b) 5X  2  2 5
Number of half lives be obtained by using ∴T = 2 h
n 1 1 1 1 1 1
N = N 0  
1 As, < < ⇒ 2 < t/2 < 3
 2 22 5 23 2 2 2
2< t / 2< 3 ⇒ 4< t < 6
27
Semiconductor Electronics
Materials, Devices and
Simple Circuits
1. In a p-n-p transistor working as a common 3. Assuming that the junction diode is ideal,
base amplifier, when the current gain is 0.96 the current in the arrangement shown in the
and emitter current is 7.2 mA, the base circuit diagram is [18 Sep. 2020, Shift-II]
current is [17 Sep. 2020, Shift-II]
100Ω
(a) 0.4 mA (b) 0.2 mA 3V 1V
(c) 0.29 mA (d) 0.35 mA
Sol. (c) (a) 2 mA (b) 20 mA
In common base amplifier for p-n-p transistor, (c) 30 mA (d) 10 mA
current gain, α = 0.96 Sol. (b)
Emitter current, I E = 7.2 mA The circuit diagram is given as
We know that,
I 100Ω
α= C 3V 1V
IE
I
⇒ 0.96 = C Since, ideal diode is represented as short circuit,
7.2 hence circuit diagram is represented as,
⇒ I C = 0.96 × 7.2 = 6.912 mA
100Ω
∴Base current, 3V 1V
I B = I E − I C = 7.2 − 6.912 A I B
= 0.288 mA ~ − 0.29 mA
V A − VB 3 − 1
∴ Current, I = =
2. In a photodiode, the value of the emf R 100
produced by a monochromatic light beam is 2
= = 0.02 A = 20 mA
directly proportional to [18 Sep. 2020, Shift-I] 100
(a) the barrier potential at p-n junction
4. A Zener diode is made by [21 Sep. 2020, Shift-I]
(b) the intensity of light falling on the photodiode
(a) heavily doping both p - and n-sides of a p-n
(c) the frequency of light falling on the photodiode junction diode
(d) the voltage applied at the p-n junction (b) heavily doping the p-side and lightly doping the
Sol. (b) n-side of a p-n junction diode
In a photodiode, the value of the emf produced (c) lightly doping the p-side and heavily doping the
by monochromatic light beam is directly n-side of a p-n junction diode
proportional to the intensity of light falling on (d) lightly doping both p - and n-sides of a p-n
the photodiode. junction diode
Semiconductor Electronics Materials, Devices and Simple Circuits 239

Sol. (a) (Planck’s constant, h = 6 .6 × 10 −34 Js and


A Zener diode is made by heavily doping both p speed of light in vacuum, c = 3 × 10 8 ms −1 )
and n-sides of the junction. Because of heavily
doping, a very thin (∠10−6 m) depletion region is [20 April 2019, Shift-I]
formed between the p and n-sides, and hence (a) 0.0248 µm (b) 0248
. µm
the electric field of the junction is extremely (c) 2.48 µm (d) 24.8 µm
high even for a small reverse bias voltage.
Sol. (d)
5. In the following circuit, the equivalent Given,
resistance between A and B is p-type semiconductor donor energy level,
[23 Sep. 2020, Shift-I] E = 50 meV = 50 × 10−3 × 1.6 × 10−19 V
4Ω 6Ω Planck’s constant, h = 6.6 × 10−34 Js
speed of light in vacuum, c = 3 × 108 m/s
Now, for the maximum wavelength of light
A 2Ω B photon’s required (p).
–10 V –2 V According to the Planck’s quantum theory,
hc c
∴ E = hν ⇒ E = [Q ν = ]
8Ω 12Ω λ λ
(a) (20/3) Ω (b) 10 Ω (c) 16 Ω (d) 20 Ω Putting the given values, we get
6.6 × 10−34 × 3 × 108
Sol. (c) 50 × 10−3 × 1.6 × 10−19 =
λ
Given figure,
6.6 × 10−34 × 3 × 108
1 4Ω 6Ω
λ=
50 × 10−3 × 1.6 × 10−19
6.6 × 3 × 10−34 × 108 6.6 × 3 × 10−4
= =
5 × 16 × 10−22 5 × 16
A 2Ω B
–10 V –2 V = 2.475 × 10−5 m
or λ = 24.75 × 10−6 = 24.75 µm
8Ω 2 12Ω 7. In a transistor, the value of α varies between
20 100
In the figure, diodes 1 and 2 both are in reverse and . Then the value of β varies
21 101
bias and in reverse bias, diode does not conduct,
between [20 April 2019, Shift-II]
so we may replace diodes 1 and 2 as open circuit
(OC) (a) 1 and 10 (b) 0.95 and 0.99
4Ω 6Ω (c) 20 and 100 (d) 200 and 300
OC
Sol. (c)
Given, in a transistor current gain,
20 100
A 2Ω B α= and
21 101
Now, common emitter transistor
OC α
8Ω 12Ω DC current gain, β =
1−α
Here, resistors of resistances 8 Ω, 2 Ω and 6 Ω 20
are in series. So, equivalent resistance between 20
points A and B Case I If α = , then β = 21 = 20
21 20
R AB = 8 + 2 + 6 1−
21
R AB = 16 Ω 100
100
6. In a p-type semiconductor the donor level is Case II α = , then β = 101 = 100
101 100
at 50 meV above the valence band. To 1−
produce one electron, the maximum 101
wavelength of light photon required is ∴Value of β varies between, β ∈[20, 100].
240 AP EAMCET Chapterwise Physics

8. For an LED to emit light in visible region of Applying KVL,


the electromagnetic spectrum, it can have ⇒ −15 + 2KI C + VCE = 0
energy band gap in the range of, ⇒ −15 + 2KI C + 7 = 0 ⇒ I C = 4 mA
I
(Plank’s constant, h = 6 .6 × 10 − 34 Js and Q Common-emitter current gain, β = C
IB
speed of light, e = 3 × 10 8 ms − 1 in vacuum)
IC 4 × 10−3
[21 April 2019, Shift-I] or IB = = = 0.04 mA
β 100
(a) 0.1 eV to 0.4 eV (b) 0.9 eV to 1.6 eV
Hence, the base current is 0.04 mA.
(c) 1.7 eV to 3.1 eV (d) 0.5 eV to 0.8 eV
Sol. (c) 10. Three amplifiers with voltage gains 10, 20
Key Idea Frequency of visible rays range from and 30 are connected in series. If the input
4 × 1014 Hz to 7 × 1014 Hz. signal has a peak value of 1 mV then the
Maximum and minimum energy of photon in peak value of the output voltage is
visible light, [22 April 2019, Shift-I]
6.6 × 10− 34 × 7 × 1014 (a) 6 V (b) 60 V (c) 0.6 V (d) 0.06 V
Emax = hν =
1.6 × 10− 19 Sol. (a)
= 2.88 eV ~ − 3 eV Given, voltage gains of three amplifiers,
6.6 × 10− 34 × 4 × 1014 A1 = 10 , A2 = 20 and A3 = 30
and Emin =
1.6 × 10− 19 peak value of input signal
= 1.65 eV ~− 1.7 eV Vi = 1mV = 10−3 V
So, the correct option matches between the Total gain of three amplifiers connected in
energy 1.7 eV to 3 eV is (c), series is given by
9. In the following common-emitter circuit, A = A1 × A2 × A3 = 10 × 20 × 30 ⇒ A = 6000
β = 100 and VCE = 7 V. If VBE is negligible, V
But, A = 0 ⇒ V0 = A × Vin = 6000 × 10−3 = 6 V
Vin
then the base current is [21 April 2019, Shift-II]
11. In the circuit given, the current through
2 kΩ Zener diode is [22 April 2019, Shift-II]
RB
I
15V
R1 500 Ω

15 V
1500 Ω R2 VZ=10 V

(a) 0.01 mA (b) 0.04 mA


(c) 0.02 mA (d) 0.03 mA (a) 10 mA (b) 6.67 mA
Sol. (b) (c) 3.33 mA (d) 5 mA
According to the question, Sol. (c)
According to the question, the circuit is
2 kΩ IC I
RB 15V
VCE=7V

IB

VBE=0 15V R1=500Ω


I2
I1
R2=
V2 V2=10V
1500 Ω
Given, β = 100, VCE = 7 V, VBE = 0
Semiconductor Electronics Materials, Devices and Simple Circuits 241

Since, potential drop across a Zener diode is ∴Effective resistance, R = 3 + 2 + 3 + 2 = 10Ω


constant. So, applying KVL in the circuit.
Current through the cell,
15 = IR1 + Vz ⇒ 15 = I(500) + 10 (Q Vz = 10 V)
(15 − 10) I = V / R = 20 / 10 = 2A
⇒ I= = 1 × 10− 2 A = 10 mA
500 14. An n-p-n transistor is connected in
There is potential drop of 10 V across the 1500 Ω common-emitter configuration as shown in
resistor.
the figure. If the collector current is 5 mA,
V 10
So, I1 = = = 6.66 mA VBE = 0 .6 V, VCE = 3 V and common-emitter
R2 1500
current amplification factor is 50, then the
Therefore, the current through the zener diode, values of R1 and R2 are respectively.
I z = I − I1 = 10 − 6.66 = 333
. mA [22 April 2018, Shift-II]
∴The correct option is (c).
R1 R2
12. The value of Y1 and Y2, respectively in the +
8V
following logic circuit if both A and B are 1. –
[22 April 2018, Shift-II]
A
B
Y1
(a) 1 k Ω, 74 k Ω (b) 74 k Ω, 1 k Ω
(c) 37 k Ω, 2 k Ω (d) 2 k Ω, 37 k gΩ
Sol. (b)
In given circuit, VCC = i B RB + VBE
Y2 V − VBE
⇒ RB = R1 = CC
iB
(a) 1, 1 (b) 1, 0 (c) 0, 1 (d) 0, 0 i 5 × 10− 3
As, i B = C = = 1 × 10− 4 A
Sol. (c) β 50
8 − 0.6
The logical circuit can be explained as, ⇒ R1 = = 7. 4 × 104 = 74 × 103 Ω
1 × 10 − 4
1
A=1 P=1+1=1 = 74 kΩ
B=1 1 and by KVL in closed collector loop, we get
1
Y1=1.0=0
VCC = i C RL + VCE
1 V − VCE
P=1.1=0 ⇒ RL = CC
iC
1 Q
8−3
=
1 5 × 10− 3
Y2=1+0=1
0 So, R2 = RL = 1.0 kΩ

So, Y1 = 0 and Y2 = 1 15. The two combinations of NAND gates shown


Hence, the correct option is (c). in the figure are equivalent to
[22 April 2018, Shift-II]
13. If the diodes are ideal in the circuit given
below, then the current through the cell is A
[22 April 2018, Shift-I]
(i) C
(a) 4 A (b) 1.5 A (c) 2 A (d) 3 A
Sol. (c) B
Diode, D1 = reverse biased (OFF)
and diode, D2 = forward biased (ON).
(ii) A
Because diodes are ideal, so voltage drop across C
B
D2 is zero.
242 AP EAMCET Chapterwise Physics

(a) (i)-Or gate, (ii)-AND gate Y1 = A + B; Y2 = A + B, as gate 1 and 2


(b) (i)-AND gate, (ii)-NOT gate are OR gates.
(c) (i)-NOT gate, (ii)-AND gate As gate 3 is NAND gate, its output will be
(d) (i)-AND gate, (ii)-OR gate Y = Y1 Y2 = Y1 + Y2 = A + B ⋅ A + B
Sol. (a) = A + B + A + B= A + B
The output of combination of NAND gate shown Using De-Morgan’s theorem and identity
in circuit diagram (i) is given as
A + A = A.
A So, given output is that of NOR gate.
A
17. In the common-base configuration, a
C transistor has current amplification factor
0.95. If the transistor is used in
B common-emitter configuration and base
B
current changes by
Fig. (i) 2 µA, then the change in the collector
current is [23 April 2018 Shift-II]
C = A ⋅ B = A + B [Q x ⋅ y = x + y ] (a) 19 µA (b) 0.91 µA (c) 1.9 µA (d) 38 µA
C = A + B = Output of OR gate Sol. (d)
The output of combination of NAND gate shown Given, α = 0.95
in circuit diagram (ii) is given as α 0.95
So, β= = = 19
1 − α 1 − 0.95
A AB
C ∆I
B = β = C = 19
Fig. (ii)
∆I B
or ∆I C = 19 × 2 = 38 µA
C = AB = AB = Output of AND gate
18. Match the following List I and List II.
16. For the combination of logic gates shown in List I List II
the figure, the equivalent logic gate is
[23 April 2018 Shift-I] A. Small Scale Integration I. Logic gates < 100
(SSI)
A
B B. Medium Scale II. Logic gates > 1000
Y Integration (MSI)
A C. Large Scale Integration III. Logic gates ≤ 10
B (LSI)

(a) AND (b) NOT D. Very Large Scale IV. Logic gates < 1000
(c) NAND (d) NOR Integration (VLSI)

Sol. (d) The correct answer is [23 April 2018 Shift-II]


A Y1 A B C D A B C D
OR (a) I III I IV (b) IV I II III
B 1
(c) I IV III II (d) III I IV II
NAND Sol. (d)
3 SSI ⇒ logic gates < 10
A Y2
OR MSI ⇒ logic gates < 100
B 3 LSI ⇒ logic gates < 1000
VLSI ⇒ logic gates > 100
28
Communication Systems
1. An AM wave has 1800 W of total power 3. A signal of frequency 10 kHz and peak
content for 100% modulation the carrier voltage 10V is used to amplitude modulate a
should have power content equal to carrier of frequency 1 MHz and peak voltage
[18 Sep. 2020, Shift-I]
20 V. The side-band frequencies in kHz are
[20 April 2019, Shift-I]
(a) 1000 W (b) 1200 W
(c) 1500 W (d) 1600 W (a) 1010, 990 (b) 910, 1090
(c) 10, 11 (d) 1.01, 0.99
Sol. (b)
Sol. (a)
Total power of AM wave, Pt = 1800 W
Given, frequency of signal, fs = 10 kHz
For 100% modulation, depth of modulation,
peak voltage, Vp = 10 V
m =1
carrier frequency, fc = 1 mHz
If Pc be the carrier power, then we know that,
and peak voltage, Vc = 20 V
 m2 
Pt = Pc 1 +  ∴The side band frequencies are given as
 2
f = fc ± fs
Pt = Pc 1 + 
1
⇒ Putting the given values, we get
 2
f = 1 × 106 ± 10 × 103 Hz = (1000 ± 10) 103 Hz
3
⇒ Pt = Pc So, the side band frequencies in kHz is1010 kHz
2
2 2 and 990 kHz.
⇒ Pc = Pt = × 1800 = 1200 W
3 3 4. A TV tower has a height of 5 m in a region of
1000
2. A speech signal of 3 kHz is used to modulate population density per square
a carrier signal of frequency 1 MHz, using π
amplitude modulation. The frequencies of kilometre. Number of people that can receive
the side bands will be [22 Sep. 2020, Shift-II] the transmission is nearly, (in thousands)
[20 April 2019, Shift-II]
(a) 1.003 MHz and 0.997 MHz
(b) 3001 kHz and 2997 kHz (a) 128 (b) 64
(c) 1003 kHz and 1000 kHz (c) 256 (d) 32
(d) 1.0 MHz and 0.997 MHz Sol. (b)
Sol. (a) Given, height of a TV tower, h = 5 m
Frequency of carrier signal, fc = 1 MHz 1000
population density, n =
Frequency of message (speech) signal, π
fm = 3kHz = 0.003 MHz Now, the maximum range of this transmission
depends upon the height of transmitting
∴ Frequency of upper side band = fc + fm
antenna and is given by d = 2 h Re .
= 1 + 0.003 = 1.003 MHz
where, h = height of tower
Frequency of lower side band = fc − fm
and Re = Radius of earth (R > > h)
= 1 − 0.003 = 0.997 MHz
244 AP EAMCET Chapterwise Physics

So, the number of people received the 7. A TV tower has a height 160 m. Its coverage
transmission,
range is nearly (Earth’s radius, Re = 6400 km)
Population covered, (Pc) = population density [22 April 2019, Shift-I]
(n) × area of transmission range (A)
(a) 45255 m (b) 55265 m
Population covered, (Pc) = n × A
(c) 452.55 km (d) 552.65 m
1000
Pc = × π d2
π Sol. (a)
Area of transmission range, Given, height of a TV tower, h = 160 m
A = πd 2 or Pc = 1000 × 2h Re [Q d 2 = 2h Re ] Radius of earth, R = 6400 km = 6.4 × 106 m
−3
or Pc = 1000 × 2 × 5 × 10 × 6400 Coverage range of TV tower is given by
or Pc = 64000 d = 2Rh = 2 × 6.4 × 106 × 160
So, the number of people received the = 45254.8 = 45255 m
transmission is 64000.
8. Co-axial cable, a widely used wire medium
5. A transmitting antenna of height 20 m and for transmission of signals offers a bandwith
the receiving antenna of height h are of approximately. [22 April 2019, Shift-II]
separated by a distance of 40 km for (a) 600 kHz (b) 750 MHz
satisfactory communication in line of sight (c) 850 GHz (d) 500 Hz
(Los) mode. Then the value of h is Sol. (b)
(Give, radius of earth is 6400 km.) Co-axial cable primarily used for transmission
[21 April 2019, Shift-I] of radio, television signals, which are in range
of 3 MHz to 3 GHz.
(a) 40 m (b) 45 m
So, the band width,
(c) 30 m (d) 25 m
∆f = f1 − f2 = (3000 − 3) × MHz ≈ 3000 MHz
Sol. (b) Hence, the nearby option is (b).
Given, height of transmitting antenna
hT = 20 m, distance of LOS, dm = 40 km 9. If E c and E m are peak values of carrier and
As maximum distance of line of sight mode modulating signals, respectively then for
dm = 2RhT + 2RhR 100% modulation, [21 April 2019, Shift-II]
Em E2
putting the given values, we get (a) Ec = (b) c = Em2 (c) Ec = Em (d) Ec = 2 Em
2 2
40 = 2 × 6.4 × 103 × 20 × 10− 3
Sol. (c)
+ 2 × 6.4 × 103 × hR × 10− 3 Key idea Modulation index of a modulated signal is
E
40 = 16 + 2 × 6.4 × 10 × 10
3 −3
× hR µ= m
Ec
⇒ 2 × 6.4 × hT = 24 × 24
Where, E m and E c are peak values of modulating
⇒ hT = 45 m signal and carrier signal.
Hence, the correct option is (b). For 100% modulation, modulation index should
be unity.
6. The frequency suitable for
E
beyond-the-horizon communication using Hence, µ = 1 = m ⇒ Em = Ec
sky waves is [21 April 2019, Shift-II]
Ec
(a) 1012 Hz (b) 109 Hz ∴The correct option is (c).
(c) 107 Hz (d) 104 Hz 10. If a message signal of frequency 10 kHz and
Sol. (c) peak voltage 12 V is used to modulate a
The frequency suitable for beyond-the-horizon carrier wave of frequency 1 MHz, the
communication using sky wave is 10 MHz (107 Hz). modulation index is 0.6. To make the
Sky wave is radio wave which is directed towards modulation index 0.75, the carrier peak
the sky and reflected back by the ionosphere voltage should be [22 April 2018, Shift-I]
towards the desired location on the earth. (a) decreased by 25% (b) increased by 25%
(c) decreased by 20% (d) increased by 20%
Communication Systems 245

Sol. (c) 12. A TV transmitter has a range of 50 km. The


Vpeak, signal height of the TV transmitter is _______
Modulation index, M =
Vpeak, carrier (Radius of the earth, Re = 6 ⋅ 4 × 10 6 m)
Let’s write Vpeak, carrier = V [23 April 2018, Shift-I]
12 (a) 195.3 m (b) 186.5 m (c) 206 m (d) 175 m
In first case, 0.6 =
V Sol. (a)
⇒ V = 20 V Range, d = 2Re hT
12
In second case, 0.75 = 50 km = 2 × 6.4 × 106 × hT
V′
12 2500 × 106 = 128 × 105 hT
⇒ V′ = = 16 V
25000
0.75 = hT
Change in peak voltage of carrier wave 128
∆V = 20 − 16 = 4V hT = 195.3
4
% change = × 100% = 20% (decrement) 13. If the height of the transmitting tower is
20 increased by 30%, then the area covered by it
11. The maximum distance between the increases by [23 April 2018, Shift-II]
transmitting and receiving TV towers is (a) 10% (b) 21% (c) 30% (d) 60%
65 km. If the ratio of the heights of the TV Sol. (c)
transmitting tower to receiving tower is Area covered by a transmission tower of height
36 : 49, the heights of the transmitting and h is
receiving towers respectively are (radius of A = 2πRh
earth = 6400 km) [22 April 2018, Shift-II] ⇒ A∝h
(a) 51.2 m, 80 m (b) 70.3 m, 95.7 m So, if height is increased by 30%, area of
(c) 30 m, 65 m (d) 25 m, 75 m coverage also increases by 30%.
Sol. (b) 14. A modulated signal is given by
hT 36
Here, = … (i) t = Ac sin ω ct + µAc sinω mt sinω ct, where µ
C m()
hR 49 is modulation index. To keep the signal
without distortion, the value of µ should be
[24 April 2018, Shift-I]
(a) > 1 (b) ≥ 1 (c) = 1 (d) ≤ 1
hT Sol. (d)
hR
Modulation index µ is given by
A
65 km µ= m
Ac
dmaximum = 2RhT + 2RhR where, Am and Ac are amplitude of modulating
wave and carrier wave.
or 65 = 2RhT + 2RhR … (ii) ⇒ To keep signal without distortion,
From Eqs. (i) and (ii), we get Am ≤ Ac
hT = 70.3 m i.e. µ ≤1
and hR = 957
. m
Practice Set 1 247

Practice Set 1
(a) 12.56 × 10−23 Am2
1. An electric field E = $i x exists in space. Find
(b) 1.256 × 10−23 Am2
the potential at the origin, taking the
potential at (10 m, 20 m) to be zero. (c) 125.6 × 10−23 Am2
(a) 500 V (b) 600 V (d) 1256 × 10−24 Am2
(c) 525 V (d) 5.25 V
6. An X-ray tube operates at 10 kV. Find the
2. A charged duel particle of radius 5 × 10 −7 m is ratio of X-ray wavelength to that of de-Broglie
located in a horizontal electric field having an wavelength.
intensity of 6 .28 × 10 5 Vm −1 . The surrounding (a) 10.062 (b) 11.062
(c) 100.62 (d) 110.62
medium is air with coefficient of viscosity
. × 10 −5 N-s/m 2 . If this particle moves
η = 16 7. Critical angle for a certain wavelength of light
with a uniform horizontal speed of 0.02 m/s, in glass is 40°. Calculate the polarising angle
then find the number of electrons in it. and angle of refraction in glass corresponding
(a) 15 (b) 30
to this.
(c) 32 (d) 35 (a) 57.3°, 32.7° (b) 32.7°, 57.3°
(c) 47.3°,22.7° (d) 22.7°,47.3°
3. One metre long metallic wire is broken into
two unequal parts P and Q. P part of the wire 8. A thermally insulated piece of metal is heated
is uniformly extended into another wire R. under atmosphere by an electric current so
Length of R is twice the length of P and the that it receives electric energy at a constant
resistance of R is equal to that of Q. Find the power P. This leads to an increase of the
ratio of the resistance of P and Q and also the absolute temperature T of the metal with time
ratio of length of P and R t as follow T = at1 / 4
(a) 1 : 4, 1 : 4 (b) 4 : 1, 4 : 1 Then, the heat capacity C P is
(c) 1 : 3, 1 : 3 (d) 3 : 1, 3 : 1 4PT 3 4PT 2
(a) (b)
4. V-I graph for a conductor (platinum wire) at a4 a3
temperatures T1 and T2 is as shown Figure (c) 4PT 2 (d) None of these
T2 − T1 is proportional to 9. A convex lens has its radii of curvature equal.
V T2 The focal length of the lens is f. If it is divided
vertically into two identical plano-convex
lenses by cutting it, then the focal length of
T1 the plano-convex lens is (µ = the refractive
θ index of the material of the lens)
f
(a) f (b)
θ 2
I (c) 2f (d) (µ −1) f
O
(a) cos2θ (b) sin2θ 10. Between two large parallel plates, a uniform
(c) cot2θ (d) tan2θ vertical field (E) is set up as shown in figure.
Find the period of oscillation of the
5. In a hydrogen atom, the electron moves in an
pendulum, if the pendulum of length L
orbit of radius 0.5 Å, making 10 16 rps. having a small conducting sphere of mass m
Calculate the magnetic moment associated and charge + q is oscillating between the
with the orbital motion of electron. plates
248 AP EAMCET Chapterwise Physics

+ + + + + + + + then the threshold frequency of the metallic


surface is
E v1 − v 2 nv1 − v 2
L (a) (b)
n−1 n−1
+q
θ nv 2 − v1 v1 − v 2
m (c) (d)
n−1 n
– – – – – – –

(a) T = 2 π
L
(b) T = 2 π
L 16. A wire in the form of a circular loop of radius r
g+
qE
g−
qE lies with its plane normal to a magnetic field
m m B. If the wire is pulled to take a square shape
(c) T = π
L
(d) T = π
L in the same plane in time t, the emf induced
qE qE in the loop is given by
g+ g−
m m πBr 2  π πBr 2  π
(a) 1 −  (b) 1 − 
11. Three identical blocks of masses m = 2 kg are t  10  t  8
drawn by a force F = 10 .2 N with an πBr 2 1 − π  πBr 2 1 − π 
(c)   (d)  
acceleration of 0.6 ms −2 on a frictionless t  6 t  4
surface, then what is the tension (in N) in
17. When photons of energy 4.25 eV strike the
string between the blocks B and C?
surface of a metal, the ejected photoelectrons
C B A F have maximum kinetic energy T A eV and de
Broglie wavelength λ A . The maximum kinetic
energy of photoelectrons liberated from
(a) 9.2 (b) 7.8 (c) 4 (d) 9.8
another metal B by photons of energy 4.70 eV
12. In a Young’s experiment, one of the slit is is TB = (T A − 150. ) eV.If the de Broglie
covered with a transparent sheet of thickness wavelength of these photoelectrons is
3.6 × 10 −3 cm due to which position of central λ B = 2 λ A , then
fringe shift to a position originally occupied by (a) the work function of A is 4.25 eV
30th bright fringe. The refractive index of the (b) the work function of B is 4.20 eV
sheet if λ = 6000 Å is (c) TA = 3.00 eV
(a) 1.5 (b) 1.2 (c) 1.3 (d) 1.7
(d) TB = 2.75 eV
13. The saturation current of a p-n junction
germanium at 27°C is 10 −5 A. What will be
18. If four non-zero vectors satisfy
a × b = c × a and a × c = b × d with
required potential to be applied in order to
| a | ≠ | d | and| b |≠| c | then
obtain a current of 250 mA in forward bias
(a) 0.26 V (b) 0.36 V (c) 0.30 V (d) 0.29 V (a) (b − d) and (b − c ) are perpendicular
(b) (b − d) and (b − c ) are parallel
14. A crystal of intrinsic silicon at room (c) (a − d) must equal (b − c )
temperature has a carrier concentration of (d) (a − d) must equal − (b − c )
1.6 × 10 16 / m3 . If the donor concentration
19. Specific heat of argon at constant pressure is
level is 4.8 × 10 20 / m3 , then the concentration 0.125 cal-g −1 K −1 and at constant volume
of holes in the semiconductor is
0.075 cal-g −1K −1 . Calculate the density of
(a) 53 × 1012 / m3 (b) 4 × 1011 /m3
argon at NTP. Given, J = 4.18 × 10 7 erg cal −1
(c) 4 × 1012 / m3 (d) 5.3 × 1011 /m3
. × 10 6 dyne cm −2 ,
and normal pressure = 101
15. Photoelectric emission is observed from a
(a) 1.77 × 10−4 g / cm3
metallic surface for frequencies v1 , and v2 of
(b) 1.77 × 10−5 g / cm3
the incident light (v1 > v2). If the maximum
values of kinetic energy of the photoelectrons (c) 1.77 × 10−3 g / cm3
emitted in the two cases are in the ratio 1 : n, (d) 1.77 × 10−2 g / cm3
Practice Set 1 249

the length which the liquid will occupy in the


20. The vertical motion of a huge piston in a tube.
machine is approximately simple harmonic
(a) 5 cm (b) 2 cm (c) 5.5 cm (d) 3 cm
with a frequency of 0.50 s −1 . A block of 10 kg
is placed on the piston. What is the. maximum 25. In the potentiometer circuit shown in figure,
amplitude of the piston’s SHM for the block the balance point with R = 10 Ω when switch
and the piston to remain together? S1 is closed and S2 is open is 50 cm, while that
(a) 0.99 m (b) 0.100 m (c) 1.00 m (d) 9.9 m when S2 is closed and S1 is open is 60 cm.
What is the value of X ?
21. A 50 m long wire of mass 20 g supports a
mass of 1.6 kg as shown in Figure. Find the C J J¢
fundamental frequency of the portion of the A B
string between the wall and the pulley. (Take S1
g = 10 ms −2 )
40 cm 10.0W S2

R C¢ X

(a)1 Ω (b) 2 Ω (c) 3 Ω (d) 4 Ω


10 cm
26. A boy stands at 78.4 m from a building and
throws a ball which just enters a window 39.2
1.6 kg m above the ground. Calculate the velocity of
(a) 20 Hz (b) 30 Hz (c) 35 Hz (d) 25 Hz projection of the ball.
22. Write down the equation for a wave
propagating with velocity 330 ms −1 and u
having frequency 110 Hz. The amplitude is W
0.05 m.
(a) y = 0.05 sin2 π  220 t
x
±  ± φ0 39.2m
 3
(b) y = 0.05 sin2 π  200 t ± 
x A B
± φ0 78.4m
 4

(c) y = 0.05 sin2 π  110 t ± 
x
± φ0
(a) 39.2 m/s (b) 33.2 m/s
 3 (c) 32.5 m/s (d) 34.5 m/s
(d) y = 0.05 sin2 π  110 t ± 
x
± φ0  l
 27. t = 2π   is the dimensionally incorrect
4  g
23. A voltmeter having a resistance of 1800 Ω is formula, where l is length, t is time period of a
employed to measure the potential difference simple pendulum and g is acceleration due to
across a 200 Ω resistor which is connected to gravity. Find the correct form of the relation,
the terminals of a DC power supply having an dimensionally.
emf of 50 V and an internal resistance of 20 Ω. (a) t =
l
(b) t =
4l
(c) t = k
l
(d) t =
3l
What is the percentage decrease in the g g g 2g
potential difference across the 200 Ω resistor 28. A solid sphere of radius R and made of
as a result of connecting the voltmeter across material of bulk modulus K is completely
it? immersed in a liquid in a cylindrical
(a) 1% (b) 5% (c) 10% (d) 25% container. A massless piston of area A floats
24. A tube of 1 mm bore is dipped into a vessel on the surface of the liquid. When a mass M is
containing a liquid of density ρ = 800 kgm −3 placed on the piston to compress the liquid,
the fractional change in the radius of the
and surface tension, S = 49 × 10 −3 Nm −1 and sphere, δR / R is given by
angle of contact θ = 0 °. The tube is held (a) Mg / KA (b) Mg / 2 KA
inclined to the vertical at an angle of 60°, find (c) Mg / 3KA (d) Mg / 4KA
250 AP EAMCET Chapterwise Physics

29. Two concentric hollow spherical shells have 34. The intrinsic conductivity of germanium at
radii r and R (R >> r). A charge Q is distributed 27° is 2.13 mho m −1 and mobilities of
on them such that the surface charge electrons and holes are 0.38 and 0.18
densities are equal. The electric potential at m 2 V −1s −1 respectively. The density of charge
the centre is
carrier is
Q (R + r ) Q (R 2 + r 2 )
(a) (b) (a) 2.37 × 1019 m− 3 . × 1019 m− 3
(b) 328
4 π ε0 ( R + r )
2 2
4 π ε0 ( R + r )
(c) 7.83 × 1019 m− 3 (d) 8.47 × 1018 m− 3
Q
(c) (d) zero
(R + r ) 35. If the volume of a block of metal changes by
0.12% when it is heated through 20°C, what is
30. An n-type semiconductor has resistivity
the coefficient of linear expansion of metal?
0.1 Ω-m. The number of donor atoms which
must be added to achieve this is (a) 2.0 × 10−5 / ° C (b) 3.0 × 10−6 / ° C
−6
(µ 2 = 0.05 m 2 V −1s −1 ) (c) 2.0 × 10 /°C (d) 3.0 × 10−5 / ° C

(a) 1.25 × 1027 (b) 1.25 × 1023 36. A bullet is fired vertically upwards with an
(c) 1.25 × 10 21
(d) 1.25 × 1022 initial velocity of 50 ms −1 . If g = 10 ms −2 what
is the ratio of the distances travelled by the
31. Three rods A, B and C of same length and
bullet during the first and the last second of
cross-sectional area are joined in series.
Their thermal conductivities are in the ratio its upward motion?
1 : 2 : 1.5 if the open ends of A and C are at (a) 9 : 1 (b) 9 : 2 (c) 3 : 1 (d) 9 : 4
200°C and 18°C, respectively. At equilibrium 37. The coil in a tangent galvanometer is 16 cm in
the temperature at the junction of A and B is radius. How many turns of the wire should be
(a) 74°C (b) 127°C (c) 156°C (d) 148°C wound on it if a current of 20 mA is to
32. A wire of length L0 is supplied heat to raise its produce a deflection of 45°?
temperature by T. If γ is the coefficient of (Take, horizontal component of earth’s field
volume expansion of the wire and y is the = 0 .36 × 10 −4 T)
young’s modulus of the wire then the energy
density stored in the wire is (a) 469 (b) 458 (c) 569 (d) 400
1 1
(a) y2T 2 γ (b) y2T 2 γ 38. Let F, FN and f denote the magnitudes of the
2 3
1 y2T 2 1
contact force, normal force and the frictional
(c) (d) γ 2T 2 y force exerted by one surface on the other kept
18 γ 18
in contact. If one of these is zero, then
33. A conducting square loop of side L and (a) F > FN (b) F > f
resistance R moves in its plane with a uniform (c) FN − f < FN + f (d) All of these
velocity v perpendicular to one of its sides. A
magnetic induction B, constant in time and 39. A light source is placed at distance b from b
space, pointing perpendicular and into the from a screen. The power of the lens required
plane of the loop exists everywhere. The to obtain k-fold magnified image is
current induced in the loop is k+1 (k + 1)2
(a) (b)
kb kb
kb kb
(c) (d)
k+1 (k −1)2
v
40. The refractive index of a lens material is µ and
B focal length f Due to some chemical changes
in the material its refractive index has
BLv BLv
(a) clockwise (b) anticlockwise increased by 20%. The percentage decrease, in
R R
2BLv focal length for µ = 15
. will be
(c) anticlockwise (d) zero (a) 4% (b) 2% (c) 6% (d) 8%
R
Answers
1. (a) 2. (b) 3. (a) 4. (c) 5. (b) 6. (a) 7. (a) 8. (a) 9. (c) 10. (a)
11. (b) 12. (a) 13. (a) 14. (d) 15. (b) 16. (d) 17. (b) 18. (b) 19. (c) 20. (a)
21. (d) 22. (c) 23. (a) 24. (a) 25. (b) 26. (a) 27. (c) 28. (c) 29. (a) 30. (c)
31. (b) 32. (d) 33. (d) 34. (a) 35. (a) 36. (a) 37. (b) 38. (c) 39. (b) 40. (c)

Solutions
1. (a) Here, E = 10 $i x When wire P is extended into another wire R of
length 2l, then its resistance
Let origin A(0, 0) and B (10 m, 20 m). ρ(2l) 4ρl
RR = =
At x = 0, E = 0 A/2 A
At x = 10 m, According to the question,
E = 10 $i × 10 ⇒ RR = RQ
0 + 100$i 4ρl ρ(1 − l)
Average electric field = = 50$i =
2 A A
VB − V A = E (AB) After solving, we get
= − E [(x 2 − x1) $i + (y2 − y1) $j] ⇒ l = 0.2 m
RP ρl / A 1
0 − V A = − 50 $i [(10 − 0) $i + (20 − 0)$j] So, = =
RQ ρ(1 − l) / A 4
− V A = − 500
RP 1 0.2 1
V A = 500 V And, = = = =1: 4
RQ (1 − l) 1 − 0.2 4
2. (b) Here, r = 5 × 10−7 m RP ρl / A 1
Also, = = =1: 4
E = 6.28 × 10 Vm 5 −1 RR 4ρl / A 4

η = 1.6 × 10−5 Nsm−2 4. (c) R1 = tanθ = R0 (1 + αT1)


v = 0.02 m/s and R2 = cot θ = R0 (1 + α T2)
Let the dust particle carry n electrons. cot θ − tanθ = R0 (1 + αT2) − R0 (1 + αT1)
q = ne = R0α(T2 − T1)
∴ Force due to electric field = viscous force 1
T2 − T1 = (cot θ − tanθ)
q(E) = ne(E) αR0
eE = 6πηrv 1  cosθ sinθ 
= −
n=
6πηrv αR0  sinθ cosθ 
eE 2cos 2θ 2
= = cot 2θ
6 ×3.14 × 1.6 × 10−5 × 5 × 10−7 × 0.02 αR0 sin 2θ αR0
n=
1.6 × 10−19 × 6.28 × 105 T2 − T1 ∝ cot 2θ
n = 30
5. (b) Here, r = 0.5 Å = 0.5 × 10−10 m, v = 1016 rps
3. (a) Let l be the length of piece P. Then length of
piece Q = (1 − l). M =?
ρl Charge on electron, e = 1.6 × 10−19 C
So, Rp =
A Equivalent current,
ρ(1 − l) e Q ν = 1 
and RQ = I = = eν
A T  T 
252 AP EAMCET Chapterwise Physics

Area of the orbit


A = π r 2 ⇒ M = IA
M = ev(πr 2)
= 1.6 × 10−19 × 1016 × 314
. × (0.5 × 10−10)2
When convex iens is complete
M = 1.256 × 10−23 Am2
Lens Maker’s formula,
6. (a) Given, V = 10 kV = 10 × 103 V = 104 V
1 1 1 
For X-rays, eV = hc / λ = (µ − 1)  − 
hc
F  R1 R2 
λ=
= (µ − 1)  + 
eV 1 1 1
For de-Broglie wavelength, f  R R 
1 R
eV = mv2 f =
2 2(µ − 1)
mv = 2eVm R = 2 f (µ − 1) ...(i)
h h Now, it is divided vertically into two identical
λ′ = =
mv 2eVm plano convex lens
λ hc 2eVm 2m
Hence, = =c
λ′ evh eV
2 × 9 × 10−31
= 3 × 18
1.6 × 10−19 × 10 × 103
λ Plano convex lens
= 10.062
λ′ 1 1 1 
= (µ − 1)  −  [Q R1 = R, R2 = ∞]
7. (a) We have, f′  1
R R2
1 1 1
= (µ − 1)  − 
µ= = = = 1.56 1 1 1
sin C sin 40° 0.6427 f1  R ∞ 
If i P be the polarising angle, then R 2 f(µ − 1)
f1 = =
µ = tan i P ⇒ tan i P = 1.56 (µ − 1) (µ − 1)
∴ i P = tan−1 (1.56) = 57.3° f1 = 2 f
Angle of refreaction, r = 90° − i P 10. (a) Let sinθ = x / L
= 90° − 57.3° = 32.7° Where x is a displacement. A force qE acts on the
8. (a) dH = Pdt sphere and thus the net force on the sphere is
(mg + qE)
Pdt P
CP = = The restoring force is − (mg + qE) sinθ
dT dt
F = − (mg + qE) sinθ
Here, T = at1 / 4
(mg + qE) x
−  g +
qE  x 2
T T4 F=−  = −ω x
or = t1 / 4 ⇒ t= L  m L
a a4
1 qE 
−3/ 4 −3 ω2 = g+ 
dT a  T 4  a4 L m
=  
a T
=   =
dt 4  a 4  4 a 4T 3 2π
but, T=
P 4PT 3 ω
CP = = 4
dT / dt L
a ⇒ T = 2π
qE
g+
9. (c) The given, R1 = R, R2 = − R m
f =F
Practice Set 1 253

2.56 × 1032
11. (b) The system of masses nh = = 5.3 × 1011 / m3
4.8 × 1020
T2 T1
C B A F
15. (b) E1 = h(v1 − v0) and E 2 = h(v2 − v0)
From the figure Diving them, we get
F − T1 = ma … (i) E 2 v2 − v0
=
And T1 − T2 = ma … (ii) E1 v1 − v0
10.2 − T1 = 2 × 0.6 Given, E 2 = nE1 . Hence, we have
T1 = 10.2 − 1.2 = 9 N v − v2
n= 2
Again from Eq. (ii), we get v1 − v0
9 − T2 = 2 × 0.6 ⇒ T2 = 9 − 1.2 ⇒ T2 = 7.8 N nv1 − v2
which gives, v0 =
12. (a) The position of 30th bright fringe y30 = 30λD n −1
d
Now, position shift of central fringe is
16. (d) Induced emf (e)
D magnetic field × change in area B∆A
y = (µ − 1)t = =
d time t
30λD D Since, the circumference of the circular loop = 2πr,
⇒ y30 = = (µ − 1)t 2πr πr
d d the side of the square loop = = .
30λ 4 2
(µ − 1) =
t Therefore,
−10 2
30 × 6000 × 10 πr π
(µ − 1) = = 0.5 ∆A − πr 2 −   = πr 2 1 − 
. × 10−5
36  2  4
µ = 1.5 B(πr 2) 1 − π 
e=  
t  4
 eV 
13. (a) I = I 0  e kT − 1
  17. (b) hv = KE(T ) + work function
ev hv = T + W
I
or e kT = +1 4.25 = TA + W A [for metal A]
I0
4.70 = TB + WB [for metal B]
250 × 10−3
= + 1 = 25001 Since, TB = (TA − 1.5) eV
10−5
h
eV Also, λ=
or = log e (25001) = 10126
. p
kT
10126
. × kT h
V= λ=
e 2mT
10126
. ×1.38 × 10−23 × 300 λA TB
= ⇒ =
1.6 × 10−19 λB TA
= 0.26 V 1
Since, λ A = λB
2
14. (d) Given, n1 = 1.6 × 1016 / m3
T = 4TB ⇒ TB = TA − 1.50
ne = 4.8 × 1020 / m3
TB − 4TB − 1.5 ⇒ TB = 0.5 eV
nh = ? TA = 2eV ⇒ W A = 225
. eV
The concentration of holes in the semiconductor WB = 4.20 eV
n12 = ne × nh
18. (b) a × b = c × d … (i)
(1.6 × 1016)2 = 4.8 × 1020 × nh
a ×c=b×d … (ii)
254 AP EAMCET Chapterwise Physics

Subtracting (ii) from (i), we get 9.8


a= ⇒ a = 0.99 m
(a × b) − (a × c) = (c × d) − (b × d) . )2 × (0.50)2
4 × (314
a × (b − c) = (c − d) × d
21. (d) Here, mass per unit length is given by
a × (b − c) = − (b − c) × d
20 × 10−3
a × (b − c) + (b − c) × d = 0 m= = 0.04 kg m−1
50 / 100
a × (b − c) − d × (b − c) = 0
T = 1.6 kg = 1.6 × 10 N = 16 N
(a − d) × (b − c) = 0
Length the vibrates
So, (a × b) (a − d) and (b − c) are parallel.
L = 50 − 10 = 40 cm= 0.4 m
. cal g −1 K−1
19. (c) Cp = 0125 1 T
v=
Cv = 0.075 cal g −1 K−1 2L m

. × 107 erg cal −1


J = 418 1 16
v=
2 × 0.4 0.04
Normal pressure, p = 1.01 × 106 dyne cm−2
20
Normal temperature, T = 273K v= ⇒ v = 25Hz
0.8
We have to calculate density, ρ = ?
As gas constant for 1 g of gas 22. (c) Here, v = 330 ms −1
pV v = 110 Hz
R=
T v 330
λ= = = 3m
p v 110
R= …(i)
ρT Also, r = 0.05 ,
where V = volume occupied by 1 gram of gas The Eq. may be written as
R  2π 2π  
Cp − CV = y = r sin  vt ± x  ± φ0
J   λ λ  
R = J(Cp − Cv) …(ii)
 
y = 0.05 sin 2π 110t ±  ± φ0
x
From Eqs. (i) and (ii), we get   
3
p
= (0125
. . × 107
− 0.075) × 418 50 5
ρT 23. (a) Current, I = = A
(200 + 20) 22
1.01 × 106
. ×107
= 0.05× 418 ∴Potential drop across 200 Ω resistor (V)
ρ × 273
5 500
ρ = 1.77 × 10−3 g /cm3 = × 200 = V
22 11
20. (a) Given, frequency, ν = 0.5 s −1 When a voltmeter of resistance 1800 Ω is
connected across the 200 Ω resistor, the effective
Mass of block, m = 10 kg ⇒ a =? resistance R is given by
1 k 1 1 1
As v= = +
2π m R 1800 200
k = 4 π 2mv2 … (i) which gives, R = 180 Ω
For maximum displacement The current in the circuit becomes
50 5
ymax = a I′ = = A
180 + 20 20
Maximum restoring foce,
F = − ka ⇒ − ka = − mg The potential drop becomes
5
a=
mg V′ = × 180 = 45V
k 20
500 5
Substitutind the value of k from Eq. (i) Potential difference, (V − V ′) = − 45 = V
mg g 11 11
a= ⇒ a= 2 2
4 π 2mv2 4π v
Practice Set 1 255

5 11 And horizontal range


Percentage decrease =× × 100 = 1%
11 500 u2 sin 2θ
= 2 × 78.4 m … (ii)
24. (a) Here, r = 1 mm ⇒ r = 1 × 10− 3 m g
2 2
Dividing Eq. (i) and (ii)
ρ = 800 kgm− 3
u2 sin2 θ g 39.2
× 2 =
S = 49 × 10−3 Nm−1 2g u sin 2θ 2 × 78.4
θ = 0° sin2 θ 1
or =
then to find h. 2 × 2sinθ . cosθ 4
Angle with the horizontal of inclined tube tanθ = 1 ⇒ θ = 45°
S cosθ
α = 90 − 60 = 30 ⇒ h = Substituting in Eq. (ii), we get
rρg
u2 sin2 90°
2 × 49 × 10−3 × cos 0° = 2 × 78.4
h= 9.8
(1 / 2) × 10−3 × 800 × 9.8 or u= 2 × 78.4 × 9.8 or u = 39.2 ms −1
h = 0.025 m
If l is the length occupied by the liquid in the tube, 27. (c) Let t = kl a gb … (i)
then [M 0 L0 T1 ] = La(LT −2)b = La + b T −2b
h
sinα = h / l or l = Using principle of homogeneity of dimensions
sinα
a + b = 0 ⇒ − 2b = 1
.
25
= . × 2 ⇒ I = 5cm
= 25 1
sin 30° b=−
2
25. (b) Let V and V ′ be the terminal voltages of cells C
a = − b = −  −  =
1 1
and C′ respectively. Let k be the resistance per unit  2 2
length of wire AB, and J′ the balance point when S1
is closed. This happens when the potential from (i) t = kl1 / 2 g−1 / 2
difference across AJ due to C equals potential l
difference across R due to C′ ⇒ t=k
g
V V′R
i.e. kAJ = … (i)
kAB R+ X 28. (c) Pressure exerted by the piston on the liquid
Let J′ be the balance point when switch S1 is open when a mass M is placed on the piston, P = Mg / A.
and S2 is closed. Then This pressure is exerted by the liquid equally in all
V directions. Therefore, the surface of the sphere
kAJ ′ = V ′ … (ii) experiences a force P per unit area. The stress on
kAB
the sphere is P = Mg / A. Now, the volume of the
Dividing Eq. (i) and (ii), we get sphere is
R+ X AJ ′
= 4 πR 3
R AJ V=
3
10 + X 60
or = Due to stress, the change in the volume of the
10 50
sphere is
⇒ x= 2Ω
 4 πR 3  4 π 2
δV = δ  = 3R δR = 4 πR2δR
26. (a) Let a boy standing at A throws a ball with a  3  3
velocity u at an angle θ with the horizontal which
δV 3δR
just enters window W. As the boy is at 78.4 m from ∴Volume strain, =
the building and the ball just enters the window V R
39.2 m above the ground, therefore By definition, bulk modulus
2 2
u sin θ stress Mg / A δR Mg
Maximum height, = 39.2 m … (i) K = = or =
2g strain 3δR / R R 3KA
256 AP EAMCET Chapterwise Physics

YT
29. (a) We know that, the surface charge densities is Stress = Y(stress) =
3
Q
σ= ...(i) 1 Y 2γ 2T 2 1 2 2
4 π (r 2 + R2) U= ⇒ U= γ T Y
2 9Y 18
The electric potential at the centre
1  σ × 4 πr 2 σ × 4 πR 2  33. (d) Since, there exists no change in flux linked
V=  +  with the coil, hence induced emf equal zero.
4 πε0  r R 
 r 2 R2 
34. (a) Conductivity of an intrinsic semiconductor
1
= × σ × 4π  +  σ = e(neµ e + nhµ h)
4 πε0 r R
In an intrinsic semiconductor
σ
= (r + R) ne = nh = ni
ε0
σ
⇒ ni =
Q e (µ e + µ h)
and V= ⋅ (r + R)
ε0 ⋅ 4 π (r 2 + R2) 213
.
ni =
⇒ V=
Q

(r + R) 1.6 × 10−19 (0.38 + 018
. )
4 πε0 (r 2 + R2) . × 1019
213
ni =
30. (c) Conductivity, σ = nd eµ e 1.6 × 0.56

σ 1 ⇒ . × 1019 m− 3
ni = 237
nd = =
eµ e ρeµ e
35. (a) Coefficient of cubical expansion of metal γ is
1
nd = given by
. × 1.6 × 10−19 × 0.05
01 ∆V ∆V 012 .
21
γ= ⇒ =
nd = 1.25 × 10 Vt V 100
012
.
A B C ∴ γ= = 6.0 × 10−5/°C
31. (b) 100 × 20
K 2K K Coefficient of liner expansion
γ 6.0 × 10−5
200°C T1 T2 18°C α= =
3 3


KA(200 − T1) 2KA(T1 − T2) KA(T2 − 18)
= = = 2.0 ×10−5/°C
l l l
KA(200 − T1) 2KA(T1 − T2) 36. (a) The maximum height h attained by the bullet is
or = given by
l l
v2 − u2 = − 2gh
200 − T1 − 2T1 − 2T2
or 3T1 − 2T2 = 200 … (i) At the maximum height final velocity (v) becomes
zero.
2KA(T1 − T2) KA(T2 − 18)
and = u2
l l Hence, h= (v = 0)
2T1 − 3T2 = − 18 … (ii) 2g
50 × 50
Solving Eq. (i) and (ii, we get or h= = 125m.
2 × 10
T = 127°C
The total time taken by the stone to attain this
32. (d) Since, α = γ u 50
3 height is given by t = = = 5 s. During the first
g 10
(stress)2 1
Energy density = = Y (strain)2 second (t = 1 s), the stone covers a distance h1 ,
2Y 2 given by
∆l γT 1 1
Strain = = α∆T = αT = h1 = ut − gt 2 = 50 × 1 − × 10 × ()1 2 = 45 m
l 3 2 2
Practice Set 1 257

During the first four seconds (t = 4 s), the stone Q image is formed on screen. Hence,(b − x 0) is also
covers a height h given by the image distance
1 Since, image if formed on screen, so m will be
h = 50 × 4 − × 10 × (4)2 = 120 m
2 negative
∴ distance travelled by the stone during the last m= − k=
v
(i.e., fifth) second of its upwards motion is u
h2 = 125 − 120 = 5 m or v = − ku
Hence, h1 / h2 = 45 / 5 = 9 : 1 and u = − x0
37. (b) Here, r = 16 × 10 m −2 and v = b − x0
b
n=? ∴ b − x 0 = kx 0 or x0 =
11 + k
I = 20 mA = 20 × 10−3 amp
According to lens formula
θ = 45° 1 1 1
− =
H = 0.36 × 10−4 tesla v u f
2rH 1 1
From, I= tanθ or − =P
µ 0n b − x0 − x0
2rH b
n= tanθ P=
µ 0I x 0 (b − x 0)
2 × 16 × 10−2 × 0.36 ×10− 4 Putting the value of x 0
= × tan 45°
4 π × 10−7 × 20 × 20 × 10−3 (k + 1)2
P=
n = 458 turns kb

38. (c) The contact force is resultant of force of friction 40. (c) From lens Maker’s formula
and normal reaction 1 1 1 
= (µ − 1)  − 
∴From situation shown in figure f  1
R R2

FN 1
= k(µ − 1)
f
F= FN2 + f 2 Differentiating the above Eq. w.r.t. f
v
d ( f −1) d
= k (µ − 1)
df df
1 dµ
or − = k
F= FN2 + f 2 f2 df
Here, F > f and F > FN ∆f
or = − kf∆µ
as FN > FN − f f
and FN + f > FN − f ⇒ FN − f < FN + f  ∆f  1
or   =− × f∆ µ
 f  (µ − 1)
39. (b) If the distance of light source from lens is x 0 ,
then the distance of screen from lens is (b − x 0) ∆f 1
or =− × ∆µ
f (µ − 1)
∆f  µ   ∆µ 
or × 100 = −    ×100
f  µ − 1  µ 
 1.5 
∴ % ∆f = −   × 2%
 1.5 − 1 
X0 (b–X0) = − 3 × 2% = − 6% = 6%
b
258 AP EAMCET Chapterwise Physics

Practice Set 2
1. A particle of mass 1 g executes an oscillatory 6. When an automobile moving with a speed of
on the concave surface of a spherical dish of 36 km/h reaches an upward inclined road of
radius 2 m, placed on a horizontal plane. If angle 30°, its engine is switched OFF. If the
the motion of the particle starts from a point coefficient of friction is 0.1. How much
on the dish at the height of 1 cm from the distance will the automobile move before
horizontal plane and the coefficient of friction coming to rest? (Take, g = 10 ms − 2 )
is 0.01, how much total distance will be (a) 8.53 m (b) 0.853 m
moved by the particle before it comes to rest? (c) 85.3 m (d) 853 m
(a) 1 m (b) 0.1 m (c) 1.05 m (d) 0.01 m
7. The length of a sonometer wire is 0.75 m and
2. Three identical spheres each of mass M are density 9 × 10 3 kg m − 3 . It can bear a stress of
placed at the corners of a right angled triangle
with mutually perpendicular sides equal to 8.1 × 10 8 Nm − 2 without exceeding the elastic
2m. Taking their point of intersection as the limit. What is the fundamental frequency
origin, the position vector of the centre of that can be produced in the wire?
mass is (a) 200 Hz (b) 300 Hz
2 2 (c) 20.0 Hz (d) 0.300 Hz
(a) ($i + $j ) (b) ($i − $j )
3 3 8. A man of mass M stands at one end of a plank
1 1
(c) ($i + $j ) (d) ($i − $j ) of length L which lies at rest on a frictionless
3 3 surface. The man walks to the other end of
3. A body of mass 2 kg is moving with velocity the plank. If the mass of plank is M / 3, then
10 ms − 1 towards east. Another body of same the distance that the man moves relative to
mass and same velocity moving towards the ground is
north collides with former and coalesces and 3L L 4L L
(a) (b) (c) (d)
moves towards north-east. Its velocity is 4 4 5 3
(a) 10 m/s (b) 5 m/s (c) 2.5 m/s (d) 5 2 m/s 9. 3 moles of a gaseous mixture having volume
4. If a surface has a work-function 4.0 eV, what  1
V and temperature T are compressed to   th
is the maximum velocity of electrons  5
liberated from the surface when it is of its initial volume. The change in its
irradiated with ultraviolet radiation of adiabatic compressibility, if gas obeys the
wavelength 0.2 µm? equation pV 19 / 13 = constants, is
(a) 4.4 × 105 m/s (b) 8.8 × 107 m/s (R = 8.3 J mol − 1 K − 1 )
(c) 8.8 × 105 m/s (d) 4.4 × 107 m/s V 2 −1
(a) ∆C = − 0.0248 mN
5. Match List-I and List-II T
V
List-I List-II (b) ∆C = − 0.035 m2N− 1
T
1. Half-wave rectifier (i) Full wave rectifier V
(c) ∆C = − 0.0426 m2N− 1
2. Output is (ii) Efficiency is 40.6% T
V
discontinuous (d) ∆C = − 0.0137 m2N− 1
3. Output is (iii) Efficiency is 81.2% T
continuous 10. Assertion (A) A wheel moving down a
4. Full wave rectifier (iv) Half-wave rectifier perfectly frictionless inclined plane will
(a) 1-ii, 2-iv, 3-i, 4-iii (b) 1-iii, 2-i, 3-ii, 4-iv undergo slipping (not rolling motion).
(c) 1-iv, 2-iii, 3-iv, 4-i (d) 1-i, 2-iii, 3-iv, 4-ii
Practice Set 2 259

Reason (R) Friction does no work in pure (a) T = mg cos θ for all values of θ
rolling. (b) T = mg cos θ, only for θ = φ
(c) T = mg , for θ = cos − 1  (2 cos φ + 1)
1
(a) Both (A) and (R) are true and (R) is the correct
explanation of (A).  3 
(d) T will be large for small values of θ
(b) Both (A) and (R) are true and (R) is not the
correct explanation of (A). 15. A ray of light from a denser medium strikes a
(c) (A) is true but (R) is false. rarer medium at angle of incidence i. The
(d) (A) is false but (R) is true. reflected and the refracted ray make an angle
of 90° with each other. The angles of
11. The ratio of capacitance of two capacitors reflection and refraction are r and r′,
filled with dielectrics of same dimensions but respectively. The critical angle is
of different value K and K / 4 arranged in two (a) sin− 1 (tan r ) (b) sin− 1 (tan i )
ways as shown in figure (i) and (ii) is −1
(c) sin (tan i − r ) (d) tan− 1 (sin i )

16. The field intensity of two points on the axial


(i) (ii) line of a bar magnet at distances 40 cm and
20 cm is 10 : 125. The length of the magnet is
(a) 10 cm (b) 5 cm (c) 15 cm (d) 20 cm
(a) 5:2 (b) 25: 16 (c) 5 : 4 (d) 2 : 5 17. A thin magnet is cut into two equal parts by
12. In the following circuit a 10 m long cutting. What is the new magnetic moment of
−1 each part? What is the time period of each
. Ωm , a
potentiometre wire with resistance12
part as compared to that of original magnet if
resistance R1 and an accumulator of emf 2 V vibrated in the same magnetic field?
are connected in series. When the emf of M T M
(a) M ′ = , T′ = (b) M ′ = , T′ =T
termocouple is 2.4 mV, then the deflection in 2 2 2
galvenometer is zero. The current supplied by (c) M ′ = M, T ′ = T (d) M ′ = M, T ′ =2T
the accumulator will be
18. The maximum horizontal range of a particle is
+ – R1 400 m. The maximum value of height
I attained by it will be
A B (a) 100 m (b) 200 m (c) 400 m (d) 800 m

G 19. Thomson coefficient of a conductor is


10 µ VK − 1 . The two ends of it are kept at 50°C
and 60°C, respectively. Amount of heat
absorbed by the conductor when a charge of
Hot junction Cold junction 10 C flows through it is
−4
(a) 4 × 10 A (b) 8 × 10− 4 A (a) 1000 J (b) 100 J (c) 100 mJ (d) 1 mJ
(c) 4 × 10− 3 A (d) 8 × 10− 3 A 20. A spring balance is attached to the ceiling of a
lift. A man hangs his bag on the spring and
13. Two drops of equal size are falling through air
the spring reads 49 N, when the lift is
with a constant speed of 10 cm/s. If the drops
stationary. If the lift moves downward with
coalesce the new constant speed will be
(a) 20 cm s − 1 (b) 10 2 cm s − 1
an acceleration of 5 ms − 1 , the reading of the
spring balance will be
(c) 10 × 21/ 3 cm s − 1 (d) 10 × 2 2 / 3 cm s − 1
(a) 24 N (b) 74N (c) 15 N (d) 49 N
14. A simple pendulum has a bob of mass m and 21. A lead bullet penetrates into a solid object and
swings with an angular amplitude φ. The melts. Assuming that 50% of its KE was used
tension in the thread is T. At a certain time, to heat it, calculate the initial speed of the
the string makes an angle θ with the vertical bullet. The initial temperature of bullet is 27°C
(θ ≤ φ) and its melting point is 327°C. Latent heat of
260 AP EAMCET Chapterwise Physics

fusion of lead = 2.5 × 10 4 J kg − 1 and specific


27. Two particles each of mass m and charge q are
heat capacity of lead = 125 J kg − 1 K − 1 .
−1 −1 attached to the two ends of a light rigid rod of
(a) 5 × 10 ms
3
(b) 5 × 10 ms
2
length 2R. The rod is rotated at constant
(c) 6 × 103 ms −1 (d) 6 × 102 ms −1 angular speed about a perpendicular axis
22. A body of mass 1.0 kg is suspended from a passing through its centre. The ratio of the
weightless spring having force constant magnitudes of the magnetic moment of the
system and its angular momentum about the
600 Nm − 1 . Another body of mass 0.5 kg
centre of the rod is
moving vertically upwards hits the suspended
q q 2q q
body with a velocity of 3.0 ms − 1 and gets (a) (b) (c) (d)
2m m m πm
embedded in it. Find the frequency of
oscillations and amplitude of motion. 28. A thin air film between a plane glass plate and
(a) 3.2 Hz, 6 cm (b) 3.2 Hz, 5 cm a convex lens is irradiated with parallel beam
(c) 3.3 Hz, 6 cm (d) 3.2 Hz, 6.2 cm of monochromatic light and is observed
23. A spaceship is launched into a circular orbit under a microscope. We see
close to earth’s surface. What additional (a) uniform brightness
velocity has to be imparted to the spaceship in (b) complete darkness
the orbit to overcome the gravitational pull? (c) field crossed over by concentric bright and dark
(Take, radius of earth, Re = 6400 km and fringes
g = 9.8 ms −2) (d) None of the above

(a) 3.0 × 103 ms −1 (b) 3.25 × 104 ms −1 29. Two spheres of radii in the ratio 1 : 2 and
(c) 3.45 × 105 ms − 1 (d) 3.2805 × 103 ms −1 densities in the ratio 2 : 1 and of same specific
heat are heated to the same temperature and
24. The diameter of a circular hole in an left in the same surroundings. Their rates of
aluminum plate is 2.0 cm at 20°C. On heating
cooling will be in the ratio
to 120°C. diameter of the hole will be
(a) 1 : 2 (b) 1 : 1
(α = 23 × 10 − 6 /°C)
(c) 1 : 4 (d) 2 : 1
(a) 2.203 cm (b) 2.23 cm
(c) 2.023 cm (d) 2.0046 cm 30. A sample of oxygen at NTP has volume V and
a sample of hydrogen at NTP has volume 4V.
25. A uniform electric field and a uniform Both the gases are mixed and mixture is
magnetic field are acting along the same maintained at NTP. If the speed of sound in
direction in a certain region. If an electron is
hydrogen at NTP is 1270 ms − 1 , that in the
projected along the direction of the field with
mixture will be
a certain velocity then
(a) 317 ms − 1 (b) 635 ms − 1
(a) its velocity will decrease
(c) 830 ms − 1 (d) 950 ms − 1
(b) its velocity will increase
(c) it will turn towards right of direction of motion 31. In a p-n-p transistor working as a common
(d) it will turn towards left of direction of motion. base amplifier, current gain is 0.96 and emitter
26. A simple pendulum is vibrating with an current is 7.2 mA. The base current is
angular amplitude of 90° as shown in the (a) 0.40 mA (b) 0.35 mA
figure. For what value of α, the acceleration is (c) 0.29 mA (d) 0.60 mA
directed horizontally? 32. A parachutist drop freely from an airplane for
90° 10 s before the parachute opens. He then
descends with a uniform retardation of
2.5 ms − 2 . If he bails out of the plane at a
height of 2495 m and g is 10 ms − 2 , his
(a) 0 (b) 90° velocity on reaching the ground will be
−1 1   1  (a) 5 ms − 1 (b) 10 ms − 1
(c) cos   (d) sin− 1  
 3  3 (c) 15 ms − 1 (d) 20 ms − 1
Practice Set 2 261

33. A cubical block of wood of density 0.5 kg m −3 37. If A = 3$i + 4 $j and B = 7 $i + 24 $j. Find a vector
−3 having the same magnitude as B and parallel
and chunk of concrete of density 2.5 kg m
are fastened together. The ratio of the mass of to A
wood to the mass of concrete which makes (a) 16$i + $j (b) 15$i + 20$j
the combination float with its entire volume (c) 20$i + 15$j (d) 14$i + 15$j
submerged under water, is
1 1 2 3 38. A stone of mass 100 g is suspended from the
(a) (b) (c) (d)
5 3 3 5 end of a weightless string of length 100 cm
and is allowed to swing in a vertical plane.
34. Two factories are sounding their sirens at The speed of the mass is 200 cms − 1 when the
800 Hz. A man goes from one factory to the string makes an angle of 60° with the vertical.
other at a speed of 2 ms −1 . The velocity of Calculate the tension in the string at θ = 60 °.
sound is 320 ms −1 . The number of beats heard Also calculate the speed of the stone when it
by the person in one second will be is in the lowest position given g = 980 cms − 1
(a) 2 (b) 4 (c) 8 (d) 10
(a) 1 × 104 dyne, 3.7 ms −1
35. Figure represents a convergent lens placed (b) 8.9 × 104 dyne, 3.7 ms −1
inside a cell filled with a liquid. The lens has a (c) 9.8 × 104 dyne, 3.7 ms −1
focal length 20 cm when in air and its (d) 8.9 × 104 dyne, 3.8 ms −1
material has refractive index 1.50. If the
liquid has a refractive index 1.60, the focal 39. A lift is going upwards with an acceleration of
length of the system is 4.9 ms − 2 . What will be the apparent weight of
a 60 kg person sitting in the lift when the lift
Material
Liquid
µ = 1.60
acquires a uniform velocity of 4.9 ms − 2 , if the
rope of the lift is broken? (Take, g = 9.8 ms − 2 )
µ=
(a) 0 (b) 2
1.50
(c) 2.5 (d) 0.2
40. A 50 g lead bullet, specific heat 0.02 is initially
at 30°C. It is fired vertically upwards with a
(a) − 80 cm (b) + 80 cm speed of 840 ms − 1 and on returning to the
(c) − 160 cm (d) − 24 cm starting level strikes a cake of ice at 0°C.How
36. A force F = (− 3$i + $j + 5k$ ) N acts at a point much ice is melted? Assume that all energy is
spent in melting only (L = 80 cal g − 1 )
(7, 3, 1). The torque about the origin (0, 0, 0)
will be (a) 60.28 g (b) 58.88 g
(a) 14$i − 38$j + 16 k$ (b) 14$i + 38$j − 16 k$ (c) 52.88 g (d) 50.28 g
(c) 14$i − 38$j − 16 k$ (d) 14$i + 38$j + 16 k$

Answers
1. (a) 2. (a) 3. (d) 4. (c) 5. (a) 6. (a) 7. (a) 8. (b) 9. (a) 10. (b)
11. (b) 12. (a) 13. (d) 14. (c) 15. (a) 16. (a) 17. (b) 18. (b) 19. (d) 20. (a)
21. (b) 22. (b) 23. (d) 24. (d) 25. (a) 26. (c) 27. (a) 28. (c) 29. (b) 30. (b)
31. (c) 32. (a) 33. (d) 34. (d) 35. (c) 36. (a) 37. (b) 38. (b) 39. (a) 40. (c)
Solutions
1. (a) Here, mass of particle, m = 1 g, 4. (c) Work-function, W = 4 eV = 4 × 1.6 × 10−19 J
Radius of dish, R = 2m, = 6.4 × 10−19 J
Height, h = 1 cm Wavelength of incident radiation,
and coefficient of friction, µ = 0.01 λ = 0.2 µm = 0.2 × 10−6 m
Potential energy of particle = mgh
Maximum KE of liberated electron,
If d is the distance moved by the particle before hc
coming to rest, then work done against friction is (KE)max = −W
λ
W = µmg × d 1 hc
As potential energy is spent in doing this work
2
mvmax = −W
2 λ
h
µmg × d = mgh or d = 1 6.6 × 10−34 × 3 × 108
µ ⇒ × 9 .1 × 10−31 vmax
2
=
2 0.2 × 10−6
1 − 6.4 × 10−19
or d= = 100 cm or d =1 m
0.01
= 9.9 × 10−19 − 6.4 × 10−19
2. (a) y = 3.5 × 10−19

(0, 2) 3.5 × 10−19 × 2 7


∴ vmax = = × 1012
9.1 × 10−31 9.1
= 8.8 × 105 m/s

5. (a)
(0, 0) x 1. Efficiency of half-wave rectifier is 40.6%.
(2, 0)
2. Half-wave rectifier converts only half cycle
Here, r1 = 0$i + 0$j, r2 = 2$i + 0$j of alternating current. So, it output is in the
and r3 = 0$i + 2$j discontinuous form.
3. Full wave rectifier converts both half cycle
M r1 + M r2 + M r3
∴ rCM = of alternating current. So, it output is in the
M+ M+ M continuous form.
M(0i$ + 0$j) + M(2i$ + 0$j) + M(0i$ + 2$j) 4. Efficiency of full wave rectifier is 81.2%.
rCM =
3M Hence, option (a) is correct.
M(2$i + 2$j) 2 $ 2 $ 2 $ $ 6. (a) Here, initial speed, u = 36 km h− 1
= = i + j = (i + j)
3M 3 3 3 36 × 1000
= = 10 ms − 1
60 × 60
3. (d) Initial momentum, p = mv$i + mv$j
θ = 30°, µ = 0.1, s =?
| p | = 2 mv
Final velocity, v = 0
Final momentum = 2m × v Work done in moving up the inclined road
By the law of conservation of momentum, = KE of the vehicle
Final momentum = initial momentum R
2m × v′ = 2mv
v
⇒ v′ =
2
Here, v =10 m/s q q
sin
10 mg mg cos q
∴ v′ = = 5 2 ms − 1 q mg
2
Practice Set 2 263
1
(mg sin θ + F) × s = mu 2 9. (a) Adiabatic elasticity, E φ = γp
2
1 Compressibility,
or (mg sin θ + µR) × s = mu 2
2  1  1
C =   =   ⇒ ∆C = C′ − C
1
mu 2  Eφ   γp 
or s= 2
mg(sin θ + µ cos θ) 1 1 1
=  p′ − p  ...(i)
2 γ  
u
or s= γ
2 g(sin θ + µ cos θ) V
For adiabatic change, pV γ = p′  
10 × 10  5
or s=
2 × 10(sin 30° + 1.0 cos 30 °) p′ = 5γ p ...(ii)
⇒ s = 8.53 m From Eqs. (i) and (ii), we get
1  1  19
7. (a) Let a be the area of cross-section of the wire. ∆C =  γ
− 1 (Q given, γ = )
γp (5)  13
Given, l = 0.75 m, d = 9 × 103 kg m− 3
− 13 × 0.905
Stress = 8.1 × 10 8 Nm− 2 =
19p
The fundamental frequency of a vibrating
string is given by pV = nRT
− 13 × 0.905V
1 T 1 stress × a ∆C =
v= = 19 × 3 × 8.31T
2l m 2l a ⋅1 ⋅ d
V 2 −1
1 8.1 × 108 = − 0.0248 mN
= = 200 Hz T
2 × 0.75 9 × 103
10. (b) Rolling occurs only on account of friction
8. (b) Since, external force on system is zero. So, no which is a tangential force capable of providing
change takes place in the centre of mass. torque. When the inclined plane is perfectly
smooth, body will simply slip under the effect of its
m1 x1 + m2 x 2
x CM = own weight.
m1 + m2
m1 ∆x1 + m2∆x 2 11. (b) Here, C1 = ε0 K1 A + ε0 K 2 A
∆x CM = 2d 2d
m1 + m2 ε 0 KA ε 0 KA  K
= + K1 = K , K 2 =
As ∆x CM = 0 2d 8d  4 
⇒ m1 ∆x1 + m2∆x 2 = 0 ε 0 KA 1 1 
= +
or M∆x1 +
M
∆x 2 = 0 d  2 8 
3 ε KA  4 + 1 
−M ∆x = 0
or ∆x1 = ∆x 2 or ∆x1 = − 2 ...(i) d  8 
3M 3
5ε 0 KA
or x rel = ∆x1 − ∆x 2 = ...(i)
8d
L = ∆x1 − ∆x 2
ε0 A ε0 A (2K)ε 0 A
∆x and C 2 = = = ...(ii)
∴ L = − 2 − ∆x 2 d d d d 5d
3 + +
2K1 2K 2 2K 2K
3
or ∆x 2 = − L 4
4 Dividing Eq. (i) by Eq. (ii), we get
Putting this value in Eq. (i), we get 5ε 0 KA
 −3  C1 5 × 5 25
−  L = 8d = =
− ∆x 2  4  L C 2 2Kε 0 A 2 × 8 16
∆x1 = = =
3 3 4 5d
264 AP EAMCET Chapterwise Physics

3 cos θ = 1 + 2 cos φ
12. (a) E = xl = Iρl ⇒ I = E
ρl 1 
θ = cos − 1 (1 + 2 cos φ)
2.4 × 10−3  3 
I= = 4 × 10−4 A
1.2 × 5 15. (a) sin C = µ rarer = µ r
µ denser µd
13. (d) Let the radius of each drop be r.
Reflected ray
Then, we have
2 r 2 (ρ − σ) g
10 = ...(i)

r
i=
9 η i
Denser
If R is the radius of the combined drop, then
4 3 4 Rarer
πR = 2 × πr 3 r¢
3 3
R = 21 / 3 r Refracted ray
If v is the new terminal velocity, then
i + r′ = 90°
2 r 2 ⋅ 22/ 3(ρ − σ) g
v= ...(ii) According to Snell’s law,
9 η
µ d sin i = µ r sin (r′)
From Eqs. (i) and (ii), we get sin i
v ⇒ = sin C
= 22/ 3 ⇒ v = 10 × 22/ 3 cm s −1 sin (r′)
10
sin i µ
14. (c) By law of conservation of energy, = r
sin(90° − i) µ d
loss in kinetic energy
⇒ sin C = tan i
= gain in potential energy
C = sin − 1 (tan r) (Q i = r)
1
mv 2 = mgl cos θ − mgl cos φ
2 16. (a) Baxial = µ 0 2Mr
4 π (r 2 − l 2)2
v 2 = 2 gl(cos θ − cos φ)
where, r is the distance of the observation point
from the centre of the magnet and l is the
θ φ length of the magnet.
l
l cos φ T µ 2 M ⋅ 40
B1 = 0 ...(i)
l cos θ T 4π [(402) − l 2]2
µ0 2 M ⋅ 20
B1 = ...(ii)
4 π [(20)2 − l 2]2
From Eqs. (i) and (ii), we get
mg cos θ 10 40 [(202) − l 2]2
mg = ⋅
125 [(402) − l 2]2 20
According to figure,
mv 2 ⇒ l = 10 cm
T − mg cos θ =
l 17. (b) If M = ml is magnetic moment of original
m magnet, then magnetic moment of each part is M′
T − mg cos θ = 2 gl (cos θ − cos φ)
l given by
T − mg cos θ = 2mg cos θ − 2mg cos φ  m ml M
M′ =   l = =
T = 3mg cos θ − 2mg cos φ  2 2 2
For θ = φ, m0 l 2
Moment of inertia of original magnet, I =
T = mg cos θ 12
If T = mg, then (m0 / 2)l 2 I
Moment of inertia of each part = =
1 = 3 cos θ − 2 cos φ 12 2
Practice Set 2 265

Time period of each part,


22. (b) Here, inertia factor = m + M = 1 + 0.5 = 1.5kg
I/2
T ′ = 2π
M Spring factor = k = 600 Nm− 1
BH
2 1 k
Frequency, v =
Time period of each part, 2π (m + M)
I 7 600
T = 2π = = 3.2 Hz
MBH 2 × 22 1.5
Then, T ′ =T Using law of conservation of linear momentum
Time period remains unchanged. in the collision,
mv 0.5 × 3
2
18. (b) Rmax = u = 400 m (for θ = 45°)
mv = (m + M)V or V = = = 1 ms −1
(m + M) 1 + 0.5
g
u2 Now, just after collision the system will have
H max = 1
2g maximum KE = (m + M)V 2. Using law of
2
400 conservation of mechanical energy, we have
= = 200 m (for θ = 90°)
2 (KE)max = (PE)max = E
19. (d) By using, H = σQθ 1 1
So, (m + M)V 2 = kA 2
2 2
⇒ H = (10 × 10− 6) × 10 × (60° − 50°)
m+ M
= 10− 3 J = 1 mJ or A=V
k
20. (a) In stationary position, 1.5 1
49 or A =1 = m = 5 cm
mg = 49 ⇒ m = = 5 kg 600 20
9.8
When lift moves downwards, 23. (d) Orbital velocity of spaceship close to earth,
mg − T = ma vo = gR
Reading of balance, Escape velocity, v e = 2 gR
T = mg − ma
Additional velocity required
= 5(9.8 − 5) = 5 × 4.8 = 24.0 N = v e − v o = 2gR − gR
21. (b) Here, let m be the mass of the bullet.
= 2 × 9.8 × 6.4 × 106 − 9.8 × 6.4 × 106
Heat required to raise its temperature from
27°C to 327°C, = 3.2805 × 103 ms − 1
∆Q1 = cm∆T = 125 × m(327 − 27) 24. (d) On heating its diameter increases,
= 3.75 × 104 mJ ...(i) (∴since α is + ve)
Heat required to melt the bullet, D 2 − D1 = D1α∆t
∆Q2 = mL = m × 2.5 × 104 J ...(ii) = 2 × 23 × 10− 6 × 100 = 46 × 10− 4
As heat developed, ∴ D 2 = 2 + 0.0046 = 2.0046 cm
1 1 1
KE = × mv 2 ...(iii) 25. (a) When electron is projected in an electric field,
2 2 2
then velocity of electron will decrease.
From Eqs. (i), (ii) and (iii), we get
1 26. (c) For horizontal acceleration net force in vertical
m(3.75 × 104 + 2.5 × 104) = mv 2 direction should be zero.
4
1 T cos α = mg
or (6.25 × 10 ) m = mv 2
4
Loss in potential energy = Gain in kinetic energy
4
1
or v = 4 × 6.25 × 104 mgh = mv 2
2
or v = 5 × 102 ms − 1
266 AP EAMCET Chapterwise Physics

1 (4 V)ρ + V (16ρ)
or mgl cos α = mv 2 or v = 2 gl cos α ρmixture =
2 4V + V
or ρmixture = 4 ρ
a γp
v H 2 = 1270 =
h T cos a ρ
γp 1
a vmixture = = (1270)
T sin a 4ρ 2
a
vmixture = 635 ms − 1
v mg cos a
mg 31. (c) Given, current gain (α) = 0.96
Refering to the figure, and emitter current (I e) = 7.2 mA
mv 2 We know that, current gain,
T − mg cos α =
l I I
α = 0.96 = c = c
m2 gl cos α I e 7.2
= = 2mg cos α
l ⇒ I c = 0.96 × 7.2 = 6.91 mA
or T = 3mg cos α Therefore, base current,
Again, T cos α = mg I b = I e − I c = 7.2 − 6.91 = 0.29 mA
1
or 3 mg cos 2 α = mg ⇒ α = cos − 1 where, I c = collector current.
3
32. (a) Initial velocity of dropping = 0
27. (a) Magnetic moment, Let v1 be velocity at end of10 s.
2q
M = IA or M= (πR 2) v1 = gt
 2π 
  v1 = 100 ms − 1
ω
Distance travelled during this time is
or M = qωR 2 ...(i)
v 2 (100)2
Further, angular momentum L is h1 = 1 = or h1 = 500 m
2 g 2 × 10
L = Iω
So, a remaining distance of 2495 − 500 = 1995 m
where, I = moment of inertia of system.
has to be travelled with a retardation of
or L = (mR 2 + mR 2)ω 2.5 m/s 2. Let the parachutist strike the ground
or L = (2mR 2)ω ...(ii) with velocity v. Then,
From Eqs. (i) and (ii), we get v 2 − v12 = 2a(h − h1)
M qωR 2 M q or v 2 − (100)2 = 2(− 2.5) (1995)
= or =
L 2mR 2ω L 2m or v 2 = 10000 − 9975
28. (c) These concentric bright and dark fringes are or v 2 = 25 or v = 5 ms − 1
called Newton’s Rings.
33. (d) Let the mass of wood be m1 and that of
29. (b) Rate of cooling per unit area,
concrete be m2. Then,
mc  dθ  4 r 3ρc  dθ 
R=  = π −  Volume of wood, V1 =
m1
A  dt  3 4 πr 2  dt  0.5
or R ∝ rρ m2
Volume of concrete, V2 =
R1  r1   ρ1  1 2 2.5
∴ =    = × = 1 :1
R2  r2   ρ2  2 1 Upthrust = weight of water displaced
m m 
30. (b) Let density of hydrogen be ρ, then that of U =  1 + 2 × 1 × g
 0.5 2.5
oxygen is16ρ.
Practice Set 2 267

Since, the combination is floating, so


38. (b) Here, m = 100 g, r = 100 cm
Upthrust = weight of combination
v = 200 cm s − 1 , θ = 60°
 m1 m 
 + 2  g = (m1 + m2) g
 0.5 2.5 mv 2
T = + mg cos θ
m1 3 r
=
m2 5 (100) × (200)2
= + 100 × 980 cos 60°
100
34. (d) Number of beats = ∆n = 2vs n
v = 8.9 × 104 dyne
2 × 2 × 800 At θ = 60°, height above the lowest point
= =10
320 r
h = r − r cos 60° or h =
2
35. (c) When a lens is immersed in a liquid
As total energy at L = total energy at 60°
f l µ g − 1 1.5 − 1
= = × 20 1 1  r
fa µ g 1.5
−1
mv L2 = mv 2 + mg  
 2
−1 2 2
µ1 1.6
0.5 × 1.6 or v L2 = v 2 + gr
⇒ fl = − × 20 = − 160 cm
0.1 or v L = 22 + 9.8 × 1 = 3.71 ms − 1
36. (a) Given, F = (− 3$i + $j + 5k$ ) N 39. (a) When rope of the lift is broken, it falls freely
Now, ∆r = 7$i + 3$j + k$ − 0i$ − 0$j − 0k$ under gravity, ie, a = g
w′ = m(g − a) = m(g − g)
or ∆r = (7i$ + 3$j + k$ )
w′ = 0
Now, τ=r×F
∴ = (7i$ + 3$j + k$ ) × (− 3i$ + $j + 5k$ )
40. (c) Here, m = 50 g = 50 × 10− 3 kg
$i $j k$ c = 0.02, θ1 = 30° C, v = 840 ms − 1

or = 7 3 1 θ 2 = 0° C, M =? and L = 80 cal g − 1
−3 1 5 In going up, kinetic energy is converted into
potential energy
or = $i(15 − 1) − $j(35 + 3) + k$ (7 + 9) Energy converted into heat
⇒ = 14 $i − 38$j + 16k$ 1 1 mv 2
= mv 2 J = cal
2 2 J
37. (b) The magnitude of A is Heat lost by bullet to come to 0°C = cm∆θ
| A | = 32 + 4 2 = 5 = 0.02 × 50 × (30 − 0) = 30 cal-s
A unit vector parallel to A will be Heat spent in melting M gram of ice = M × L
$ $ mv 2
A$ = A = 3i + 4 j + 30 = M × L
|A| 5 2J
50 × 10− 3 × (840)2
Now, magnitude of B is + 30 = M × 80
2 × 4.2
| B| = (7)2 + (24)2 = 25
4200 + 30 = 80 M
Therefore, a vector parallel to A and having 4230
magnitude of B will be 25 A.$ ⇒ M=
$ $ 80
(3i + 4 j) or M = 52.88 g
= 25 = 15$i + 20$j
5
268 AP EAMCET Chapterwise Physics

Practice Set 3
1. The angle which the velocity vector of a 5. A particle moving in a straight line has
projectile thrown with a velocity v at an angle velocity v given by ν 2 = α − βy 2 , where α and β
θ to the horizontal will make with the are constants and y is its distance from a fixed
horizontal after time t of its being thrown up is point in the line. Find the time period and
θ
(a) θ (b) tan−   amplitude of motion.
t 
2π α 2π α
 v cos θ   v sin θ − gt  (a) , (b) ,
(c) tan− 1   (d) tan− 1   β β β β
 v sin θ − gt   v cos θ 
2π α 2π α
(c) , (d) ,
2. The system is pushed by a force F as shown in β β β β
figure. All surfaces are smooth except between
B and C. Friction coefficient between B and C is 6. A body of mass m falls from a height h on to
µ. Minimum value of F to prevent block B from the pan of a spring balance. The masses of the
downward slipping is pan and spring are negligible. The spring
A C constant of the spring is k. The body get
F B attached to the pan and starts executing SHM
2m m 2m in the vertical direction. Find the amplitude.
mg 2 kh (mg )2
(a) 1+ (b) mgh +
k mg 2k
 3   5 
(a)   mg (b)   mg k 2 hk 2k
 2µ  2 µ  (c) 1+ (d) mgh +
mg mg (mg )2
(c)   µ mg (d)   µ mg
5 3
2 2 7. The force required to move a body up a rough
inclined plane is double the force required to
3. When a tension F is applied the elongation
prevent the body from sliding down the
produced in uniform wire of length l, radius r
plane. The coefficient of friction, when the
is e. When tension 2F is applied, the
angle of inclination of the plane is 60° is
elongation produced in another uniform wire 1 1
of length 2l and radius 2r made of same (a) (b)
3 2
material is 1 1
(a) 0.5 e (b) 10
. e (c) 15
. e (d) 2.0 e (c) (d)
3 2
4. The maximum intensity in Young’s double az
α kθ
slit experiment is I 0 . Distance between the 8. In the relation p = e , where
slits is d = 5λ is the wavelength of β
monochromatic light used in the experiment. p is the pressure, z is distance
What will be the intensity of light in front of
k is the Boltzmann constant and
one of the slits on a screen at a distance
D = 10 d? θ is the temperature. The dimensional
I0 3I0 formula of β will be
(a) (b)
2 4 (a) [M0L2 T0 ] (b) [M1L2 T- 1 ]
I0 1 0 -1
(c) [M L T ] (d) [M0L2 T- 1 ]
(c) I0 (d)
4
Practice Set 3 269

9. A rectangular coil 3 cm × 3 cm consisting of 15. A drop of liquid pressed between two glass
100 turns carries a 0.1 A. If it produces a plates spreads into a circle of diameter 10 cm.
deflection 10 0 in a field of induction 0.1 T the Thickness of the liquid film is 0.5 mm and
couple per unit turns is coefficient of surface tension is 70 × 10 −3 Nm −1 .
(a) 9 × 10− 2 N-m/degree The force required to pull them apart
(a) 4.4 N (b) 1.1 N (c) 2.2 N (d) 3.6 N
(b) 9 × 10− 5 N-m/degree
(c) 9 × 10−6
N-m/degree
16. A steel ball of mass 0.1 kg falls freely from a
height of 10 m and bounces to a height of 5.4 m
(d) 0.9 N-m/degree from the ground. If the dissipated energy in
10. Assertion (A) In still air, a hydrogen-filled this process is absorbed by the ball, the rise in
balloon rises up to a certain height and then its temperature is
stops rising. (specific heat of steel = 460 Jkg − 10 C − 1 ,
Reason (R) The upthrust depends on the g = 10 ms − 2 )
density of hydrogen relative to that of air. (a) 0.01 °C (b) 0.1 °C (c) 1 °C (d) 1.1°C
(a) Both (A) and (R) are true and (R) is the correct 17. A tuning fork is found to give five beats in
explanation of (A). three seconds when sounded in conjunction
(b) Both (A) and (R) are true and (R) is not the with a stretched string vibrating transversely
correct explanation of (A). under a tension of either 10.2 kgf or 9.9 kgf.
(c) Only (A) is correct, but (R) is wrong. The frequency of the fork is approximately
(d) Both (A) and (R) are wrong.
(a) 237 Hz (b) 235 Hz (c) 223 Hz (d) 225 Hz
11. A block of mass m is pulled along a horizontal 18. In Millikan’s oil drop experiment an oil drop
surface by applying a force at an angle θ with having a charge is held stationary with an
the horizontal. If the block travels with a external potential difference of 400 V. If the
uniform velocity and has a displacement d size of the drop is doubled without any
and the coefficient of friction is µ, then work change in charge, the potential difference
done by the applied force is required to keep the drop stationary is
µ mgd µ mgd cos θ (a) 800 V (b) 1600 V (c) 3200 V (d) 200 V
(a) (b)
cos θ + µ sin θ cos θ + µ sin θ 3
µ mgd sin θ µ mgd cos θ 19. Two identical glass µ g = equiconvex lenses
(c) (d) 2
cos θ + µ sin θ cos θ − µ sin θ of focal length f are kept in contact. The space
12. The mass and diameter of a planet are two between the two lenses is filled with water
 4
times those of earth. If a seconds pendulum is µ w =  . The focal length of the combination
taken to it, the time period of the pendulum  3
(in second) is is
1 1 f 4f 3f
(a) (b) (a)f (b) (c) (d)
2 2 2 3 4
(c) 2 (d) 2 2 20. A conductor of resistance 3Ω is stretched
13. A bullet is fired on to a wooden plank. In uniformly till its length is doubled. The wire
is now bent in the form of an equilateral
penetrating through the plank it loses 10% of
triangle. The effective resistance between the
its KE. How many planks are required to just
ends of any side of the triangle (in ohm) is
stop the bullet? 9 8
(a) 20 (b) 15 (c) 10 (d) 5 (a) (b) (c) 2 (d) 1
2 3
14. Bulk modulus of water is 2 × 10 9 Nm − 2. The 21. A wheel which is initially at rest is subjected
change in pressure required to increase to a constant angular acceleration about its
density of water by 0.1% is axis. It rotates through an angle of 15° in time
(a) 2 × 109 Nm − 2 (b) 2 × 108 Nm − 2 t second. The increase in angle through which
−2 it rotates in the next 2t second is
(c) 2 × 10 Nm
6
(d) 2 × 104 Nm − 2
(a) 90° (b) 120° (c) 30° (d) 45°
270 AP EAMCET Chapterwise Physics

22. The potential energy of certain spring when 28. Five moles of hydrogen initially at STP is
stretched through a distance s is 10 J, the compressed adiabatically so that its
amount of work (in joule) that must be done temperature becomes 673 K. The increase in
on this spring to stretch it through additional internal energy of the gas (in kilojoule) is
distance s will be (Take, R = 8.3 J/ mol-K; γ = 1.4 for diatomic
(a) 30 (b) 40 (c) 10 (d) 20 gas)
(a) 80.5 (b) 21.55
23. Two particles of masses m1 and m2 are
(c) 41.50 (d) 65.55
connected by a rigid massless rod of length r
to constitute a dumb-bell which is free to 29. The magnetic induction at the centre of
move in the plane. The moment of inertia of current carrying circular coil of radius 10 cm
the dumb-bell about an axis perpendicular to is 5 5 times the magnetic induction at a point
the plane passing through the centre of mass is on its axis.The distance of the point from the
m1m2 r 2 centre of the coil (in cm) is
(a) (b) (m1 + m2 ) r 2
m1 + m2 (a) 5 (b) 10 (c) 20 (d) 25
2
(c)
m1m2 r
(d) (m1 − m2 ) r 2
30. Two short bar magnets of magnetic moment
m1 − m2 M each are arranged at the opposite corners of
a square of side such that their centres
24. A satellite with kinetic energy E is revolving coincide with the corners and their axis are
round the earth in a circular orbit. The parallel. If the like poles are in the same
minimum additional kinetic energy required direction, the magnetic induction at any of
for it to escape into outer space is the other corners of the square is
(a) 2 E (b) 2 E µ0 M µ 0 2M µ M µ M2
E (a) (b) (c) 0 (d) 0
(c) (d) E 4π d 3 4π d 3
4 π 2d 3
4 π 2d 3
2
31. In nuclear fission the percentage of mass
25. The densities of a liquid in gcm − 3 at 0°C and converted into energy is about
100°C are respectively 1.0127 and 1. A specific (a) 10% (b) 0.01% (c) 0.1% (d) 1%
gravity bottle is filed with 300 g of the liquid
at 0°C upto the rim and it is heated to 100°C. 32. For a transistor amplifier in common emitter
Then the mass of the liquid expelled configuration for load impendence of 1 kΩ
(in gram) is [Take, coefficient of linear (h fe = 50 and hoe = 25 × 10 − 6 ) the current gain
expansion of glass = 9 × 10 − 6 /°C] is
3 3 (a) − 52 (b) − 157
. (c) − 24.8 (d)− 4878
.
(a) (b)
101
. 101.
33. Determine the speed with which the earth
3.81 3.81
(c) (d) have to rotate on its axis so that a person on the
10127
. 0.0127  3
equator weigh   th as much as at present.
26. A gas under constant pressure of 4.5 × 10 5 Pa  5
when subjected to 800 kJ of heat, changes the Take the equatorial radius 6400 km.
volume from 0.5 m 3 to 2.0 m 3 . The change in (a) 7.826 × 10− 4 rad/s (b) 7.826 × 10− 5 rad/s
−6
internal energy of the gas is (c) 7.826 × 10 rad/s . × 10− 4 rad/s
(d) 7826
. × 10 J
(a) 675 5
. × 10 J
(b) 525 5

34. A spot light S rotates in a horizontal plane


. × 105 J
(c) 325 (d) 1. 25 × 105 J
with a constant angular velocity of 0.1 rad/s.
27. A spherical droplet having a potential of 2.5 V The spot P moves along the wall at a distance
is obtained as a result of merging of 125 3 m. What is the velocity of the spot P when
identical droplets. Find the potential of the θ = 45 °? Where θ is the angle between the line
constituent droplet joining the spot light to spot and the wall.
(a) 0.4 V (b) 0.5 V (c) 0.2 V (d) 0.1 V (a) 0.5 ms − 1 (b) 0.6 ms − 1
(c) 5.0 ms − 1 (d) 6.0 ms − 1
Practice Set 3 271

35. A compound microscope is used to enlarge an 38. In a new system of units called star units.
object kept at a distance of 0.03 m from its 1 kg* = 10 kg; 1 m* = 1 km and 1 s*= 1 min.
objective, which consists of several convex What will be the value of 1 J of energy in the
lenses in contact, and has focal length 0.02 m. new system?
If the lens of focal length 0.1 m is removed (a) 3.6 × 10− 4 ms − 1 J* (b) 6.3 × 10− 4 ms − 1 J*
from the objective, find out the distance by (c) 3.6 × 10− 5 ms − 1 J* (d) 6.3 × 10− 5 ms − 1 J*
which the eye-piece of the microscope must
be moved to refocus the image. 39. A particle is moving eastwards with a velocity
of 5 ms − 1 . In 10 s, the velocity changes to
(a) 9 cm (b) 8 cm
(c) 9.2 cm (d) 9.5 cm
5 ms − 1 north-wards. The average acceleration
in this time is
36. A liquid drop of diameter D breaks up into 27 (a) zero
equal tiny drops. If σ is the surface tension of (b)
1
ms − 2 towards north-west
liquid, then increase in surface energy is 2
1
(a) 2 πD2 σ (b) 4 πD2 σ (c) ms − 2 towards north-east
2
(c) 6 πD2 σ (d) πD2 σ
1
(d) ms − 2 towards north
37. Carnot engine takes in a thousand kilo 2
calories of heat from a reservoir at 627°C and
40. When 0.005 A current flows through a
exhausts it to a sink at 27°C. What is the moving coil galvanometer to read 5 V using
efficiency of the engine? an external resistance of 975 Ω. The
(a) 66.67% (b) 6.667% resistance of the galvanometer (in ohm) is
(c) 666.7% (d) None of these (a) 5 (b) 10 (c) 15 (d) 25

Answers
1. (d) 2. (b) 3. (b) 4. (a) 5. (a) 6. (a) 7. (c) 8. (a) 9. (b) 10. (b)
11. (b) 12. (d) 13. (c) 14. (c) 15. (c) 16. (b) 17. (d) 18. (c) 19. (d) 20. (b)
21. (b) 22. (a) 23. (a) 24. (d) 25. (c) 26. (d) 27. (d) 28. (c) 29. (c) 30. (a)
31. (c) 32. (d) 33. (a) 34. (b) 35. (a) 36. (a) 37. (a) 38. (a) 39. (b) 40. (d)

Solutions
1. (d) Dividing Eq. (ii) by Eq. (i), we get
v′ sin α v′ sin α v sin θ − gt
v′ =
v′ cos α v cos θ
v α v′ cos α v sin θ − gt
v sin θ or tan α =
v cos θ
 v sin θ − gt 
or α = tan− 1  
θ  v cos θ 
v cos θ
According to the figure, 2. (b) Horizontal acceleration of the system is
v′ cos α = v cos θ F F
…(i) α= = …(i)
and v′ sin α = v sin θ − gt …(ii) 2m + m + 2m 5m
272 AP EAMCET Chapterwise Physics

Let N be the normal reaction between B and C. dv


or = − βy …(ii)
C dt
It means acceleration
N
dv
a= or a = − βy
dt
a As, a ∝ y and − ve sign
Free body diagram of C gives It show that acceleration is directed towards mean
2 position. So, if the particle is left free, it will
N = 2ma = F [from equ. (ii)] execute SHM.
5
Here, ω2 = β or ω = β
Now B will not slide downward if

µ N ≥ mB g or µ  F  ≥ mg T=
2 Time period,
5  ω

or F≥
5
mg ⇒ Fmin =
5
mg T=
2µ 2µ β
We know that, v = 0
3. (b) We know that,
When, y= A
e1 F1 L1 r22
= × × 0 = α − βA2
e 2 F2 L2 r12
α
e1 F l 4r 2 or A=
⇒ = ⋅ =1 β
e 2 2F 2l r 2
α
⇒ e1 = e 2 ⇒ e 2 = e Amplitude, A=
β
4. (a) Path difference,
yd d 5λ 6. (a) When a body of mass m falls from height h on
∆x = ⇒ y= = the pan of spring balance, let the spring of spring
D 2 2 balance gets compressed by a length x.
and D = 10d = 50λ
5λ 5λ
So, ∆x = × m
2 50λ
λ
or ∆x =
4 k
Corresponding phase difference will be
2π 2π λ π
∆φ = ⋅ ∆x =     =
λ  λ   4  2
∆φ π
or =
2 4 The loss of PE of the mass = mg (h + x). The gain is
From law of Malus, elastic potential energy by the spring due to
∆φ compression
I = I 0 cos2   1
 2 = kx 2
2
π
= I 0 cos2   According to law of conservation of energy, we
 4
have
I0
or I= 1
mg (h + x) = kx 2
2 2
5. (a) Given, v2 = α − βy2 …(i) or
1 2
kx − mgx − mgh = 0
2
Differentiating it w.r.t. time t, we have
2mg 2mgh
dv dy x2 − x− =0
2v = − β 2y k k
dt dt
Practice Set 3 273

Solving the quadratic equation, we have 9. (b) Couple per unit turn,
2
2mg
±  2mg  +  8mgh  . × 9 × 10− 4 × 01
NIAB 100 × 01 .
    C= =
k  k   k  θ
x= 10
2 = 9 × 10− 5 N-m/degree
mg mg 2kh
= ± 1+
k k mg 10. (b) Density of hydrogen is less than that of air, so
that balloon filled with air gets upthrust, in this
In equilibrium position, the spring will be way it rises. But, at the certain height it stop rising
mg as its density will much the density of air.
compressed by distance . If A is the amplitude
k
of oscillation then 11. (b) Because the block moves with a uniform
velocity, the resultant force is zero.
mg mg mg 2kh
+ A= ± 1+ R F
k k k mg
mg 2kh q
or A= 1+ F′ F cos q
k mg
mg
7. (c) For upward motion,
Resolving F into horizontal component F cos θ and
Fup = mg(sin θ + µ cos θ) ...(i)
vertical component F sin θ, we get
For downward motion,
R + F sin θ = mg
Fdown = mg(sinθ − µ cosθ) ...(ii)
or R = mg − F sin θ
Also, F ′ = µR
= µ (mg − F sin θ)
µ
But, F cos θ = F ′
or F cos θ = µ (mg − F sin θ)
mg sinθ mg cosθ or F (cos θ + µ sin θ) = µ mg
mg
µ mg
or F=
θ cos θ + µ sin θ

According to the question, Work, W = F ⋅ s cos θ


Fup = 2Fdown µ mgd cos θ
∴ W= [Q s = d]
cos θ + µ sin θ
mg(sin θ + µ cos θ) = 2mg(sin θ − µ cos θ)
sin θ + µ cos θ = 2 sin θ − 2µ cos θ 12. (d) Given, mp = 2me
3µ cos θ = sin θ and dp = 2De or rp = 2re
1 1
µ = tan θ ⇒ µ = × tan 60° T1 = 2 s
3 3
= time period of second pendulum
1 1
⇒ µ= 3= I
3 3 As T1 = 2π
ge
8. (a) Argument of exponential term must be
dimensionless I
and T2 = 2π
α z  gp
 k θ  = [M L T ]
0 0 0

  T12 gp
∴ =
Now, [kθ] = [Energy] = [ML2T− 2] T22 ge
Gm
[k θ] [ML2T− 2] Now, ge = 2 e
[α] = = = [MLT – 2] re
[z] [L]
Gmp
[α] [MLT− 2] and gp = 2
[β] = = = [M 0L2T0 ] rp
[p] [ML– 1 T − 2]
274 AP EAMCET Chapterwise Physics

gp mp re2 2me r2 1 1−
stress di 1000
= =
∴ = = × e2 = As
ge me rp2 me 4re 2 B d f 1001

T12 1 stress 1
or = =
B 1001
T22 2
B 2 × 109
4 1 or stress = =
or = 1001 1001
T22 2
= 2 × 106 Nm − 2
⇒ T2 = 2 2 s
15. (c) Force required to pull the glass plates apart
13. (c) Let x be the thickness of the plank. Velocity
2πr 2T
after penetrating one plank will be F=
1 9 1 d
mv′2 = × mv2
2 10 2 2 × 22 × (5 × 10− 2)2 × 70 × 10− 3
=
9 2 7 × 0.5 × 10− 3
or v′ 2 = v
10 10 × 22 × 10− 2
= = 22
. N
Retardation will be 1
9 2
v − v2 = − 2ax 16. (b) Loss in energy
10
2 = mg (h − h′)
v
or a= = 01
. × 10 × (10 − 54
. ) = 4.6 J
20 x
Total distance travelled before coming to rest will Now, 4.6 J = ms∆θ
be given by = 01
. × 460 × ∆θ
v2 or ∆θ = 01
. °C
v = 2aX = 2 ⋅
2
⋅X
20 x 17. (d) If n be the frequency of tuning fork, then
X
or = 10 5 1 10.2 × g
x n+ = …(i)
3 2L m
So, 10 planks will be required.
5 1 9.9 × g
and n− = …(ii)
14. (c) Initial density, di = M …(i) 3 2L m
V
M Dividing Eq. (i) by Eq. (ii), we get
Final density, d f = …(ii) 5
V−u n+
3 = 10.2 = 1 + 1
Dividing Eq. (i) by Eq. (ii), we get 5 9.9 66
n−
di V−u u 3
= =1 −
df V V 5
n+
u stress or 3 = 67
But, = strain = 5 66
V B n−
3
di stress
∴ =1 − or n = 225Hz
df B
18. (c) For oil drop to be stationary,
Percentage change in density
Eq = mg
 d − di 
= f  × 100 = 01
. V 4
 di  ⇒ q = πR3ρg
d 3
df 1 V ∝ r3
or −1 =
di 1000 If size is doubled potential difference becomes 8
df 1001 times to the original value.
or =
di 1000 ∴ V ′ = 8 V = 8 × 400 = 3200 V
Practice Set 3 275
1 2
19. (d) From lens Maker’s formula, 15° = 0 + αt
2
= (1.5 − 1)  + 
1 1 1
or R= f 1 2
f  R R ⇒ 15° = αt ...(i)
2
For the water lens, For the second conditions (time = 3t second)
1 4 1  2
=  − 1  − −  =  − 
1 1 1 1
θ1 = α (3t)2 = (α) 9 t 2 ...(ii)
f′  3   R R 3  f  2 2
1
1
=
2 So, ∆θ = θ1 − αt 2
f ′ − 3f 2
1 2 1 2
Now using, ∆θ = 9 × αt − αt
2 2
1 1 1 1
= + + 1 2
= 8 αt
F f1 f2 f3 2
or
1 1 1 1 2
= + + = −
2 = 8 × 15° = 120° (from Eq. (i))
F f f f f 3f
22. (a) Work done to stretch a spring by distance x
1 4 3f
or = or F = 1 2 1 2
F 3f 4 U= kx = ks = 10 J
2 2
20. (b) Resistance, R = ρl × l = ρl = ρ l 2
2
1
= k (s + s)2
A l Al V 2
(here, volume) 1
= k (4s 2) = 40 J
R = 3Ω 2
If the length is doubled, let the new resistance be ∴ ∆W = U ′ − U = 40 J − 10 J = 30 J
R′.
ρ 23. (a) Suppose C be the centre of mass of the
R′ = (2l)2 dumb-bell. Let the particles of masses m1 and m2 be
V
placed at distance r1 and r2 from C. Hence,moment
ρ
= 4  l 2  = 4R = 4 × 3 = 12 Ω of inertia of given system about an axis passing
V  through CM
The wire is bent into an equilateral triangle then I = m1 r12 + m2r22 …(i)
resistance of each side
According to definition of centre of mass
12
= =4Ω m1 r1 = m2r2 …(ii)
3
A r1 + r2 = r …(iii)
Solving for r1 and r2
From Eqs. (ii) and (iii), we get
4Ω 4Ω
m2r
r1 = …(iv)
m1 + m2
C B m1 r
4Ω r2 = …(v)
m1 + m2
Sides AB and BC are in series and their equivalent
is in parallel with CA. Putting Eqs. (iv) and (v) in Eq. (i), we get
8×4 8 m1 m2r 2
∴ Effective resistance = = Ω I=
8+ 4 3 m1 + m2
21. (b) If angular acceleration is constant, we have 24. (d) For a satellite revolving round the earth in
1 2
θ = ω0 t + αt circular orbit radius r
2
mv02 GMm GM
The given θ = 15° = or v0 =
r r2 r
ω0 = 0 1 1 GMm
As E = mv0 =
2
For the first conditions (time = t second) 2 2 r
276 AP EAMCET Chapterwise Physics

2GM 4 4
and ve2 = ∴ πR3 = 125 × πr 3
r 3 3
1 1 m × 2GM or R = 5r
E e = mve2 = ×
2 2 r 125q
As . V=
V = 25
=
mGM
= 2E 4 πε0 (5r)
r 125 q
or . =
25 ×
Total energy needed = 2E = E + E 5 4 πε0 r
Extra energy needed = E
or . = 25 × V0
25
25. (c) Density of liquid at 0°C .
25
or V0 =
ρ0 = 1.0127 5
and density of liquid at 100°C ⇒ V0 = 01
. V
ρ100 = 1 28. (c) Work done by an ideal gas is adiabatic
m0 = 300 g expansion
Volume of the bottle = V dU = n
R
dT
Mass left at 100°C = m γ −1
m 300 R
ρ0 = 0 = = 1.0127 dU = n (T2 − T1)
V V γ −1
300 T1 = 273 K
V= Given,
1.0127
m T2 = 673 K
and ρ100 =
V R = 8.3J/mol-K
m
or 1= n= 5
300
1.0127 γ = 1.4
300 8.3
or m= ∴ dU = 5 × (673 − 273)
1.0127 1.4 − 1
300 381
.
Mass expelled = m0 − m = 300 − = =
41.5
× 400 J
1.0127 1.0127 0.4
26. (d) Given, p = 4.5 × 105 Pa = 415 × 100 J = 41.50kJ
dQ = 800 kJ 29. (c) Magnetic field at any point on the axis of
V1 = 0.5 m 3 current carrying coil at a distance x from the centre
is given by
V2 = 2 m 3
µ 0 Ir 2
Work done, dW = p (V2 − V1) Ba = 3
…(i)

= 4.5 × 10 (2 − 0.5)
5 2(r +
2
x 2) 2

= 6.75 × 10 J 5 At the centre x = 0


µ 0 NIr 3 µ 0 NI
Change in internal energy Bc = = …(ii)
2r 3 2r
dU = dQ − dW
Dividing Eq. (ii) by Eq. (i), we get
= 800 × 103 − 6.75 × 105 3

= 1.25 × 105 J Bc µ 0 NI 2(r 2 + x 2) 2


= ×
q Ba 2r µ 0 NIr 2
27. (d) Potential of drop, V =
4 πε0 r Given, Bc = 5 5 Ba
3
As volume before merging
(r 2 + x 2) 2
= volume after merging ∴ 5 5=
r3
Practice Set 3 277
3
As, w′ = w − mRω2
(100 + x 2) 2
or 5 5= 3
1000 or mg = mg − mRω2
5
Squaring both sides, we get
3 2
125 × 106 = (100 + x 2)3 mRω2 = mg − mg = mg
5 5
or 100 + x 2 = 5 × 102 = 500 or x = 20 cm 2g 2 × 9.8
or ω= =
5R 5 × 6400 × 103
30. (a) Magnetic induction at point A due to magnet at
B (axial point) = 7.826 × 10− 4 rad/s
µ 0 2M
B1 = (along BA) 34. (b) In ∆OSP , tan φ = OP = x
4 π d3 OS 3
S N or x = 3 tan φ
D
P

B θ
B2 x
B1
A
d φ
S
Magnetic induction at point A due to magnet at D 3m O
(equatorial point). Differentiating, we get
µ M dφ
B2 = 0 3 dx
= 3 sec2 φ
4π d dt dt
µ
Resultant, B = B1 − B2 = 0  3 
M or v = 3 sec 2 φ(ω)
4π  d 
As φ = 90° − θ = 90° − 45°
31. (c) Nuclear fission reaction is φ = 45°
92 U
235
+ 0 n1 → (92 U236) secφ = sec 45° = 2
→ 56 Ba 144
+ 36Kr + 30 n + energy
92 1
⇒ v = 3( 2) × 01
.2

Mass of reactants = 236.0526 amu or v = 0.6 ms − 1


Mass of products = 2358293
. amu
Therefore, difference in mass between reactants
35. (a) Here, f = 0.02 m
and products is converted into energy u = − 0.03 m
= 236.0526 − 2358293
. Frm lens formula,
= 0.2233 amu 1 1 1
− =
Thus, percentage of mass converted into energy v u f
0.2233 1 1 1 1 1 1
= × 100 ≈ 01
.% or = + = − =
236.0526 v f u 0.02 0.03 0.06

32. (d) In CE configuration, ⇒ v = 0.06 m


− h fe When one lens of f1 = 01 . m is removed. The new
Ai = focal length ( f2) of objective will be given by
1 + hoe RL
1 1 1
− 50 + =
= = − 48.78 f1 f2 f
1 + 25 × 10−6 × 103
1 1 1 1 1 4
or = − = − =
33. (a) Here, w = mg f2 f f1 0.02 01. 01
.
.
01
and w′ =   mg f2 = = 0.025 m
3 or
 5 4
278 AP EAMCET Chapterwise Physics

If v′ is distance of new image, then from 38. (a) Joule is a unit of energy.
1 1 1
+ =  1  kg  1 m* 
2
v ' u f2    
1 kg (1 m)  2
 1000 
1 1 1 ∴ 1J= = 10
or = + (1 s) 2
 1 s *
v′ f2 u  
 60 
1 1
= + . × 10− 4 kg* m *2s *− 2
= 36
0.025 0.03
. × 10− 4 ms−1 J*
= 36
⇒ v′ = 015
. m
Displacement of eye-piece = v′ − v 39. (b) According to the figure,
= 015
. − 0.06 = 0.09 m = 9 cm ∆v = v 2 − v1
36. (a) Let r be the radius of each tiny drop. = v12 + v22 − 2v1 v2 cos 90°
Therefore,
3
–v1
4  D 4
π   = 27 × πr 3
3  2 3
1  D v2 = 5 m/s
∆v
or r=  
3  2 90°
D
or r= v1 = 5 m/s
6
Increase in surface energy = 52 + 52 = 5 2
= surface tension × increases in surface area Average acceleration
 D 
2
= σ ×  27 × 4 πr 3 − 4 π    =
∆v 5 2
= =
1
ms − 2
 2 ∆t
  10 2
 2
D2  directed towards north-west.
= σ 4 π  27 ×   −
D
 6 
 4 40. (d) Given, V = 5volt
2D 2 I g = 0.005 A
= σ 4π ×
4 and R = 975 Ω
= 2π D 2σ External resistance
V
37. (a) Given, Q1 = 1000 k-cal = 106 cal R= −G
Ig
T1 = 627° C = 627 + 273 = 900 K V
GF = −R
T2 = 27°C = 27 + 273 = 300 K Ig
T 300 2
Also, η =1 − 2 =1 − = 5
T1 900 3 G= − 975
0.005
2
= × 100 or η = 66.67% G = 25 Ω
3

You might also like